Anda di halaman 1dari 358

1

Autopsy of a 70-year-old woman reveals a papillary growth within the left ventricular chamber. The
growth consists of a small mass of finger-like projections attached to the mitral valve,
without associated valvular or other cardiac abnormalities. Histologically, each papillary
structure is composed of a core of fibrous tissue lined by thickened endothelium. The patient
did not have any history of cardiac disease or evidence of thromboembolism. Which of the
following is the most likely diagnosis?
A. Acute mural thrombus
B. Cardiac myxoma
C. Infective endocarditis
D. Nonbacterial thrombotic endocarditis
E. Papillary fibroelastoma
Explanation:
The correct answer is E. Gross and microscopic features of this small mass in the left
ventricle are entirely consistent with papillary fibroelastoma. This lesion is not neoplastic,
despite the sound of the name. It probably results from organized thrombi forming on the
endocardial surfaces of the mitral valve or left ventricular cavity. Papillary fibroelastomas
are usually clinically silent and are discovered at autopsy as an incidental finding.
An acute mural thrombus (choice A) would not have a core of fibrous tissue. Acute mural
thrombosis usually develops as a result of stasis in the ventricular cavities, in association
with ventricular enlargement, myocardial infarction, or ventricular aneurysm, for example.
Thrombosis often develops in the atria when there is atrial fibrillation.
Cardiac myxoma (choice B) is the most frequent primary cardiac neoplasm. It is benign and
consists of stellate mesenchymal cells within a myxoid background. Since the left atrium is the
most frequent location, this tumor can produce mitral stenosis by a ball-valve effect.
Both forms of endocarditis are associated with formation of vegetations attached to the surface
of the atrioventricular valves. Vegetations of infective endocarditis (choice C) are bulky and
composed of fibrin, bacteria, and inflammatory cells. Since nonbacterial thrombotic
endocarditis (choice D) is caused by hypercoagulable states, the vegetations consist of
aggregates of fibrin but few inflammatory cells and no bacteria.
Note that all of the above conditions may lead to systemic embolization. Fragments of
vegetations, thrombi, myxoma, and papillary fibroelastoma may detach and be released into the
bloodstream, causing infarcts.
A 45-year-old man presents to his physician with hematuria. Renal biopsy demonstrates a focal
necrotizing glomerulitis with crescent formation. The patient has a history of intermittent
hemoptysis and intermittent chest pain of moderate intensity. A previous chest x-ray had
demonstrated multiple opacities, some of which were cavitated. The patient also has chronic
cold-like nasal symptoms. Which of the following is the most likely diagnosis?
A. Aspergillosis
B. Polyarteritis nodosa
C. Renal carcinoma metastatic to the lung

D. Tuberculosis
E. Wegener's granulomatosis
Explanation:
The correct answer is E. While in real life, other diseases (or combinations of diseases) may
occasionally cause concurrent pulmonary, sinus, and renal involvement, if you see this pattern
on a test question, you should immediately think of Wegener's granulomatosis. This is a rare
focal necrotizing vasculitis of still unclear etiology, which also features prominent
granulomas, some of which are centered on the vascular lesions. The vasculitis and granulomas
can involve the entire respiratory tract, and an easily obtained nasal biopsy may sometimes
yield the diagnosis. The renal involvement is usually in the form of a necrotizing
glomerulonephritis. The disease typically affects middle aged men, and its formerly poor
prognosis has been improved by corticosteroid and cyclophosphamide therapy.
Aspergillosis (choice A) can cause prominent lung disease, but does not usually have renal
involvement.
Polyarteritis nodosa (choice B) is a possibility, but usually spares the lungs.
Renal cell carcinoma (choice C) might cause lung nodules, but there is no evidence of tumor in
the kidney.
Tuberculosis (choice D) can cause prominent lung disease, but does not usually have renal
involvement.
A 55-year-old man presents to a physician with jaundice. Ultrasonography demonstrates a 5 cm mass in the
head of the pancreas. Endoscopic retrograde cholangiopancreatography with cytologic sampling
demonstrates cells with large hyperchromatic nuclei and a high nuclear/cytoplasmic ratio. A few
small glands composed of these cells are also seen in the cytologic preparation. The overall
prognosis for this man will be most similar to that of a patient with which of the following
malignancies?
A. Adenocarcinoma of the breast
B. Adenocarcinoma of the colon
C. Adenocarcinoma of the esophagus
D. Adenocarcinoma of the prostate
E. Primary gastric lymphoma
Explanation:
The correct answer is C. The patient probably has pancreatic adenocarcinoma. This cancer
carries one of the worst prognoses, with a 3.5% overall 5 year survival rate despite all
attempts at aggressive management. The prognosis is also bleak with adenocarcinoma of the
esophagus, with a 10% overall 5 year survival rate.
Adenocarcinoma of the breast (choice A) now has an overall 5 year survival rate of 60-70%.
Adenocarcinoma of the colon (choice B) now has an overall 5 year survival rate of 50-60%.

3
Adenocarcinoma of the prostate (choice D) now has an overall 5 year survival rate of 50-70%.
Primary gastric lymphoma (choice E) has an overall survival 5 year survival rate of 75-85%.
Biopsy of a persistent exophytic area on the vulva of a 60-year-old woman demonstrates a squamous
epithelial lesion. No koilocytes are seen. The lesions show papillary projections composed of
disordered, squamous epithelium with well-differentiated cells. The basement membrane at the
dermal-epidermal junction is focally disrupted by squamous cell groups extending deep into the
dermis. Which of the following diagnoses is most accurate?
A. Condyloma acuminatum
B. Extramammary Paget's disease
C. Vulvar intraepithelial neoplasia
D. Vulvar melanoma
E. Vulvar squamous cell carcinoma
Explanation:
The correct answer is E. The disruption of the basement membrane with cell groups extending
deep into the dermis indicates that this is invasive squamous cell carcinoma of the vulva,
which may arise in vulvar intraepithelial neoplasia, in condyloma, or spontaneously. The latter
type (which is likely here) tends to be well differentiated, while the former two are often
poorly differentiated.
Condyloma acuminatum (choice A) would contain koilocytes and would not cross the basement
membrane.
Extramammary Paget's disease (choice B) would probably contain individual tumor cells that
stain for mucin.
The lesion of vulvar intraepithelial neoplasia (choice C) does not cross the basement membrane.
Vulvar melanoma (choice D) is composed of melanocytes that would mark with S100 or HMB-45.
Which of the following conditions is associated with overexpression of bcl-2?
A. Acute lymphoblastic leukemia
B. Burkitt lymphoma
C. Follicular lymphoma
D. Multiple myeloma
E. Small lymphocytic lymphoma
Explanation:
The correct answer is C.bcl-2 inhibits apoptosis; a (14;18) translocation resulting in
overexpression of the bcl-2 protein in B lymphocytes causes apoptosis of neoplastic cells to be
permanently inhibited, producing follicular lymphoma. Follicular lymphoma involves the lymph
nodes and bone marrow of older people and is characterized by slow, but relentless, growth.

4
Thus, this lymphoma allows a long survival even without any treatment. Indeed, follicular
lymphoma is resistant to aggressive therapy because of its low mitotic activity.
Acute lymphoblastic leukemia (ALL) (choice A) is a neoplasm of pre-B cells that affects
children. Different molecular mechanisms probably account for different cases. Hyperdiploidy,
the Philadelphia chromosome, t(12;21), and t(4;11) are variably found in patients with ALL.
Burkitt lymphoma (choice B) occurs in different clinical settings, including the African
endemic, sporadic nonendemic, and AIDS-associated forms. Histologically, all these forms are
identical and are derived from neoplastic transformation of B cells expressing surface IgM.
Common to all forms are translocations involving the c-myc oncogene (chromosome 8) and Ig loci
(chromosome 14). These translocations lead to overexpression of c-myc, which encodes a nuclear
transcription protein, resulting in sustained cell replication. The African form is associated
with Epstein-Barr virus infection.
Multiple myeloma (choice D) is a neoplasm of terminally differentiated B lymphocytes that
secrete large amounts of monoclonal Ig or its fragments (light or heavy chains). Approximately
25% of cases have translocations that lead to overexpression of FGFR3, a receptor for
fibroblast growth factor implicated in control of cell proliferation.
Small lymphocytic lymphoma (choice E), a B cell neoplasm, is equivalent to chronic lymphocytic
leukemia. Neoplastic cells resemble a normal subset of circulating mature B cells that express
CD5. Various types of chromosomal abnormalities have been found in small lymphocytic
lymphoma/chronic lymphocytic leukemia, but none affecting the bcl-2 locus.
A 10-year-old boy with history of epilepsy and mental retardation is brought to a specialty clinic for
evaluation. Physical examination is remarkable for several ovoid hypopigmented areas on the
trunk and large numbers of red and yellow papules on the face, particularly near the mouth.
Biopsy of the papules demonstrates angiofibromata. This patient is most likely to have which of
the following central nervous system pathologies?
A. Acoustic neuromas
B. Capillary hemangioblastomas
C. Herniation of cerebellar tonsils into foramen magnum
D. Large cortical hamartomas
E. Leptomeningeal angiomatosis
Explanation:
The correct answer is D. The disease is tuberous sclerosis. The facial angiofibromata are also
called adenoma sebaceum, and the hypopigmented patches on the trunk are called ash-leaf spots.
This disease is one of the neurocutaneous disorders called phacomatoses. Tuberous sclerosis is
inherited as an autosomal dominant trait, and epilepsy and mental retardation are commonly seen
in this disorder. Large, firm, white hamartomatous nodules (tubers) are seen in the cortex and
in subependymal sites. The tubers consist of aberrantly arranged neurons and/or glia. Patients
may also have pancreatic cysts, renal angiomyolipomas, and cardiac rhabdomyomas. Rarely, an
astrocytoma will arise in a tuber.
Acoustic neuromas (choice A) are a feature of neurofibromatosis Type II.
Capillary hemangioblastomas (choice B) are a feature of Von Hippel-Lindau syndrome.

5
Herniation of cerebellar tonsils (choice C) is a feature of Arnold-Chiari malformation.
Leptomeningeal angiomatosis (choice E) is a feature of Sturge-Weber disease.
A 30-year-old African American woman has a chest x-ray that shows hilar lymphadenopathy and diffuse
abnormalities of the lung parenchyma. Biopsy reveals non-caseating granulomas. Acid fast,
silver, and PAS stains on the granulomas are negative. Which of the following is the most likely
diagnosis?
A. Coccidioidomycosis
B. Histoplasmosis
C. Leprosy
D. Sarcoidosis
E. Tuberculosis
Explanation:
The correct answer is D. Sarcoidosis is a common granulomatous disease, the etiology of which
remains unclear. It is a diagnosis of exclusion, and mycobacterial (acid fast) and fungal
(silver stain, PAS) are used to rule out infectious etiologies. The granulomas are
characteristically small, non-caseating (hard) granulomas that may contain giant cells.
Sarcoidosis has an initial predilection for the lungs and the hilar lymph nodes, but with time
may spread to involve many organs. In the United States, sarcoidosis is much more common in
people of African ancestry. In Europe, it mainly affects whites. There is a slight female
preponderance. Patients can have a progressive course or experience exacerbations and
remissions. Some patients recover with little residual damage; those who die typically do so
from pulmonary fibrosis and cor pulmonale.
The causative agent of coccidioidomycosis (choice A), Coccidioides immitis, or the causative
agent of histoplasmosis (choice B), Histoplasma capsulatum, would stain with PAS and silver
stain.
The causative agent of leprosy (choice C), Mycobacterium leprae, or the causative agent of
tuberculosis (choice E), Mycobacterium tuberculosis, would stain with acid fast stain.
A patient presents with a severe form of atopic asthma. Which of the following changes would most likely
be found in this patient's blood?
A. Basophilic leukocytosis
B. Eosinophilic leukocytosis
C. Lymphocytosis
D. Monocytosis
E. Neutrophilic leukocytosis
Explanation:
The correct answer is B. An increased number of eosinophils (AKA eosinophilia) occurs in
association with several conditions, the most frequent of which are immune-mediated diseases

6
(e.g., asthma, hay fever, and pemphigus vulgaris) and parasitic infestations. This is due to an
absolute increase in the number of circulating eosinophils, brought about by IL-5, which
stimulates differentiation of eosinophilic precursor cells in the bone marrow.
Basophilic leukocytosis (choice A) is a rare event that is sometimes observed in association
with chronic myelogenous leukemia. It is not seen in patients with asthma.
Lymphocytosis (choice C) may result from a vast array of conditions, but it is not typical of
asthma or other allergic diseases. Lymphocytosis may develop in response to a number of
infections (e.g., brucellosis, whooping cough, hepatitis, infectious mononucleosis, and
tuberculosis) or manifest as part of chronic lymphocytic leukemia.
Monocytosis (choice D) refers to an increase in number of monocytes, which are circulating
macrophages. Chronic infections (e.g., tuberculosis, rickettsiosis, and malaria) and chronic
inflammatory conditions (e.g., collagen vascular diseases and inflammatory bowel disease) are
the most common underlying causes.
Neutrophilic leukocytosis (choice E) is a typical systemic reaction to acute and chronic
infections, especially those due to bacteria. The increase in neutrophil number is mediated by
IL-1 and TNF, which induce a rapid release of neutrophils from the bone marrow in acute
infections and stimulate proliferation of bone marrow precursors in chronic infections.
A patient with long term severe hypertension develops progressive dementia. CT scan of the head
demonstrates a diffuse loss of deep hemispheric white matter. Which of the following terms best
describes the pathological process that is occurring?
A. Anemic infarcts
B. Hemorrhagic infarcts
C. Hypertensive encephalopathy
D. Lacunae
E. Subcortical leukoencephalopathy
Explanation:
The correct answer is E. This patient has subcortical leukoencephalopathy (Binswanger's
disease), which is one of the neurologic syndromes associated with hypertension. It is
uncommon, but obviously devastating. The histologic findings are diffuse, irregular loss of
axons and myelin accompanied by widespread gliosis. Small infarcts may be seen in the frontal
lobes. The pathologic mechanism may be damage caused by severe arteriolosclerosis. None of the
other choices would produce diffuse subcortical white matter involvement.
Anemic infarcts (choice A) can be seen in hypertensive patients as a consequence of
atherosclerotic thromboembolic events.
Hemorrhagic infarcts (choice B) can also be seen in hypertensive patients as a consequence of
atherosclerotic thromboembolic events followed by reperfusion. They tend to occur in gray
matter or at the gray-white junction.
Hypertensive encephalopathy (choice C) is an acute generalized dysfunction of the brain that
can occur in malignant hypertension or other very severe hypertensive processes. The primary
pathology is seen in cerebral vessels, although cerebral edema may be present.

7
Lacunae (choice D) are small necrotic foci in deep gray matter (especially basal ganglia and
thalamus) seen in some hypertensive patients.
What percentage of Down's syndrome patients also have congenital cardiovascular disease?
A. 0.1%
B. 3%
C. 20%
D. 50%
E. 90%
Explanation:
The correct answer is C. This fact is worth remembering: one-fifth of Down's syndrome patients
have congenital cardiovascular disease, most commonly an ostium primum type of ASD and/or a
ventricular septal defect. This is a sufficiently high incidence to justify at least one
echocardiogram in each of these children's lives. Affected children are also particularly
vulnerable to seizures, and as adults may develop an Alzheimer-like dementia in their mid 40s.
A 50-year-old man presents with renal colic and an intravenous pyelogram demonstrates "clumps" of
contrast medium limited to the medulla. Multiple small stones are also seen. Blood chemistries
are all within normal limits. What is the most likely explanation for these findings?
A. Adult polycystic renal disease
B. Horseshoe kidney
C. Infantile polycystic renal disease
D. Medullary sponge kidney
E. Renal dysplasia
Explanation:
The correct answer is D. This is a classic presentation for medullary sponge kidney, which has
a male predominance and typically presents at 40-60 years of age. Histologically, medullary
sponge kidney shows multiple small cysts lined by columnar or cuboidal epithelium localized to
the medullary collecting tubules. The cysts can contain laminated concretions of calcium
phosphates. Renal failure is rare in patients with medullary sponge kidney and the pathogenesis
for the lesion has not been clarified. The rare uremic medullary sponge kidney is distinguished
from the more common form by occurrence in 20-30 year-olds, salt-losing nephropathy, and
progression to renal failure.
The kidney of adult polycystic disease (choice A) is massively enlarged and filled throughout
with round cysts of varying sizes. Adult polycystic kidney disease often presents with
hypertension rather than renal failure or stones.
Horseshoe kidney (choice B) involves fusion of the upper or lower (most common) pole of the
kidney. It is fairly common (as high as 1:500), and is typically an incidental finding at
autopsy.

8
Infantile polycystic kidney (choice C) produces a small kidney with round medullary cysts and
"radiating" linear cortical cysts.
Renal dysplasia (choice E) can also cause cystic change in a kidney, but typically involves
only the medulla and cortex of part of the kidney.
A 75-year-old female presents to the doctor with a chief complaint of vaginal spotting. She has been
post-menopausal for 25 years and does not take hormones. An ultrasound is performed, and shows a
mass in the uterine fundus. A hysterectomy is performed, and pathologic examination of the
removed uterus reveals a malignant tumor of the endometrial glands and stroma. Which of the
following is the most likely diagnosis?
A. Endolymphatic stromal myosis
B. Endometrial carcinoma
C. Endometrial stromal sarcoma
D. Leiomyosarcoma
E. Malignant mixed mllerian tumor
Explanation:
The correct answer is E. Malignant mixed mllerian tumor is a tumor with 2 components, stromal
and epithelial (endometrial glands), both of which are malignant. This is a rare and highly
aggressive tumor that has a 25% 5-year survival rate. It usually affects older patients and
presents with post-menopausal bleeding. The stromal component can contain metaplastic
components such as cartilage and bone. Interestingly, usually only the epithelial component
metastasizes.
Endolymphatic stromal myosis (choice A) is a type of endometrial stromal tumor of intermediate
malignancy. It appears as small pieces of stroma between myometrial bundles that infiltrate
lymph channels. Patients may have pain or bleeding, or may be asymptomatic. Recurrences happen
late in the course of the disease (years) in half of patients and metastasis occurs in 15%.
There is no epithelial component, so this is an incorrect choice.
Endometrial carcinoma (choice B) is a malignancy of the epithelial glandular component of the
endometrium. Abnormal bleeding is the usual presentation. High estrogen states cause this
tissue to proliferate. There is no stromal component of this tumor, so this is an incorrect
choice.
Endometrial stromal sarcoma (choice C) is a true sarcoma arising from the endometrial stroma
that infiltrates the myometrium and invades vessels. There is no epithelial component.
Leiomyosarcoma (choice D) is a true sarcoma arising from the uterine smooth muscle. It commonly
has satellite lesions within the uterus. Leiomyosarcomas usually recur after removal; survival
is greater with well differentiated lesions. Poorly differentiated lesions have a 10 to 15% 5year survival rate. Distant metastasis is via blood vessels. There is no epithelial component.
An 80-year-old man has low back pain. An x-ray of the lower back and pelvis shows sclerotic changes in
the lower vertebrae and in focal areas throughout the pelvis. The radiologist's report states
that the sclerotic changes may represent osteoarthritis; however, metastatic prostate cancer
cannot be excluded. Which of the following is most cost-effective in the initial work-up of this
patient?

9
A. Bone marrow aspirate and biopsy
B. Digital rectal exam
C. Prostate-specific antigen
D. Radionuclide bone scan
E. Serum alkaline phosphatase
Explanation:
The correct answer is B. Osteoarthritis is the most common rheumatologic disease, the
prevalence of which increases with age. It primarily involves weight-bearing joints, hence its
distribution in the lower vertebrae, pelvic bones, and proximal femur. Sclerotic bone,
representing reactive bone formation, develops as a reaction to injury and is responsible for
the slightly elevated serum alkaline phosphatase levels that normally occur in much of the
elderly population. If prostate cancer with osteoblastic (bone-forming) metastases to the
vertebral column and pelvis were present in this patient (stage D disease), a digital rectal
exam would be the most cost-effective initial step in the work-up. With advanced prostate
cancer, the gland would very likely be enlarged and hard ("stony").
A bone marrow aspirate and biopsy (choice A) is not usually part of the normal work-up of
possible metastatic prostate cancer and has no place in the evaluation of osteoarthritis.
A prostate-specific antigen (PSA; choice C) level should be ordered in this patient, but not as
the initial step in the work-up, since it does not distinguish hyperplasia from cancer and is
fairly expensive. In known cases of prostate cancer, the PSA is a measure of tumor burden and
is used to monitor recurrences when following patients who have been treated for prostate
cancer.
A radionuclide bone scan (choice D) is commonly used to rule out metastasis in patients with
prostate cancer. It is expensive and is not used as a screening test for prostate cancer.
The serum alkaline phosphatase (choice E) is typically elevated in metastatic prostate cancer
due to osteoblastic activity in the metastatic foci. However, an elevated serum alkaline
phosphatase is non-specific, since it may be slightly increased in osteoarthritis (reactive
bone formation) as well as in liver disease.
A 40-year-old woman presents to the emergency department with severe abdominal pain localized to the
right upper quadrant. A urine sample is taken for rapid dipstick reagent strip analysis. A
positive result for which of the following substances would most strongly suggest gallstone
disease as a possible cause of her abdominal pain?
A. Bilirubin
B. Glucose
C. Nitrite
D. Protein
E. Urobilinogen
Explanation:

10
The correct answer is A. A small gallstone passing into the common bile duct can cause
obstructive jaundice with conjugated hyperbilirubinemia. The conjugated bilirubin will spill
into the urine, causing the bilirubin square on the reagent strip to react. This strip may also
react with other causes of intrahepatic or extrahepatic obstructive jaundice, so it is not
completely specific. It is worth becoming very familiar with the strengths and weaknesses of
reagent strip technology, however, since this information may be available hours before serum
chemistry values are reported.
High glucose (choice B) in urine suggests diabetes mellitus.
High nitrite (choice C) in urine suggests urinary tract infection.
High protein (choice D) in urine suggests renal disease or myeloma.
Urobilinogen levels (choice E) in obstructive jaundice can be normal, raised, or lowered;
consequently, they are not diagnostically helpful in the setting described in the question stem
unless urinary bilirubin is negative.
A 60-year-old man presents to his physician after a routine screening test indicates hyperlipidemia.
Physical examination reveals raised, irregular, yellow papules in the skin of the soft tissues
below the eyes. Biopsy of these lesions would most likely show which of the following?
A. Benign nevus cells
B. Malignant nevus cells
C. Microscopic blisters
D. Munro microabscesses
E. Multinucleated giant cells
Explanation:
The correct answer is E. The lesions are xanthomas, which are tumor-like dermal collections of
foamy histiocytes containing cholesterol and lipids. The lesions may also contain
multinucleated giant cells (Touton giant cells) with clustered nuclei and foamy cytoplasm.
Xanthomas may be idiopathic or may be related to hyperlipidemia or lymphoproliferative
malignancies (e.g., leukemias and lymphomas).
Nevus cells are a type of melanocyte. Benign nevus cells (choice A) are a feature of the common
nevus, or mole. Malignant nevus cells (choice B) are a feature of malignant melanoma.
Microscopic blisters (choice C) at the dermal/epidermal junction are a feature of dermatitis
herpetiformis.
Munro microabscesses (choice D), small collections of neutrophils in the cornified epidermis,
occur in psoriasis.
A 17-year-old male develops a painless, firm mass beneath the nipple of his left breast. The mass is
mobile, and no fluid can be expressed from the breast. The right breast is normal to
examination. Which of the following conditions does this mass most likely represent?
A. Fibrocystic changes
B. Gynecomastia

11

C. Intraductal papilloma
D. Invasive duct carcinoma
E. Invasive lobular carcinoma
Explanation:
The correct answer is B. The most common breast mass in men, especially under 25 years of age,
is gynecomastia-a benign proliferation of ductal and stromal elements of the breast. It is
generally an idiopathic condition, probably related to pubertal hormonal changes.
Fibrocystic changes (choice A) in the breast, which reflect physiological responses in the
breast to cyclical levels of sex hormones, are not observed in men. Fibrocystic changes are
most common in the late reproductive years and include fibrosis, cyst formation, and a variety
of epithelial changes such as hyperplasia and apocrine metaplasia.
Intraductal papillomas (choice C) are benign neoplasms commonly evolving in the major
lactiferous ducts beneath the nipple. They most commonly present with a bloody nipple discharge
and are rare in men.
Carcinoma of the male breast (choices D and E) is rare, and almost always develops in the
breasts of elderly men. Male breast carcinomas have a somewhat worse prognosis than their
female counterpart. Grossly and microscopically they resemble ductal carcinoma in the female.
Physical examination of a 45-year-old man who looks much older than his stated age demonstrates thin
arms and legs, a swollen abdomen, red tongue, dry, thin, and slightly yellow skin, gynecomastia,
testicular atrophy, multiple spider angiomas, tremor, yellow discoloration of sclera, and shortterm memory loss. Which of the following conditions is most strongly suggested by these
findings?
A. Bronchogenic carcinoma
B. Colon carcinoma
C. Congestive heart failure
D. Glomerulonephritis
E. Hepatic cirrhosis
Explanation:
The correct answer is E. The physical examination is typical for an advanced alcoholic with
hepatic cirrhosis. It is important to recognize these symptoms, as these patients are notorious
for "underestimating" and even denying their alcohol use.
Bronchogenic carcinoma (choice A) typically presents with cough and/or respiratory changes, but
can present with mass effects in the chest or involvement of mediastinal nerves or vessels.
Colon cancer (choice B) typically presents with changes in the stool or bowel habits.
Congestive heart failure (choice C) is typically heralded by shortness of breath or peripheral
edema, or both.

12
Glomerulonephritis (choice D) typically presents with changes in quality or quantity of urine,
and, possibly, fluid retention.
A 54-year-old white male presents with gradual onset of mild dementia, ataxic gait, and startle
myoclonus. An MRI scan is normal, and an examination of his cerebrospinal fluid reveals no
abnormalities, but the patient's EEG is remarkable for recurrent bursts of high-voltage slow
waves. Over the next 6 months, the patient's dementia rapidly worsens, accompanied by general
hypertonicity and profound dysarthria. The patient dies shortly thereafter. Which of the
following is the mostly likely neuropathological finding on autopsy?
A. Cerebellar hyperplasia
B. Diffuse spongiform change
C. Multiple lacunar infarcts
D. Negri bodies
E. Neurofibrillary tangles
Explanation:
The correct answer is B. The rapidly progressive dementia in this case is characteristic of
Creutzfeldt-Jakob disease (CJD). The dementia is usually accompanied by motor dysfunction and
abnormal EEG activity, as described in the question stem. The pathological hallmark of this
disease is spongiform change in the gray matter. Death usually occurs within 6-12 months of
disease onset.
At autopsy, the cerebellum in CJD appears atrophic, not hyperplastic (choice A).
Multiple lacunar infarcts (choice C) are seen with vascular dementia, and patients typically
present with focal neurological signs. Additionally, vascular dementia typically presents with
a more gradual decline in cognitive function, measured in years rather than months.
Negri bodies (choice D) are pathognomic for rabies, which does not cause dementia.
Neurofibrillary tangles (choice E) are often seen in patients with Alzheimer's disease.
Alzheimer's disease primarily affects the higher order association cortex; motor dysfunction is
not generally observed. Additionally, in Alzheimer's disease, there is a more gradual decline
in cognitive function, measured in years rather than months.
A 38-year-old woman is in her first pregnancy, which has been uneventful until the 34th week, when she
develops swelling of feet and hands. An obstetric check-up reveals that she also has
hypertension and proteinuria. Laboratory analysis shows elevated aspartate aminotransferase
(AST) and alanine aminotransferase (ALT) and slightly decreased platelets. The initial event in
the pathogenesis of her condition is thought to be which of the following?
A. Chorioamnionitis
B. Disseminated intravascular coagulation
C. Maternal hypertension
D. Maternal renal ischemia
E. Placental ischemia

13

Explanation:
The correct answer is E. This patient's condition is a classic third trimester complication
referred to as toxemia of pregnancy, or preeclampsia. It occurs in 6% of all pregnancies but is
more frequent in primiparas. Although the pathogenesis is still unclear, the first event
appears to be placental ischemia, probably due to abnormalities in the trophoblast and
alterations in the maturation of placental vessels. The trophoblast of invading placental
vessels fails to acquire the characteristics of normal endothelial cells, with subsequent
alterations in blood flow. Placental ischemia then triggers the release of thromboplastic
substances, increases renin synthesis, and reduces prostaglandin E levels.
Chorioamnionitis (choice A) is an infection of chorioamnionic membranes due to bacteria that
ascend through the vaginal canal. This is an important cause of spontaneous abortion in the
second and third trimester, but it plays no role in the pathogenesis of toxemia of pregnancy.
The release of thromboplastic substances may cause disseminated intravascular coagulation (DIC)
(choice B).
Increased renin and reduced prostaglandin E mediate increased sensitivity to angiotensin,
leading to maternal hypertension (choice C).
Ischemic damage to maternal organs, including brain, liver, and kidneys (choice D), results
from thrombotic occlusion of arterioles and capillaries as a consequence of DIC.
Overall, the clinical picture of toxemia is due to DIC-mediated ischemic damage to brain
(changes in mental status and convulsions), liver (elevated liver enzymes), and kidneys
(proteinuria, leading to peripheral edema). A manifestation of toxemia is HELLP syndrome, which
stands for hemolysis, elevated liver enzymes, and low platelets.
Molecular studies on an abdominal lymph node containing lymphoma demonstrate t(2;8)(p12;q24)
translocation. This is most compatible with which of the following diseases?
A. Burkitt's lymphoma
B. Mantle cell lymphoma
C. Multiple myeloma
D. Small cell lymphoma
E. Small cleaved cell lymphoma
Explanation:
The correct answer is A. Burkitt's lymphoma is actually associated with three translocations.
The common variant t(8;14)(q24;q32), involving the oncogene myc on chromosome 8, and the heavy
immunoglobulin chain on chromosome 14. The other two variants are: t(8;22)(q24;q11), involving
myc and the lambda light chain immunoglobulin site, and t(2;8)(p12;q24), involving the kappa
light chain and myc.
Mantle cell lymphoma (choice B), multiple myeloma (choice C), and small (not cleaved) cell
lymphoma (choice D) are associated with the t(11;14)(q13;q32) translocation involving bcl -1
and the heavy chain site.
Small cleaved cell lymphoma (choice E) is associated with t(14;18)(q 32;q21), involving the

14
immunoglobulin chain site and bcl-2.
A newborn infant is noted to have numerous, light brown macules dispersed across her skin. The
significance of this feature is due to its strong association with the development of which of
the following tumors?
A. Basal cell carcinoma
B. Neuroblastoma
C. Neurofibroma
D. Retinoblastoma
E. Wilms' tumor
Explanation:
The correct answer is C. Congenital "cafe au lait" spots are present in more than 90% of
patients with neurofibromatosis. This autosomal dominant disorder is characterized by multiple
neural tumors, especially neurofibromas, pigmented hamartomas of the iris (Lisch nodules), and
cafe au lait spots, which usually occur over nerve trunks. Although the majority of
neurofibromas in this disease are benign, the tumors can be quite disfiguring and
psychologically damaging.
Basal cell carcinoma (BCC; choice A) is a common, slow-growing tumor of sun-exposed skin. It
develops in adulthood after years of chronic sun damage. Although malignant melanoma may arise
from large congenital nevi, BCC does not.
Neuroblastoma (choice B) is a common childhood tumor that arises anywhere along the sympathetic
chain, and most commonly in the adrenal medulla. The tumor usually presents as an abdominal
mass and is not associated with skin findings.
The ocular neoplasm associated with neurofibromatosis is the Lisch nodule, not the
retinoblastoma (choice D). Retinoblastoma is a neuroepithelial tumor usually identified by
funduscopic examination of a child with visual changes.
Wilms' tumor (choice E) is a neoplasm of primitive renal blastema that may be associated with
congenital malformations of visceral organs, notably the adrenals and gonads, but not skin. It
also presents as an abdominal mass, and is highly survivable with modern treatment modalities.
A neonate develops bile-stained vomiting and progressive abdominal distention, and does not pass
meconium over the first two days of life. The anus is patent, and the bowel loops are palpable.
Plain radiograph shows bubbly meconium in the right lower quadrant. No localized areas of
constriction or other abnormalities are noted. Which of the following is most likely
etiologically related to this infant's condition?
A. Cystic fibrosis
B. Hirschsprung's disease
C. Meckel's diverticulum
D. Omphalocele
E. Polycystic kidney disease

15

Explanation:
The correct answer is A. The baby has meconium ileus, which is a manifestation of cystic
fibrosis due to the abnormally viscid pancreatic secretions which "get stuck" in the small
bowel. Meconium ileus can cause gut perforation with peritonitis and intraperitoneal
calcifications (that may be visible on plain film). Meconium ileus complicated by intestinal
perforation or formation of fistulas to the bladder or vagina must be treated surgically.
Medical treatments for uncomplicated meconium ileus are now available, which use enemas,
mucolytic agents, or pancreatic enzymes.
Hirschsprung's disease (choice B) is a cause of congenital constipation related to absence of
ganglion cells in a segment of bowel. The aganglionic bowel segment is narrowed because the
lack of peristalsis keeps stool from moving into the segment. The distal rectum is always
involved, and the lesion may extend proximally as far as the small intestine. The bowel
proximal to the lesion is usually dilated.
A Meckel's diverticulum (choice C) can form due to the persistence of the vitelline duct, which
connects the developing gut to the yolk sac. They are classically located in the distal ileum
within 30 cm of the ileocecal valve, and may contain ectopic pancreatic tissue or gastric
mucosa.
Omphalocele (choice D) is characterized by herniation of abdominal viscera through the
abdominal wall near the umbilicus.
In polycystic kidney disease (choice E), cysts of the liver and pancreas may occur; meconium
ileus is not associated with this disorder.
A 69-year-old woman is brought to the emergency room after falling off a step stool and fracturing her
hip. Her past medical history is significant for several bouts of pneumonia during the past
year. Laboratory results indicate a normal white blood cell count, but platelets are decreased,
and the erythrocyte sedimentation rate (ESR) is elevated. X-rays reveal multiple lytic bone
lesions. Serum electrophoresis demonstrates an M-protein spike. Which of the following is the
most likely diagnosis?
A. Chronic lymphocytic leukemia
B. Monoclonal gammopathy of uncertain significance
C. Multiple myeloma (plasma cell myeloma)
D. Plasmacytoma
E. Waldenstrm's macroglobulinemia
Explanation:
The correct answer is C. The patient is suffering from multiple myeloma, a neoplastic
proliferation of plasma cells (or their precursors) found within the bone marrow. These
malignant cells are responsible for the production of excessive amounts of immunoglobulin
(usually IgG or IgA), producing an M- protein spike, and increasing the ESR. The decreased
platelet count probably reflects infiltration of the bone marrow by myeloma cells. It is not
unusual for patients with malignant myeloma to have recurring bacterial infections,
particularly pneumococcal pneumonia, because the overall production of normal immunoglobulins
of all isotypes is decreased. Lytic bone lesions due to infiltration by myeloma cells may lead
to pathological fractures. All of the other disorders listed can be associated with an M-

16
protein spike, but this is a nonspecific finding.
Chronic lymphocytic leukemia (CLL; choice A) is ruled out by the normal white count, since this
type of leukemia is generally associated with an absolute lymphocytosis.
Patients with monoclonal gammopathy of undetermined significance (choice B) are generally
asymptomatic, although they have a predisposition for subsequently developing myeloma,
lymphoma, amyloidosis, or Waldenstrm's macroglobulinemia.
Plasmacytoma (choice D) (solitary myeloma) is a rare, isolated plasma cell neoplasm in bone or
soft tissues. If the primary cancer is in bone, it is likely to disseminate; extraosseous
tumors tend to remain localized. Since this patient had multiple lytic bone lesions, multiple
myeloma is the correct diagnosis.
Waldenstrm's macroglobulinemia (choice E) is a disorder involving neoplasms of lymphocytoid
plasma cells that produce monoclonal IgM. Hypergammaglobulinemia produces hyperviscosity of the
blood which, along with infiltration by tumor produce the characteristic signs and symptoms.
Weakness, weight loss, bone pain, hepatosplenomegaly, and lymphadenopathy occur commonly.
A 55-year-old woman complains to her physician that the skin of her armpits and groin "keeps getting
darker and darker." Physical examination demonstrates velvety brown and warty skin in the axilla
and groin. Biopsy of these lesions shows a variably hyperplastic epidermis with many sharp peaks
and valleys. Aside from cosmetic considerations, which of the following is the primary medical
significance of these lesions?
A. They may be a sign of immunosuppression
B. They may be a sign of visceral carcinoma
C. They may be easily superinfected
D. They may be malignant
E. They may be premalignant
Explanation:
The correct answer is B. The lesions are acanthosis nigricans, which looks somewhat like a mole
or wart, but is actually due to epidermal hyperplasia. Acanthosis nigricans can be seen in
obesity, diabetes, and in patients with underlying cancers, often adenocarcinomas of the chest
or abdomen.
The lesions are not characteristic of immunosuppression (choice A) and are not easily
superinfected (choice C). They are also neither malignant (choice D) nor premalignant (choice
E).
A 47-year-old woman undergoes endometrial biopsy because she has had repeated episodes of irregular
spotting between periods lately. The biopsy shows strips of endometrium bearing long, narrow,
coiled glands lined by a single layer of columnar epithelium showing regular, uniform, small
nuclei and clear apical vesicles. Which of the following is the most likely diagnosis?
A. Atypical hyperplasia
B. Complex hyperplasia without atypia
C. Proliferative endometrium

17

D. Secretory endometrium
E. Simple hyperplasia
Explanation:
The correct answer is D. Endometrial biopsies are often performed to evaluate patients with
menstrual bleeding abnormalities, particularly in perimenopausal or postmenopausal patients.
While the biopsies are done to rule out hyperplasias or cancer, most of the specimens actually
show only proliferative or secretory (as in this patient) endometrium. Secretory endometrium
has the features noted in the question stem.
Atypical hyperplasia (choice A) is characterized by complex glands lined by cells showing
features of atypia, such as cytomegaly, increased nuclear cytoplasmic ratio, prominent
nucleoli, and increased mitotic index.
Complex hyperplasia without atypia (choice B) is characterized by complex, branching glands
without cellular atypia.
Proliferative endometrium (choice C) is characterized by smaller, noncoiled, glands lined with
columnar epithelium without apical (secretory) vesicles.
Simple (cystic) hyperplasia (choice E) is characterized by cystically dilated glands without
cellular atypia.
Which of the following conditions is the most frequent cause of spontaneous abortion in the first
trimester of pregnancy?
A. Abruptio placentae
B. Chorioamnionitis
C. Chromosomal abnormalities
D. Placenta previa
E. Trauma
Explanation:
The correct answer is C. At least 10% to 15% of normally fertilized and implanted ova are lost
in the first trimester of pregnancy because of spontaneous abortion. Studies using immunoassay
of human chorionic gonadotropin (hCG) for early diagnosis of pregnancy suggest that the
percentage of fertilized ova lost in the first trimester might be even higher. The great
majority of these cases are attributable to chromosomal abnormalities. Chromosomal studies are
not routinely performed in such cases, but they are recommended when a malformed fetus has been
identified or when habitual or recurrent abortions occur.
Abruptio placentae (choice A), a complication of the third trimester, occurs when the placenta
detaches prematurely from its implantation site. Retroplacental hemorrhage develops within the
space between placenta and uterine wall, leading to interruption or severe reduction in the
blood supply to the fetus.
Chorioamnionitis (choice B), a complication of the second and third trimesters, results from
ascending infections through the vaginal canal. Infection of chorioamnionic membranes may lead

18
to premature rupture of membranes and abortion or premature labor.
Placenta previa (choice D) is a placenta implanted in the lower segment of the uterus. When
dilatation of this segment begins in late pregnancy, a placenta previa may cause severe
bleeding and lead to premature labor.
Surprisingly, trauma (choice E) is a rare cause of spontaneous abortion.
A 65-year-old woman has a long-standing dementing disorder characterized by deterioration in
personality, neglect of personal hygiene, impaired judgment, and disinhibited behavior. MRI
demonstrates severe cortical atrophy limited to the frontal lobes and anterior two thirds of the
temporal lobes, while the remaining cortex is preserved. No evidence of recent or remote
infarcts is found. Which of the following diagnoses is most consistent with these pathologic and
clinical features?
A. Alzheimer disease
B. Creutzfeldt-Jacob disease
C. Dementia with Lewy bodies
D. Frontotemporal dementia
E. Vascular dementia
Explanation:
The correct answer is D. Not all dementing disorders manifest with the same clinical features.
Although there is considerable overlap in clinical symptomatology among different types of
dementias, making clinical diagnosis somewhat problematic, there are classic presentations that
allow identification of a specific form of dementia with a high degree of confidence. In this
case, the patient has symptoms due to frontal lobe damage, eg, disinhibition, impaired
judgment, and personality changes. Furthermore, MRI demonstrates a specific pattern of cortical
atrophy, restricted to the frontal lobes and anterior portion of the temporal lobes. This
combination points toward a group of dementias called frontotemporal dementia, the most
frequent form of which is Pick disease. Other forms of frontotemporal dementia are very
infrequent. Remember: frontal symptoms in conjunction with frontotemporal atrophy =
frontotemporal dementia/Pick disease.
Alzheimer disease (choice A) is the most frequent form of dementia in industrialized countries.
Although symptoms due to frontal damage may be present in Alzheimer disease, they are usually
associated with a more generalized impairment of higher neurologic functions, eg, language,
memory, and learned movements. In addition, cortical atrophy in Alzheimer disease is widespread
and not limited to the frontal and anterior temporal lobes.
Creutzfeldt-Jacob disease (choice B) represents the prototype of prion diseases. Cortical
atrophy is not a prominent feature of Creutzfeldt-Jacob disease, which manifests with
personality changes, memory loss, and seizures, leading to death after a rapid clinical course
(a few months to 1 year).
Dementia with Lewy bodies, also known as diffuse Lewy body disease (choice C), is one of the
most common forms of dementia in Western countries, possibly more common than vascular
dementia. It is characterized by widespread formation of Lewy bodies in the substantia nigra,
limbic cortex, and subcortical nuclei, such as the basal nucleus of Meynert. Extrapyramidal
symptoms similar to Parkinson disease manifest in this form of dementia as a result of
degeneration of dopaminergic pathways.

19

Vascular dementia (choice E) is an umbrella term encompassing dementing conditions that arise
from pathology of large or small cerebral vessels. It manifests with memory loss associated
with focal neurologic symptoms depending on the location of damage. MRI would identify old or
recent infarcts, as well as white matter disease. Conditions associated with vascular dementia
include the following: Multi-infarct dementia, which is caused by multiple, scattered brain
infarcts secondary to atherosclerosis of large arteries of the circle of Willis and/or carotid
arteries. Binswanger disease, which involves rarefaction of cerebral white matter and is caused
by hypertension-related arteriolosclerosis. Lacunar infarcts, which consist of small (< 1 cm)
infarcts in the striatum and thalamus; this condition is related to arteriolosclerosis.
Biopsy of a reasonably well-demarcated mass of the nasopharynx demonstrates a plasma cell proliferation.
Serum electrophoresis shows a small monoclonal IgG spike. Bone marrow evaluation fails to
demonstrate plasma cell proliferation and no lesions are seen on extensive skeletal x-rays.
Which of the following is the most likely diagnosis?
A. Heavy chain disease
B. Monoclonal gammopathy of undetermined significance
C. Multiple myeloma
D. Plasmacytoma
E. Waldenstrm's macroglobulinemia
Explanation:
The correct answer is D. Plasmacytoma (solitary myeloma) involving soft tissue (lungs,
nasopharynx, nasal sinuses) is a plasma cell proliferation resembling multiple myeloma but
without significant metastatic potential. In contrast, some plasmocytomas involving bone
eventually (up to 10 to 20 years) develop into frank multiple myeloma.
Heavy chain diseases (choice A) constitute a group of rare lymphoplasmacytic malignancies in
which excessive amounts of a defective heavy immunoglobulin chain are produced. They may take
the form of gamma heavy-chain disease (from IgG), alpha heavy-chain disease (from IgA) or mu
heavy-chain disease (from IgM); malignant cells are usually present in marrow in all of these
conditions.
Monoclonal gammopathy of undetermined significance (choice B) is a disease of elderly patients
with a monoclonal spike on serum or urine electrophoresis, but no identifiable mass or bone
marrow lesion; 20% of these patients eventually develop one of the other diseases listed in the
answer choices.
Multiple myeloma (choice C) is a malignancy derived from a single plasma cell clone with
significant metastatic potential. Multiple lytic bone lesions are usually seen.
Waldenstrm's macroglobulinemia (choice E) is a malignancy of lymphoplasmacytic cells that
secrete IgM. In this disorder, the bone marrow is diffusely rather than focally infiltrated by
lymphocytes, plasma cells, and hybrid forms.
A 65-year-old woman presents with a 30-lb weight loss and malaise. Cancer is suspected. Which of the
following is the correct list, starting with the most prevalent, of the three most common
causes of cancer in women?
A. Breast, lung, colon and rectum

20

B. Breast, uterus, lung


C. Colon and rectum, lung, ovary
D. Lung, breast, ovary
E. Ovary, uterus, lung
Explanation:
The correct answer is A. The correct female incidence sequence is breast (32%), lung (13%),
and colon and rectum (13%). The two major causes of cancer death in women are lung (23%) and
breast (18%). In men, the incidence sequence is prostate (32%), lung (16%), and colon and
rectum (12%). The two leading causes of male cancer deaths are lung (33%) and prostate (13%).
The uterus and ovary (choices B, C, D, and E) are not among the three organs most frequently
affected by cancer in women.
A 50-year-old woman presents with a 5-year history of headaches, generalized tonic-clonic seizures, and
bilateral leg weakness. Skull films reveal hyperostosis of the calvarium. Biopsy of the
responsible lesion shows a whorling pattern of the cells. Which of the following is the most
likely diagnosis?
A. Arachnoid cyst
B. Glioblastoma multiforme
C. Meningioma
D. Metastatic breast cancer
E. Oligodendroglioma
Explanation:
The correct answer is C. The most likely diagnosis is an intracranial meningioma. Meningiomas
are slow-growing, benign tumors comprising 15% of intracranial tumors; they are most common in
the elderly. They originate from either dura mater or arachnoid and are sharply demarcated from
brain tissue. Meningiomas often incite an osteoblastic reaction in the overlying cranial bones.
Microscopically, the meningioma cells have a tendency to encircle one another, forming whorls
and psammoma bodies. Clinically, they present as mass lesions; seizures may occur. The superior
parasagittal surface of the frontal lobes is a favorite site of origin. This can often produce
leg weakness, since the leg motor fibers that pass down through the internal capsule originate
in parasagittal cortical regions. Treatment of meningiomas is usually surgical.
Arachnoid cysts (choice A) are formed by splitting of the arachnoid membrane; most arachnoid
cysts arise near the Sylvian fissure. They may present with mass effect, but would be unlikely
to produce seizures, prominent focal signs, or reactive hyperostosis.
Glioblastoma multiforme (choice B) is an aggressive malignant astrocytoma that would likely
have killed the patient long before 5 years had elapsed.
Metastatic breast cancer (choice D) would generally look different microscopically (the
whorling cell pattern is characteristic of meningioma). It would be unlikely for metastatic
cancer to cause a reaction in the overlying bone, or to be present long enough to cause

21
symptoms for 5 years.
Oligodendrogliomas (choice E) are glial tumors that could produce the described clinical
picture, but usually do not cause hyperostosis of the calvarium or exhibit the characteristic
whorling cell pattern microscopically.
When a histologic section is taken of an abscess, many of the observed neutrophils show a degenerative
change in which the nucleus has undergone fragmentation. This process is known as
A. caseous necrosis
B. coagulative necrosis
C. karyolysis
D. karyorrhexis
E. pyknosis
Explanation:
The correct answer is D. Karyorrhexis refers to a pattern of nuclear degradation in which a
pyknotic or partially pyknotic nucleus undergoes fragmentation followed by complete lysis. This
pattern is common in the neutrophils present in acute inflammation.
The type of necrosis seen in an abscess is liquefactive necrosis. Caseous necrosis (choice A)
is seen in tuberculosis and some other granulomatous diseases; coagulative necrosis (choice B)
is seen following infarctions of many organs (other than the brain).
Karyolysis (choice C) is also a degenerative change affecting nuclei. In this case, however, it
is seen as a decrease in nuclear basophilia, which is presumably the result of DNAse activity.
Pyknosis (choice E) is characterized by nuclear shrinking and basophilia, apparently as a
result of DNA condensation.
A 52-year-old woman has recently undergone a breast resection for carcinoma. Based on the statistics for
breast cancer incidence, which of the following types of carcinoma does this patient most
likely have?
A. Colloid (mucinous)
B. Invasive ductal
C. Invasive lobular
D. Medullary
E. Metastatic bronchogenic
Explanation:
The correct answer is B. Invasive ductal carcinoma is the most likely candidate. Of the various
types of breast adenocarcinoma, invasive ductal carcinoma is by far the most common variant,
accounting for approximately 75% to 80% of all invasive breast carcinomas. Invasive ductal
carcinoma develops from epithelial cells of the terminal duct. Histologically, it is composed
of small, glandular, ductlike structures, lined by variably anaplastic cells. The most common

22
mode of presentation is a palpable mass in the breast. Its prognosis depends mostly on staging
(spread of cancer) rather than grading (degree of differentiation).
The colloid (mucinous) variant (choice A) is relatively rare (about 1% to 2%) and occurs more
frequently in older women. Histologically, this carcinoma is characterized by abundant mucin
secretion. It is associated with a better prognosis than the ductal type.
Invasive lobular carcinoma (choice C) is the second most frequent histologic type of breast
adenocarcinoma, accounting for approximately 10% of all cases. Its presumed cell of origin is
the lobular cell. The most typical histologic characteristic is the presence of cancer cells
lined up in orderly rows ("single-file").
Medullary carcinoma (choice D) tends to occur in younger women and is associated with a
slightly better prognosis. Although a malignant tumor, medullary carcinoma is well
circumscribed and surrounded by a florid lymphoplasmacytic reaction. The name is due to its
soft consistency.
Metastatic cancer may involve the breast like any other organ. Bronchogenic carcinoma (choice
E) may also spread to the breast by lymphatic route or by contiguity, but this would be less
likely than primary breast cancer.
A 55-year-old man is brought to his physician's office with a 3-month history of progressive mental
deterioration in the form of memory loss, mood changes, and errors in judgment. His gait is
unsteady, and he requires assistance to prevent falling. He has no history of seizures, head
trauma, or incontinence. Computed tomography (CT) scan and lumbar puncture are unremarkable.
Physical examination reveals hypertonicity of all extremities, bilateral equivocal plantar
response, ataxic gait, and myoclonic jerks in the lower extremities. What is the mechanism by
which this infectious agent causes its pathology?
A. Amyloid deposition
B. Autoimmune destruction
C. Chronic inflammation
D. Embolization and infarction
E. Toxin production
Explanation:
The correct answer is A. This is the classic presentation of Creutzfeldt-Jacob disease (CJD).
Although the pathogenesis is incompletely understood, these patients develop extracellular
deposition of abnormal fibrillar proteins in the brain, ie, amyloid.
Autoimmune destruction (choice B) is not indicated because there is no immunologic response to
the deposition of these extracellular proteins; thus there is no chronic inflammation (choice
C)
Although embolization and infarction (choice D) could complicate the presentation in the age
group typically afflicted with CJD, these processes are not believed to have any direct role in
this pathology.
No toxin is produced (choice E) to account for the presentation in CJD.
A patient consults a dermatologist about a skin lesion on her neck. Examination reveals a 1-cm diameter,

23
red, scaly plaque with a rough texture and irregular margins. Biopsy demonstrates epidermal and
dermal cells with large, pleomorphic, hyperchromatic nuclei. Which of the following conditions
would most likely predispose this patient to the development of this lesion?
A. Actinic keratosis
B. Compound nevus
C. Dermal nevus
D. Junctional nevus
E. Melanoma
Explanation:
The correct answer is A. The lesion is a squamous cell carcinoma of the skin. Actinic
keratosis, which is a hyperplastic lesion of sun-damaged skin, predisposes for squamous cell
carcinoma. Another predisposing condition to remember is xeroderma pigmentosum, which
predisposes for both squamous cell and basal cell carcinomas of skin.
A nevus is a mole, containing characteristic cells called nevocellular cells. If the
nevocellular cells are located at the dermal-epidermal junction (junctional nevus, choice D),
in the dermis (dermal nevus, choice C), or both (compound nevus, choice B), they do not
predispose for squamous cell carcinomas of the skin. Malignant melanoma (choice E), however,
can arise in pre-existing nevi.
A 24-year-old woman gives birth to an apparently normal infant. The neonate begins feeding well by the
second day, then at ten days, suddenly develops gastrointestinal obstruction. Which of the
following is the most likely cause of this presentation?
A. Adhesions
B. Congenital pyloric stenosis
C. Hirschsprung's disease
D. Intussusception
E. Volvulus
Explanation:
The correct answer is D. All of the conditions listed can cause gastrointestinal obstruction,
but the clinical presentation is most suggestive of intussusception. In intussusception, there
is telescoping of one bowel segment into another, more distal segment. The disorder is
relatively common in infants and children due to the poor support offered by their thin
mesentery. Intussusception produces intestinal obstruction, and it may produce bowel ischemia
or infarction by trapping mesenteric vessels along with the affected segment. In some cases,
the intussusception may be reduced by diagnostic barium enema.
Adhesions (choice A) can cause bowel obstruction following surgery or inflammatory bowel
diseases.
Congenital pyloric stenosis (choice B) typically presents as projectile vomiting in a 3-4 week
old baby.

24

Hirschsprung's disease (choice C), caused by absence of ganglion cells in the distal bowel, is
usually diagnosed in the first few days of life when there is a failure to pass meconium.
Volvulus (choice E) is due to rotation of bowel segments. This is usually a disease of the
elderly.
A middle-aged man with a long history of sexual activity slowly develops testicular enlargement. Needle
biopsy demonstrates the presence of obliterative endarteritis with perivascular cuffing of
lymphocytes and plasma cells. A diffuse interstitial inflammation with edema and prominent
plasma cell infiltrate is also present. Which of the following is the most likely diagnosis?
A. Gonorrhea
B. Mumps
C. Nonspecific orchitis
D. Syphilis
E. Tuberculosis
Explanation:
The correct answer is D. This is one presentation of syphilis, which can involve the testis and
epididymis in both the acquired and congenital types of the disease. Characteristically, the
testis is usually involved before the epididymis. Microscopically, the testis may show either
gumma formation or the findings described in the question stem.
Gonorrhea (choice A) usually causes an acute epididymitis and orchitis with prominent
neutrophils.
Mumps orchitis (choice B) would usually be accompanied by parotitis.
Nonspecific orchitis (choice C) is characterized by prominent neutrophils.
Tuberculosis orchitis (choice E) is characterized by granulomas with acid-fast bacteria.
A 70-year-old woman dies in a nursing home after a heart attack. The time of onset of her clinical
symptomatology and the cause of death are uncertain; furthermore, the possibility of neglect is
being considered. Therefore, an autopsy investigation is arranged. The forensic pathologist
discovers acute thrombosis involving the posterior descending branch of the right coronary
artery with resultant myocardial infarction (MI) in the posterior third of the interventricular
septum. Histologically, there is coagulation necrosis with associated abundant neutrophilic
infiltration. Histiocytes and lymphocytes are scanty. Which of the following is the approximate
period between the onset of pain (ie, beginning of ischemic injury) and death?
A. 1 hour
B. 12 hours
C. 2 days
D. 5 days
E. 10 days

25

Explanation:
The correct answer is C. Following irreversible ischemic injury, the heart (and any other
organ) displays an orderly sequence of events that progresses from necrosis of parenchymal
cells to inflammatory reaction, granulation tissue, and scar healing.
Although ischemic injury manifests with pain almost immediately following vascular occlusion,
histologic evidence of necrosis lags behind the clinical symptoms. At 1 hour (choice A) after
ischemia, there is no morphologic change indicative of necrosis. The first signs of necrosis
appear 12 hours (choice B) after irreversible ischemia: myocytes appear intensely eosinophilic
and wavy, but there is no inflammatory reaction yet. Acute inflammatory cells (neutrophils)
infiltrate the infarcted area beginning 1 day and peaking at approximately 2-3 days after
injury. This acute inflammatory response partially overlaps with the subsequent influx of
lymphocytes and histiocytes. Reabsorption of necrotic myofibers by histiocytes, as well as
proliferation of small blood vessels, marks early formation of granulation tissue at around 5
days (choice D). Granulation tissue is advanced at 10 days (choice E) and consists of
fibroblasts, small blood vessels, and residual chronic inflammatory cells within a matrix of
young collagen matrix.
A 54-year-old man presents with a chief complaint of "burning" abdominal pain in the epigastric region.
Endoscopy demonstrates a well-defined, regular gastroesophageal junction located 3 cm above the
esophageal hiatus in the diaphragm. Biopsy of the distal side of the junction demonstrates
normal gastric mucosa. This lesion is best classified as which of the following?
A. Achalasia
B. Esophageal ring
C. Esophageal web
D. Paraesophageal hernia
E. Sliding hernia
Explanation:
The correct answer is E. This patient has a sliding hiatal hernia, which is the most common
(90%) form of hiatal hernia. This condition is frequently associated with gastric reflux.
Achalasia (choice A) is actually a physiologic, rather than an anatomic variation. In this
disorder, the lower esophageal sphincter fails to relax adequately, and esophageal peristalsis
is often abnormal.
Esophageal rings (choice B) are mucosal folds in the esophagus. They are called esophageal webs
(choice C) in the upper esophagus. Schatzki rings are mucosal rings in the lower esophagus, at
the gastroesophageal junction.
In a paraesophageal hernia (choice D), an area of gastric cardia rolls along with the esophagus
through an incompetent hiatus into the thorax. In a paraesophageal hernia, the gastroesophageal
junction would not be displaced.
Which of the following pulmonary conditions is associated with widespread formation of hyaline membranes
in the alveolar cavities?
A. Asthma

26

B. Bacterial pneumonia
C. Desquamative interstitial pneumonitis
D. Diffuse alveolar damage
E. Hemodynamic pulmonary edema
Explanation:
The correct answer is D. Diffuse alveolar damage, clinically referred to as adult respiratory
distress syndrome (ARDS), is characterized by diffuse damage to the alveolar/capillary barrier,
which may result from diverse acute conditions. The four most frequent causes are trauma,
sepsis, shock, and gastric aspiration. The pathogenesis is not entirely clear, but influx of
neutrophils and release of cytokines, eicosanoids, and free radicals seem to be crucial in
promoting alveolar damage. The most characteristic histopathologic hallmark of diffuse alveolar
damage is formation of hyaline membranes within the alveolar cavities. These consist of
proteinaceous material of plasma origin and necrotic debris from desquamated epithelium. The
condition has a 60% mortality and manifests with acute respiratory failure.
The pathologic features of asthma (choice A) are relatively nonspecific and are similar, for
example, to those of chronic bronchitis, including chronic inflammatory infiltration,
hyperplasia of mucous glands, and hypertrophy of smooth muscle. The presence of numerous
eosinophils, however, is more characteristic of asthma.
Bacterial pneumonia (choice B) is characterized by intra-alveolar exudation of neutrophils,
fibrin, and erythrocytes. Bacteria are also present within the alveolar cavities.
Desquamative interstitial pneumonitis (DIP) (choice C) is a form of interstitial disease
referred to as idiopathic pulmonary fibrosis. In contrast to usual interstitial pneumonitis
(another form of idiopathic pulmonary fibrosis), DIP is more responsive to steroid treatment.
Histopathologically, DIP leads to hyperplasia of pneumocytes and accumulation of histiocytes
that fill the alveolar cavities.
Hemodynamic pulmonary edema (choice E) is caused by increased hydrostatic pressure, as occurs
in acute left ventricular failure. It is due to escape of fluid from the intravascular
compartment into the alveoli.
A baby is born with a flat facial profile, prominent epicanthal folds, and simian crease. She vomits
when fed, and upper GI studies demonstrate a "double bubble" in the upper abdomen. Which of the
following cardiovascular abnormalities might this child also have?
A. Atrial septal defect
B. Berry aneurysm
C. Coarctation of the aorta
D. Endocardial cushion defect
E. Tetralogy of Fallot
Explanation:
The correct answer is D. The disease is Down syndrome (trisomy 21). In addition to mental

27
retardation and the characteristic physical findings described in the question stem, duodenal
atresia is fairly common, as evidenced by the "double bubble" sign on x-ray. These children are
also likely to have various cardiac anomalies; endocardial cushion defect is the most common.
Atrial septal defect (choice A) is one of the most common genetic defects in the general
population, but is less common than endocardial cushion defect in patients with Down syndrome.
Berry aneurysms (choice B), also known as saccular aneurysms, are typically located in the
circle of Willis on the ventral surface of the brain. They occur more frequently in patients
with adult polycystic disease. Rupture can produce subarachnoid hemorrhage.
Coarctation of the aorta (choice C) occurs more commonly in females with a 45, XO genotype
(Turner syndrome).
Tetralogy of Fallot (choice E) is the most common cause of early cyanosis, consisting of a
ventricular septal defect, right ventricular outflow tract obstruction, an overriding aorta,
and right ventricular hypertrophy.
A 47-year-old man presents to the emergency room with sudden onset of severe upper abdominal pain with
vomiting. The pain is focused in the epigastrium with radiation to the back. Serum amylase
levels are 2000 U/L. Which of the following are the most commonly encountered predisposing
factors for this patient's condition?
A. Alcohol use and gallstones
B. Helicobacter pylori infection and excess gastric acid secretion
C. Hepatitis B infection and iron overload
D. Obesity and high serum cholesterol
E. Stress and cigarette use
Explanation:
The correct answer is A. The clinical scenario is typical of acute pancreatitis. The
overwhelmingly most important contributing factors for development of acute pancreatitis are
gallstones (particularly small ones) and alcohol abuse.
Helicobacter pylori infection and excess gastric acid secretion (choice B) are predisposing
factors for peptic ulcer disease of the stomach and duodenum, respectively.
Hepatitis B infection and iron overload (choice C) predispose for cirrhosis.
Predisposing factors for myocardial infarction include obesity, high serum cholesterol (choice
D), stress, and cigarette smoking (choice E).
A 37-year-old woman complains to her gynecologist of discomfort during intercourse and placement of a
tampon. Physical examination demonstrates a flocculent swelling below the skin of the
posterolateral part of one labium majora.Which of the following is the most likely diagnosis?
A. Bartholin's gland cyst
B. Condyloma acuminatum
C. Lichen sclerosis

28

D. Vestibular adenitis
E. Vulvar squamous hyperplasia
Explanation:
The correct answer is A. This is a Bartholin's gland cyst, which is a relatively common lesion
occurring when Bartholin's duct becomes obstructed, typically a sequela to a previous
infection. The cysts can enlarge to 3 to 5 cm in diameter. They are lined by either
transitional epithelium or metaplastic squamous epithelium. Treatment is by excision or
marsupialization (permanent opening).
Condyloma acuminatum (choice B) usually produces a papillary lesion (venereal wart).
Lichen sclerosis (choice C) usually produces a gray, parchment-like thinned epidermis.
Vestibular adenitis (choice D) usually produces an exquisitely tender posterior introitus with
focal ulcerations.
Vulvar squamous hyperplasia (choice E) usually produces a white plaque.
Which of the following types of hepatitis is associated with an immune-mediated vasculitis characterized
by p-ANCA antibodies?
A. Hepatitis A
B. Hepatitis B
C. Hepatitis C
D. Hepatitis D
E. Hepatitis E
Explanation:
The correct answer is B. Hepatitis B is associated with polyarteritis nodosa (PAN), a
necrotizing vasculitis of small- and medium-sized muscular arteries involving all organ
systems. A significant percentage of patients with PAN have hepatitis B antigenemia. They also
have circulating immune complexes containing hepatitis B antigen. Hepatitis B antigen, IgM, and
complement can be demonstrated in blood vessel walls. P-ANCA (perinuclear-antinuclear
cytoplasmic antibody) is a marker for polyarteritis nodosa.
Hepatitis A (choice A) is not associated with a vasculitis.
Hepatitis C (choice C) accounts for 50-70% of chronic hepatitis. Chronic hepatitis C can be
associated with immune-complex mediated extrahepatic complications, but is less common than
hepatitis B. Hepatitis C does have a significant association with essential mixed
cryoglobulinemia, which presents with glomerulonephritis, arthralgias, hepatosplenomegaly, and
lymphadenopathy, in addition to a vasculitis. However, there is no association with p-ANCA.
Hepatitis D (choice D) requires that the patient be co-infected with hepatitis B. As such, it
does not independently cause disease, but it can produce a worsening of the liver disease.
Hepatitis E (choice E) resembles hepatitis A due to its primarily enteric mode of spread. It is

29
not associated with chronic hepatitis and does not have a predisposition for vasculitis.
Testicular biopsy of an infertile man demonstrates a complete absence of sperm or sperm precursors in
spermatic tubules that have a regular, round cross-section and are closely packed together. The
most probable etiology is
A. diabetes mellitus
B. maturation arrest
C. seminoma
D. Sertoli-only syndrome
E. tuberculosis
Explanation:
The correct answer is D. All of the conditions listed can cause infertility due to a low or
absent sperm count. Only in Sertoli-only syndrome is there a complete absence of sperm
precursors in an undamaged tubule. There is no known method to correct Sertoli-only syndrome
(or maturation arrest) that is not due to a treatable chronic disease.
Chronic diseases such as diabetes mellitus (choice A) or tuberculosis (choice E) can arrest the
maturation of sperm, but do not usually show a complete absence of sperm precursors.
In maturation arrest (choice B), mature sperm are absent, but precursors are found.
Tumors such as seminomas (choice C) cause infertility by occluding the flow of semen or by
replacing the seminiferous tubules. Sampling of a seminiferous tubule not replaced by tumor
would probably still demonstrate sperm.
A 50-year-old woman who works as a paralegal in a law firm comes to her local doctor because of problems
with sleep. The patient says that over the past several weeks, she hasn't slept well, feels
tired, and has had headaches. She does not smoke or drink alcohol, except on special occasions,
and does not take any medications. The patient's pupils are 5 mm in size, equal and reactive,
with both the direct and consensual light reflexes intact. Accommodation is unimpaired.
Examination of the visual fields and funduscopy are unremarkable. Extraocular movements reveal
normal conjugate, oblique, and downward movement, but she is unable to look upwards. No other
abnormalities are found on the neurological examination. Which of the following is the most
likely diagnosis?
A. Acoustic neuroma
B. Astrocytoma in the cerebellum
C. Craniopharyngioma
D. Parasagittal meningioma
E. Pinealoma
Explanation:
The correct answer is E. This patient has a pinealoma. Tumors of the pineal gland compress the
vertical gaze center in the tectum of the midbrain. The pineal gland manufactures melatonin

30
from its precursor serotonin; an inadequate supply of melatonin results in insomnia. Tumors of
the pineal gland will not compress the cerebral cortex or the rest of the brainstem.
Frequently, the only physical sign noted is failure of upward gaze.
An acoustic neuroma (choice A) is a schwannoma of the eighth cranial nerve. It results in
deafness, ataxia, and dysarthria. Nystagmus may be present. The gaze centers are not affected.
Astrocytomas of the cerebellum (choice B) are usually seen in children. These tumors present
with headache, nausea, vomiting, papilledema, and cerebellar signs such as ataxia, dysarthria,
nystagmus, and intention tremor. The gaze centers are not affected.
Craniopharyngiomas (choice C) are usually seen in children. There is failure of growth,
headaches, and bitemporal hemianopia.
Parasagittal meningiomas (choice D) usually result in headache, spastic paresis, and urinary
incontinence.
A 25-year-old man experiences the gradual onset of intermittent diarrhea, which over years, progresses
to severe diarrhea, alternating with constipation, rectal bleeding, and passage of mucus. On
physical examination, the abdomen is tender over the colon. Stool examination fails to reveal
parasites. Colonoscopy demonstrates inflammation limited to the rectum, with no higher lesions.
Which of the following diseases would most likely be seen in a close relative of the patient?
A. Celiac disease
B. Crohn's disease
C. Hirschsprung's disease
D. Tropical sprue
E. Whipple's disease
Explanation:
The correct answer is B. The presentation is classic for ulcerative colitis. Family members
have an increased incidence of both ulcerative colitis and Crohn's disease, supporting the idea
that these two diseases are actually different ends of the same spectrum. In contrast to
Crohn's disease, in which the lesions may be patchy and involve the distal ileum and even the
esophagus, in ulcerative colitis, the lesions involve the rectum and may extend continuously
proximally for varying distances up to the cecum and very distal end of the ileum.
Celiac disease (choice A) is a small intestinal disease related to gluten intolerance.
Flattening of villi, elongated crypts and marked inflammation in the lamina propria are noted
histologically.
Hirschsprung's disease (choice C) is a congenital cause of severe constipation and megacolon
due to a lack of ganglion cells in the distal colon.
Tropical sprue (choice D) clinically resembles celiac disease, but may be related to infection.
Whipple's disease (choice E) is an intestinal diarrheal disease that has been shown to be due
to a bacterial infection.
A surgeon operating on a 47-year-old woman finds cystic masses in both ovaries. Each mass consists of a
unilocular cyst containing clear fluid. The entire wall is covered with papillary excrescences.

31
Which of the following is the most likely diagnosis?
A. Endometrioid adenocarcinoma
B. Granulosa cell tumor
C. Mature cystic teratoma
D. Mucinous cystadenocarcinoma
E. Serous cystadenocarcinoma
Explanation:
The correct answer is E. Similar to testicular tumors, ovarian tumors can be classified
according to cell of origin. There are three main categories: tumors of surface epithelium,
tumors of germ cell origin, and tumors of sex cord-stromal origin. Ovarian surface epithelium
(coelomic mesothelium) may differentiate along tubal (serous), cervical (mucinous), or
endometrial lines, giving origin to serous cystadenoma/cystadenocarcinoma, mucinous
cystadenoma/cystadenocarcinoma and endometrioid tumors, respectively. Serous tumors represent
40% of all ovarian tumors, and serous cystadenocarcinoma is the most frequent serous tumor.
Serous cystadenocarcinomas occur primarily in women aged 40 to 65 years. About two thirds of
these tumors are bilateral. As the name suggests, it is a cystic tumor containing clear fluid.
The cystic wall is lined by malignant epithelial cells forming papillary fronds.
Less frequent than serous cystadenocarcinoma, endometrioid adenocarcinoma (choice A) and
mucinous cystadenocarcinoma (choice D) also derive from surface epithelium. Endometrioid
carcinoma is histologically similar to endometrial adenocarcinoma, whereas mucinous
cystadenocarcinoma is composed of mucin-producing cells similar to cervical epithelium. Both
these neoplasms have solid and cystic areas (mucinous cystadenocarcinoma more so than
endometrioid carcinoma) and may be bilateral (endometrioid carcinoma more frequently than
mucinous cystadenocarcinoma).
Granulosa cell tumors (choice B) originate from ovarian stroma and consist of variable mixtures
of granulosa cells and theca cells. Since they frequently produce large amounts of estrogens,
these tumors manifest with precocious puberty in preadolescent girls. On the contrary, mature
women with granulosa cell tumors develop endometrial hyperplasia and fibrocystic change of
breast. Histologically, these neoplasms are composed of uniform cuboidal cells, forming
structures reminiscent of ovarian follicles (Call-Exner bodies).
Mature cystic teratoma (choice C) is the most frequent neoplasm derived from germ cells.
Teratomas can be further classified into mature cystic, immature, and monodermal teratomas. The
great majority of teratomas are mature cystic. Since they originate from more than one germ
layer, these neoplasms contain an amazing mixture of mature tissue components, often including
skin, teeth, neural epithelium, thyroid, cartilage, and intestinal tissue, for example.
A 24-year-old woman is seen by her family practitioner. Her urine sample has a stable, frothy white foam
on top. Which of the following substances is likely to be present in her urine in significant
amounts?
A. Bilirubin
B. Blood
C. Glucose

32
D. Ketones
E. Protein
Explanation:
The correct answer is E. Reagent strips of various types are commonly used both in physicians'
offices and in hospital laboratories for rapid semiquantitative urinalysis. The Multistix
strip, which is one of the more commonly used strips, contains reagent squares for glucose,
bilirubin, ketones, specific gravity, blood, pH, protein, urobilinogen, nitrite, and
leukocytes. Each of these squares undergoes a chemical change when dipped in urine, causing the
color of the square to change. The result is "read" by comparing the new color to reference
colors on the bottle. In this case, you need to know that a stable froth on urine is usually
due to proteinuria (more than several grams per 24 hr); therefore, the protein indicator would
be positive on the dipstick.
High levels of bilirubin (choice A) in urine can cause an unusual yellow foam.
Blood in the urine (choice B) might be present in some forms of renal disease, but would not
explain the stable foam.
High levels of glucose (choice C) in urine can cause it to develop a sweet smell and taste;
smelling and tasting urine was an ancient method of diagnosing diabetes mellitus, but is no
longer recommended for obvious reasons.
Ketones (choice D) may give urine an acetone-like odor, but testing for ketones in this manner
is no longer recommended for obvious reasons.
A patient with a long-standing intrauterine contraceptive device develops chronic pelvic pain. The
device is removed, and a biopsy of the endometrium is performed. The biopsy specimen shows a
prominent infiltrate composed of lymphocytes, plasma cells, and histocytes. Which of the
following is the most likely diagnosis?
A. Acute endometritis
B. Adenomyosis
C. Chronic endometritis
D. Endometriosis
E. Simple hyperplasia of endometrium
Explanation:
The correct answer is C. This is chronic endometritis, evidenced by the chronic inflammatory
infiltrate of lymphocytes, plasma cells, and histiocytes. This disorder may be idiopathic but
is more often associated with an obvious predisposing factor, such as chronic pelvic
inflammatory disease, tuberculosis, retained gestational tissue, or, as in this case, an
intrauterine contraceptive device. Chronic endometritis can cause abnormal bleeding, pain, and
infertility.
Acute endometritis (choice A) is characterized by a prominent neutrophilic infiltrate and
usually occurs after delivery or miscarriage.
Adenomyosis (choice B) refers to endometrium abnormally located in myometrium.

33

Endometriosis (choice D) refers to abnormally located patches of endometrium (except in the


myometrium, where it would be called adenomyosis).
Simple hyperplasia of endometrium (choice E) causes cystically dilated glands in endometrium.
Autopsy of a 72-year-old man demonstrates the presence of deep venous thrombosis of the legs and
multiple acute brain infarcts due to thromboembolic occlusion of penetrating arteries. Which of
the following pathologic conditions would most likely account for these findings?
A. Atherosclerosis of penetrating cerebral arteries
B. Endocarditis of the tricuspid valve
C. Patent foramen ovale
D. Pulmonary thromboembolism
E. Trousseau syndrome
Explanation:
The correct answer is C. Persistence of patent foramen ovale is found in a significant
proportion of healthy subjects. A widely patent foramen ovale may allow emboli originating from
the veins in the legs to bypass the pulmonary circulation and reach the systemic arteries,
thereby producing infarcts (paradoxical embolism) in the brain as well as in other organs.
Interatrial or interventricular defects can have the same effect. None of the other answer
choices would explain the development of embolic infarcts in the cerebral parenchyma.
Atherosclerotic changes are frequently found in the circle of Willis and its major branches,
but not in the small-caliber penetrating arteries of the brain (choice A).
Endocarditis of the tricuspid valve (choice B) may give rise to emboli resulting from
fragmentation of valvular vegetations. Emboli from the tricuspid valve, however, would enter
the pulmonary circulation, possibly leading to infarcts of the lungs.
Pulmonary thromboembolism (choice D) frequently occurs as a result of deep venous thrombosis,
especially after immobilization, bed rest, obstetric delivery, and surgery. However,
thromboemboli that become lodged in the pulmonary arteries cannot pass through the pulmonary
capillary filter and cause systemic embolization.
Trousseau syndrome (choice E), also known as migratory thrombophlebitis, occurs in association
with disseminated cancers, especially mucinous adenocarcinomas.This condition is probably due
to release of procoagulant factors by the tumor and manifests with recurrent episodes of
thrombosis affecting veins (but not arteries) in both limbs and visceral organs.
A patient complains of difficulty breathing through his nose and bony pain in his cheeks, near his nose. Physical examination
and CT of the head reveal mass lesions involving the nose, pharynx, and sinuses. CT-guided biopsy demonstrates a nonkeratinizing, squamous cell carcinoma. Which of the following disorders is associated with the same oncogenic virus that is the
likely cause of this patient's cancer?
A. Adult T-cell leukemia
B. Burkitt's lymphoma

34
C. Cervical carcinoma
D. Hepatocellular carcinoma
E. Kaposi's sarcoma
Explanation:
The correct answer is B. The disease is nasopharyngeal carcinoma, which is associated with the Epstein-Barr Virus (EBV).
This virus is also associated with the African form of Burkitt's lymphoma that characteristically involves the jaw.
HTLV-1, or human T-lymphocyte virus, is associated with adult T-cell leukemia (choice A).
HPV, or human papillomavirus, is associated with cervical carcinoma (choice C), penile carcinoma, and anal carcinoma.
Hepatitis B virus (HBV) is associated with hepatocellular carcinoma (choice D). HHV 8, a member of the herpes family, is
associated with Kaposi's sarcoma (choice E).
A 34-year-old man develops pulmonary hemorrhage and glomerulonephritis. Lung biopsy with
immunofluorescence demonstrates IgG deposition along the basement membrane. These antibodies are most likely
directed against which of the following types of collagen?
A. Type I
B. Type II
C. Type III
D. Type IV
E. Type X
Explanation:
The correct answer is D. The disease described is Goodpasture's syndrome, in which autoantibodies to
basement membrane proteins cause damage to the lungs and kidneys. Pulmonary hemorrhage (especially in smokers) and
rapidly progressive glomerulonephritis are common. The characteristic autoantibody present is directed against Type IV
collagen, a component of the basement membrane.
Type I collagen (choice A) is found in bone, skin, tendon, dentin, fascia, and late wound repair.
Type II collagen (choice B) is found in cartilage (including hyaline cartilage), the vitreous body of the eye, and the nucleus
pulposus of the intervertebral disk. Type III collagen (choice C) is found in skin, blood vessels, uterus, fetal tissue, and
granulation tissue.
Type X collagen (choice E) is found in epiphyseal plates.
A patient has long-standing severe hemolytic anemia characterized by hypochromic cells. Electrophoresis studies demonstrate
a near complete absence of beta chains. Several years later, the patient develops cardiac failure. Intracardiac deposition of
which of the following would be most likely to contribute to the cardiac failure?
A. Calcium
B. Iron
C. Magnesium
D. Potassium
E. Sodium
Explanation:
The correct answer is B. The disease is beta thalassemia major, which is a severe hemolytic anemia

35
characterized by a failure to produce the beta chains of hemoglobin (some HbF, the fetal form of hemoglobin, is produced).
The excess alpha chains are insoluble, leading to intra- and extravascular hemolysis. These patients require large numbers of
transfusions, and iron overload with resulting secondary hemochromatosis can contribute to eventual cardiac failure. The heart
is also damaged by the chronic high output state needed to compensate for the anemia.
Calcium (choice A) deposition is seen in damaged tissues and states with high serum calcium, such as hyperparathyroidism.
Magnesium (choice C), potassium (choice D), and sodium (choice E) are highly soluble and do not usually precipitate in
tissues.
A 30-year-old woman presents with complaints of weakness and headaches. Her friends say she has been
irritable and depressed lately. On physical examination, the patient is jaundiced, and her liver is small and firm. Neurologic
examination is remarkable for choreoathetotic movements and a fine tremor that, when her upper limbs are extended,
resembles a bird flapping its wings. Which of the following tests would most likely lead to to correct diagnosis?
A. Nerve conduction studies
B. Prussian blue stain of liver biopsy specimen
C. Serum alkaline phosphatase
D. Serum transaminases
E. Slit-lamp examination of the eyes
Explanation:
The correct answer is E. This patient is exhibiting symptoms of Wilson's disease, which is due to inadequate
copper excretion by the biliary system. Mutations in a copper-transporting ATPase, coded for by the ATP7B
gene on chromosome 13, appear to underlie this autosomal recessive disorder. Accumulation of copper in the
liver initially produces fatty change, followed by hepatocellular necrosis, inflammation, bile duct proliferation,
and cirrhosis. Eventually, the copper spills out of the liver to deposit in other tissues, notably the brain. The
caudate nucleus and putamen are generally most affected; injury to these structures produces an
extrapyramidal movement disorder that most commonly presents with choreoathetosis and tremor but which
may produce cerebellar signs or parkinsonism. Various psychiatric symptoms can accompany neurological
involvement. Copper deposition in Descemet's membrane in the cornea produces the nearly pathognomonic
Kayser-Fleischer ring, which can be seen with slit-lamp examination of the eyes. Failure to demonstrate a
Kayser-Fleischer ring in a patient with hepatic disease and neurological impairment virtually excludes the
diagnosis of Wilson's disease. Low serum ceruloplasmin and increased urinary copper, or increased copper
levels on liver biopsy, are diagnostic.
Nerve conduction studies (choice A) would be of little value in the diagnosis of Wilson's disease, although they
are valuable in detecting dysfunction of peripheral nerves that result, for example, from demyelination, loss of
nerve axons, failure of conduction, or neuromuscular junction failure.
A Prussian blue stain performed on a liver biopsy (choice B) would demonstrate increased iron stores in a
patient's liver secondary to hemochromatosis.
Serum alkaline phosphatase (choice C) would be elevated in patients with biliary tract disease, although other
types of liver disease, bone disease, or pregnancy can produce elevations as well.
Serum transaminases (choice D) would be increased in patients with a variety of diseases causing
hepatocellular injury, including hepatitis and cirrhosis, but this finding is not particularly specific.

36
A physician in the emergency department is evaluating a patient with severe chronic obstructive pulmonary disease
prior to oxygen supplementation. The physician decides to draw arterial blood for blood gas studies. His technique
is faulty, however, and he introduces room air into the syringe while pulling on the plunger as he is drawing the
syringe out of the patient. Which of the following patterns of changes would be most likely to be produced by this
exposure of arterial blood to room air?

PO2
PCO2
pH
A. Decreased
Decreased
Decreased
B. Decreased
Elevated
Decreased
C. Elevated
Decreased
Decreased
D. Elevated
Decreased
Elevated
E. Elevated
Elevated
Elevated
Explanation:
The correct answer is D. The technical part of the collection of arterial blood samples is difficult. Some hospitals
allow only physicians to collect the samples, while other hospitals allow nurses or technicians with additional
special training to collect the samples. No matter who performs the arterial draw, care must be taken to avoid
exposing the blood to room air, as such exposure tends to cause the blood to partially equilibrate with the room
air. Room air would have a higher PO2 and a lower PCO2 than this patient's blood, so the sample would have a
higher PO2 and a lower PCO2. In the atmosphere, PO2 = 150 mm Hg and PCO2 is near 0 mm Hg; in the arterial
blood of a healthy patient, PO2 = 100 mm Hg, PCO2 = 40 mm Hg (PO2 could be lower and PCO2 higher in a
diseased individual). Because CO2 is decreased, there will be less carbonic acid present in the blood, thus
raising the pH.

In which of the following sites do myxopapillary ependymomas most frequently occur?


A. Cerebellum
B. Conus medullaris
C. 4th ventricle

37

D. Lateral ventricles
E. Midbrain
Explanation:
The correct answer is B. Myxopapillary ependymoma is a variant of ependymoma, a tumor arising from
ependymal cells. Histologically, myxopapillary ependymoma contains a myxoid (mucus-rich) intercellular matrix,
in which spindly neoplastic ependymal cells are arranged in a fascicular and papillary pattern (hence its
designation). It is a benign tumor that almost always occurs in the distal segment of the spinal cord, ie, the
conus medullaris. Once excised, the patient is cured.
The cerebellum (choice A) is the favorite site for pilocytic astrocytomas, medulloblastomas, and
hemangioblastomas, but not ependymomas.
In general, classic ependymomas occur in close proximity to the ventricular cavities, specifically, the 4th
ventricle (choice C) in children and the lateral ventricles (choice D) in adults. The myxopapillary variant does
not occur in either location.
A midbrain location (choice E) would be truly exceptional for any type of ependymoma.

A 54-year-old woman with chronic microcytic hypochromic anemia also has a sore, smooth, red tongue and a
sense of dysphagia midway during swallowing. This patient is at increased risk for developing which of the
following conditions?
A. Adenocarcinoma of the esophagus
B. Barrett's esophagus
C. Candida esophagitis
D. CMV esophagitis
E. Squamous cell carcinoma of esophagus
Explanation:
The correct answer is E. The patient has Plummer-Vinson syndrome, characterized by atrophic glossitis,
esophageal webs, and iron-deficiency anemia. Patients with this syndrome are at increased risk of developing
squamous cell carcinoma of the esophagus.
Barrett's esophagus (choice B) and adenocarcinoma of the esophagus (choice A) are associated with reflux
esophagitis.
Candida(choice C) and CMV (choice D) esophagitis can be seen in immunosuppressed patients, including AIDS
patients.

Which of the following locations is most likely for the development of carcinoma in a 32-year-old baseball player

38
who has chewed tobacco for 15 years?
A. Floor of the mouth
B. Lower lip
C. Tongue
D. Tonsils
E. Upper lip
Explanation:
The correct answer is B. Oral cancer is most strongly related to tobacco chewing, with weaker associations with
cigarette smoking, pipe smoking, and alcohol use. Unfortunately, many teenagers believe that chewing tobacco
is "harmless" because it does not cause lung cancer, and the case illustrated in the question is unfortunately
not uncommon. Oral cancers tend to occur on the lower lip (40%; choice B), tongue (20%; choice C), floor of
the mouth (15%; choice A), with other oral sites (choices D and E) being less common. They are usually
squamous cell carcinomas and unlike their skin counterparts, frequently cause both extensive morbidity and
mortality.

A 67-year-old male smoker presents to his physician for a routine physical examination. Chest x-ray demonstrates
a 2-cm density on the left side. Laboratory studies are remarkable for a serum sodium of 134 mEq/L. The findings
may be attributable to tumor cell secretion of
A. adrenocorticotrophic hormone (ACTH)
B. antidiuretic hormone (ADH)
C. melanocyte-stimulating hormone (MSH)
D. parathyroid hormone (PTH)
E. vasoactive intestinal polypeptide (VIP)
Explanation:
The correct answer is B. All of the hormones listed can be secreted by bronchogenic carcinoma, and may cause
a paraneoplastic syndrome. Of the answer choices provided, only ADH (antidiuretic hormone) causes
hyponatremia.
ACTH (choice A) causes Cushing's syndrome.
MSH (choice C) causes increased skin pigmentation.
PTH (choice D) causes hypercalcemia.
VIP (choice E) causes diarrhea and hypokalemia.

39
Other hormones that can be produced include human chorionic gonadotropin (hCG; gynecomastia), prolactin
(lactation), and calcitonin (hypocalcemia).

A 48-year-old female presents to the doctor with lower back pain. She states that she has had the pain for about
two weeks and that it has become steadily more severe. An x-ray shows a lytic bone lesion in her lumbar spine.
Review of systems reveals the recent onset of mild headaches, nausea, and weakness. Her CBC shows a
normocytic anemia, and her erythrocyte sedimentation rate is elevated. Urinalysis shows heavy proteinuria, and a
serum protein electrophoresis shows a monoclonal peak of IgG. Which of the following is responsible for this
patient's spinal lesion?
A. Bence-Jones proteins
B. Lymphoplasmacytoid proliferation
C. Osteoblast activating factor
D. Osteoclast activating factor
E. Primary amyloidosis (AL)
Explanation:
The correct answer is D. First of all, the disease described above is multiple myeloma. Multiple myeloma is a
plasma cell neoplasm in which the plasma cells proliferate a single, or monoclonal, type of immunoglobulin. In
this case, and most commonly, IgG is produced. Patients with this disease are usually over 40 and may have
normocytic anemias. They often complain of skeletal pain from lytic bone lesions and may report headaches
and nausea caused by hyperviscosity of the blood due to the excessive amounts of immunoglobulins. The lytic
bone lesions are caused by the production of osteoclast activating factor by the neoplastic plasma cells. This
can also lead to hypercalcemia.
Bence-Jones proteins (choice A), are immunoglobulin light chains. They are often overproduced in multiple
myeloma and are filtered in the urine. They are not usually detected in serum unless there is renal impairment,
but they can be detected in the urine by electrophoresis and immunofixation. They do not cause bony lytic
lesions.
Lymphoplasmacytoid proliferation (choice B), describes a normal type of B lymphocyte which is morphologically
between a lymphocyte and a plasma cell. Lymphoplasmacytoid lymphocytes produce IgM, and in Waldenstrom's
macroglobulinemia, they undergo neoplastic proliferation and produce IgM peaks. Bone lesions are not seen in
this disease.
Osteoblast activating factor (choice C), would not produce osteolytic lesions and is not seen in multiple
myeloma. There is a rare osteoblastic variant of multiple myeloma with dense bony osteosclerosis rather than
lytic lesions, but osteoblast activating factor has not been shown to be involved.
Primary amyloidosis (AL) (choice E), is a primary light-chain type of amyloidosis associated with multiple
myeloma. The insoluble proteinaceous deposits occur in the tongue, heart, kidney, and skin. This does not
cause bony lytic lesions.

A 35-year-old man who recently traveled to a third world country develops chronic, severe dysentery.
Colonoscopy demonstrates ulceration of the cecum, and a cecal biopsy reveals 15-to-40 micron amoebae with

40
ingested erythrocytes and small nuclei with distinctive tiny central karyosomes. Which of the following organisms
is the most likely culprit?
A. Acanthamoeba sp.
B. Balantidium coli
C. Entamoeba histolytica
D. Giardia lamblia
E. Naegleria fowleri
Explanation:
The correct answer is C.Entamoeba histolytica is the usual cause of intestinal amebiasis, and has the
microscopic features described in the question stem. A particularly helpful (but not always present) feature of
this organism is the presence of ingested red blood cells within the amoebae. These amoebae cause
flask-shaped ulceration of the intestinal mucosa and submucosa, with a particular propensity for involving the
cecum and ascending colon. The disease manifestations range from none (asymptomatic carriers) to mild
chronic diarrhea, to severe, purging dysentery. In symptomatic cases, the liver may develop destructive
amoebic liver abscesses that tend to become secondarily (and potentially life-threateningly) infected by
bacteria.
Acanthamoeba(choice A) is a free-living amoebae that can cause amoebic meningocephalitis.
Balantidium coli(choice B) is a large ciliated intestinal parasite that can occasionally cause colonic disease
resembling that caused by Entamoeba histolytica.
Giardia lamblia(choice D) is a small intestinal protozoa with a distinctive pear-shaped morphology that appears
to have a "face."
Naegleria fowleri(choice E) is a free-living amoebae that can cause amoebic meningoencephalitis.

A 27-year-old man develops bilateral parotid gland swelling and orchitis, and is generally ill with fever of 102 F.
Which of the following substances is most likely to be significantly elevated in the patient's serum?
A. Alanine aminotransferase (ALT)
B. Amylase
C. Aspartate aminotransferase (AST)
D. Ceruloplasmin
E. Creatine phosphokinase, MB isoenzyme (CPK-MB)
Explanation:
The correct answer is B. The disease is mumps, caused by a paramyxovirus. In children, mumps causes a

41
transient inflammation of the parotid glands, and less commonly, the testes, pancreas, or central nervous
system. Mumps tends to be a more severe disease in adults than in children. Mumps in adults involves the
testes (causing orchitis) and pancreas with some frequency. Pancreatic involvement can cause elevation of
serum amylase.
ALT (choice A) and AST (choice C) are markers for hepatocellular damage.
Ceruloplasmin (choice D) is a copper-carrying protein that is decreased in Wilson's disease.
CPK-MB (choice E) is the isoenzyme of CPK that is relatively specific for the myocardium. This enzyme is
increased in the early stages of a myocardial infarction.

A 50-year-old hypertensive man develops very severe, "tearing" chest pain, which migrates from his upper back
to mid-back over the period of an hour. Pathologic examination of a specimen removed from the patient during
emergency surgery would most likely demonstrate which of the following?
A. Cystic medial degeneration
B. Infarction
C. Plasma cells around the vasa vasorum
D. Severe atherosclerosis
E. Tree-barking
Explanation:
The correct answer is A. This is a classic description of a dissecting aortic aneurysm, a very important condition
that may cause death if missed or misdiagnosed. Dissecting aneurysms are actually dissecting hematomas, with
the blood located between the middle and outer thirds of the media of the aorta. Dissecting aneurysms are
associated with hypertension in many cases; they are also associated with cystic medial degeneration of the
wall of the aorta (seen in Marfan's syndrome). Often, an intimal tear is present; these are thought to represent
the starting point for the dissection. Unlike abdominal aortic aneurysm and syphilitic aneurysm, aortic dissection
is not usually associated with aortic dilatation.
The pain of myocardial infarction (choice B) does not usually move.
Plasma cells around the vasa vasorum (choice C) and "tree-barking" (wrinkling of the aortic intima; choice E )
are features of syphilitic aneurysms.
Atherosclerotic (choice D) aneurysms typically affect the abdominal aorta.

The presence of the Philadelphia chromosome is associated with a more favorable prognosis in patients with
which of the following diseases?
A. Acute lymphoblastic leukemia
B. Acute myelogenous leukemia

42

C. Chronic lymphocytic leukemia


D. Chronic myelogenous leukemia
E. Hairy cell leukemia
Explanation:
The correct answer is D. The presence of the Philadelphia chromosome, a translocation from the long arm of
chromosome 22 to chromosome 9 [t(9;22)], is associated with a more favorable prognosis in patients with
chronic myelogenous leukemia.
Acute lymphoblastic leukemia (ALL; choice A) is the most common cause of leukemia in children. The presence
of the Philadelphia chromosome is associated with a worse prognosis for the patient. This form of leukemia is
also associated with a B-ALL translocation of the c-myc proto-oncogene of chromosome 8 to chromosome 14
[t(8;14)(q24;q32)].
Acute myelogenous leukemia (AML; choice B) is the most common acute leukemia in adults. The M2 subtype is
associated with the t(8;21) translocation and the M3 subtype is associated with the t(15;17) translocation.
Over half of patients with chronic lymphocytic leukemia (choice C) display one of several chromosomal
abnormalities. This includes trisomy 12 (involves the h-ras proto-oncogene), translocation t(11;14) (involves
k-ras and bcl-1 proto-oncogenes), and deletion (14q-) or inversion (14q) (involves immunoglobulin heavy chain
gene).
Hairy cell leukemia (choice E) is associated with the expression of tartrate-resistant acid phosphatase (TRAP)
on the surface of B cells.

A 32-year-old woman visits her gynecologist for a Pap smear. On physical examination, her gynecologist palpates
a large adnexal mass on the right. After ultrasound confirmation of a large ovarian mass, a laparotomy is
scheduled, and the mass is removed. Pathologic examination of the mass demonstrates a cystic cavity filled with
hair and keratin debris, and the wall contains skin, adnexal tissue, thyroid tissue, and neural tissue. All of the
tissues are similar to those normally found, and no malignant changes are seen. Which of the following is the
most likely diagnosis?
A. Immature teratoma
B. Leiomyoma
C. Leiomyosarcoma
D. Mature teratoma
E. Rhabdomyosarcoma
Explanation:
The correct answer is D. The lesion is a mature teratoma. Teratomas located in the ovary and containing a hair
and keratin filled cyst are sometimes called dermoid cysts. Teratomas contain cells of a variety of types, often
including skin, skin adnexal structures (hair follicles, sweat glands, sebaceous glands), connective tissues,

43
neural tissue, muscle, and thyroid tissue. If immature tissues such as primitive neuroepithelial cells or
developing skeletal muscle cells are seen, the lesion is considered potentially malignant and classified as an
immature teratoma (choice A).
Leiomyomas (choice B) are benign tumors of smooth muscle (e.g., uterine "fibroids"), usually in the female
genital tract.
Leiomyosarcomas (choice C) are rare malignant tumors of smooth muscle, usually in the female genital tract.
Rhabdomyosarcomas (choice E) are malignant skeletal muscle tumors with a predilection for the head and neck
and urogenital regions in children.

An endocrinologist examines a patient suspected of having Riedel thyroiditis. Which of the following findings on
physical examination would best help confirm the diagnosis?
A. Eyeball protrusion
B. Massive soft thyroid gland
C. Single large thyroid nodule
D. Very tender and painful thyroid
E. "Woody" thyroid gland
Explanation:
The correct answer is E. Riedel thyroiditis, also called ligneous (rocklike) stroma, is a rare form of chronic
thyroiditis characterized microscopically by a marked fibrous reaction that destroys most or all of the thyroid
gland and may involve adjacent structures. The etiology is unknown. Clinically, this disease tends to affect
middle-aged and older, mostly female patients and causes the thyroid to have a firm "woody" texture. It may be
clinically mistaken for a neck malignancy and can cause symptoms of stridor, dyspnea, dysphasia, laryngeal
nerve paralysis, or hypothyroidism.
Eyeball protrusion (choice A) suggests the hyperthyroidism of Graves disease.
A massive, soft thyroid gland (choice B) suggests multinodular goiter.
A single large thyroid nodule (choice C) could be due to either a thyroid adenoma or thyroid cancer.
A very tender and painful thyroid (choice D) suggests subacute granulomatous (de Quervain) thyroiditis.

A 40-year-old woman with polycythemia vera develops progressive severe ascites and tender hepatomegaly over
a period of several months. Liver function tests are near normal. Which of the following tests would be most likely
to establish the diagnosis?
A. Endoscopic retrograde cholangiopancreatography (ERCP)
B. Hepatic venography

44

C. Serum alpha-fetoprotein
D. Serum ceruloplasmin
E. Serum iron studies
Explanation:
The correct answer is B. The clinical presentation is most consistent with Budd-Chiari syndrome (hepatic vein
obstruction), which may occur as a complication of thrombogenic and myeloproliferative disorders, including
polycythemia vera. The presentation illustrated is the most common; alternative presentations include fulminant
liver failure and cases in which intractable abdominal pain is the most prominent initial finding. The best method
listed to establish the diagnosis of Budd-Chiari syndrome is hepatic venography to demonstrate the occlusion of
the hepatic venous system. Liver biopsy to provide evidence of centrilobular congestion and sinusoidal dilatation
(in the absence of right-sided heart failure) is definitive, but more invasive.
Endoscopic retrograde cholangiopancreatography (choice A) is most useful in demonstrating lesions of the
biliary tree.
Serum alpha-fetoprotein (choice C) is a marker for hepatocellular carcinoma.
Ceruloplasmin levels (choice D) are altered in Wilson's disease, in which cirrhosis and brain damage occur
secondary to abnormalities in the metabolism of copper.
Serum iron studies (choice E) are useful when considering hemochromatosis as a cause of cirrhosis.

A 49-year-old homemaker presents to her physician because she noticed a lump in her breast during
self-examination. Biopsy of the lump demonstrates invasive ductal carcinoma. The connective tissue adjacent to
the nests of tumor is very densely collagenous. This is an example of which of the following processes?
A. Anaplasia
B. Carcinoma in situ
C. Desmoplasia
D. Dysplasia
E. Metaplasia
Explanation:
The correct answer is C. This is an example of desmoplasia, which is excessive fibrous tissue formation in the
stroma of a tumor. The abundant fibrous tissue growth is, itself, benign.
Anaplasia (choice A) is a term used for tumors that show severe loss of cell differentiation and tissue
organization; anaplastic tumors typically are much more clinically aggressive than their well-differentiated
counterparts.
Dysplasia (choice D) is atypical cellular proliferation (without being so severe as to qualify for the diagnosis of

45
cancer); an example is the epithelium seen in tubular adenomas of the colon. In contrast, carcinoma in situ
(choice B) is a similar change that is severe enough to be classified as cancer, but is confined to the epithelium
with no invasion of underlying tissue. Carcinoma in situ can be found throughout the epithelial surfaces of the
body.
Metaplasia (choice E) is the replacement of one type of differentiated cell or tissue by another not normally
present at that site; an example is the replacement, in smokers, of the normal, ciliated, columnar epithelium of
the respiratory tract with squamous epithelium.

Autopsy of an elderly individual who died in a nursing home with no known genetic diseases reveals small
amounts of amyloid deposition in the heart. Amyloid deposition is not seen in other organs. There is no history of
long-standing inflammatory disease. This type of amyloid would be most likely to be composed of which of the
following proteins?
A. Amyloid-associated protein
B. Amyloid light chain protein
C. Beta-2-amyloid protein
D. Beta-2-microglobulin
E. Transthyretin
Explanation:
The correct answer is E. This patient has senile cardiac amyloidosis, which is usually a clinically insignificant
condition due to deposition of structurally normal transthyretin (formerly called prealbumin). The transthyretin is
a normal serum protein used to transport thyroxin and retinal. In addition to causing senile cardiac amyloidosis,
transthyretin, in a mutant rather than normal form, is deposited as amyloid in the familial amyloid
polyneuropathies.
Amyloid-associated protein (choice A) is a more common protein deposited as amyloid, and precipitates in
secondary amyloidosis associated with underlying chronic inflammatory conditions.
Amyloid light chain protein (choice B) is a common protein deposited as amyloid, and precipitates in amyloidosis
related to multiple myeloma and other monoclonal B cell proliferations.
Beta-2-amyloid protein (choice C) is deposited as amyloid in the brain (notably in blood vessels and cerebral
plaques) of patients with Alzheimer's disease.
Beta-2-microglobulin (choice D) is a normal serum protein that is deposited in amyloidosis complicating
long-term hemodialysis.

Routine physical examination of a patient demonstrates proteinuria by dipstick method. No glucose is detected.
Urine protein electrophoresis demonstrates a monoclonal spike. A tumor of which cell line would most likely
produce these findings?
A. Plasma cells

46

B. Renal tubular cells


C. Smooth muscle
D. T-lymphocytes
E. Transitional epithelium
Explanation:
The correct answer is A. The tumor is multiple myeloma, a neoplasm of plasma cells. The monoclonal spike on
urine protein electrophoresis is typically due to excess light chains (associated with urinary Bence-Jones
protein).
Monoclonal proteinuria would not be produced by tumors of the the other cell lines listed.
Renal cell carcinoma is a tumor of renal tubular cells (choice B).
Leiomyoma and leiomyosarcoma are tumors of smooth muscle (choice C).
Cutaneous T-cell lymphomas and T-cell leukemia are tumors of T-lymphocytes (choice D).
Transitional cell carcinoma is a tumor of transitional epithelium (choice E).

An 83-year-old female has a biopsy of an ulcerated nipple lesion that is interpreted as Paget's disease. A biopsy
of the underlying breast tissue will most likely show which of the following?
A. Acute mastitis
B. Ductal carcinoma in situ
C. Intraductal papilloma
D. Invasive lobular carcinoma
E. Normal breast tissue
Explanation:
The correct answer is B. Paget's disease of the breast is a form of ductal carcinoma in which neoplastic cells
involve the squamous epithelium of the skin by direct extension through the lactiferous ducts. Underlying breast
tissue shows the origin of the ductal carcinoma-usually ductal carcinoma in situ and less frequently invasive
ductal carcinoma.
Acute mastitis (choice A) is a disease of nursing women in which bacteria gain entry to the breast tissue via
cracks in the traumatized nipple. It is characterized by acute inflammation and tissue necrosis.
Intraductal papilloma (choice C), a papillary mass arising within the ducts, usually presents as a single
subareolar tumor that may produce a bloody or serous nipple discharge. Most intraductal papillomas are
benign and are cured with complete excision.

47

Invasive lobular carcinoma (choice D) is a tumor of the terminal ductules of the breast. It presents as a poorly
circumscribed, rubbery breast mass, unlike invasive ductal carcinoma, which tends to appear as a hard,
stellate, and fibrous tumor. Lobular carcinoma does not produce Paget's disease.
Paget's disease of the breast always reflects underlying duct cancer. This is in marked distinction from
extramammary Paget's disease, which may arise without an identifiable underlying malignancy (choice E).

A 52-year-old male presents with epigastric pain that improves with meals. Endoscopy demonstrates a 2 cm
ulcerated area located 3 cm distal to the pyloric junction. Basal acid output is within normal limits. Which of the
following is most likely to have made the strongest contribution to the development of this disease?
A. Aspirin use
B. Chronic antacid use
C. Drinking alcohol
D. Helicobacter pylori infection
E. Smoking
Explanation:
The correct answer is D. The patient has a duodenal peptic ulcer. The strongest risk factor for duodenal peptic
ulcer is Helicobacter pylori infection, which is found in almost 100% of these cases (contrast to 70% infection
rate in gastric peptic ulcer). The basal acid output is normal in many patients with duodenal ulcer.
Aspirin use (choice A) and ethanol use (choice C) are more strongly implicated in gastric ulcer disease than
duodenal ulcer disease.
Chronic antacid use (choice B) is seen as a result of peptic ulcer disease, not as a cause of it.
Smoking (choice E) may also be a lesser contributing factor to the development of peptic ulcer.
A 42-year-old woman is noted to have mildly elevated creatinine and blood urea nitrogen on routine physical
exam. She recalls that her father also had kidney trouble and died in kidney failure. Workup reveals persistent
azotemia and microscopic hematuria without evidence of urinary tract infection. An ultrasound of the kidneys
identifies bilaterally enlarged and multicystic kidneys. In addition to chronic renal failure, the clinician should also
be concerned about her risk of
A. liver failure
B. pancreatic insufficiency
C. portal hypertension
D. renal cell carcinoma
E. subarachnoid hemorrhage

48

Explanation:
The correct answer is E. Multicystic kidneys, slowly progressive renal failure, and a positive family history are
characteristics of autosomal dominant (adult) polycystic kidney disease (APKD). This disease typically presents
in the 40s to 60s and is characterized by marked renal enlargement due to numerous fluid-filled cysts, which
develop between the normally functioning nephrons. APKD is highly associated with hepatic cysts, and berry
aneurysms in the circle of Willis that may rupture, producing spontaneous subarachnoid hemorrhage.
Infrequently, APKD also produces cysts in the pancreas, spleen, or lungs, but these are not clinically relevant.
Hepatic cysts in adult polycystic disease do not ordinarily produce symptoms of hepatic failure (choice A).
Pancreatic cyst formation in APKD is not generally associated with pancreatic insufficiency (choice B).
Children with autosomal recessive polycystic kidney disease may develop congenital hepatic fibrosis with
hypertension and splenomegaly, but this is not part of APKD (choice C).
APKD is not considered a risk factor for renal cell carcinoma or any other type of cancer (choice D).

A 32-year-old African American female presents with pelvic pain, low back pain, and a sensation of "pulling" or
"stretching" in her groin. Bimanual examination reveals a firm mass in the right adnexa. An ultrasound
examination reveals the presence of fluid in the abdominal cavity and the right thoracic cavity. Which of the
following conditions is most strongly suggested by this patient's presentation?
A. Ectopic pregnancy
B. Endometrial implant
C. Ovarian fibroma
D. Pelvic inflammatory disease
E. Uterine leiomyoma
Explanation:
The correct answer is C. Meigs syndrome is the unusual combination of hydrothorax (often right sided), ascites,
and an ovarian tumor (often a benign fibroma). Low back pain and a stretching or pulling sensation are
characteristic of ascites in some patients (ascites can also be asymptomatic). The etiology of the fluid
accumulation in Meigs syndrome remains a mystery.
Ectopic pregnancy (choice A) would not be associated with hydrothorax. If an ectopic pregnancy ruptured,
blood could accumulate in the abdominal cavity, but the patient would likely be hypotensive or dead.
An endometrial implant (choice B) could cause pain, or impair fertility, but would not be expected to produce the
combination of ascites and isolated right-sided hydrothorax.
Pelvic inflammatory disease (choice D) would be unlikely to produce the combination of ascites and isolated
right-sided hydrothorax, although it might cause low back pain or pelvic pain.

49
A uterine leiomyoma (choice E) would not be expected to produce an adnexal mass, nor would it be likely to
cause ascites and hydrothorax.
A 3-year-old child develops severe generalized edema following a viral infection. On the basis of clinical
chemistry tests, a renal biopsy is performed, with normal light microscopic findings. Which of the following
abnormal laboratory values might be expected in this individual?
A. Decreased alpha globulin levels
B. Decreased fibrinogen
C. Increased serum calcium levels
D. Low serum albumin levels
E. Red blood cell casts in the urine
Explanation:
The correct answer is D. This child has minimal change disease, which is the major cause (over 90% of cases)
of nephrotic syndrome in children aged 2 to 6 years. The most prominent clinical chemistry finding in these
patients is massive proteinuria. The urinary protein in minimal change disease, in contrast to other causes of
nephrotic syndrome, is often composed predominantly of albumin. Many other clinical chemistry changes may
also be seen, including decreased serum albumin levels, hyperlipidemia, increased serum levels of alpha2- and
beta-globulins, decreased IgG, and increased fibrinogen. Minimal change disease characteristically shows
normal or near normal appearance of the glomeruli by light microscopy and extensive fusion of foot processes
of the glomerular podocytes by electron microscopy. A point not always recognized by beginners is that the
podocyte alterations may represent a reaction to, rather than a cause of, the proteinuria (e.g., an attempt to
"seal the holes" in the glomerulus), since varying degrees of foot process fusion (together with more specific
features) may sometimes be seen in other glomerular diseases associated with the nephrotic syndrome.
Alpha-globulin levels (choice A) would be increased, rather than decreased, in minimal change disease.
Fibrinogen levels are increased, rather than decreased (choice B).
Serum calcium levels (choice C) are typically decreased in the nephrotic syndrome, possibly due to renal loss
of vitamin D binding protein.
Red blood cell casts in the urine (choice E) are indicative of glomerulonephritis, rather than the nephrotic
syndrome.

A 42-year-old African-American man sustains severe injuries in an automobile accident and is admitted to the
intensive care unit. Examination of a peripheral blood smear on the 3rd day of admission reveals helmet cells,
schistocytes, and decreased platelets. Which of the following is most strongly suggested by these findings?
A. Autoimmune hemolysis
B. Disseminated intravascular coagulation (DIC)
C. Hereditary spherocytosis

50

D. Megaloblastic anemia
E. Sickle cell anemia
Explanation:
The correct answer is B. The findings suggest disseminated intravascular coagulation (DIC), which is a feared
complication of many other disorders, such as obstetrical catastrophes, metastatic cancer, massive trauma, and
bacterial sepsis. The basic defect in DIC is a coagulopathy characterized by bleeding from mucosal surfaces,
thrombocytopenia, prolonged PT and PTT, decreased fibrinogen level, and elevated fibrin split products.
Helmet cells and schistocytes (fragmented red blood cells) are seen on peripheral blood smear.
Autoimmune hemolysis (choice A) and hereditary spherocytosis (choice C) would be characterized by
spherocytes in the peripheral smear.
Macro-ovalocytes and hypersegmented neutrophils can be seen in megaloblastic anemia (choice D).
Sickle cells are seen in sickle cell anemia (choice E).
A 52-year-old man is found dead in his home. Autopsy reveals hemopericardium secondary to ventricular wall
rupture. Roughly how long before his death did the man probably have a myocardial infarction?
A. 2 days
B. 7 days
C. 12 days
D. 20 days
E. 60 days
Explanation:
The correct answer is B. Unsuspected (or denied) myocardial infarction is not uncommon, and death may occur
because of untreated complications. A number of serious complications can occur between 5 and 10 days
following infarction, due to marked weakening of the necrotic myocardium. These include rupture of the
ventricular wall leading to hemopericardium and cardiac tamponade (as this patient had), rupture of the
interventricular septum, and rupture of the papillary muscle.
Arrhythmias are the most common complication 2 days post-infarction (choice A).
Fibrinous pericarditis secondary to an autoimmune phenomenon (Dressler's syndrome) can be seen several
weeks after infarctions (choices C and D).
By 60 days after infarction (choice E), the contracted scar is usually complete, and residual complications
include left ventricular failure and arrhythmias.

51
A 25-year-old man presents to a rheumatologist with complaints of joint pain involving the large joints of the legs.
On questioning, the patient indicates that exacerbations in the joint pain are frequently accompanied by diarrhea.
Which of the following gastrointestinal diseases is most likely to be implicated as the cause of the patient's joint
problems?
A. Amebic colitis
B. Chronic appendicitis
C. Diverticulosis
D. Pseudomembranous colitis
E. Ulcerative colitis
Explanation:
The correct answer is E. Several gastrointestinal diseases are associated with rheumatologic complaints. The
most frequent of these are the chronic inflammatory bowel diseases, ulcerative colitis and Crohn's disease,
which can be associated with sacroiliitis (related to HLA-B27) or lower limb arthritis. Other GI diseases
associated with arthropathy include bypass surgery, Whipple's disease, Behcet's syndrome, and celiac disease.
Amebic colitis (choice A) is caused by ingestion of infectious cysts (typically from Entamoeba histolytica).
Symptoms include abdominal pain and diarrhea; malaise and weight loss may occur. Cecal amebiasis can
resemble acute appendicitis.
Chronic appendicitis (choice B) may be asymptomatic or cause poorly defined abdominal pain.
Diverticulosis (choice C) is usually a disease of older adults. It is often asymptomatic unless inflammation
supervenes.
Pseudomembranous colitis (choice D) is a severe form of diarrhea usually seen in the setting of prior antibiotic
use. The causative organism is almost always Clostridium difficile.

A 45-year-old woman complains of difficulty speaking, chewing, and swallowing. She experiences generalized
weakness that increases with effort and as the day goes on. Symptoms are significantly improved after taking
neostigmine. Autoantibodies responsible for causing the patient's condition are directed against
A. acetylcholine receptors
B. double-stranded DNA
C. dystrophin
D. erythrocyte surface antigens
E. myelin
Explanation:

52

The correct answer is A. The patient has myasthenia gravis (MG), which typically produces weakness
worsening over the course of the day. It often affects the eye muscles and can produce diplopia. Neostigmine,
an acetylcholinesterase inhibitor, would temporarily improve the patient's condition, which is associated with
antibodies against nicotinic acetylcholine receptors present on skeletal muscle.
Antibodies to double-stranded DNA (choice B) as well as anti-Smith antibodies are found specifically in systemic
lupus erythematosus. Peripheral nuclear staining is observed on immunofluorescence.
Antibodies to dystrophin (choice C) are not a recognized pathology. Abnormal or absent dystrophin, resulting
from mutations in the X chromosome, is associated with Becker's and Duchenne muscular dystrophy,
respectively. Pelvic girdle weakness and ataxia are classic symptoms.
Antibodies to erythrocyte surface antigens (choice D) can be found in warm antibody autoimmune hemolytic
anemia. Patients with this condition would have a positive direct Coomb's test.
Antibodies to myelin (choice E) may play a role in multiple sclerosis, which is presumed to be of autoimmune
etiology. This demyelinating disease is characterized by the spontaneous appearance and remission of
symptoms such as hyperreflexia, weakness, spasticity, dysarthria, tremor, ataxia, and visual disturbances.
Neostigmine would not produce any improvement.

A 40-year-old woman with polycythemia vera develops progressive severe ascites and tender hepatomegaly over
a period of several months. Liver function tests are near normal. Which of the following tests would be most likely
to establish the probable diagnosis?
A. Endoscopic retrograde cholangiopancreatography
B. Hepatic venography
C. Serum alpha fetoprotein
D. Serum ceruloplasmin studies
E. Serum iron studies
Explanation:
The correct answer is B. The clinical presentation is most consistent with Budd-Chiari syndrome (hepatic vein
obstruction), which may occur as a complication of thrombogenic and myeloproliferative disorders including
polycythemia vera. The presentation illustrated is the most common; alternative presentations include fulminant
liver failure and cases in which intractable abdominal pain is the most prominent initial finding. Hepatic
venography is the best technique of those listed to demonstrate the occlusion of the hepatic venous system.
Endoscopic retrograde cholangiopancreatography (choice A) is most useful in demonstrating lesions of the
biliary tree.
Serum alpha fetoprotein (choice C) is a marker for hepatocellular carcinoma.
Ceruloplasmin (choice D) levels are decreased in Wilson's disease.
Serum iron studies (choice E) are useful when considering hemochromatosis as a cause of cirrhosis.

53

A 10-year-old boy develops an itchy, vesicular rash, which is maximal on his face and trunk. Physical examination
demonstrates a mixture of lesions, with macules, papules, vesicles, and crusted lesions. The mother reports that
the lesions seem to be occurring in crops. Which of the following is the most likely diagnosis?
A. Herpes simplex I
B. Herpes simplex II
C. Measles
D. Shingles
E. Varicella
Explanation:
The correct answer is E. This is varicella (chicken pox), which is the primary form of infection by the herpes
zoster (varicella-zoster) virus. Recurrence due to virus harbored in neurons tends to be dermatomal in
distribution and is called shingles. Fever, malaise, headache, and myalgia may also be present, particularly in
the prodromal phase. Tzanck smear of the base of a vesicle may demonstrate multinucleated giant cells.
Immunocompromised patients can be treated with acyclovir to prevent dissemination. Chicken pox may be
complicated by secondary bacterial infection, pneumonia, systemic spread (immunosuppressed patients),
neurologic involvement (rare), Reye's syndrome (rare), and hemolytic anemia (rare).
Herpes simplex I (choice A) causes oral vesicles and ulcers.
Herpes simplex II (choice B) causes genital vesicles and ulcers.
Measles (choice C) causes a blotchy, nonvesicular rash.
Shingles (choice D) is the recurrent form of herpes zoster infection and usually is localized to a single
dermatome.

Which of the following complications is currently the major limitation to the long-term success of cardiac
transplantation?
A. Allograft rejection
B. Graft arteriosclerosis
C. Graft atherosclerosis
D. Opportunistic infections
E. Lymphoma
Explanation:

54
The correct answer is B. Currently, graft arteriosclerosis (AKA graft vascular disease) is the most important limit
to the long-term success of heart transplantation. For unknown reasons, the coronary arteries of transplanted
hearts undergo intimal thickening associated with hyperplasia of myocytes and fibroblasts and deposition of
matrix. This results in luminal stenosis and myocardial ischemia. Patients may develop myocardial infarction,
which is clinically silent because the heart is denervated. The overall survival after heart transplantation is 80%
at 1 year and 60% at 5 years. Do not confuse graft arteriosclerosis with graft atherosclerosis (choice C).
Atherosclerosis is caused by accumulation of cholesterol esters and development of atheromas.
Atherosclerosis may recur in the coronary arteries of transplanted hearts, but is not a limiting factor in long-term
success of heart transplantation.
Allograft rejection (choice A) is certainly a major postoperative problem. However, thanks to early diagnosis
based on periodic endomyocardial biopsy and the availability of immunosuppressant therapy, this complication
can be prevented or successfully treated.
Although opportunistic infections (choice D) and development of Epstein-Barr related lymphomas (choice E) are
undesired effects of profound immunosuppression, these complications do not constitute a significant limitation
to the overall outcome of cardiac transplantation.

A mother takes her 4-year-old to a pediatrician because the child is having chronic, severe headaches. Physical
examination demonstrates poor visual tracking with one eye, which had not been present 1 year previously. The
pediatrician orders a CT scan of the head, which demonstrates a cystic 4-cm mass above the pituitary gland.
Resection of the tumor reveals a cystic lesion filled with dark, oily fluid containing granular debris. Histologic
examination of the tumor would most likely demonstrate a tumor with areas resembling which of the following?
A. Autonomic ganglion
B. Brain
C. Skin
D. Thyroid
E. Tooth enamel organ
Explanation:
The correct answer is E. The tumor is a craniopharyngioma, which is also called an ameloblastoma because of
its histologic resemblance to tooth enamel organ, which contains ameloblasts. The resemblance is not merely
coincidental, because the embryologic development of the pituitary involves both downward growth from the
brain, forming the posterior lobe of the pituitary, and upward growth from the mouth (from remnants of Rathke's
pouch), forming the anterior lobe of the pituitary. Craniopharyngiomas may occupy the sella turcica or may be
found in a suprasellar location, often in the hypothalamus. The tumor may present with mass effects (as in this
child) or pituitary insufficiency.
Ganglioneuromas contain tissue resembling autonomic ganglia (choice A).
Dermoid cysts (mature teratomas) of the ovary can contain tissue resembling brain (choice B), skin (choice C),
or thyroid (choice D).
A 67-year-old male develops severe chest pain. He is admitted to the hospital, and diagnosed with a myocardial
infarction based on his electrocardiogram and serial CK-MB levels. One week later, he again complains of

55
precordial pain and develops a fever of 102F (38.9C). Physical examination is remarkable for a loud friction
rub. Which of the following is the most likely diagnosis?
A. Caseous pericarditis
B. Fibrinous pericarditis
C. Hemorrhagic pericarditis
D. Purulent pericarditis
E. Serous pericarditis
Explanation:
The correct answer is B. Different types of pericarditis can be seen in different settings. Fibrinous and
serofibrinous pericarditis may follow acute myocardial infarction (Dressler's syndrome) and can be seen in
uremia, chest radiation, rheumatic fever, systemic lupus erythematosus, and following chest trauma (including
chest surgery).
Caseous pericarditis (choice A) is generally due to tuberculosis.
Hemorrhagic pericarditis (choice C) can be seen with tuberculosis, malignant tumors, patients with bleeding
diatheses, and following chest surgery.
Purulent pericarditis (choice D) is seen when pyogenic infections involve the pericardium, e.g., after
cardiothoracic surgery.
Serous pericarditis (choice E) is seen in non-infectious inflammations (rheumatic fever, lupus, scleroderma,
tumors, uremia).

A 25-year-old man presents with bilateral hearing loss. MRI reveals bilateral tumors within the cerebellopontine
angles. Surgery is performed, and the tumors are removed. Both are found to be neurilemomas ("schwannoma").
Which of the following is the most likely diagnosis?
A. Metastatic disease
B. Multiple sclerosis
C. Neurofibromatosis type 1
D. Neurofibromatosis type 2
E. Tuberous sclerosis
Explanation:
The correct answer is D. Neurofibromatosis type 2 is an autosomal dominant condition caused by mutations of a
gene on chromosome 22 coding for a cytoskeleton-related protein called merlin. Much less common than

56
neurofibromatosis type 1, it manifests with multiple CNS tumors, the most frequent of which are schwannomas of
the 8th cranial nerve and meningiomas. Bilateral schwannomas are virtually pathognomonic (ie, diagnostic) of
neurofibromatosis type 2.
Metastases to the CNS (choice A) are often multiple and usually involve the gray-white matter junction. Besides
the unusual location, the young age would make this diagnosis highly improbable.
Multiple sclerosis (choice B) is a chronic remitting/relapsing demyelinating disease. It manifests with focal
neurologic deficits caused by well-circumscribed areas of myelin loss in the white matter of the brain (usually
periventricular), brainstem, spinal cord, or optic nerves. It is not associated with an increased incidence of any
type of brain tumor.
Neurofibromatosis type 1 (choice C) is also autosomal dominant and is caused by mutations of a gene on
chromosome 17 coding for neurofibromin, a protein involved in signal transduction. The most characteristic
clinical features include caf-au-lait spots, neurofibromas (tumors of peripheral nerves different from
schwannomas), Lisch nodules (pigmented nodules of the iris), and CNS tumors, but not schwannomas.
Tuberous sclerosis (choice E), like neurofibromatosis, is a "neurocutaneous syndrome," ie, a condition
characterized by concomitant neurologic and skin lesions. Tuberous sclerosis is caused by mutations in two loci,
either TS1 or TS2. Multiple hamartomas of the brain (cortical "tubers") and other organs, shagreen patches,
ash-leaf patches, and other skin lesions constitute the clinical findings in this disorder.

A 35-year-old man is referred to a psychiatrist because of erratic behavior. The man had been adopted in
infancy, so his family history is not known. Over the next year, he develops uncontrollable erratic movements,
such that attempts to pick up a cup or use a pencil produce sudden uncontrolled lurches. When he tries to walk,
he staggers, thrusts, and abruptly changes direction. Eventually, with disease progression, he develops
increasing rigidity and is unable to move, and finally dies ten years after the onset of symptoms. Which of the
following changes would most likely be seen on examination of his brain at autopsy?
A. Depigmentation of the substantia nigra and locus ceruleus
B. Diffuse cortical atrophy with relative sparing of primary motor and sensory areas
C. Selective frontal and temporal lobe atrophy
D. Striking degeneration of the caudate nucleus
E. Widespread neuronal loss and gliosis in subcortical sites
Explanation:
The correct answer is D. The disease is autosomal dominant Huntington's chorea. The question stem describes
a typical clinical progression (the family history is usually strikingly positive). Pathological findings include
severe atrophy of the caudate nucleus (with loss of medium-sized spiny neurons), less severe involvement of
the putamen and cerebral cortex, and dilation of the lateral ventricles apparent on CT and MRI studies. The
disorder is known to be caused by expansion of a CAG trinucleotide repeat in a gene on the short arm of
chromosome 4 coding for a protein called huntingtin. There is no effective therapy.
Choice A is characteristic of Parkinson's disease, characterized by bradykinesia, pill-rolling tremor, and
cogwheel rigidity.
Choice B is characteristic of Alzheimer's disease, a degenerative dementing disorder.

57

Choice C is characteristic of Pick's disease, a dementing disorder that may be confused with Alzheimer's
disease. Microscopically, there is gliosis, neuronal loss, and swollen neurons, which may contain characteristic
Pick bodies (silver-staining cytoplasmic inclusions).
Choice E is characteristic of progressive supranuclear palsy. Patients exhibit an extrapyramidal syndrome
accompanied by dystonias of the neck and paralysis of downward gaze.

The presence of which of the following features in an atherosclerotic plaque indicates that it has become a
complicated lesion?
A. Cholesterol crystals
B. Chronic inflammatory cells
C. Intimal smooth muscle
D. Lines of Zahn
E. Necrotic cell debris
Explanation:
The correct answer is D. Complicated lesions indicate advanced atherosclerotic disease. They arise in
atherosclerotic plaques, and render them more susceptible to sudden occlusion and acute infarction of the
supplied tissues. Commonly, the plaque ulcerates or ruptures, and the exposed surfaces, being highly
thrombogenic, precipitate thrombus formation. Thrombi are typified by the lines of Zahn, alternating layers of
platelets and fibrin (the pale lines) and layers of blood (the dark lines). Beyond thrombus formation, other
features of a complicated plaque include hemorrhage into the lesion itself, and microembolism by cholesterol
crystals or calcified debris. Furthermore, the weakened media underlying the plaque may develop an
aneurysmal dilatation. In general, the clinical significance of atherosclerosis is related to the consequences of
complicated lesions.
The incorrect options all include features of atheromatous plaques, but do not indicate complicated lesions:
Beneath the endothelium of a plaque there is a fibrous cap composed of smooth muscle (choice C), chronic
inflammatory cells (choice B) and lipid laden macrophages (foam cells), as well as extracellular material.
The core of the lesion, which lies between the intima and the media, is composed of necrotic cellular debris
(choice E), with cholesterol crystals (choice A), calcium, and more foam cells.

A 72-year-old female patient with Alzheimer's disease, but no other medical problems, suddenly becomes
comatose and dies due to an intracranial hemorrhage that caused severe damage to her entire left cerebral
hemisphere. There was no evidence or history of trauma. What is the most likely cause of this hemorrhage?
A. Epidural hematoma
B. Subdural hematoma

58
C. Amyloid angiopathy
D. Rupture of berry aneurysm
E. Rupture of Charcot-Bouchard aneurysm
Explanation:
The correct answer is C. Alzheimer's disease patients are prone to large "lobar" hemorrhages that are usually
centered in the parietal lobe (thus the name "lobar") and may spread to totally destroy an entire cerebral
hemisphere, resulting in death. This is due to amyloid deposition into the walls of cerebral blood vessels
(amyloid angiopathy), similar to the amyloid plaques seen in the parenchyma of the brain with this disease.
Amyloid makes these vessels weak and prone to rupture.
An epidural hematoma (choice A) is a collection of blood above the dura mater, usually due to a blow to the side
of the head that fractures the temporal bone of the skull and shears the middle meningeal artery. Even though
this fast-flowing arterial blood usually causes symptoms within the first 24 hours and can cause life-threatening
mass effects, no traumatic event was involved in this case.
A subdural hematoma (choice B) forms when slow-flowing venous blood collects below the dura mater due to
leakage from stretched cortical veins as they drain into the superior sagittal sinus. Even though it is true that
Alzheimer's patients are more susceptible to these bleeds due to brain atrophy, causing cortical veins to be
maximally stretched and prone to tearing, subdural hematomas are usually associated with some sort of trauma.
Furthermore, a subdural hematoma causes gradual symptoms over time, rather than presenting as a sudden
devastating event as in this case.
Rupture of a berry aneurysm (choice D) usually causes a subarachnoid hemorrhage, in which blood leaks into
the space between the arachnoid membrane and the brain. The blood may also gain access to the ventricles,
but does not generally damage the cerebral hemispheres. Subarachnoid hemorrhages can be sudden and
deadly, but usually present with a severe headache ("the worst headache of my life"). These aneurysms are not
especially associated with Alzheimer's disease.
Rupture of a Charcot-Bouchard aneurysm (choice E) might cause a sudden hemorrhage within the brain
causing coma and death, but it is not especially associated with Alzheimer's disease. Charcot-Bouchard
aneurysms are primarily seen in patients with severe hypertension.

A 37-year-old woman presents to her physician complaining of difficulty reading and fatigue. She reports having a
"pins and needles" feeling in her left arm several months ago that resolved without treatment. On examination,
visual field deficits and mild hyperreflexia are noted. MRI confirms the suspected diagnosis. Which of the following
is the underlying mechanism of this patient's disease?
A. Antibodies to acetylcholine receptors
B. Axonal degeneration
C. Demyelination of the peripheral nerves
D. Loss of oligodendrocytes
E. Loss of Schwann cells

59
Explanation:
The correct answer is D. This woman presents with the classic signs and symptoms of multiple sclerosis (MS). A
key to this disease is different neurological signs that are separated by space and time. (Another classic clue
might have been oligoclonal bands on electrophoresis of the CSF.) MS is a demyelinating disease of the central
nervous system, characterized by loss of oligodendroglial cells, which are the cells that are responsible for
producing myelin in the central nervous system. Diagnosis can be confirmed by an MRI revealing sharply
delineated regions of demyelination (plaques) throughout the central nervous system white matter (especially in
periventricular areas).
Antibodies to acetylcholine receptors (choice A) have been implicated in the etiology of myasthenia gravis, not
multiple sclerosis.
MS is generally characterized by axonal preservation, rather than degeneration (choice B).
Demyelination of peripheral nerves (choice C) occurs in a number of diseases (e.g., Guillain-Barr), but not in
MS. Guillain-Barr is characterized by ascending muscle weakness, areflexia, and paralysis.
Oligodendrocytes are responsible for producing myelin in the central nervous system; Schwann cells (choice E)
are responsible for myelination in the peripheral nervous system, and are not affected in multiple sclerosis.

Which of the following endometrial lesions is associated with the highest risk of developing endometrial
adenocarcinoma?
A. Chronic endometritis
B. Complex hyperplasia with atypia
C. Complex hyperplasia without atypia
D. Simple hyperplasia
E. Squamous metaplasia
Explanation:
The correct answer is B. In general, any condition characterized by excessive estrogenic stimulation is
associated with some degree of endometrial hyperplasia and increased risk of endometrial cancer. Endometrial
hyperplasia is a histologic precursor of endometrial adenocarcinoma. Hyperplasia may progress to invasive
adenocarcinoma through progressive degrees of cellular and architectural atypia. The grade of hyperplasia,
therefore, is related to the severity of alterations of gland architecture, growth pattern, and cytologic features.
The most severe changes are present in complex hyperplasia with atypia. Disorganization and crowding of
glands, high mitotic activity, and nuclear atypia characterize this change. Longitudinal studies show that 25% of
women with this form of hyperplasia develop adenocarcinoma. Complex hyperplasia without atypia (choice C) is
characterized by similar crowding of glands and epithelial cells in the absence of cellular atypia: progression to
cancer occurs in 5% of cases. In simple hyperplasia (choice D), the glands are dilated and irregular, but
cytologic atypia and mitoses are absent and transformation is rare.
Chronic endometritis (choice A) has no relationship with endometrial adenocarcinoma and refers to a condition
of chronic inflammatory infiltration of the endometrium. A diagnosis of chronic endometritis is made only when
plasma cells are found on biopsy. (Lymphocytes are normally present in the endometrial mucosa.) This
condition is associated with pelvic inflammatory disease (PID), intrauterine contraceptive devices (IUD), and

60
retention of gestational tissue within the uterine cavity.
Squamous metaplasia (choice E) is occasionally found in endometrial biopsy and results from transformation of
the normal columnar epithelium into squamous epithelium. It is not associated with endometrial
adenocarcinoma.
A patient comes to medical attention after he sets off the metal detector at the airport, despite removing his
watch, belt buckle, and every other obvious source of metal. Which of the following diseases might be responsible
for this phenomenon?
A. Argyria
B. Gall stones
C. Hemochromatosis
D. Kidney stones
E. Wilson's disease
Explanation:
The correct answer is C. Hemochromatosis is an iron storage disorder that can cause cirrhosis (with increased
risk of hepatocellular carcinoma), skin pigmentation, pancreatic damage leading to diabetes mellitus, and
congestive heart failure. These complications are due to damage caused by deposition of iron in tissues; the
total body iron in some of these individuals may reach 50 g, large enough to set off some airport metal
detectors.
Argyria (choice A) is a blue-gray skin discoloration related to silver poisoning.
Neither gallstones (choice B) nor kidney stones (choice D) contain metal.
In Wilson's disease (choice E), copper is deposited in liver and brain, but this would not be detected by metal
detectors.

A Pap smear of a 23-year-old woman demonstrates squamous cells with enlarged, hyperchromatic nuclei and
prominent perinuclear halos. The Pap smear is graded as cervical intraepithelial neoplasia, grade II (CIN II).
Which of the following viruses is most likely to be etiologically related to this neoplastic growth?
A. Epstein-Barr virus (EBV)
B. Hepatitis B virus (HBV)
C. Human herpesvirus 8 (HHV8)
D. Human papillomavirus (HPV)
E. Human T-cell lymphotropic virus -1 (HTLV-1)

61
Explanation:
The correct answer is D. CIN II corresponds to moderate dysplasia of the cervix. The CIN lesions of all grades
(including condyloma) and the cervical cancers that can arise from them appear to be associated with infection
with certain subtypes of human papilloma virus (HPV). Koilocytotic atypia (enlarged, hyperchromatic nuclei and
prominent perinuclear halos) is commonly observed with HPV infection. HPV is usually spread through sexual
contact and also causes penile and anal condyloma and carcinomas.
EBV (choice A) is associated with nasopharyngeal carcinoma and Burkitt's lymphoma.
HBV (choice B) is associated with hepatocellular carcinoma.
HHV8 (choice C) is associated with Kaposi's sarcoma.
HTLV-1 (choice E) is associated with adult T-cell leukemia.

A young man presents to his physician's office for a physical exam. He is concerned because his father died of a
heart attack in his late 40's. The physician finds that he has elevated serum cholesterol and LDL levels, but his
VLDL and triglycerides are normal. Further investigation reveals an LDL receptor deficiency. This patient has
which of the following types of hyperlipidemia?
A. Type I
B. Type IIa
C. Type IIb
D. Type III
E. Type IV
F. Type V
Explanation:
The correct answer is B. There are many clues in the question that should have guided you to this choice. The
laboratory findings are classic for Type IIa hyperlipidemia. These patients have LDL receptor deficiencies and
are at a great risk of advanced coronary atherosclerosis. Since it is autosomal dominant, the patient's father
could have been affected as well.
Type I hyperlipidemia (choice A), or familial hyperchylomicronemia, is caused by a lipoprotein lipase deficiency.
These patients have high serum triglycerides and normal cholesterol. They do not have a substantially higher
risk of atherosclerosis.
Type IIb hyperlipidemia (choice C), or familial combined hyperlipidemia, presents as elevated serum LDL, VLDL,
cholesterol, and triglycerides. These patients do have an increased incidence of atherosclerosis.
Type III hyperlipidemia (choice D), or familial dysbetalipoproteinemia, presents as increased serum cholesterol
and triglycerides. The mode of inheritance is not understood, but apoprotein E is affected and the risk of
atherosclerosis is great.

62
Type IV hyperlipidemia (choice E), or familial hypertriglyceridemia, presents as increased triglycerides with
normal cholesterol and LDL. The disease may be sporadic and is possibly associated with an increased risk for
atherosclerosis.
Type V hyperlipidemia (choice F), or mixed hypertriglyceridemia, is not common. Cholesterol is slightly
increased and triglycerides are greatly increased. There is deficient apoprotein CII. The risk of atherosclerosis
is not clear.

A pregnant woman develops deep, boring pain of her left thigh muscles associated with swelling and enhanced
warmth of the same leg. The pain is worsened by extending the foot. The superficial veins of the leg are
engorged. Her condition puts her at risk for which of the following?
A. Acute renal failure
B. Cerebral hemorrhage
C. Hepatic infarction
D. Myocardial infarction
E. Pulmonary embolus
Explanation:
The correct answer is E. The patient has clinical findings strongly suggestive of deep venous thrombosis (DVT).
Pain that increases upon extension of the foot is referred to as Homans' sign. Pregnancy, particularly during the
third trimester, induces a hypercoagulable blood state (possibly to limit the chance of fatal hemorrhage during
delivery), which can manifest as DVT. The major complication of DVT is pulmonary embolism, which may be
massive and can cause sudden death.
Renal failure (choice A) can be a serious problem in pregnancy, but is not related to deep vein thrombosis.
Cerebral hemorrhage (choice B) and hepatic infarction (choice C) are complications of preeclampsia.
The hypercoagulability that predisposes for deep vein thrombosis also predisposes for myocardial infarction
(choice D), but MI is not considered a complication of DVT.

A 31-year-old African-American female dancer injures herself during a dance routine and is seen in the
emergency room for tender ribs. A chest radiograph is performed which shows no rib fractures, but does reveal
significant bilateral hilar adenopathy and middle and upper zone linear streaks and nodules. Upon further
questioning she recalls an episode approximately one year ago, in which she experienced fever, ankle swelling,
and tender red bumps on her lower extremities. She currently has no medical complaints and is able to rehearse
without any difficulty breathing. Which of the following is the most likely diagnosis?
A. No disease, the patient is healthy
B. Pneumocystis carinii pneumonia
C. Sarcoidosis

63

D. Systemic sclerosis
E. Wegener's granulomatosis
Explanation:
The correct answer is C. This patient has sarcoidosis, a multi-system disease of unknown etiology
characterized by noncaseating granulomas on histological examination of various organs. Sarcoidosis occurs
mostly in blacks in the third to fifth decades. Approximately 90% of cases involve the lungs, with findings such
as hilar adenopathy and pulmonary infiltration. Other findings include uveitis, erythema nodosum (the lesions
on this patient's lower extremities), arthritis, central and peripheral neuropathies, cardiomyopathy, and
hypercalcemia. Symptoms include fatigue, exertional dyspnea, and non-productive cough. Diagnosis is made
after excluding other causes by examination of a tissue biopsy. Radiographic staging is performed by serial
chest x-rays.
Pneumocystis carinii pneumonia (PCP) (choice B) is a disease almost exclusively of HIV-infected patients,
usually with a characteristic chest radiograph appearance of bilateral fluffy infiltrates.
Systemic sclerosis (choice D), or scleroderma, is a systemic collagen-vascular disease that involves the skin
nearly 100% of the time, and the esophagus in approximately 75% of patients.
Wegener's granulomatosis (choice E) is a disorder characterized by focal necrotizing vasculitis in the lung and
upper airways associated with granuloma formation, and necrotizing glomerulitis.
Following a respiratory infection, a 20-year-old man goes to his physician for a follow-up visit. Physical
examination is unremarkable, but dipstick analysis of his urine reveals marked proteinuria and microscopic
hematuria. The young man is referred to a specialist, who performs a renal biopsy. Immunofluorescence
microscopy of the biopsy tissue demonstrates IgA deposition in the glomerular mesangium. These results are
most consistent with which of the following disorders?
A. Berger's disease
B. Goodpasture's syndrome
C. Minimal change disease
D. Poststreptococcal glomerulonephritis
E. Systemic lupus erythematosus
Explanation:
The correct answer is A. Berger's disease, or Ig A nephropathy, may develop after a respiratory infection. It is a
major cause of recurrent microscopic hematuria, and may progress to renal failure in a number of cases. (Note:
don't confuse Berger's disease with Buerger's disease-a vasculitis that occurs in smokers.)
Questions about Goodpasture's syndrome (choice B) often contain a clue about linear deposition of IgG
anti-basement membrane antibodies.
You should associate fusion of podocyte foot processes with minimal change disease (choice C).

64
Classic poststreptococcal glomerulonephritis (choice D) may follow pharyngitis caused by group A
streptococcus. This type of glomerulonephritis is associated with granular immunofluorescence and
subepithelial humps by electron microscopy.
Renal involvement in lupus (choice E) can have many manifestations, but there will usually be evidence of a
systemic inflammatory disease.

A 63-year-old African-American male presents with vague complaints of abdominal and back pain, malaise,
nausea, and weakness, which have been present for 3 or 4 months. Review of systems reveals a 15 pound
weight loss, occasional vomiting, and several episodes of unilateral leg swelling, which have involved both legs at
different times. These findings are most consistent with which of the following diagnoses?
A. Pancreatic cancer
B. Primary sclerosing cholangitis
C. Pyelonephritis
D. Reflux esophagitis
E. Splenic infarction
Explanation:
The correct answer is A. Pancreatic carcinoma often presents with vague abdominal, back, and gastrointestinal
complaints; and physical examination is generally unrevealing. The tremendous weight loss is very suspicious
for carcinoma, and the migrating thrombophlebitis (Trousseau's sign) is extremely helpful in making the
diagnosis, which should be confirmed with ultrasonography or CT. Although any carcinoma can elicit migratory
thrombophlebitis, it is mostly associated with tumors of the pancreas, lung, and colon.
Primary sclerosing cholangitis (choice B) is an inflammatory fibrosing disease of the biliary tree of unknown
etiology, although highly associated with inflammatory bowel disease. It presents with symptoms of liver failure
(jaundice, pruritus) and progresses to biliary cirrhosis.
Pyelonephritis (choice C) may present with back pain that usually localizes to the costovertebral angle, and is
generally associated with fevers and dysuria. Chronic pyelonephritis proceeds to hypertension and renal
failure.
Reflux esophagitis (choice D) presents with heartburn, regurgitation, and dysphagia and may occasionally be
confused with a heart attack by the patient. Long term consequences include bleeding, strictures, and Barrett's
esophagus.
Splenic infarction (choice E), typically associated with arterial thromboembolic events, generally is an
asymptomatic occurrence that does not produce clinical symptoms unless the entire spleen is lost.
A 37-year-old man presents to a physician because of a lesion on the shaft of his penis. On physical
examination, a solitary, thickened, whitish plaque with a slightly ulcerated, crusted surface is observed. Biopsy
reveals markedly dysplastic epithelial cells, many mitoses (some of which are abnormal), disordered epithelial
maturation, and an intact basement membrane with no evidence of stromal invasion. Which of the following is the
most likely diagnosis?

65

A. Bowenoid papulosis
B. Bowen's disease
C. Condyloma acuminatum
D. Erythroplasia of Queyrat
E. Squamous cell carcinoma
Explanation:
The correct answer is B. This is Bowen's disease, which is one clinical form of penile carcinoma in situ (the
other forms are Bowenoid papulosis and erythroplasia of Queyrat). Untreated Bowen's disease will, over a
period of years, progress to frank invasive carcinoma in 10% to 20% of patients. Bowen's disease can affect
the genital region of both men and women.
Bowenoid papulosis (choice A) is another form of penile carcinoma in situ, characterized clinically by multiple
reddish-brown papular lesions.
Condyloma acuminatum (choice C) usually produces a papillary lesion.
Erythroplasia of Queyrat (choice D) is another form of penile carcinoma in situ, characterized by single or
multiple shiny red plaques.
Squamous cell carcinoma (choice E) will be frankly invasive on biopsy.
A physician has been treating a 60-year-old patient with renal failure due to polycystic kidney disease. The
physician should be specifically concerned about the possible coexistence of which of the following conditions?
A. Aneurysm of aortic root
B. Atherosclerotic aneurysm
C. Berry aneurysm
D. Cystic medial necrosis
E. Mycotic aneurysm
Explanation:
The correct answer is C. There is a specific association between the adult form of polycystic kidney disease
and congenital berry aneurysms of the circle of Willis. These aneurysms can rupture, producing a
subarachnoid hemorrhage, and possibly causing death.
Aneurysms of the aortic root (choice A) are associated with syphilis.
Atherosclerosis (choice B) causes abdominal aortic aneurysms, many of which are seen in diabetics.
Cystic medial necrosis (choice D) can cause dissecting aneurysms (e.g., in Marfan's syndrome).

66
"Mycotic" aneurysms (choice E) are due to bacterial, not fungal, infection.

A 50-year-old woman with a swan-neck deformity of the hands and enlarged knuckles develops large
subcutaneous nodules near her elbows. If the nodules were biopsied, which of the following best describes their
likely histological appearance?
A. Amorphous crystalline mass surrounded by macrophages
B. Cystic space caused by myxoid degeneration of connective tissue
C. Darkly pigmented synovium with an exuberant, villous growth
D. Fibrinoid necrosis surrounded by palisading epithelioid cells
E. Well-encapsulated nodule of polygonal cells within a tendon sheath
Explanation:
The correct answer is D. Swan-neck deformity, enlarged knuckles, and subcutaneous nodules are classic clues
for rheumatoid arthritis. The subcutaneous rheumatoid nodules are composed histologically of areas of fibrinoid
necrosis surrounded by palisading epithelioid cells.
Gout tophi are amorphous crystalline masses surrounded by macrophages (choice A).
Ganglion cysts are small cystic spaces caused by myxoid degeneration of connective tissue (choice B).
Pigmented villonodular synovitis causes darkly pigmented synovium with exuberant villous growth (choice C).
Nodular tenosynovitis causes a well-encapsulated nodule of polygonal cells within a tendon sheath (choice E).

A 21-year-old female with a several month history of easy bruising and increased menstrual flow is evaluated for
a bleeding disorder. Her platelet count is 31,000/mm3. Subsequent investigations determine that she has
idiopathic thrombocytopenic purpura (ITP). In this disorder, the low platelet count is due to which of the following?
A. Antiplatelet antibodies
B. Defective platelet aggregation
C. Hypersplenism
D. Ineffective megakaryopoiesis
E. Mechanical trauma
Explanation:

67
The correct answer is A. ITP is a chronic autoimmune disorder in which antibodies against platelet glycoproteins
cause platelet destruction and removal by the reticuloendothelial system. Secondary thrombocytopenia can also
be produced by lupus, viral infections, and drugs. Only when secondary thrombocytopenia has been ruled out,
can the diagnosis of ITP be made.
Defective platelet aggregation (choice B) is responsible for thrombasthenia, an autosomal dominant disease
that causes prolonged bleeding time but normal numbers of platelets.
Hypersplenism (choice C) causes thrombocytopenia when an enlarged spleen traps normal platelets in the
absence of other specific platelet disorders. This type of thrombocytopenia can be cured with splenectomy.
Although the thrombocytopenia in ITP often improves with splenectomy, ITP does not cause splenomegaly.
Megakaryopoiesis (choice D) is disturbed in any disorder that causes bone marrow failure, including drug
toxicity, leukemia, and infections. In this setting, thrombocytopenia is often part of a pancytopenia.
Mechanical trauma (choice E) causes thrombocytopenia in disseminated intravascular coagulation (DIC),
thrombotic microangiopathies (thrombotic thrombocytopenic purpura and hemolytic-uremic syndrome), and with
giant hemangiomas.

A patient with prolonged diarrhea undergoes esophagogastroduodenoscopy. Biopsy of the small intestine
demonstrates numerous crescent-shaped protozoa adjacent to the epithelial brush border. Which of the following
organisms is the most likely pathogen?
A. Entamoeba histolytica
B. Escherichia coli
C. Giardia lamblia
D. Naegleria fowleri
E. Trichomonas vaginalis
Explanation:
The correct answer is C. The probable organism is Giardia lamblia, which characteristically infects the small
intestine. When seen in smears from duodenal aspirates, this flagellated organism has a characteristic
"face-like" appearance. However, in biopsy specimens, the organism is often caught at an angle, and the
characteristic appearance and location is as described in the question stem. Giardia is a common contaminant
of water supplies (even in the United States), and patients who ingest the cysts may be asymptomatic or may
occasionally develop prolonged diarrhea or intestinal malabsorption. Some patients with severe disease have
low serum IgA or low overall immunoglobulin levels.
Entamoeba histolytica (choice A) usually infects the large intestine and/or liver.
Escherichia coli (choice B) is a bacterial cause of diarrhea.
Naegleria fowleri (choice D) causes meningoencephalitis.
Trichomonas vaginalis (choice E) causes vaginitis.

68

A 60-year-old man presents to the emergency room with severe, tearing pain radiating to the upper back. Over a
period of hours the pain moves to the mid back and then involves both flanks. Hematuria develops shortly
thereafter. Which of the following renal complications has most likely occurred?
A. Acute glomerulonephritis
B. Bilateral renal infarction
C. Polycystic kidney disease
D. Pyelonephritis
E. Sickle cell crisis
Explanation:
The correct answer is B. The patient has a dissecting aortic aneurysm that occluded the renal arteries, causing
bilateral renal infarction with flank pain and hematuria. This is a very dangerous complication of dissecting aortic
aneurysm, which classically presents as described in the question stem.
Acute glomerulonephritis (choice A) is characterized by hematuria, red cell casts, and often, proteinuria and
edema. This syndrome typically develops over days, not hours, and would not be expected to result from
dissecting aortic aneurysm.
Polycystic kidney disease (choice C) is a lifelong condition characterized by bilateral polycystic changes in the
kidneys. It is associated with hypertension and eventual renal failure.
Pyelonephritis (choice D) is an infection of the kidney that can develop as a complication of bacterial
endocarditis, or as a consequence of vesicoureteral reflux, but would not be expected to result from aortic
dissection.
Sickle cell crisis (choice E) can cause papillary necrosis of the kidneys with hematuria, but there is no indication
that the patient has sickle cell anemia.

During a routine preemployment physical examination, an apparently healthy 24-year-old man is found to have
increased serum levels of unconjugated bilirubin. Conjugated bilirubin and transaminases are normal. Careful
questioning reveals no evidence of recent illnesses. Which of the following is the most likely diagnosis?
A. Crigler-Najjar syndrome
B. Dubin-Johnson syndrome
C. Gilbert's syndrome
D. Rotor syndrome
E. Wilson's disease

69

Explanation:
The correct answer is C. Gilbert's syndrome is a relatively common hereditary condition that is so mild, a case
can be made for classifying it as a normal variant rather than a true disease. It produces asymptomatic
unconjugated hyperbilirubinemia, and is important to know about to avoid unnecessary diagnostic procedures.
Crigler-Najjar syndrome (choice A) represents several hereditary diseases characterized by severe
unconjugated hyperbilirubinemia. The type I form, in particular, can be fatal.
Dubin-Johnson (choice B) and Rotor (choice D) syndromes are relatively mild hereditary conjugated
hyperbilirubinemias.
Wilson's disease (choice E) is a serious disorder of copper metabolism associated with hepatic cirrhosis,
movement disorder, and pathognomonic gold-colored rings (Kayser-Fleischer rings) in the iris. Diagnosis is
based on reduced serum ceruloplasmin (a copper-binding protein), increased hepatic copper content, and
increased urinary copper excretion.
Which of the following CNS tumors has the worst prognosis and is associated with the shortest survival?
A. Anaplastic astrocytoma (WHO grade III)
B. Glioblastoma multiforme
C. Meningioma
D. Oligodendroglioma
E. Well-differentiated astrocytoma (WHO grade II)
Explanation:
The correct answer is B. Glioblastoma multiforme (GBM) is the most common and most malignant of the
primary CNS neoplasms. GBM belongs to the category of gliomas called astrocytomas, which originate from
neoplastic transformation of astrocytes. There is a continuum in the anaplastic features of astrocytomas, from
well-differentiated astrocytoma, characterized by minimal atypia and mitotic activity, to GBM, characterized by a
brisk mitotic rate, multifocal necrosis, and extreme nuclear pleomorphism. Since histologic atypia has been
shown to correlate with biologic behavior, several grading systems have been developed to help predict the
prognosis of astrocytomas. The most common of such grading systems (WHO and Saint Anne-Mayo) divide
astrocytomas into four grades, from grade I (benign, well-differentiated) to grade IV (malignant, poorly
differentiated). GBM is grade IV astrocytoma. It is associated with a poor prognosis; most patients die within
approximately 12 months of diagnosis.
Well-differentiated astrocytoma (choice E) is basically a synonym for grade II astrocytoma, while anaplastic
astrocytoma (choice A) refers to grade III astrocytoma. Survival is better than GBM, although definitive cure is
rare, even for well-differentiated astrocytomas.
Meningioma (choice C) is a benign tumor of meningothelial origin that grows from the dura mater, pushing,
rather than infiltrating, the underlying brain. Nevertheless, meningiomas may infrequently show histologic
features of malignancy and infiltrate the brain, in which case the prognosis is worse (though still better than the
dreadful GBM).
Oligodendroglioma (choice D) arises from oligodendroglial cells and usually lacks histologic markers of

70
malignancy (e.g., mitotic activity, nuclear atypia, and necrosis). Owing to its infiltrative pattern of growth,
however, complete excision is virtually impossible (remember, there is no basement membrane or other
anatomic boundary in the CNS to contain the spread of a glial neoplasm). Thus, oligodendrogliomas recur over
and over following each surgical resection. After each recurrence, these tumors acquire progressively
increasing degrees of anaplasia, ultimately becoming similar to GBM. Survival can be very long (7-10 years),
but definitive cure is exceptional.

Following penicillin treatment for pneumococcal pneumonia, a patient develops a palpable purpuric rash. Biopsy of
the rash demonstrates vasculitis with hemorrhage into the skin. The involved arterioles and venules show fibrinoid
necrosis and a neutrophilic infiltrate into the wall. Many of the neutrophils are fragmented. Which of the following is
the most likely diagnosis?
A. Allergic granulomatosis and angiitis
B. Giant cell arteritis
C. Leukocytoclastic angiitis
D. Polyarteritis nodosa
E. Wegener's granulomatosus
Explanation:
The correct answer is C. The correct diagnosis is leukocytoclastic angiitis, which is also known as
hypersensitivity angiitis or &ldquo;microscopic&rdquo; polyarteritis nodosa. Affected vessels are usually smaller
(arterioles, venules, and capillaries) than those of classic polyarteritis nodosa, and consequently, infarction is
less common. The microscopic appearance is that described in the question stem; the fragmentation of the
neutrophils is &ldquo;leukocytoclasis.&rdquo; Cases can be either limited to the skin or generalized. Specific
antigens (e.g., penicillin) may be suspected as possible triggers, leading to the alternative term
&ldquo;hypersensitivity angiitis.&rdquo; Removal of the offending antigen in these cases may induce clinical
improvement.
Allergic granulomatosis and angiitis (choice A) is an alternative name for Churg- Strauss disease, which affects
the respiratory and renal systems.
Giant cell arteritis (choice B) is an alternative name for temporal arteritis, which affects the temporal and
ophthalmic arteries.
Polyarteritis nodosa (choice D) primarily affects vessels larger than arterioles.
Wegener's granulomatosus (choice E) affects the respiratory and renal systems.

A 60-year-old man suddenly becomes completely blind in one eye, and angiography demonstrates occlusion of
the central retinal artery. Which of the following is the most likely cause of the occlusion?
A. Atheroma or embolism

71
B. Cranial (temporal) arteritis
C. Hypertension
D. Polycythemia vera
E. Tumor
Explanation:
The correct answer is A. The point of this question is that sometimes the obvious explanation is the correct one.
Occlusion of the central retinal artery rapidly causes irreversible blindness with loss of the inner retinal layers.
(The photoreceptor rod and cone cells are maintained by the pigment epithelium.) The site of occlusion is
typically just posterior to the cribriform plate. A garden-variety atheroma or embolism is overwhelmingly the
most common cause of central retinal artery occlusion.
Despite all of the teaching about the risk of blindness in temporal arteritis (choice B), this disorder causes only
10% of central retinal artery occlusions.
Hypertension (choice C) is more apt to cause bleeding than thrombosis.
Polycythemia vera (choice D) could (rarely) cause occlusion because of increased blood viscosity and a
tendency for thrombosis.
Tumor (choice E) might also cause retinal artery thrombosis, but this would be far rarer than atheroma.

During a pre-employment physical, a 45-year-old man is noted to have a 3 cm palpable nodule in one lobe of an
otherwise normal sized thyroid gland. Needle aspiration of the nodule demonstrates polygonal tumor cells and
amyloid, but only very scanty colloid and normal follicular cells. Which of the following is the most likely diagnosis?
A. Follicular thyroid carcinoma
B. Hashimoto's disease
C. Medullary thyroid carcinoma
D. Papillary thyroid carcinoma
E. Thyroid adenoma
Explanation:
The correct answer is C. The most distinctive clue is the presence of amyloid, which specifically suggests
medullary thyroid carcinoma. This carcinoma is a tumor of the neuroendocrine parafollicular cells of the thyroid,
rather than the follicular lining epithelium. These cells produce calcitonin, the precursor protein of which can
precipitate, forming bands and nodules of amyloid that appear histologically identical to other forms of amyloid.
The other thing to remember about medullary carcinoma of the thyroid is that it can be a component of multiple
endocrine neoplasia (MEN) syndromes type IIa (parathyroid disease, pheochromocytomas, medullary
carcinoma) and type IIb (parathyroid disease, pheochromocytomas, medullary carcinoma, mucocutaneous
ganglioneuromas, Marfanoid habitus).

72
Because follicular thyroid carcinoma (choice A) closely resembles normal thyroid tissue, it usually cannot be
reliably diagnosed based on fine needle aspiration alone.
Hashimoto's disease (choice B) would show lymphocytes, plasma cells, and macrophages on aspiration.
Papillary thyroid carcinoma (choice D) can be diagnosed by aspiration if papillary clusters are seen.
Thyroid adenoma (choice E) cannot be reliably distinguished from well-differentiated thyroid carcinoma on
aspiration.

A 56-year-old alcoholic man is brought in to the emergency room after being found unconscious by his daughter,
who called the paramedics. Paramedics report finding the man in a stuporous condition in the bathtub, covered
with vomit. On arrival to the emergency room, the man is clammy and his blood pressure is 85/50. Which of the
following conditions is the most likely cause of his hypotension?
A. Acute hemorrhagic pancreatitis
B. Chronic calcifying pancreatitis
C. Chronic obstructive pancreatitis
D. Cystic fibrosis
E. Pancreatic pseudocyst
Explanation:
The correct answer is A. Acute hemorrhagic pancreatitis is a life-threatening abdominal emergency that is most
often seen in the setting of excessive acute alcohol or food ingestion. In this condition, activated pancreatic
enzymes are released into the tissues, where they cause severe local damage to the pancreas, with pain
radiating to the back. The enzymes are also released into the blood stream. Shock may result from
hemorrhage, activation of bradykinin and related peptides, and/or release of proteolytic and lipolytic enzymes
into the circulation. Other systemic manifestations include hypocalcemia, glucose intolerance, and jaundice.
Chronic calcifying pancreatitis (choice B) is seen in chronic alcoholics, but does not cause the dramatic
presentation of acute hemorrhagic pancreatitis.
Chronic obstructive pancreatitis (choice C) is seen in gallstone disease.
Cystic fibrosis (choice D) is an inherited disease that usually causes death by age 30.
Pancreatic pseudocyst (choice E) is an acquired loculation of fluid that may be seen after pancreatitis or
trauma.

An abdominal x-ray performed on a 54-year-old man demonstrates a large, irregular, calcified mass with multiple
broad projections filling one renal pelvis. Which of the following laboratory findings might be expected in this
patient?
A. Decreased urine pH

73

B. Hypercalcemia
C. Hyperuricemia
D. Increased ammonia concentration in the urine
E. Increased cystine concentration in the urine
Explanation:
The correct answer is D. The patient has a stag-horn calculus. These very large calculi are almost always
composed principally of magnesium ammonium phosphate (often with enough calcium to be radio-opaque) and
form in the setting of infection by urea-splitting bacteria such as Proteus.
Increased urine ammonia concentrations are a byproduct of the bacterial metabolism of urea, and tend to
increase urine pH (compare with choice A).
Hypercalciuria, with or without hypercalcemia (choice B), is a cause of calcium oxalate stones.
Uric acid stones can be seen in patients with hyperuricemia (choice C) secondary to gout, or in conditions in
which a very rapid cell turnover occurs (e.g., leukemias).
Genetically determined defects in the renal transport of amino acids are associated with cystine stones (choice
E).

Oral examination of a 57-year-old female reveals a 1-cm, flat, white patch on the buccal mucosa. Which of the
following diagnoses indicates the greatest likelihood that this lesion will progress to an oral malignancy?
A. Hairy leukoplakia
B. Leukoplakia
C. Lichen planus
D. Oral thrush
E. Squamous papilloma
Explanation:
The correct answer is B. Leukoplakia is a white plaque on the oral mucosa for which a more specific diagnosis
cannot be rendered. Leukoplakia is often associated with hyperkeratosis and may or may not show dysplastic
squamous epithelium. On average, 5% of leukoplakias contain in situ or overt carcinoma.
Hairy leukoplakia (choice A) is an oral infective lesion seen almost exclusively in HIV infection. It is a fluffy, white
hyperkeratotic lesion in which a destructive piling up of keratotic squames is seen. Hairy leukoplakia is
associated with viral infection, mostly EBV, HPV, and/or HIV. It does not progress to cancer.
Lichen planus (choice C) is a dermatologic condition that may present with white oral plaques. Microscopically,
the lesion is characterized by intense lymphocytic infiltration of the dermoepidermal (or mucosal-submucosal)

74
junction with destruction of the basal layer of cells. This inflammatory condition has not been shown to be
premalignant.
Oral thrush (choice D) is a superficial candidal infection, typically occurring in the immunosuppressed, or very
young. Thrush is an infectious, non-neoplastic disease.
Squamous papilloma (choice E) is a benign human papillomavirus (HPV) infection of the oral mucosa. Typically
associated with HPV genotypes 6 and 11, squamous papilloma only rarely progresses to squamous carcinoma.

A 71-year-old white male has had polycythemia vera for 12 years. Throughout this period he was treated by
phlebotomy alone, and has remained stable. Recently he has noted a dragging sensation in his abdomen, and
physical examination reveals massive splenomegaly, palpable to 7 cm below the ribs. This finding may represent
the conversion of polycythemia vera to which of the following hematological disorders?
A. Chronic myeloid leukemia
B. Hairy cell leukemia
C. Myelodysplastic syndrome
D. Myeloid metaplasia with myelofibrosis
E. Waldenstrm's macroglobulinemia
Explanation:
The correct answer is D. There are four types of myeloproliferative disorders: chronic myeloid leukemia (CML),
polycythemia vera, myeloid metaplasia with myelofibrosis (MMM), and essential thrombocythemia (which is very
rare). Both CML and polycythemia vera are characterized by hypercellular bone marrow producing all three cell
lines. Over a period of years, the bone marrow may burn out, and the marrow space is replaced by fibrosis. In
this circumstance, hematopoiesis (including production of neoplastic cells) moves to extramedullary sites,
predominantly the spleen and liver. This process is called myeloid metaplasia with myelofibrosis.
Although both polycythemia vera and CML (choice A) can progress to MMM, polycythemia vera does not
progress to CML.
Hairy cell leukemia (choice B), a leukemia often associated with massive splenomegaly, is derived from a B
lymphocyte, not a myeloid stem cell. It would not progress to a myeloproliferative disorder.
Myelodysplastic syndrome (MDS; choice C) refers to myeloid disorders featuring maturation defects and
ineffective hematopoiesis. MDS produces cytopenia and acute leukemias, whereas myeloproliferative disorders
produce polycythemia and chronic leukemia.
Waldenstrm's macroglobulinemia (choice E) is a plasma cell dyscrasia that produces IgM in excess. It derives
from plasma cells and, as such, is not a myeloproliferative disease.

A 30-year-old male presents to his physician for a pre-athletic physical exam. He has no complaints or significant
past medical history. During auscultation of the heart, a faint murmur consistent with aortic regurgitation is heard.
Examination of the musculoskeletal system shows decreased range of motion of the hips bilaterally and of the
spine in rotation and forward bending. No scoliosis is seen. Examination of the eyes and mouth is unremarkable.

75
His skin is smooth and dry. It is thought that he has a spondyloarthropathy. Which of the following tests would
best differentiate this patient's disease from the other spondyloarthropathies?
A. HLA-B27
B. MRI of the femoral head
C. Rheumatoid factor
D. Spinal x-ray
E. Stool culture
Explanation:
The correct answer is D. This young man has an inflammatory arthropathy involving his hips and spine. The
disease is ankylosing spondylitis, which typically occurs in young men. Ankylosing spondylitis begins at the
sacroiliac crests, then moves upward in the spine, causing inflammation and destruction of the posterior
elements of the vertebral bodies. The posterior aspects of the vertebrae fuse, reducing the range of motion,
and partially taking the weight off the vertebrae, leading to an atrophy osteoporosis. The spinal x-ray will show
fusion of the disks and possible intervertebral disc ossification. Patients may also have associated inflammation
and fibrosis of the proximal aorta, leading to aortic regurgitation.
HLA-B27 (choice A) is a class I histocompatibility antigen that has a strong association with ankylosing
spondylitis, Reiter's syndrome, psoriatic arthritis, and certain enteropathic arthritides. It is found in all
spondyloarthropathies and therefore can not help to distinguish between them.
MRI of the femoral head (choice B) would reveal inflammatory arthritis in that location, but that would not be
enough to differentiate ankylosing spondylitis from the other entities. The spinal findings are more characteristic
of this disease.
Rheumatoid factor (choice C) is typically negative for all of the spondyloarthropathies, as are other serologic
markers of rheumatoid arthritis.
Stool culture (choice E) is not of diagnostic importance in distinguishing these entities. Ankylosing spondylitis is
not associated with diarrhea or stool pathogens.

A patient consults a physician because of a small lesion on the lips that, on biopsy, proves to be a mucosal
neuroma. The patient's mother had medullary carcinoma of the thyroid. In addition to medullary carcinoma of the
thyroid, to which of the following conditions would this patient be particularly vulnerable?
A. Gastrinoma
B. Insulinoma
C. Parathyroid adenoma
D. Pheochromocytoma
E. Pituitary adenoma

76

Explanation:
The correct answer is D. You should recognize this as a probable case of multiple endocrine neoplasia,
specifically, MEN III (formerly MEN II b). Features of this autosomal dominant condition include medullary
carcinoma of the thyroid, pheochromocytoma, and oral and intestinal ganglioneuromatosis (including mucosal
neuromas).
Gastrinomas (choice A) and insulinomas (choice B) are found in MEN I.
Parathyroid adenomas (choice C) are found in MEN I and II.
Pituitary adenomas (choice E) are found in MEN I.

A 35-year-old woman complains of severe lower abdominal pain, which is worst during menstruation.
Laparoscopic examination of the pelvis demonstrates multiple small brown spots on the surface of pelvic
structures. Most of these lesions are cauterized, but biopsy of one of the remaining lesions reveals glandular
tissue resembling normal endometrium. No cytologic atypia or abnormally shaped glands are seen. Which of the
following is the most likely diagnosis?
A. Acute endometritis
B. Adenomyosis
C. Chronic endometritis
D. Endometriosis
E. Metastatic endometrial cancer
Explanation:
The correct answer is D. This benign condition is endometriosis, rather than metastatic endometrial cancer.
Endometriosis is defined as a benign growth of endometrium at sites at which it does not normally occur
(excluding the myometrium, at which site it is called adenomyosis). Endometriosis is common and is a significant
cause of both pain and pelvic scarring. Most of the problems occur because the abnormally located
endometrium responds to hormonal control and may menstruate, producing a very irritating fluid. Endometriosis
may apparently be started either by seeding of menstrual cells in the pelvis (entering through the open end of
the fallopian tubes) or by metaplasia of mesothelial or other cells (accounting for rare, well-documented cases
of endometriosis of bizarre sites such as nasal mucosa or lungs).
Acute endometritis (choice A) is acute inflammation of the endometrium.
Adenomyosis (choice B) is characterized by foci of endometrium deep in the myometrium.
Chronic endometritis (choice C) is chronic inflammation of the endometrium.
Metastatic endometrial cancer (choice E) would be characterized by abnormal glands and cytologic atypia.
A 32-year-old man presents to his physician for a routine physical examination. The man admits to recent loss of
10 pounds and occasional fatigue, but ascribes these to increases in his workload. On physical examination

77
there is a 2-3 cm firm, freely moveable, nontender mass in his neck on the right side. Biopsy of the neck mass
reveals Reed-Sternberg cells in a mixed inflammatory infiltrate. CT scan reveals marked enlargement of
mediastinal nodes and the presence of a single nodule in his liver. How should his disease be staged?
A. Stage IA
B. Stage IB
C. Stage IIA
D. Stage IIB
E. Stage IIIA
F. Stage IIIB
G. Stage IVA
H. Stage IVB
Explanation:
The correct answer is F. This is a classic presentation for Hodgkin's disease, a form of lymphoma characterized
by neoplastic proliferation of Reed-Sternberg cells admixed with variable numbers of reactive lymphocytes,
neutrophils, and eosinophils. About half the patients have usually painless adenopathy in the neck or
supraclavicular area at the time of diagnosis. A minority have constitutional symptoms such as fatigue, weight
loss, and night sweats, but these can be important clues. Staging of Hodgkin's disease is based on the extent
of spread and the presence or absence of constitutional symptoms. The man in question has involvement of
cervical lymph nodes, mediastinal nodes, and the liver at the time of diagnosis, so his disease would be stage III
(involvement of lymph nodes or extralymphatic organs on both sides of the diaphragm). The presence of
constitutional symptoms makes this stage IIIB (if constitutional symptoms were absent, it would be IIIA [choice
E]).
Stage I disease (choices A and B) is limited to a single lymph node region or a single extralymphatic organ.
Stage II disease (choices C and D) can involve two or more lymph node regions on one side of the diaphragm,
or can involve contiguous extralymphatic organs or tissues.
Stage IV disease (choices G and H) is defined by the presence of multiple or disseminated disease foci in
extralymphatic organs or tissues. Lymphatic involvement may occur, but need not be present for the diagnosis.

In which of the following sites is embryonal rhabdomyosarcoma most likely to occur?


A. Gastrointestinal tract
B. Head and neck
C. Kidneys
D. Liver

78
E. Lungs
Explanation:
The correct answer is B. Embryonal rhabdomyosarcoma is the most common form of rhabdomyosarcoma and is
composed of cells resembling those found in developing muscle, predominately small, round cells. In embryonal
rhabdomyosarcomas that protrude into an open space, the malignant cells immediately below the surface may
be more densely packed, forming a "cambium" layer. The head and neck (particularly the nose, nasopharynx,
and orbit) is the most frequent site for embryonal rhabdomyosarcoma; other favored sites are the genitourinary
tract and the extremities.

In which of the following neurodegenerative conditions would you expect to observe the phenomenon known as
anticipation?
A. Familial Alzheimer disease (FAD)
B. Familial amyotrophic lateral sclerosis (ALS)
C. Huntington disease
D. Pick disease
E. Progressive supranuclear palsy
Explanation:
The correct answer is C. Anticipation is a phenomenon in which the phenotype of a disease worsens over
successive generations. This has been observed in families affected by a hereditary disorder because of an
expansion of unstable sequences of nucleotide repeats (triplet repeat expansion). Clinical features worsen with
each successive generation as the number of triplet repeats increases. Huntington disease is caused by
expansion of an unstable CAG repeat in a gene encoding a protein called huntingtin, of unknown function. All
the unstable triplet-repeat disorders identified so far are associated with neurodegenerative conditions. Other
examples are fragile X syndrome, myotonic dystrophy, and Friedreich ataxia.
FAD (choice A) comprises 5% to 10% of all cases of Alzheimer disease and is due to autosomal dominant
mutations of three different genes: amyloid precursor protein (APP) gene, presenilin-1 gene, and presenilin-2
gene. The e4 allele of the gene encoding apolipoprotein E increases the risk for FAD. Unstable repeat
expansion is not a cause of FAD.
The great majority of cases of ALS (choice B)are sporadic. A small subset of familial ALS is caused by
mutations in the gene coding for superoxide dismutase on chromosome 21.
Pick disease (choice D) and progressive supranuclear palsy (choice E) are usually sporadic. The genetic
alterations of the very few familial cases reported have not been elucidated.

A 37-year-old woman is in a serious automobile accident and sustains a closed head injury. She does not
immediately seek medical attention, but is brought to the emergency room two hours later by her brother. On
physical examination, there is mydriasis and loss of the pupillary light reflex. Several hours later, she is unable to

79
follow a flashlight with her eyes. Which of the following types of herniation is most likely occurring in this patient?
A. Cerebellar tonsils into the foramen magnum
B. Cerebellum upward past the tentorium
C. Cingulate gyrus under the falx
D. Medulla into the foramen magnum
E. Temporal lobe under the tentorium
Explanation:
The correct answer is E. Head trauma can cause subdural or epidural hematomas that force the medial aspect
of the temporal lobe (uncus) under the free edge of the tentorium cerebelli (an uncal herniation). The resulting
compression of the oculomotor (III) nerve characteristically affects the peripheral areas of the nerve, which
carry parasympathetic information, before affecting the central areas of the nerve, which carry somatomotor
information. Recognizing the development of symptoms related to this damage may permit life-saving medical
or surgical intervention before irreversible damage is done to the brainstem.
Tonsillar herniation into the foramen magnum (choice A) can compress the medulla, causing respiratory failure
and death.
Upward herniation of the cerebellum (choice B) is occasionally seen in patients with posterior fossa lesions.
Herniation of the medially located cingulate gyrus under the falx cerebri (choice C), also called subfalcine or
cingulate herniation, can be seen with cerebral hemisphere expansion and can compress the anterior cerebral
artery.
The medulla (choice D) is normally in the foramen magnum.

Following a fall on a basketball court, a 20-year-old man develops a swollen and painful upper arm. Over the next
several weeks, the involved area becomes more circumscribed and firm, and then later evolves to a painless,
hard, well-demarcated mass. X-ray of the arm at this point would most likely demonstrate which of the following?
A. Benign-appearing bony outgrowths from the humerus
B. Dislocation of the shoulder
C. Flocculent radiodensities surrounding a radiolucent center
D. Healing fracture
E. Malignant-appearing bony outgrowths from the humerus
Explanation:
The correct answer is C. This is a typical presentation of myositis ossificans, which usually occur in athletic
adolescents and young adults following trauma. Following muscle trauma, an area of damage heals with a

80
fibroblastic proliferation that then ossifies, even though there is no connection to bone. Flocculent
radiodensities surrounding a radiolucent center are seen on x-ray. The lesion can microscopically resemble
extraskeletal osteosarcoma, but the clinical history is usually quite different (osteosarcoma is a disease of the
elderly). Simple excision usually cures myositis ossificans.

A pregnant woman presents at 22 weeks gestation for a routine prenatal visit. Physical examination
demonstrates ankle edema and new onset hypertension. Urinalysis reveals marked proteinuria.
Development of which of the following would justify a diagnosis of eclampsia?
A. Diabetes mellitus
B. Hyperuricemia
C. Seizures
D. Systemic lupus erythematosus
E. Thrombocytopenia
Explanation:
The correct answer is C. A pregnant patient is considered to be in preeclampsia if she develops
hypertension, proteinuria, and edema. Eclampsia includes the addition of seizures to the triad.
Approximately 7 percent of pregnant women develop preeclampsia, typically between 20 weeks
gestation to 6 weeks postpartum. Predisposing conditions include pre-existing hypertension,
diabetes (choice A), and autoimmune diseases such as lupus (choice D). Laboratory features can
include hyperuricemia (choice B) and thrombocytopenia (choice E), but these are not used to
define the presence of eclampsia.
A 65-year-old woman presents to the emergency room with a pathologic fracture of the shaft of her
humerus. X-ray studies demonstrate multiple lytic and blastic bone lesions. Biopsy of one of
these lesions shows adenocarcinoma. Which of the following is the most likely source of the
primary tumor?
A. Breast
B. Colon
C. Kidney
D. Lung
E. Thyroid
Explanation:
The correct answer is A. Breast cancer is unusual in that it produces both lytic and blastic
metastases to bone. Breast and prostate cancers are the most common sources of bone metastases,
but prostate metastases are usually blastic.
Colon cancer (choice B) does not usually metastasize to bone.

81
Kidney cancer (choice C), lung cancer (choice D), and thyroid cancer (choice E) produce lytic
lesions when they metastasize to bone.
Physical examination of a 14-year-old boy demonstrates six coffee-colored skin macules up to 3 cm in
diameter. No other skin lesions are noted, but a small mass lesion is felt in the subcutaneous
tissues below two of the macules. These masses are most likely closely associated with which of
the following structures?
A. Arrector pili
B. Hair follicles
C. Peripheral nerves
D. Sebaceous glands
E. Sweat glands
Explanation:
The correct answer is C. The lesions are caf au lait spots. Isolated caf au lait spots are
common in the general population, but more than 3 or 4 caf au lait spots suggests the
possibility of neurofibromatosis. Apparently, the peripheral nerve tumors (neurofibromas) that
are part of this syndrome sometimes induce hyperpigmentation of the overlying skin, producing
the caf au lait spots.
The arrector pili (choice A) are the small muscles of the hair follicle that give rise the
sensation of "goose-bumps"; it is thought that some leiomyomas of the skin may arise from these
tiny muscles.
Hair follicles (choice B), sebaceous glands (choice D), and sweat glands (choice E) can give
rise to a variety of benign and, very rarely, malignant tumors of the skin that do not usually
involve subcutaneous tissues.
During a routine physical examination, a 45-year-old woman is noted to have a ruddy complexion. Her
hematocrit is 52%. Her lungs are clear and she does not smoke. Serum erythropoietin levels are
elevated. Cancer of which of the following organs is the most likely cause of her increased
hematocrit?
A. Breast
B. Colon
C. Kidney
D. Stomach
E. Thyroid
Explanation:
The correct answer is C. The kidney normally produces erythropoietin, which stimulates the
erythroid cell line in the bone marrow. Renal cell carcinoma can lead to overproduction of
erythropoietin and thereby cause secondary polycythemia. Other causes of secondary polycythemia
are diseases that impair oxygenation, including pulmonary diseases (including smoking) and
congestive heart failure.

82

Breast cancer (choice A) can present as an axillary mass or as Paget's disease of the nipple.
Colon cancer (choice B) and stomach cancer (choice D) can present with anemia secondary to
blood loss.
Thyroid cancer (choice E) can present with hypothyroidism.
A patient with respiratory symptoms undergoes bronchoalveolar lavage. Which of the following findings
would be most suggestive that congestive heart failure is the cause of the symptoms?
A. Ciliated bronchial epithelial cells
B. Encapsulated pairs of small cocci
C. Hemosiderin-laden macrophages
D. Rhomboid-shaped crystals
E. Single hat-shaped structures that stain with silver stains
Explanation:
The correct answer is C. Hemosiderin-laden macrophages are sometimes called "heart failure
cells" because they are so often seen in congestive heart failure. The congestive heart failure
causes increased pulmonary capillary pressure with tiny hemorrhages. The pulmonary alveolar
macrophages phagocytize the dead erythrocytes and retain the iron from the hemoglobin in the
form of hemosiderin.
Ciliated bronchial epithelial cells (choice A) are a normal finding.
The cocci described in choice B are pneumococcus, and can cause pneumonia.
Choice D describes Charcot-Leyden crystals, found in allergic asthma.
Choice E describes the Pneumocystis organism, which can infect AIDS patients.
A 78-year-old woman is brought to the emergency department because of slowly developing confusion over
the past ten days. Careful physical examination demonstrates a relatively small laceration of
the scalp. CT of the head would be most likely to reveal which of the following?
A. Epidural hematoma
B. Mixed parenchymal and subarachnoid hemorrhage
C. Multiple tiny hemorrhages of the putamen
D. Subarachnoid hemorrhage
E. Subdural hematoma
Explanation:
The correct answer is E. This history is fairly typical for subdural hematoma. Usually, the
patient is elderly or alcoholic and has some degree of brain atrophy, causing the brain to pull
slightly away from the skull. This stretches the bridging (penetrating) veins that cross the

83
skull to connect to the cranial venous sinuses, leaving them very vulnerable to rupture after
minor head trauma. This type of hematoma is due to venous, rather than arterial, hemorrhage and
thus tends to develop rather slowly.
Epidural hematoma (choice A) is seen following severe head trauma causing laceration of the
middle meningeal artery.
Mixed parenchymal and subarachnoid hemorrhage (choice B) can be seen with bleeding from AV
malformations.
Multiple tiny hemorrhages of the putamen (choice C) are intraparenchymal hemorrhages associated
with severe hypertension.
Subarachnoid hemorrhage (choice D) can be seen with bleeding from ruptured berry aneurysms.
A liver biopsy from a 54-year-old man shows many Mallory bodies. This finding is most suggestive of
which of the following diseases?
A. Alcohol abuse
B. Alpha1-antitrypsin deficiency
C. Hepatitis A
D. Hepatitis B
E. Wilson's disease
Explanation:
The correct answer is A. Mallory bodies are eosinophilic cytoplasmic inclusions ("alcoholic
hyaline") that are found in the largest numbers in alcoholic hepatitis. They were originally
considered to be pathognomic of alcohol abuse, but have since been found (in much smaller
numbers) in many other liver conditions.
Alpha1-antitrypsin deficiency (choice B) involvement of the liver is characterized by periodic
acid Schiff (PAS)-positive cytoplasmic granules in hepatocytes.
Hepatitis A (choice C) and hepatitis B (choice D) infections are definitively established with
serologic markers.
In Wilson's disease (choice E), there is excess copper deposition in the liver.
A pathologist examining electron micrographs of glomeruli sees prominent deposits between the podocytes
and the basement membrane of the glomerular capillaries. These findings are most likely related
to prior infection with which of the following genera?
A. Escherichia
B. Klebsiella
C. Neisseria
D. Pseudomonas
E. Streptococcus

84

Explanation:
The correct answer is E. The disease is post-infectious glomerulonephritis, which is usually
related to prior sore throat or skin infection by Streptococcal species; Staphylococcus also
causes some cases. The prominent deposits are also known as subepithelial humps, and consist of
immunoglobulin and complement.
Gram-negative rods such as Escherichia(choice A), Klebsiella (choice B), and Pseudomonas
(choice D) have not been implicated as significant causes of post-infectious
glomerulonephritis.
The gram-negative cocci Neisseria(choice C) have not been implicated in post-infectious
glomerulonephritis.
About one week after birth, a premature infant develops acute abdominal distress with gastrointestinal
bleeding, large intestinal perforation, and sepsis. Which of the following conditions should be
suspected?
A. Crohn's disease
B. Diverticulosis
C. Necrotizing enterocolitis
D. Pseudomembranous colitis
E. Ulcerative colitis
Explanation:
The correct answer is C. This question illustrates the typical presentation of necrotizing
enterocolitis. This frequently fatal condition is unfortunately fairly common in premature and
low-birth-weight infants and appears to have a multifactorial origin, with intestinal ischemia,
microbial agents, and a poor gastrointestinal immune response being major contributing factors.
None of the other conditions listed in the choices would usually be seen in a neonate.
Crohn's disease (choice A), also known as regional enteritis, is a type of inflammatory bowel
disease that affects the colon and small intestine. It usually manifests in the teens or early
twenties, but milder cases may not be diagnosed until later in life.
Diverticulosis (choice B) is prevalent in older individuals.
Pseudomembranous colitis (choice D) is seen in the setting of broad-spectrum antibiotic use.
Ulcerative colitis (choice E) is a form of inflammatory bowel disease typically diagnosed in
young adults.
An infant born to German-American parents presents with mild anemia, jaundice, and splenomegaly. A
complete blood count with differential reveals spherocytosis; the reticulocyte count is
elevated. The parents state that several relatives have also suffered from a similar illness.
The infant's condition is most likely caused by defective
A. clathrin
B. connexon

85

C. dynein
D. spectrin
E. tubulin
Explanation:
The correct answer is D. The infant is suffering from hereditary spherocytosis, a disease that
can result from defective erythrocyte cytoskeletal proteins such as spectrin or ankyrin.
Spectrin is a cytoskeletal protein in the erythrocyte that binds to the ankyrin/band 3 protein
complex. It also binds to a protein 4.1/actin complex to form a stable supporting cytoskeletal
network just below the plasma membrane. This helps to maintain the biconcave shape of the cell.
Defective spectrin can lead to a destabilization of this network and a tendency of the affected
cells to assume a spherical shape and become osmotically fragile. Such cells often become
trapped in the splenic cords, and are subsequently destroyed by phagocytes. This can lead to
hemolytic anemia, jaundice and splenomegaly.
Clathrin (choice A) is a protein found on the cytoplasmic side of the plasma membrane. It is
located within the coating of the so-called coated pits seen in electron micrographs of cells
engaged in receptor-mediated endocytosis.
Connexon (choice B) is a protein found in gap junctions, usually aggregated around the aqueous
pores within the junctional area.
Dynein (choice C) is an ATPase protein responsible for the interaction and sliding of
microtubule doublets, resulting in the bending of cilia and flagella.
Tubulin (choice E) is a protein that exists in alpha and beta forms that polymerize to form
microtubules.
A 43-year-old man presents to his physician with complaints of fatigue. Physical examination is
remarkable for hypertension. Laboratory examination reveals anemia, elevated blood urea nitrogen
(BUN), and elevated creatinine. Which of the following conditions would best account for this
presentation?
A. Adult polycystic kidney disease
B. Lipoid nephrosis
C. Poststreptococcal glomerulonephritis
D. Renal cell carcinoma
E. Wilms' tumor
Explanation:
The correct answer is A. Adult polycystic disease is characterized by kidneys that
progressively enlarge as a consequence of multiple renal cysts. Presenting signs (typically
beginning when patients are in their 40s), include hypertension (secondary to renin
production), renal failure (with elevated BUN and creatinine), and anemia (secondary to a
failure of renal erythropoietin production). This autosomal dominant condition also causes
cysts in other organs (liver, pancreas, spleen, gonads), berry aneurysms (the rupture of which
may cause death), and abnormalities of cardiac valves.

86

Lipoid nephrosis (choice B), or minimal change disease, is the most common cause of nephrotic
syndrome in children, and is not usually associated with renal failure.
Poststreptococcal glomerulonephritis (choice C) usually presents with hematuria following a
bout of streptococcal pharyngitis.
Renal tumors typically present either as a mass or with hematuria. Renal cell carcinoma (choice
D) generally affects adults, while Wilms' tumor (choice E) is an early childhood tumor.
A hospital laboratory is sent blood samples from six patients with six different diagnoses. A blood
smear from one sample shows a reduced number of normochromic, normocytic erythrocytes, with
numerous schistocytes. This blood sample has been probably drawn from the patient with
A. acute blood loss
B. chronic autoimmune gastritis
C. disseminated intravascular coagulation
D. immune-mediated hemolysis
E. iron deficiency
F. thalassemia minor
Explanation:
The correct answer is C. Simply put, schistocytes are fragments of red blood cells of different
shapes and sizes. They are derived from mechanical damage to circulating erythrocytes, which
results in a form of anemia referred to as microangiopathic hemolytic anemia. There are two
main causes of this form of anemia: disseminated intravascular coagulation (DIC) and prosthetic
valves. DIC is due to widespread and uncontrolled activation of the coagulation cascade,
leading to formation of microthrombi throughout the circulation. Erythrocytes become fragmented
as they squeeze through narrowed arterioles and capillaries.
Acute blood loss (choice A) is followed by reduced circulating blood volume. In the acute
phase, a sample of blood may appear entirely normal. Only following hemodilution does a
normocytic normochromic anemia becomes apparent.
Chronic autoimmune gastritis (choice B) is associated with autoantibodies that block the
binding of intrinsic factor to vitamin B12, leading to deficient B12 absorption and
megaloblastic anemia. Abnormally large (macrocytic) red blood cells and hypersegmented
neutrophils are the diagnostic features of this form of anemia.
Immune-mediated hemolysis (choice D) is due to agglutinins that can be classified as warm (IgG
type) or cold (IgM type) according to whether the Coombs test is positive at 37 C or 0-4 C.
Agglutinins binding red blood cells may cause acute or chronic anemia by intravascular or
extravascular coagulation. Unconjugated hyperbilirubinemia is usually present, and the anemia
is normochromic and normocytic. Schistocytes are not observed.
Any form of iron-deficiency anemia (choice E), whether due to decreased absorption or to
chronic blood loss, is characterized by small (microcytic) and pale (hypochromic) erythrocytes.
Thalassemia (choice F) is a genetic condition caused by mutations of the genes coding for the
globin chains. Patients homozygous for mutations in the &beta;-globin genes are affected by a

87
severe form of chronic anemia, referred to as thalassemia major, because of severely reduced or
absent &beta;-globin synthesis. Heterozygous individuals have the &beta;-thalassemia trait and
manifest a clinical picture known as thalassemia minor. They are usually asymptomatic, but the
peripheral blood smear shows microcytic erythrocytes, similar to those observed in irondeficiency anemia.
A 45-year-old man presents to a clinician because of a "bulge in his neck." Physical examination
demonstrates an enlargement of the patient's thyroid gland. Needle aspiration of the thyroid
demonstrates cell clusters that are suspicious for papillary carcinoma of the thyroid. Which of
the following findings would most strongly support this diagnosis?
A. Anitschkow cells
B. Auer rods
C. Psammoma bodies
D. Reed-Sternberg cells
E. Roth's spots
Explanation:
The correct answer is C. Features of papillary carcinoma of the thyroid on aspiration include
papillary clusters, "Orphan Annie" nuclei, and psammoma bodies. Psammoma bodies are laminated,
concentric, calcific spherules seen most frequently in papillary adenocarcinoma of the thyroid,
serous papillary cystadenocarcinoma of the ovary, meningioma, and malignant mesothelioma.
Anitschkow cells (choice A) are activated histiocytes found in rheumatic heart disease.
Auer rods (choice B) are cytoplasmic inclusions found in promyelocytic leukemia cells.
Reed-Sternberg cells (choice D) are typically binucleate (or contain bilobed nuclei) with
prominent "owl's eyes" nucleoli cells found in Hodgkin's disease.
Roth's spots (choice E) are pale retinal spots surrounded by hemorrhage seen in endocarditis.
A blood sample from a patient with polycythemia vera is sent for complete blood count. Which of the
following blood components is most likely to be reported within normal limits?
A. Lymphocytes
B. Neutrophils
C. Platelets
D. Red blood cells
E. White blood cells
Explanation:
The correct answer is A. Polycythemia vera is an example of a myeloproliferative disorder, a
neoplastic disease of multipotential myeloid stem cells with the capacity to differentiate into
erythrocytes, megakaryocytes, or granulocytes. The only hematologic cell line which is not
increased in polycythemia vera is the lymphocytic line, and the lymphocyte count is generally

88
normal. The complications of polycythemia vera are generally due to increased blood viscosity
and a tendency toward thrombosis. Without regular phlebotomy, death usually occurs within
months of diagnosis.
Increased megakaryocyte production often produces platelet counts (choice C) above 500,000
cells/ml.
Dramatic increases in erythroid cell production (choice D) dominate the hematologic picture in
polycythemia vera, and patients may have hematocrits as high as 60%.
Clonal expansion of granulocyte precursors leads to markedly increased neutrophil counts
(choice B) and white blood cell counts (choice E).
A patient presents to an ophthalmologist complaining of difficulty reading in the afternoons and
evenings because his eyelids get "too heavy" and he gets double vision. The ophthalmologist is
concerned about a paraneoplastic syndrome because the patient also has a recent unexplained 20
lb weight loss. Which of the following tumors would be most likely to produce this patient's
symptoms?
A. Breast carcinoma or renal cell carcinoma
B. Bronchogenic carcinoma or intracranial neoplasm
C. Lymphoma or myeloma
D. Renal cell carcinoma or parathyroid adenoma
E. Thymoma or bronchogenic carcinoma
Explanation:
The correct answer is E. The patient is suffering from myasthenia gravis, which is caused by an
autoimmune attack on the nicotinic acetylcholine receptor of the skeletal muscle motor end
plate. Symptoms include early muscle fatigue, often beginning with the small, active muscles
around the eyes. Myasthenia gravis can be associated with thymoma and bronchogenic carcinoma.
Hypercalcemia can be produced as a paraneoplastic effect in lytic bony metastases, squamous
cell lung cancer, breast cancer (choice A), renal cell carcinoma (choices A and D), multiple
myeloma (choice C), and parathyroid adenomas (choice D).
The syndrome of inappropriate antidiuretic hormone secretion (SIADH) can be produced by
bronchogenic carcinoma and intracranial neoplasms (choice B).
Which of the following cardiac complications may develop in a 33-year-old woman with systemic lupus
erythematosus (SLE) because of her underlying condition?
A. Hemorrhagic pericarditis
B. Infective endocarditis
C. Libman-Sacks endocarditis
D. Mitral valve prolapse
E. Myocardial fibrosis

89
Explanation:
The correct answer is C. Libman-Sacks endocarditis affects patients with systemic lupus
erythematosus (SLE). Small granular vegetations consisting of fibrin develop on the
undersurface of the leaflets of mitral and aortic valves. Fragments of vegetations may detach,
resulting in embolism. Eventually, vegetations heal by organization of fibrin and subsequent
fibrosis, leading to distortion of valve leaflets and resultant insufficiency or stenosis. The
pathogenesis is unclear but is probably related to thrombotic diathesis.
Hemorrhagic pericarditis (choice A) is usually a consequence of tuberculosis or metastatic
spread of cancer to the pericardium. An exudate containing blood accumulates in the pericardial
sac, producing pain and a characteristic auscultatory phenomenon known as a pericardial rub.
SLE is associated instead with fibrinous pericarditis, which also manifests with chest pain and
pericardial rub.
Infective endocarditis (choice B) is prevalent in patients with damaged or prosthetic valves
and in intravenous drug abusers. It is associated with formation of bulky, friable vegetations
that consist of masses of fibrin, neutrophils, and bacteria. Such vegetations cause extensive
destruction of valve leaflets and may release fragments into the bloodstream, with resultant
septic embolism.
There is no association between SLE and mitral valve prolapse (choice D), which is a frequent,
usually asymptomatic, valvular abnormality that occurs in 2% to 3% of the general population.
Mitral valve prolapse is also associated with Marfan syndrome. Complications include sudden
death, thromboembolism, and mitral regurgitation.
Myocardial fibrosis (choice E) does not occur in SLE. It develops in progressive systemic
sclerosis, a collagenopathy characterized by extensive fibrosis in many organs, especially
skin, lungs, and gastrointestinal tract. The heart is involved in 30% of cases. Myocardial
fibrosis may cause arrhythmias.
A 27-year-old female feels a lump in her breast and schedules an appointment with her doctor for
examination. A tissue biopsy is taken and examined, with multiple areas showing only benign
features. Which of the following findings implies an increased risk of subsequently developing
invasive carcinoma?
A. Apocrine metaplasia
B. Blue-domed cysts of Bloodgood
C. Duct ectasia
D. Intraductal papilloma
E. Sclerosing adenosis
Explanation:
The correct answer is E. Sclerosing adenosis is a type of proliferative fibrocystic disease
that is often seen with other variants of fibrocystic disease. It is a proliferation of small
ducts and myoepithelial cells near the terminal duct lobular unit. It also is characterized by
fibrosis that distorts the glands and lobules into a whorled pattern. The demonstration of this
on biopsy, as with other proliferative fibrocystic changes, implies an increased risk for
subsequent invasive breast cancer.
Apocrine metaplasia (choice A) is the term used to describe a transformation of ductal

90
epithelial cells to eosinophilic cells resembling apocrine sweat gland epithelium. This is a
benign, common, non-proliferative change that is not associated with increased cancer risk.
A blue-domed cyst of Bloodgood (choice B) is a fluid-filled cyst, sometimes up to 5 cm in size,
that appears blue grossly before it is incised. When incised, the fluid is serous and darkcolored. These cysts are part of benign non-proliferative fibrocystic disease and do not carry
increased cancer risk.
Duct ectasia (choice C) is characterized by a large dilated duct with inspissated material. It
is associated with periductal inflammation and fibrosis of the large collecting ducts under the
areola. It is common in elderly women, although the cause is not clear. There is no increased
risk of cancer.
Intraductal papilloma (choice D) is a benign tumor that occurs in the large subareolar ducts.
It is usually solitary and found in middle-aged-to-elderly females. Clinically, it mimics
cancer and may be associated with nipple discharge. It is covered with benign epithelium and
attached to the duct by a fibrovascular stalk. A solitary lesion is not precancerous nor is it
a risk for subsequent cancer.
A 40-year-old man develops progressive weakness culminating in paralysis. Both upper and lower motor
neurons are lost in this man's disease. Over the course of a decade, the disease progresses to
complete paralysis of all voluntary muscles, and he dies in respiratory failure. At autopsy,
which CNS sites would show the most marked neuronal loss?
A. Caudate
B. Cerebellum
C. Globus pallidus
D. Spinal cord
E. Substantia nigra
Explanation:
The correct answer is D. Amyotrophic lateral sclerosis (ALS) is a devastating disease
characterized by degeneration of both upper and lower motor neurons. Depending on the stage of
this disease, patients may primarily experience either upper motor symptoms (hyperreflexia,
spasticity, Babinski reflex) or lower motor symptoms (weakness, muscular atrophy,
fasciculations). The upper motor neurons are located in the motor area of the cerebral cortex;
the lower motor neurons are located in the anterior horn of the spinal cord. The physicist
Steven Hawking has the disease. There is as yet no effective therapy.
The caudate (choice A) is damaged in Huntington disease, characterized by choreiform movements.
The cerebellum (choice B) is damaged in spinocerebellar degenerative diseases, including
Friedreich ataxia and olivopontocerebellar atrophy, both of which have ataxia as a prominent
symptom.
The globus pallidus (choice C) degenerates in striatonigral degeneration, which resembles
Parkinson disease.
The substantia nigra (choice E) is damaged in Parkinson disease, characterized by tremor and
difficulty initiating movements.

91
A 35-year-old retarded man with a strong history of mental retardation among male relatives undergoes
genetic testing. His lymphocytes are cultured in a medium containing methotrexate and 4% of the
metaphase chromosomes in the lymphocytes show a breakpoint at q27.3 on the X chromosome. This
man is at increased risk for which of the following cardiovascular disorders?
A. Aortic stenosis
B. Atrial septal defect
C. Mitral valve prolapse
D. Tricuspid atresia
E. Ventricular septal defect
Explanation:
The correct answer is C. The disease is Fragile X Syndrome, which is a familial form of mental
retardation that is roughly as common as Down's syndrome. The phenotype has a variable
expression, but can include large head circumference at birth, perinatal complications
(premature birth, asphyxia, seizures), and possibly increased incidence of sudden infant death
syndrome. Later, mental retardation, particularly involving language, and symptoms suggestive
of attention deficit disorder and/or autism may appear. Features suggestive of connective
tissue disorder (lax skin and joints, flat feet, large ears) are common. After puberty, there
may be a long narrow face with prominent jaw and nasal bridge. Macro-orchidism is also common
after puberty. Mitral valve prolapse and aortic root dilatation, which may appear in late
adolescence or adulthood, are among the most serious complications of this disorder.
Aortic regurgitation related to aortic root dilatation, not stenosis (choice A), can be a
problem in this population.
Common congenital cardiac malformations such as atrial septal defect (choice B) or ventricular
septal defect (choice E) are not features of Fragile X syndrome, but can be seen with Down's
syndrome.
Tricuspid atresia (choice D) is a serious congenital cardiac malformation limiting flow into
the right ventricle, but it is not part of Fragile X syndrome.
A 52-year-old woman complains of generalized itching. On examination, she has mild jaundice, and
laboratory evaluation reveals elevated conjugated bilirubin and increased alkaline phosphatase.
A liver biopsy shows granulomatous inflammation centered on small bile ducts within the portal
spaces. Which of the following findings would support the diagnosis of primary biliary
cirrhosis?
A. Antimitochondrial autoantibodies
B. Decreased serum ceruloplasmin
C. Elevated liver enzymes
D. Hypoalbuminemia
E. Prolonged prothrombin time
Explanation:

92
The correct answer is A. Primary biliary cirrhosis is characterized by chronic granulomatous
inflammation leading to destruction of medium-sized intrahepatic bile ducts. A cholestatic
picture develops, with conjugated hyperbilirubinemia and increased alkaline phosphatase.
Itching is due to elevated levels of circulating bile salts. The disease predominantly affects
middle-aged women and typically progresses to cirrhosis. Ninety percent of patients have
circulating antimitochondrial autoantibodies, which may play a pathogenetic role.
Decreased serum ceruloplasmin (choice B) suggests Wilson disease, an inherited condition
characterized by accumulation of copper in the liver, brain, and eye. Increases in hepatic
copper content and urinary copper excretion are diagnostically important findings.
Elevated liver enzymes (choice C) result from any condition causing damage to hepatocytes, with
release of enzymes (such as AST and ALT) into the bloodstream. Thus, this finding is not
specific for any particular type of cirrhosis or liver disease.
Hypoalbuminemia (choice D) is caused by liver failure of any etiology and results from
decreased hepatocyte protein synthesis.
Prolonged prothrombin time (choice E) is also an effect of liver failure of any cause.
Coagulopathy is attributable to impaired synthesis of clotting factors II, VII, IX, and X.
A 45-year-old woman develops abdominal and pelvic discomfort. Physical examination reveals a large mass
in the right lower quadrant, which is surgically resected. The mass consists of a large (25 cm)
cystic sac containing thick mucinous fluid within a thin wall. On careful inspection, the
pathologist finds an area of increased thickness in the cyst wall, which is sampled for
histology. Microscopically, the tumor appears to be composed mostly of a single layer of
nonciliated columnar cells arranged in papillary projections. The thickened area, however,
displays stratification of epithelial cells, increased cytologic atypia, and high mitotic
activity. Nevertheless, no stromal invasion is found. Which of the following is the most likely
diagnosis?
A. Borderline mucinous tumor
B. Mucinous cystadenocarcinoma
C. Mucinous cystadenoma
D. Serous cystadenocarcinoma
E. Serous cystadenoma
Explanation:
The correct answer is A. Classification of ovarian tumors, like testicular tumors, is based on
putative cell of origin. Thus, ovarian tumors can be divided into neoplasms of germ cells,
surface epithelium, or stromal origin. Two thirds of all ovarian neoplasms derive from the
surface (coelomic) epithelium. These cystic tumors may contain clear serous fluid or turbid
mucinous fluid. Depending on whether a tumor is benign or malignant, surface epithelium tumors
can be differentiated into serous cystadenoma or cystadenocarcinoma, and mucinous cystadenoma
or cystadenocarcinoma. How about borderline tumors? These are intermediate cases in which the
epithelial lining shows malignant features (cytologic atypia and architectural disorganization)
in the absence of stromal invasion. Thus, microscopic features of the ovarian tumor in this
case are consistent with a borderline mucinous tumor.
Mucinous cystadenocarcinoma (choice B) shows obvious signs of malignancy, including foci of
invasion of the stroma within the cystic wall.

93

Mucinous cystadenoma (choice C) is a cystic tumor with a mucin-rich fluid content. The
epithelial lining is similar to intestinal or cervical epithelium, ie, a single layer of
columnar cells with apical mucin and no cilia.
Serous cystadenocarcinoma (choice D) is the most frequent malignant ovarian tumor. Its
epithelial lining is composed of columnar cells showing atypia and crowding. By definition,
stromal invasion is present.
Serous cystadenoma (choice E) is the benign counterpart of serous cystadenocarcinoma. Ciliated
columnar epithelial cells line the neoplasm in an orderly single layer.
CT scan of the head in a patient who had been in an automobile accident fails to demonstrate a pituitary
gland. Endocrine evaluation demonstrates serum hormones are all within normal limits. Which of
the following is the most likely diagnosis?
A. Craniopharyngioma
B. Empty sella syndrome
C. Hypothalamic suprasellar tumor
D. Large pituitary adenoma
E. Pituitary microadenoma
Explanation:
The correct answer is B. Empty sella syndrome is an uncommon but dramatic condition in which
the pituitary radiologically appears to be completely missing from the sella turcica, which is
instead filled with cerebrospinal fluid (CSF). The most common cause is herniation of the
arachnoid through the diaphragm sella. This leads to excess local CSF production, in turn
causing pressure atrophy of the pituitary. Other causes include Sheehan's syndrome, infarction
of an adenoma followed by scarring of the remaining pituitary, and surgery or radiation
therapy. Most patients with empty sella syndrome have sufficient residual pituitary parenchyma
to prevent hypopituitarism.
Craniopharyngiomas (choice A) can produce a mass that involves the pituitary.
Hypothalamic suprasellar tumors (choice C), including craniopharyngiomas, can produce a mass
that involves the pituitary.
Large pituitary adenomas (choice D) usually produce a mass lesion. They can infarct and produce
empty sella syndrome, but this is rare.
Pituitary microadenoma (choice E) would produce no visible radiographic changes.
A patient presents to a physician complaining of recurrent episodic diarrhea, triggered by eating too
much or drinking alcohol. His wife states that "he turns as red as a beet, starts wheezing, and
looks just ghastly" during these episodes. Chest x-ray demonstrates a lung mass. Which of the
following would be most likely to be present on biopsy of the lesion?
A. Bronchioloalveolar carcinoma
B. Carcinoid tumor

94
C. Primary tuberculosis
D. Recurrent tuberculosis
E. Squamous cell carcinoma
Explanation:
The correct answer is B. The patient has carcinoid syndrome, in which vasoactive mediators
secreted by carcinoid tumors (typically of the lung or small bowel) produce recurrent diarrhea,
flushing, wheezing, and carcinoid heart disease. Although many substances may be produced by
these tumors, serotonin is one of the more important mediators produced, probably accounting
for the observed diarrhea.
Bronchioloalveolar carcinomas (choice A) are not particularly likely to secrete hormones.
Primary (choice C) and recurrent (choice D) tuberculosis can be uncommonly associated with
finger clubbing, or hypertrophic osteopathy, but not with carcinoid syndrome.
Squamous cell carcinoma (choice E) can cause hypercalcemia by producing circulating hormones
(humoral hypercalcemia of malignancy).
A patient develops a persistent macrocytic anemia. Serum folate levels are normal, but serum vitamin B12
levels are low. Oral vitamin absorption studies demonstrate that the patient is unable to
absorb vitamin B12 in adequate amounts. Cancer of which of the following organs is most
strongly associated with this patient's condition?
A. Colon
B. Duodenum
C. Esophagus
D. Ileum
E. Stomach
Explanation:
The correct answer is E. Megaloblastic anemia with poor vitamin B12 absorption suggests
pernicious anemia. In this condition, an autoimmune attack on gastric parietal cells leads to
atrophic gastritis with deficient synthesis of the intrinsic factor needed for vitamin B12
absorption. Vitamin B12 is required for DNA synthesis in blood cell precursors; deficiency
produces megaloblastic anemia. Chronic atrophic gastritis predisposes an individual to the
development of gastric carcinoma.
Predisposing factors for colon cancer (choice A) include familial polyposis syndromes,
inflammatory bowel disease, isolated adenomatous polyps, and diet.
Duodenal (choice B) and ileal (choice D) cancers are uncommon.
Predisposing factors for esophageal cancer (choice C) include Barrett's esophagus and PlummerVinson syndrome.
A diabetic mother gives birth to a baby who dies in the first week of life. Autopsy reveals a severe
cardiac malformation. Which of the following is the most likely diagnosis?

95

A. Atrial septal defect


B. Coarctation of the aorta
C. Eisenmenger's syndrome
D. Tetralogy of Fallot
E. Transposition of the great arteries
Explanation:
The correct answer is E. Maternal diabetes is best known for causing large but immature-for-age
babies. There is also a specific association between maternal diabetes and transposition of the
great vessels. In transposition of the great vessels, the aorta takes off from the anterior
part of the right ventricle and the pulmonary trunk takes off from the posterior part of the
left ventricle. This produces a complete separation of the systemic and pulmonary circulations.
Without surgical correction, most affected infants die within the first months of life,
although a patent ductus arteriosus, patent foramen ovale, or ventricular septal defect may
allow enough mixing of blood to temporarily sustain life.
In atrial septal defect (choice A) blood can pass from one atrium to the other.
Associate coarctation of the aorta (choice B) with Turner syndrome.
Eisenmenger's syndrome (choice C) is a shift from a left-to-right shunt to a right-to-left
shunt secondary to developing pulmonary hypertension.
Tetralogy of Fallot (choice D) consists of a ventricular septal defect, an overriding aorta,
pulmonic stenosis, and right ventricular hypertrophy. It is the most common cause of early
cyanosis.
A 10-year-old child with new onset of visual field abnormalities and diabetes insipidus undergoes a head
CT. The CT scan demonstrates a 3 cm mass lesion with focal calcification involving the area
above the sella turcica. Needle biopsy of this mass shows that the tumor is composed of tissue
resembling tooth enamel. Which of the following diagnoses is most likely?
A. Craniopharyngioma
B. Glioblastoma multiforme
C. Large pituitary adenoma
D. Medulloblastoma
E. Pituitary microadenoma
Explanation:
The correct answer is A. The tumor is a craniopharyngioma, alternatively known as an
adamantinoma or ameloblastoma. Craniopharyngiomas may arise in, or more commonly above, the
sella turcica. The histological pattern recapitulates the enamel organ of the tooth, with nests
or cords of stratified squamous or columnar epithelium embedded in a loose fibrous stroma.
Calcification (and even metaplastic bone formation) is common in these benign tumors, which are
thought to arise from vestigial remnants of Rathke's pouch.

96

Glioblastoma multiforme (choice B) characteristically shows at least some enlarged cells with
bizarre nuclei.
Large pituitary adenomas (choice C) contain nests of uniform glandular cells.
Medulloblastoma (choice D) is made of small basophilic cells with relatively large nuclei for
their size.
Pituitary microadenomas (choice E) contain nests of uniform glandular cells.
A 25-year-old-man presents with weight loss, abdominal pain, and bloody diarrhea.
Sigmoidoscopy/colonoscopy reveal mucosal erythema and ulceration extending in a continuous
fashion proximally from the rectum. Which of the following pathologic findings would also be
characteristic of this patient's illness?
A. Bowel wall thickening
B. Cobblestone appearance of mucosa
C. Fistulas
D. Pseudopolyps
E. Transmural lesions
Explanation:
The correct answer is D. This is a question that tests your ability to distinguish between
ulcerative colitis (UC) and Crohn's disease. First, you need to figure out which one this
patient has. The key clues here are the bloody diarrhea (much more common in UC), the rectal
involvement, and especially, the continuous nature of the mucosal damage. Once you've figured
out that the patient has ulcerative colitis, you need to identify the answer choice that is
characteristic of UC. The correct answer is pseudopolyps, which are inflammatory polyps found
in ulcerative colitis and not Crohn's disease.
All of the other choices are features of Crohn's disease. Especially diagnostic is the
transmural nature of the inflammation, which can lead to the development of fissures and
fistulas. Remember also that while Crohn's can involve any part of the GI tract, it typically
does not involve the rectum and is usually found in the terminal ileum and/or colon. In
contrast to UC, the lesions are discontinuous (skip lesions).
Biopsy of a skin lesion shows marked intercellular edema that splays apart adjacent cells, leaving only
thin dark lines between the cells. What is this process called?
A. Acantholysis
B. Acanthosis
C. Hyperkeratosis
D. Parakeratosis
E. Spongiosis
Explanation:

97

The correct answer is E. Spongiosis is the name used for intercellular edema of the epidermis.
Fluid accumulates between the cells, pulling them apart. However, the intercellular connections
(desmosomes) remain largely intact. This produces a distinctive microscopic appearance in which
each cell is surrounded by a broad white band with tiny dark cross-stripes, somewhat
reminiscent of a closed zipper. The basic terms that describe skin lesions are a favorite
target on examinations.
Acantholysis (choice A) is the process of cell separation seen in some blistering diseases. The
intercellular connections are broken in acantholysis, producing "rounded-up" cells that are
dissociated from each other.
Acanthosis (choice B) is a marked thickening of the epidermis due to an increase in the number
and size of the epithelial cells.
Hyperkeratosis (choice C) is an increase in the thickness of the superficial keratin layer of
the epidermis.
The term parakeratosis (choice D) is used when nuclei are present in the normally anuclear
superficial keratin layer.
An elderly woman with a history of multiple oral ulcers presents with flaccid bullae on her scalp, face,
and trunk. Manual pressure on the skin produces separation of the upper layer of the epidermis,
followed by eventual sloughing of the skin. The patient has been in relatively good health
until recently, and denies taking any medications. A biopsy of one of the skin lesions reveals
separation of epithelial cells above the basal layer. Autoantibodies to which of the following
components would most likely be found in this patient?
A. Epidermal basement membrane proteins
B. Glycoprotein IIb/IIIa
C. Intercellular junctions of epidermal cells
D. Intrinsic factor
E. Type IV collagen
Explanation:
The correct answer is C. Bullae with the cleavage plane above the basal layer of the epidermis
suggests pemphigus vulgaris, which is caused by autoantibodies to intercellular junctions of
epidermal cells. The autoantibodies decrease the ability of the keratinocytes to adhere to one
another, permitting formation of vesicles and bullae. Oral involvement is common, and often
precedes the characteristic skin lesions. Separation of the epidermis upon manual stroking of
the skin is known as Nikolsky's sign. This sign is present in other disorders such as StevensJohnson syndrome, but we are told the woman is not taking any medications, a typical cause of
Stevens-Johnson syndrome in the adult population.
Antibodies to epidermal basement membrane proteins (choice A) are seen in bullous pemphigoid,
which is a bullous disease characterized by blisters with a cleavage line between the epidermis
and dermis.
Antibodies to glycoprotein IIb/IIIa (choice B) are seen in autoimmune thrombocytopenic purpura.
Antibodies to intrinsic factor (choice D) are seen in pernicious anemia.

98

Antibodies to Type IV collagen (choice E) are seen in Goodpasture's syndrome.


A 60-year-old man who is being evaluated for abdominal pain and a 30-pound weight loss undergoes
endoscopy, which demonstrates a broad region of the gastric wall in which the rugae are
flattened. Biopsy of this area shows infiltration by numerous polygonal tumor cells with small,
dark, round or ovoid nuclei pushed to the margin of the cell by large, clear, cytoplasmic
structures. These cells might be expected to have which of the following properties?
A. Keratohyalin granules observed by electron microscopy
B. Melanosomes and premelanosomes by electron microscopy
C. Positive staining for gastrin by light microscopy
D. Positive staining for leukocyte common antigen by light microscopy
E. Positive staining for mucin by light microscopy
Explanation:
The correct answer is E. The tumor described is the linitis plastica form of gastric
adenocarcinoma, in which individual mucin-producing tumor cells diffusely infiltrate the mucosa
and muscularis propria to produce a rigid, thickened, "leather-bottle" gastric wall. This tumor
is poorly differentiated and has a poor prognosis.
Keratohyalin granules (choice A) are a feature of squamous cell carcinoma, which does not
usually occur in the stomach.
Melanosomes and premelanosomes (choice B) are observed with electron microscopy in melanocytic
lesions, including melanoma.
Positive immunostaining for gastrin (choice C) would be a feature of gastrin-secreting
carcinoids, which typically form small, yellow nodules composed of nests or cords of small
cells with centrally located, round-to-oval, stippled nuclei.
Positive immunostaining for leukocyte common antigen (choice D) is associated with lymphoma,
which can also affect the stomach, appearing similar to linitis plastica grossly.
Microscopically, however, the individual malignant lymphocytes usually have centrally located
nuclei, and lack the large, clear, cytoplasmic vacuoles described in this question.
Which of the following is the most frequent underlying cause of sudden cardiac death (SCD)?
A. Aortic valve stenosis
B. Congenital anomalies
C. Dilated cardiomyopathy
D. Hereditary anomalies of conduction system
E. Hypertrophic cardiomyopathy
F. Hypertrophy of heart, isolated or hypertensive
G. Ischemic heart disease

99

H. Mitral valve prolapse


I. Myocarditis
J. Pulmonary hypertension
Explanation:
The correct answer is G. About 300,000 to 400,000 individuals succumb to sudden cardiac death
(SCD) each year in the U.S. SCD is defined as any unexpected death of proven cardiac origin,
occurring either within 1 hour after the onset of symptoms or without the onset of symptoms. By
far the most common cause of SCD is ischemic heart disease (IHD), which is most frequently
related to atherosclerosis of the coronary arteries. In younger populations, the
nonatherosclerotic causes listed above are proportionately more frequent.
Aortic valve stenosis (choice A) is most commonly due to senile calcification of the valve
leaflets or a congenitally bicuspid valve.
Congenital anomalies (choice B and D) of the heart, whether involving the cardiac chambers and
valves or the conduction systems, should be strongly suspected in cases of SCD of a young
person.
Dilated cardiomyopathy (choice C) has numerous etiologies, whereas hypertrophic cardiomyopathy
(choice E) is more often familial.
Hypertrophy of the heart (increased cardiac mass) (choice F) is an independent risk factor for
SCD and may be associated with hypertension or increased physical activity (athletes).
Mitral valve prolapse (choice H) is a frequent anomaly in the general population and, although
usually asymptomatic, has been found in young victims of SCD.
Myocarditis (choice I) is usually caused by viral infections and may be asymptomatic, manifest
with slowly progressive heart failure, or lead to SCD.
Patients with pulmonary hypertension (whether primary or secondary) (choice J) are at increased
risk for SCD.
In summary, although IHD is the underlying cause in the majority of SCD cases,
nonatherosclerotic causes should be suspected in young victims or patients without evidence of
significant coronary artery disease.
A 40-year-old man with adult polycystic kidney disease is brought to the emergency room in a coma. CT
scan of the head demonstrates a subarachnoid hemorrhage without parenchymal hemorrhage. Which of
the following is the most likely source of the bleeding?
A. AV malformation
B. Bridging veins
C. Charcot-Bouchard aneurysm
D. Circle of Willis
E. Middle meningeal artery

100
Explanation:
The correct answer is D. Two relationships are useful in solving this problem. The first
relationship is that adult polycystic kidney disease has a specific association with berry
aneurysms involving the circle of Willis and its branches. The second relationship is that
spontaneous subarachnoid hemorrhages are most often the result of bleeding from berry
aneurysms. The berry aneurysms develop at sites of congenital weakness (near branch points) of
the relatively unsupported vessels of the circle of Willis.
AV malformations (choice A) tend to produce mixed parenchymal and subarachnoid hemorrhages.
Rupture of bridging veins (choice B) produces a subdural hematoma.
Rupture of Charcot-Bouchard aneurysms (choice C) can be seen with hypertension (which may
complicate adult polycystic kidney disease). Such rupture produces intraparenchymal hemorrhage,
which if severe, may spread to the subarachnoid space.
Rupture of the middle meningeal artery (choice E) produces epidural hematoma.
Following a weekend alcoholic binge, a patient is brought to the emergency room covered with vomit.
While there, he complains of severe pain of the chest and upper abdomen with radiation to the
left shoulder. The initial chest radiograph is normal. Despite an unremarkable EKG, the patient
is admitted for a presumptive or developing myocardial infarction. Over the next 24 hours, his
condition deteriorates and he goes into shock. Repeat x-ray shows extensive haziness in the lung
fields. Which of the following conditions is the most likely cause of the man's illness?
A. Boerhaave's syndrome
B. DiGeorge syndrome
C. Dressler's syndrome
D. Dubin-Johnson syndrome
E. Osler-Weber-Rendu disease
Explanation:
The correct answer is A. Boerhaave's syndrome is the eponymic name for lower esophageal
rupture. This condition is uncommon, but can follow straining and vomiting. The rupture allows
a high volume of gastric contents to enter the pleural space, and has a very high mortality
rate if the diagnosis is delayed beyond 24 hours (which is unfortunately rather common). The
pain involves the chest and upper abdomen, and can radiate to the back, left chest, or left
shoulder. The initial chest radiograph is often normal. Shock eventually develops. Treatment is
with surgical repair and drainage.
DiGeorge syndrome (choice B) is a failure of development of the third and fourth pharyngeal
pouches, which leads to absence of the parathyroid glands and thymus.
Dressler's syndrome (choice C) is pericarditis following a myocardial infarction.
Dubin-Johnson syndrome (choice D) is an inherited cause of conjugated hyperbilirubinemia.
In Osler-Weber-Rendu disease (choice E), also known as hereditary hemorrhagic telangiectasia,
multiple, small aneurysmal telangiectasias are present from birth in the skin, oral cavity,
alimentary tract, respiratory tract, urinary tract, liver, brain, and spleen.

101

A 60-year-old woman presents to a physician because her right knee is painful, swollen, and red.
Aspiration of the joint space demonstrates basophilic rhomboid crystals. Which of the following
is the most likely composition of the crystals?
A. Calcium pyrophosphate
B. Cholesterol
C. Cystine
D. Monosodium urate
E. Struvite
Explanation:
The correct answer is A. The disease is pseudogout, which is caused by deposition of calcium
pyrophosphate (basophilic rhomboid) crystals within the joint space. The disease tends to
affect older patients of both sexes and typically involves the knee or other large joints.
There is no entirely satisfactory treatment available, although colchicine administration may
protect against attacks and corticosteroids may be of some benefit.
Cholesterol (choice B) can form gallstones in association with obesity, high estrogen states,
multiparity, Crohn's disease, rapid weight loss, clofibrate therapy, and Native American
origin.
Cystine (choice C) and struvite (choice E) can form kidney stones.
Monosodium urate (choice D) is deposited in gout as needle shaped, negatively birefringent
crystals.
A 22-year-old patient presents with multiple neural tumors, Lisch nodules, and caf au lait spots. He
informs the physician that his father, as well as one uncle and his paternal grandfather, had a
similar condition. This patient likely suffers from
A. ependymoma
B. Huntington's disease
C. Marfan's syndrome
D. neuroblastoma
E. neurofibromatosis type I
Explanation:
The correct answer is E. Neurofibromatosis type 1, or von Recklinghausen's disease, is an
autosomal dominant disorder with high penetrance, but variable expressivity. The disease has
three major features: (1) multiple neural tumors anywhere on or in the body; (2) numerous
pigmented cutaneous lesions (caf au lait spots); and (3) pigmented iris hamartomas (Lisch
nodules). Electron micrographic studies show that the tumors are the result of the
proliferation of fibroblasts or Schwann cells in the peripheral nerves, possibly due to ras
inactivation. There is no treatment, except for surgical resection of symptomatic tumors.

102
Ependymoma (choice A) can occur wherever ependymal cells are found. They are more common in
children, and most often originate in the fourth ventricle. They are the most common
intramedullary glioma of the spinal cord.
Huntington's disease (choice B), an autosomal dominant disorder, is characterized by severe
degeneration of the caudate nucleus along with degenerative changes in the putamen and cortex.
In addition to chorea, these patients frequently suffer from athetoid movements, progressive
dementia, and behavioral disorders.
Marfan's syndrome (choice C) is due to a defect in the gene for fibrillin. The major clinical
findings involve the skeleton, cardiovascular system, and the eye. Affected individuals tend to
be tall with long extremities and long, tapering appendages. Mitral valve prolapse and
dilatation of the aortic valve ring or aortic dissection due to cystic medial degeneration are
common.
Neuroblastoma (choice D) is a rare tumor derived from ganglion cell precursors. It may arise
anywhere in the cerebral hemispheres and is typically encountered in children ranging in age
from 2 months to 9 years.
A 65 year-old man develops periumbilical pain which then localizes to the right lower quadrant. On
physical examination, his temperature is 100.5 degrees F rectally, and his abdomen is tender.
Which of the following is the most likely diagnosis?
A. Acute appendicitis
B. Diverticulitis
C. Gallstones
D. Pancreatitis
E. Pyelonephritis
Explanation:
The correct answer is B. Diverticulitis is a disease of the elderly, and usually involves the
distal colon. However, in severe cases, the diverticula may extend throughout the colon and up
to the cecum. Inflammation of a cecal diverticulum can closely mimic acute appendicitis.
Acute appendicitis (choice A) is usually a disease of young adults (and sometimes children).
Pancreatitis (choice D), pyelonephritis (choice E), and gall bladder disease (choice C), refer
pain to the mid back, lateral back, and right upper quadrant, respectively.
A baby is born with a large defect in the occipital bone through which the posterior portion of the
brain has herniated. Which of the following terms best describes this lesion?
A. Encephalocele
B. Meningocele
C. Myelocele
D. Spina bifida
E. Syringomyelia

103

Explanation:
The correct answer is A. The central neurons system and its overlying bones are subject to a
variety of malformations and developmental diseases. The defect described in the question stem
is a cranial encephalocele, in which brain herniates through a defect in the skull bones. The
most common site for such a herniation is the occipital bone. Small defects in the occipital
bone can be treated surgically, but large defects are very problematic, particularly if
significant herniation has occurred, since the brain becomes very vulnerable to trauma and
infection.
Meningocele (choice B) is the term used when the meninges, but not the brain or spinal cord,
herniate through a defect in the bony cranium or spinal column.
Myelocele (choice C) is the term used when the spinal cord herniates through a defect in the
spinal column.
Spina bifida (choice D) refers to vertebral defects through which the spinal cord or meninges
may herniate.
Syringomyelia (choice E) is a tubular, fluid-filled cavity within the spinal cord.
A 77-year-old smoker with hemoptysis and weight loss undergoes a left upper lobectomy for squamous cell
carcinoma. The uninvolved lung tissue shows destruction of the alveolar septae around the
respiratory bronchioles, with marked enlargement of the airspaces. Anthracotic pigment is
deposited heavily in the walls of these tissues. These findings are most compatible with
A. asthma
B. chronic bronchitis
C. emphysema
D. pulmonary hypertension
E. silicosis
Explanation:
The correct answer is C. Emphysema is a pulmonary disease characterized by enlargement of the
alveolar airspaces due to destruction of the septae without consequent fibrosis. The gross
appearance of emphysematous lungs is characteristic: alveoli are sufficiently dilated to allow
visualization with the naked eye, and destruction of structural support to lymphatic vessels
produces heavy pigment deposition in the tissue. Microscopic findings that confirm the
diagnosis include enlarged, round airspaces with club-like ends of broken septae sticking into
the alveoli.
Asthma (choice A) is a disease of airway hyperreactivity, and is characterized by hypertrophy
of the bronchial basement membranes and smooth muscle, with glandular hyperplasia and thick
mucus plugs in the bronchi.
Chronic bronchitis (choice B) produces marked hypersecretion of mucus in the large airways, and
can be identified by hypertrophy of mucous glands in the bronchi and goblet cell hyperplasia in
the smaller airways.
Pulmonary hypertension (choice D) affects neither the airways nor the alveoli. It is

104
characterized by thickening of the arterial smooth muscle with intimal hyperplasia and
fibrosis. Atherosclerotic changes in the normally plaque-free larger pulmonary arteries may be
seen.
Silicosis (choice E), one of the forms of pneumoconiosis, is an interstitial fibrosing disease
that produces thick pleural scars and dense nodules of collagen that may calcify. The silica
particles may be visualized within the nodules using polarized light.
A patient with chronic reflux esophageal symptoms undergoes endoscopy, which demonstrates islands of red
tissue above the gastroesophageal junction. Biopsy of several of these lesions demonstrates
glandular epithelium with no atypical nuclei, with no invasion into the adjacent tissue. For
which of the following cancers does this lesion predispose?
A. Adenocarcinoma of the esophagus
B. Adenocarcinoma of the stomach
C. Sarcoma of the esophagus
D. Sarcoma of the stomach
E. Squamous cell carcinoma of the esophagus
F. Squamous cell carcinoma of the stomach
Explanation:
The correct answer is A. The lesion is Barrett's esophagus, which is related to chronic reflux
of gastric contents into the esophagus, and which predisposes for the development of
adenocarcinoma of the distal esophagus.
Conditions predisposing for adenocarcinoma of the stomach (choice B) include chronic atrophic
gastritis, pernicious anemia, and post-surgical gastric remnants.
Sarcoma of the esophagus (choice C) or stomach (choice D) is rare.
Plummer-Vinson syndrome predisposes for squamous cell carcinoma of the esophagus (choice E).
Squamous cell carcinoma of the stomach (choice F) is virtually non-existent.
A 63-year-old male with a 75-pack-year history of smoking and marked chronic obstructive pulmonary
disease (COPD) develops fevers and a persistent cough frequently tinged with blood. Chest x-ray
reveals a complex cavitary lesion in the right upper lobe. Bronchoscopic examination identifies
a large lung abscess from which pure Fusobacterium nucleatum is cultured. What is the most
likely source of the Fusobacterium?
A. Blood
B. Infected aerosols
C. Oral cavity
D. Stomach
E. Upper respiratory tract

105
Explanation:
The correct answer is C. Anaerobic lung abscesses commonly arise from aspirated oral secretions
in patients with compromised neurological status (e.g., anesthesia, coma, intoxication), or in
individuals with a depressed cough reflex. The most common anaerobic organisms isolated from
lung abscesses include Bacteroides, Fusobacterium, and Peptococcus species, all of which are
typical oral flora.
Blood-borne pathogens (choice A) producing lung abscesses probably originate as emboli from
vegetations on the right heart. The most likely pathogens in endocarditis are Staphylococcus
and Streptococcus species; Fusobacterium would be very unusual.
Fusobacterium usually is not pathogenic and lives in a commensal relationship with the host.
Infective aerosols (choice B) generally contain organisms that are intrinsically pathogenic
(e.g., mycobacteria, influenza virus).
It is the acidity of the gastric aspirate that most seriously injures the lung, not the gastric
bacteria. Fusobacterium do inhabit the gastrointestinal tract, but viable bacteria are much
more likely to be transmitted from the oral secretions than from the stomach (choice D).
The upper respiratory tract (choice E) is generally sterile, although COPD is frequently
complicated by chronic bronchitis. Numerous bacterial and viral pathogens can be involved in
chronic bronchitis; however, Fusobacterium is not an upper respiratory tract pathogen.
A 39-year-old female presents to her gynecologist for a checkup. An adnexal mass is palpated and an
ultrasound shows a 10 cm right ovarian cyst. Surgery is scheduled and the tumor is removed. The
patient is informed that the tumor is not overtly malignant, but is a "borderline" malignancy
and that a future complication may be pseudomyxoma peritonei. Which of the following types of
ovarian tumors was removed?
A. Brenner tumor
B. Clear cell tumor
C. Endometrioid tumor
D. Mucinous cystadenoma
E. Serous cystadenoma
Explanation:
The correct answer is D. Mucinous cystadenoma is a tumor that arises from the ovarian surface
epithelium and can be benign, borderline, or malignant. It occurs in middle-aged females and
contains mucin-secreting epithelial cells. If it is a borderline or malignant tumor then a rare
complication is pseudomyxoma peritonei. This is a condition in which large amounts of
gelatinous, sticky, myxoid material fills the abdominal cavity and causes adhesion and
obstruction of viscera. The myxoid material itself is not malignant, but the complications from
its presence are usually fatal.
Brenner tumors (choice A) can be benign, borderline, or malignant. They are composed of
epithelial nests that look like bladder epithelium. There is also a stromal component that
looks like ovarian stroma. Mucin may be produced in the center of the epithelial cells. They
are sometimes associated with mucinous cystadenomas, but alone are not associated with
pseudomyxoma peritonei.

106
Clear cell tumor (choice B) is a malignant ovarian epithelial tumor that has clear-appearing
cytoplasm due to glycogen content. It is rare and is usually associated with endometriosis and
coexistent endometrial carcinoma. They do not lead to pseudomyxoma peritonei.
Endometrioid tumor (choice C) is a neoplasm of ovarian surface epithelium that appears
histologically like endometrial glands and is almost always malignant. It is associated with
endometriosis and coexistent endometrial carcinoma. These do not lead to pseudomyxoma
peritonei.
Serous cystadenoma (choice E) is the most common tumor of ovarian surface epithelium. It is
usually cystic and filled with serous fluid. It may be benign, borderline, or malignant and
occur mostly between 20-50 years of age. Malignant tumors are called serous cystadenocarcinomas
and show more papillary structures with invasion. They do not lead to pseudomyxoma peritonei.
A 7-year-old girl contracts chicken pox from her sister. Five days after the onset of the rash, she
begins vomiting and becomes lethargic. She is seen in the emergency room, and admitted to the
hospital, after which she becomes comatose. Laboratory examination is significant for elevated
liver enzymes and ammonia. Which of the following is the most likely diagnosis?
A. Crigler-Najjar syndrome
B. Dubin-Johnson syndrome
C. Gilbert syndrome
D. Reye syndrome
E. Rotor syndrome
Explanation:
The correct answer is D. Reye syndrome (fatty liver with encephalopathy) is an acute (and
potentially fatal) post-viral injury that is characterized by severe mitochondrial damage
affecting the liver, brain, skeletal muscle, heart, and kidneys. Most patients are children,
although adult cases have been described. Varicella and influenza A and B are the most common
precipitating illnesses. Aspirin use has been linked to the development of this disorder, but
cases occur in the absence of salicylate ingestion.
Crigler-Najjar (choice A) syndrome is a rare, mild-to-severe form of inherited unconjugated
hyperbilirubinemia.
Dubin-Johnson syndrome (choice B) is an inherited conjugated hyperbilirubinemia associated with
a darkly pigmented liver.
Gilbert syndrome (choice C) is a relatively common, benign form of inherited unconjugated
hyperbilirubinemia.
Rotor syndrome (choice E) resembles Dubin-Johnson syndrome, but is associated with a normalcolored liver.
A 30-year-old man with a 15-year history of ulcerative colitis develops intermittent cholestatic
jaundice. Ultrasonographic examination fails to reveal gallstones. Liver biopsy demonstrates a
large bile duct obstruction. Which of the following would most likely be seen on endoscopic
retrograde cholangiopancreatography (ERCP)?
A. Beading of intrahepatic bile ducts

107

B. Markedly dilated common bile duct containing irregular radiolucent masses


C. Mass at the ampulla of Vater
D. Moderately dilated intrahepatic bile ducts and stricture in the bile duct at the
porta hepatis
E. Very dilated biliary tree terminating in a blunt, nipple-like obstruction at the
lower end of the common bile duct
Explanation:
The correct answer is A. The most likely diagnosis is primary sclerosing cholangitis, a
disorder with a probable autoimmune component that is associated with ulcerative colitis (2/3
of primary sclerosing cholangitis patients have a history of ulcerative colitis). The disease
is characterized by inflammation and fibrosis of the intrahepatic and extrahepatic bile ducts,
producing alternating strictures and dilatation of the structures. These changes are seen as
"beading" on endoscopic retrograde cholangiopancreatography.
Gallstones in the biliary tree produce irregular radiolucent masses (choice B).
Choice C describes the findings associated with carcinoma of the ampulla of Vater.
Choice D describes the findings associated with carcinoma of the extrahepatic bile ducts.
Choice E describes the findings associated with carcinoma of the pancreas.
A 17-year-old girl in the 4th month of pregnancy presents with vaginal bleeding, accompanied by passage
of grapelike tissue fragments. Plasma human chorionic gonadotropin (hCG) is very high.
Following ultrasound evaluation, dilatation and curettage is performed. Histologic examination
shows diffuse trophoblastic proliferation and edema in all chorionic villi. Cytogenetic studies
demonstrate 46, XY karyotype. Clinical studies show that this condition progresses to
choriocarcinoma at a rate of
A. 0.2%
B. 2%
C. 20%
D. 50%
E. 90%
Explanation:
The correct answer is B. The condition described is a complete hydatidiform mole. Hydatidiform
mole is a spectrum of conditions due to abnormal proliferation of trophoblast and ranging from
entirely benign conditions (complete and partial mole) to very aggressive neoplasms
(choriocarcinoma). Partial moles have a triploid karyotype (69,XXY) and derive from
fertilization of an ovum with one or two sperm; fetal parts are frequently seen. Complete moles
have a diploid karyotype (46,XX or 46,XY) and derive from fertilization of an empty ovum by a
single sperm (androgenesis); no fetal parts are found. Ten percent of moles acquire a more
aggressive behavior and invade deeper parts of the myometrium, degenerating into invasive
moles. Two percent of complete moles may transform into choriocarcinoma, a very aggressive

108
neoplasm that spreads rapidly through the blood stream. Fortunately, choriocarcinomas of
placental origin are highly responsive to chemotherapy, affording up to 100% cure or remission.
Nongestational choriocarcinoma, in contrast, is poorly responsive to chemotherapy.
Remember the following important points:
-Complete mole, usually diploid karyotype: results from androgenesis (sperm fertilization of an
empty egg), no fetal parts are identified, 2% progress to choriocarcinoma.
-Partial mole, usually triploid karyotype: results from fertilization of normal egg by one or
two sperm; fetal parts are often identified; rarely progresses to choriocarcinoma.
A 4-year-old girl is brought to a specialty clinic by her foster parents because of a limp in her right
leg and a serum alkaline phosphatase (ALP) that is 10 times the adult upper limit of normal for
the test. The child is withdrawn and clinging to her foster mother. Physical examination is
remarkable for a blue-green discoloration of the skin overlying a tender, 3-cm mass on the
anterior portion of the right mid-thigh. An x-ray of the right thigh reveals a dense mass within
the femur that extends into the surrounding tissue. Which of the following best describes the
increased alkaline phosphatase and physical findings in this case?
A. ALP is abnormal for her age and the bone lesion represents an area of aseptic
necrosis
B. ALP is abnormal for her age and the bone lesion represents repair of a femoral
fracture
C. ALP is abnormal for her age and is unrelated to the bone lesion
D. ALP is normal for her age and the bone lesion is an osteogenic sarcoma
E. ALP is normal for her age and is unrelated to the bone lesion
Explanation:
The correct answer is B. The reference intervals for serum alkaline phosphatase (ALP) and serum
phosphate are higher in children than in adults. Growing bone in children requires alkaline
phosphatase secreted by osteoblasts and an increase in serum phosphate to provide a proper
solubility product ([calcium] x [phosphorus]) for mineralization of bone. However, the normal
serum ALP in children is no greater than 5-times the upper limit of normal for adults, hence
this patient's ALP is abnormally elevated. The most likely scenario is that the child is a
victim of abuse and has suffered a femoral fracture that is healing. Bone repair further
increases the serum ALP owing to the increase in osteoblastic activity.
Aseptic necrosis of bone in the leg (choice A) is more likely to occur in the femoral head
(Legg-Calve-Perthes disease). This disease is more common in boys than girls and occurs in the
3- to 10-year-old age bracket. Although aseptic necrosis is associated with dense bone
formation, it does not extend out into the soft tissue nor would it produce skin discoloration
directly over the lesion (due to trauma in this case).
Since the elevation in serum ALP is increased for age and related to the bone lesion, choice C
is incorrect.
Since the serum ALP is not normal for the age of this patient, choices D and E are incorrect.
Furthermore, osteogenic sarcoma (choice D) does not normally occur in the first decade and
favors the metaphysis of the distal femur or proximal tibia.

109
A six-month-old baby girl who was normal at birth, begins to show signs of motor retardation. While she
could sit up at 5 months, she can no longer do so. As time goes on, the child continues to
deteriorate, and eventually becomes unresponsive to visual or auditory stimuli. Funduscopic
examination reveals cherry-red macular spots in both eyes. Which of the following genetic
abnormalities is most often related to the development of this disease?
A. Confined placental mosaicism
B. Expanded trinucleotide repeat
C. Frameshift mutation
D. Nondisjunction
E. Translocation
Explanation:
The correct answer is C. Most cases of Tay-Sachs disease are caused by insertion of four
nucleotides into the coding region of the gene for the alpha-subunit of the hexosaminidase A
gene. This produces a frameshift mutation, resulting in the creation of a premature stop codon
downstream from the inserted nucleotides. The disease presents as described in the question,
and is more common in Ashkenazi Jews. The carrier rate in this population is 1 in 25. There is
presently no effective treatment and survival beyond four years of age is rare.
Confined placental mosaicism (choice A) is caused by a genetic aberration (e.g., a trisomy)
occurring in the trophoblast or extraembryonic progenitor cells in the inner cell mass of the
embryo. This leads to abnormal placental development and intrauterine growth retardation.
Expansion of tandem nucleotide repeats (choice B) is thought to be etiologically involved in
Fragile X syndrome and Huntington disease.
Nondisjunction (choice D) during meiosis is a common cause of trisomy, e.g., trisomy 21, or
Down syndrome.
Translocations (choice E) are frequently involved in leukemias, lymphomas, and solid tumors. A
minority of cases of Down syndrome are actually due to a translocation, rather than trisomy.
A 54-year-old African-American female patient undergoes a routine insurance physical examination. Chest
x-ray reveals bilateral hilar masses. Biopsy of the masses shows granulomata, but acid-fast and
fungal stains are negative for organisms. Which of the following diseases should be suspected?
A. Caroli's disease
B. Raynaud's disease
C. Sarcoidosis
D. Scleroderma
E. Systemic lupus erythematosus
Explanation:
The correct answer is C. Sarcoidosis is a multisystem disease characterized by non-caseating
granulomata in a variety of organs. The disease may be symptomatic (respiratory and

110
constitutional symptoms) or may be discovered incidentally when chest x-ray or autopsy reveals
bilateral hilar adenopathy. Definitive diagnosis is based on biopsy, which reveals noncaseating granulomata that are negative for fungi or acid-fast bacilli. Sarcoidosis is more
common in individuals of African descent.
Caroli's disease (choice A) is a congenital malformation of the bile duct system.
Raynaud's disease (choice B) is a vasospasm of vessels that causes temporary ischemia in the
hands.
Scleroderma (choice D), or progressive systemic sclerosis, is characterized by progressive
fibrosis of skin and internal organs.
Systemic lupus erythematosus (choice E) is an autoimmune disease characterized by vasculitis
(which may produce a variety of symptoms depending on the site of the lesion), rash, renal
disease, hemolytic anemia, and neurologic disturbances.
During laparoscopy performed for other reasons, a surgeon is startled to see a uniformly black liver.
Which of the following is the most likely diagnosis?
A. Angiosarcoma
B. Dubin-Johnson syndrome
C. Hemochromatosis
D. Malignant melanoma
E. Wilson's disease
Explanation:
The correct answer is B. Dubin-Johnson syndrome is a relatively mild, hereditary conjugated
hyperbilirubinemia that has the distinctive feature of causing the liver to turn black. The
same black pigment can be seen in microscopic sections. The origin of the black pigment is not
fully understood, but the genetic defect responsible for Dubin-Johnson syndrome has recently
been identified, and appears to involve a defect in the canalicular organic anion transporter.
Patients homozygous for Dubin-Johnson syndrome may also have significant amounts of
unconjugated bilirubin in the serum, presumably due to deconjugation of conjugated bilirubin in
the hepatobiliary system.
Angiosarcoma (choice A) can produce a dark red liver due to increased blood and hemorrhage.
Hemochromatosis (choice C) can produce a dark red to brown liver.
Malignant melanoma (choice D) can produce black nodules in the liver, but would not be expected
to produce a uniformly black liver.
The liver in Wilson's disease (choice E) can be cirrhotic but does not have a distinctive
color.
A 64-year-old man begins to show behavioral changes and irritability, and is found wandering in the park
near his home. On neurological examination, there is evidence of mild aphasia and cognitive
dysfunction, but motor function is preserved. CT scan of the head demonstrates selective
atrophy of the cortex of the frontal lobes. Which of the following is the most likely
diagnosis?

111

A. Alzheimer's disease
B. Friedreich's ataxia
C. Huntington's disease
D. Parkinson's disease
E. Pick's disease
Explanation:
The correct answer is E. Pick's disease is a condition that is clinically similar to
Alzheimer's disease. It differs from Alzheimer's disease in that the pronounced brain atrophy
characteristically involves the frontal and temporal lobes, with sparing of the posterior
aspects of the cortex. Microscopically, the affected cortex contains characteristic ballooned
neurons (Pick cells) or cytoplasmic inclusions (Pick bodies). Clinically, there is a slowly
progressive dementia with language disturbances and behavioral changes that may eventually lead
to mutism. The progressive nature of the dementing process has been termed a "descent into a
sea of mindlessness."
Alzheimer's disease (choice A) is characterized by dementia, but with diffuse brain atrophy.
Friedreich's ataxia (choice B) characteristically affects the spinal cord, brainstem, and
cerebellum, rather than cortex.
Huntington's disease (choice C) characteristically affects the caudate nucleus and putamen.
Motor symptoms such as choreoathetosis are prominent.
Parkinson's disease (choice D) characteristically affects the substantia nigra, and produces
prominent motor symptoms, including cogwheel rigidity, a resting tremor, and a festinating
gait.
A neonate has a 10-cm neck mass. She is diagnosed with a cystic hygroma of the neck. The physician is
concerned that she may also have which of the following abnormalities?
A. Aneurysm of the ascending portion of the aortic arch
B. Aortic dissection
C. Coarctation of the aorta
D. Cystic medial necrosis of the aorta
E. Early severe aortic atherosclerosis
Explanation:
The correct answer is C. You need to know that cystic hygromas (lymphatic malformations
resembling hemangiomas) are a feature of Turner syndrome that later contributes to the better
known "webbed neck." Turner syndrome is also associated with coarctation of the aorta, and to a
lesser extent, pulmonary stenosis.
Aneurysms involving the proximal aorta (choice A) suggest tertiary syphilis. The clues given in
a question about tertiary syphilis might be sexually transmitted disease or chronic

112
neurological signs.
Aortic dissection (choice B) and cystic medial necrosis of the aorta (choice D) would probably
be presented in a question giving you a clue about Marfan's syndrome, such as a patient with
tall stature and hyperextensible joints.
Early severe aortic atherosclerosis (choice E) can be a feature of either diabetes mellitus
(clues would include high fasting blood glucose, thirst, excessive urination, etc.) or familial
hyperlipidemias (clues include xanthomas or elevated serum lipids).
1) A 31-year-old HIV-positive man develops a severe pneumonia. Lower respiratory tract secretions
obtained by fiberoptic bronchoscopy with bronchoalveolar lavage and stained with methenamine
silver stain demonstrate cup-shaped cysts with sharply outlined walls. Which of the following
organisms is the most likely pathogen in this case?
A. Candida albicans
B. Giardia lamblia
C. Haemophilus influenzae
D. Pneumocystis carinii
E. Streptococcus pneumoniae
Explanation:
The correct answer is D. The organism described is Pneumocystis carinii, which is an
opportunistic parasite that appears to be more closely related to fungi than to protozoa. Its
cyst form, when stained with silver stains, has the distinctive appearance described in the
question stem, and is typically found in frothy material that occupies the lumen of alveoli. The
trophozoites are smaller and much harder to recognize. Bronchoalveolar lavage is considered much
more reliable than induced sputum as a diagnostic specimen. Pneumocystis pneumonia is a common
infection among AIDS patients, and is very uncommon in other clinical settings. Formerly, many
AIDS patients died with Pneumocystis pneumonia, but the combination of early drug treatment
(with trimethoprim/sulfamethoxazole or pentamidine) and prophylaxis (usually with
trimethoprim/sulfamethoxazole) has decreased the number of fatal infections. In severe cases,
Pneumocystis infection can sometimes be demonstrated in extrapulmonary sites.
Candida albicans(choice A) can infect the lung and stain with methenamine silver, but the
description of the lavage material would probably include the terms fungal hyphae and yeast
forms.
Giardia lamblia (choice B) causes diarrhea, rather than pneumonia.
Haemophilus influenzae(choice C) and Streptococcus pneumoniae(choice E) are bacteria and would
not stain with silver stains.
2) A normal birthweight, term baby with high APGAR scores fails to pass meconium within 36 hours of
birth. The neonate also has a distended abdomen and has been vomiting and feeding poorly.
Digital rectal examination temporarily relieves the obstruction, but the baby fails to pass
stool thereafter. Barium enema examination demonstrates a very narrow distal segment of rectum
with proximal dilation. Abnormalities of which of the following are most likely etiologically
related to this baby's disorder?

113
A. Chloride channels
B. Ganglion cells
C. Mucosal cells
D. Smooth muscle cells
E. Vagus nerve
Explanation:
The correct answer is B. The disease is Hirschsprung's disease, which is a congenital cause of
constipation caused by an absence of ganglion cells in both the submucosal and intermyenteric
plexus of a segment of bowel. The aganglionic bowel segment is narrowed because the lack of
peristalsis keeps stool from moving into the segment. The distal rectum is always involved, and
the lesion can extend proximally anywhere from a few centimeters past the rectum all the way up
to the small intestine. The bowel proximal to the lesion is usually dilated. In this patient's
case, rectal examination dilated the narrowed aganglionic bowel, temporarily allowing passage
of stool. Definitive treatment consists of surgical removal of the affected segment.
Failure to pass meconium is also characteristic of cystic fibrosis, a disorder of chloride
channels (choice A). The characteristic radiologic appearance of the bowel in this case
strongly suggests Hirschsprung's disease.
The mucosa (choice C) is not directly affected by Hirschsprung's disease, although a lifethreatening (20% mortality) secondary enterocolitis may develop.
Smooth muscle (choice D) changes are not usually apparent on biopsy of aganglionic segments of
bowel in Hirschsprung's disease.
Abnormalities of the vagus nerve (choice E) are not related to the aperistalsis in
Hirschsprung's disease.
3)A 20-year-old athlete dies suddenly during a basketball game. Autopsy reveals asymmetric thickening of
the interventricular septum at the level of the mitral valve. Microscopically, the myocytes in
the area are hypertrophied and arranged in a haphazard pattern. Which of the following is the
most likely diagnosis?
A. Alcoholic cardiomyopathy
B. Cardiac amyloidosis
C. Endocardial fibroelastosis
D. Idiopathic hypertrophic subaortic stenosis
E. Loeffler's endocarditis
Explanation:
The correct answer is D. The lesion described is hypertrophic cardiomyopathy, more specifically
known as idiopathic hypertrophic subaortic stenosis (IHSS). This lesion is usually seen in
young adults, although it can also be present in the elderly. A genetic predisposition
(autosomal dominant) may be present. Some patients with this condition may experience dyspnea,
angina, dizziness, or congestive heart failure. Other patients are asymptomatic until they

114
undergo sudden death, usually during strenuous exercise, possibly because the aortic outlet
becomes completely occluded as a result of muscle contraction. Infective endocarditis of the
adjacent (damaged) mitral valve and atrial fibrillation may also occur.
Alcohol abuse can produce a dilated cardiomyopathy (choice A).
Severe cardiac amyloidosis (choice B), endocardial fibroelastosis (choice C), and Loeffler's
endocarditis (choice E) can all produce a restrictive cardiomyopathy.
A 34-year-old man with AIDS suddenly falls to the floor and has a tonic-clonic seizure. His concerned
friends call paramedics, who take him to the hospital. On arrival at the hospital he is
conscious, but confused. Physical examination is remarkable for cachexia and oral thrush.
Neurological examination reveals isolated weakness of lateral gaze on the right. MRI reveals
multicentric mass lesions in the brain and meninges. One of the masses is biopsied and
appropriate immunohistochemical stains are performed. From which of the following cell types did
the masses most likely derive?
A. Astrocyte
B. B lymphocyte
C. Ependymal cell
D. Melanocyte
E. Oligodendrocyte
Explanation:
The correct answer is B. Approximately one-third of AIDS patients will experience significant
morbidity from neurologic disease. Primary CNS lymphoma is rare in the general population, but
is a relatively common type of lymphoma in AIDS patients. Presenting symptoms include seizures,
headache, and cranial nerve deficits (e.g., the abducens nerve palsy in this patient). The
lesions are often multicentric, and may involve the leptomeninges in a significant number of
cases. Primary CNS lymphoma is typically a late manifestation of AIDS (median CD4 count 40/l).
This lymphoma is usually an intermediate-to-high-grade B cell lymphoma; evidence of EpsteinBarr virus infection is commonly present.
Astrocytoma is a neoplasm derived from cells in the astrocyte (choice A) lineage. These are not
particularly associated with AIDS.
Ependymal cells line the ventricles. Tumors derived from ependymal cells (choice C), called
ependymomas, characteristically produce rosettes or perivascular pseudorosettes,
microscopically. The incidence of ependymoma is not increased in AIDS patients.
Melanoma is a neoplasm arising from transformation of melanocytic cells (choice D). Whereas the
brain is a favorite site for metastatic melanoma, the incidence of melanoma is not particularly
increased in AIDS patients.
Oligodendrocytes (choice E) are responsible for producing myelin in the central nervous system.
Neoplasms arising from cells in the oligodendrocyte lineage are called oligodendrogliomas.
Calcification is frequently present, and may be visible on CT scan. These neoplasms are not
more common in AIDS patients.
In which of the following ways does seminoma differ most significantly from dysgerminoma?

115
A. Most common age of presentation
B. Number of mitoses
C. Potential to contain foci of more aggressive tumors
D. Radiosensitivity
E. Ultrastructural appearance
Explanation:
The correct answer is A. Both men and women can develop a distinctive germ cell lesion that is
called a seminoma if it involves a testis and a dysgerminoma if it involves an ovary. Seminomas
and dysgerminomas are very similar tumors but differ in two significant respects: the most
common age of presentation in men is in the fourth decade, while in women it is in the third
decade; and seminomas are relatively common in men while dysgerminomas are rare in women (1% of
ovarian tumors).
Both of these tumors are composed of sheets of uniform polyhedral cells with intervening
fibrous septa of connective tissue, lymphocytes and multinucleated giant cells. Both tumors are
similar ultrastructurally (choice E), and the number of mitoses (choice B) is comparable
between the two tumors. These tumors in pure form are very radiosensitive (choice D), with a
90% 5-year survival, but can be much more aggressive if foci of other germ cell tumors (notably
embryonal carcinoma, choriocarcinoma and yolk sac tumors) are present (choice C).
A 35-year-old woman consults a physician because of fatigue and "feeling cold all the time". On
examination, her thyroid gland is diffusely enlarged and rubbery. Thyroid studies show low T3
and T4 and high TSH. Thyroid aspiration demonstrates large numbers of lymphocytes of all
degrees of maturation, a few abnormal follicular cells with eosinophilic granular cytoplasm,
and only rare normal follicular cells. Hyperplasia and neoplastic proliferation of which of the
following tissues or organs may be associated with this patient's disorder?
A. Colon
B. Esophagus
C. Peripheral nerve
D. Skin
E. Thymus
Explanation:
The correct answer is E. The thyroid disease is Hashimoto's thyroiditis, an autoimmune disease
in which the thyroid parenchyma is destroyed by a lymphocytic infiltrate. The infiltrate
typically contains mature follicles; the remaining scanty follicular cells often have
eosinophilic granular cytoplasm and are called Hurthle cells or oncocytes. Clinically, patients
usually have hypothyroidism, although brief periods of hyperthyroidism ("Hashitoxicosis") may
also be seen. Like myasthenia gravis, Hashimoto's disease may be associated with thymic
disorders including thymic hyperplasia, benign thymomas, and malignant thymomas.
Colon (choice A) cancer is associated with ulcerative colitis and adenomatous polyps.
The risk of esophageal cancer (choice B) is increased with Barrett's esophagus and in Plummer-

116
Vinson syndrome.
You should associate neurofibromas of peripheral nerve (choice C) with caf au lait spots on
the skin.
Skin cancer (choice D) occurs with greater frequency in association with xeroderma pigmentosa
and actinic keratosis.
A 25-year-old man undergoes orchiectomy for a testicular tumor. Microscopically, the tumor shows a
variety of patterns, including lobules containing large cells with watery cytoplasm, structures
resembling primitive glomeruli, syncytiotrophoblast, and shafts of undifferentiated cells with
focal glandular differentiation. Which of the following terms most accurately describes this
tumor?
A. Choriocarcinoma
B. Embryonal carcinoma
C. Mixed tumor
D. Seminoma
E. Yolk sac tumor
Explanation:
The correct answer is C. This is a mixed testicular tumor. Mixed patterns occur in 60% of
testicular tumors, with the most common pattern being teratoma (not present in this patient),
embryonal carcinoma, yolk sack tumor, and hCG-containing syncytiotrophoblast. This patient's
tumor also contains elements of seminoma. The prognosis of these tumors is determined by the
presence or absence of more aggressive elements, notably choriocarcinoma.
Choriocarcinoma (choice A) contains both syncytiotrophoblast and cytotrophoblast.
Embryonal carcinoma (choice B) is characterized by sheets of undifferentiated cells with focal
glandular differentiation.
Seminoma (choice D) shows lobules containing large cells with watery cytoplasm.
In about half the cases, yolk sac tumor (choice E) contains structures resembling primitive
glomeruli, known as endodermal sinuses.
A 71-year-old man has been in excellent health, and practicing competently as an attorney. He is brought
to the emergency room following a motor vehicle accident. A workup, including imaging of the
spine, thorax, and head, is negative, but the patient is admitted for overnight observation.
His injuries include several lacerations to the face and extremities as well as several
contusions to the thorax. Three weeks later, he is admitted to the hospital for confusion. A
neurologic exam is normal except that he is not oriented to time or place, and can recall only
1 out of 6 objects after 3 minutes. Which of the following is the most likely diagnosis?
A. Alzheimer's disease
B. Brain metastases
C. Epidural hematoma

117
D. Normal pressure hydrocephalus
E. Subdural hematoma
Explanation:
The correct answer is E. This is a classic history and presentation of a subdural hematoma,
which usually occurs several days to weeks following a traumatic head injury. Often the patient
will have had a hospital admission with a negative workup, including x-rays and CT scans of the
head. Either gradually or abruptly, the patient can experience decreasing mental status,
sleepiness, focal neurologic deficits, seizures, etc. The bleeding originates from the bridging
veins of the meninges and blood collects beneath the dural layer. A CT or MRI of the brain will
show a semicircular, crescent-shaped opacity (blood) just below the cranium impinging on the
brain tissue.
Alzheimer's disease (choice A) usually develops more gradually, typically over several years
(although some cases can develop over a few months).
Brain metastases (choice B) could very well present like this, however, the patient will often
have constitutional symptoms of cancer such as fatigue and weight loss. In a patient with a
recent traumatic event, however, subdural hematoma is more likely.
An epidural hematoma (choice C) differs from a subdural hematoma in that it occurs much more
rapidly, usually over several hours or less. The bleed originates from the middle meningeal
artery, which runs right next to the anterolateral cranium and often gets injured with a skull
fracture in this location. A CT or MRI will show a circular opacity within the cranium
compressing the brain tissue.
Normal pressure hydrocephalus (choice D) occurs in the elderly and is associated with urinary
incontinence, new onset of confusion, and gait disturbances.
A biopsy of a brain tumor from a 42-year-old man reveals a glial neoplasm consisting of atypical
astrocytes with scattered mitoses. Besides mitotic activity, which of the following markers can
provide information about the neoplasm's proliferative activity?
A. bcl-2
B. Glial fibrillary acidic protein (GFAP)
C. Ki-67
D. p53
E. Ubiquitin
Explanation:
The correct answer is C. Ki-67 is a nuclear factor (of uncertain function) whose expression
correlates with neoplastic replicative activity. Its expression can be visualized by
immunostaining of formalin-fixed, paraffin-embedded sections. Ki-67 labeling correlates with a
neoplasm's rate of growth and, therefore, with prognosis.
The bcl-2 gene (choice A) suppresses apoptosis by different mechanisms. Its abnormal activation
is involved in the pathogenesis of low-grade lymphomas but not astrocytomas.
GFAP (choice B) is an intermediate cytoskeletal filament (analogue of keratin and vimentin)

118
expressed exclusively by certain types of glial cells, e.g., astrocytes and ependymal cells.
Immunohistochemistry for GFAP is used diagnostically to confirm an astrocytic origin of a
neoplasm, but gives no information about mitotic or proliferative rate.
The gene p53 (choice D) encodes a protein that blocks the cell cycle when damage to DNA occurs.
If the damage is successfully repaired, p53 allows the cell cycle to resume; if not, p53
induces apoptosis, thus eliminating dangerous DNA mutations. Mutations of p53 have been found
in the great majority of human neoplasms, including gliomas. However, its expression gives no
information concerning neoplastic replicative activity.
Ubiquitin (choice E) is a low-molecular-weight heat-shock protein. Its function is to tag
aberrant proteins for degradation. It is present in many abnormal intraneuronal inclusions
associated with neurodegenerative disorders, such as Lewy bodies, Pick bodies, and
neurofibrillary tangles. It has no relationship with mitotic activity or growth rate.
Examination of a peripheral smear from a patient with a hematocrit of 30% demonstrates large numbers of
target cells. Further work-up of this patient would most likely reveal which of the following
abnormalities?
A. Abnormal hemoglobin synthesis
B. Abnormal red cell cytoskeleton
C. Cells that sickle when exposed to low oxygen tension
D. Mineral deficiency
E. Vitamin deficiency
Explanation:
The correct answer is A. Target cells are red cells with a peripherally dense rim of hemoglobin
enclosing a zone of pallor, often with a darker central punctum. They appear as a result of an
increase in the area of the red cell membrane, and/or a decrease in hemoglobin. Target cells
are most commonly seen in three conditions: thalassemia, hemoglobin C disease, and liver
disease. Of the choices given, only thalassemia involves abnormalities of hemoglobin synthesis
(either the alpha or the beta chain).
Hereditary spherocytosis is an abnormality of red cell cytoskeleton (choice B).
Sickle cell anemia and sickle cell trait are diseases characterized by sickling of cells when
exposed to low oxygen (choice C).
The microcytic anemia of iron deficiency is the best example of an anemia due to mineral
deficiency (choice D).
The macrocytic anemias of B12 and folate deficiencies are examples of anemias related to
vitamin deficiency (choice E).
A pathologist is examining five placentas from five different births. He notes the following
characteristics in the five placentas:
Patient A: fused dichorionic diamnionic
Patient B: dichorionic diamnionic

119
Patient C: circumvallate placenta
Patient D: monochorionic diamnionic
Patient E: bipartite placenta
Which of the patients unquestionably gave birth to identical twins?
A. Patient A
B. Patient B
C. Patient C
D. Patient D
E. Patient E
Explanation:
The correct answer is D. Patient D is the lucky mother of identical (monozygotic) twins. The
chorion forms before the amnion, so the possible combinations are monoamnionic and
monochorionic; diamnionic and monochorionic; and diamnionic and dichorionic (either fused or
separated). The first two of these possibilities are seen only in identical twins. However, the
latter can be seen either in fraternal or identical twins. Exactly what happens in the case of
identical twins depends on the precise point at which twinning occurs: very early separation
produces completely separate membranes with duplication of both chorion and amnion; somewhat
later separation produces one chorion and two amnions; and very late separation produces one
chorion and one amnion. A dichorionic, diamnionic placenta develops if splitting occurs early
after fertilization, before the chorion forms. This type of placenta may occur with either
monozygotic or dizygotic twins. Thus, we are unable to determine whether Patient A (choice A)
or Patient B (choice B) had identical twins from the examination of the placentas.
Neither Patient C (choice C) nor Patient E (choice E) had a twin pregnancy. A circumvallate
placenta has an extrachorial portion outside the insertion of the amnionic membranes, creating
a circumferential groove (vallum in Latin). A bipartite placenta is composed of two equal
segments. Circumvallate and bipartite placentas are examples of placental abnormalities that
usually have no clinical significance.
A 42-year-old paraplegic woman has a neurogenic bladder and requires an indwelling urinary catheter. She
develops a urinary tract infection and is seen by a urologist, who orders abdominal x-rays and
an intravenous pyelogram. The radiographic studies demonstrate a very large stone that fills
and follows the contours of the renal pelvis. The stone is most likely composed of which of the
following?
A. Calcium salts
B. Cholesterol
C. Cystine
D. Magnesium ammonium phosphate
E. Uric acid
Explanation:

120

The correct answer is D. Staghorn calculi, such as described in the question, are associated
with infection by urea-splitting bacteria (notably Proteus species) and are composed of
magnesium ammonium phosphate (struvite). They are more common in women than men, and are far
more common in patients requiring chronic bladder catheterization.
Calcium-containing stones (choice A) are typically seen in patients with hypercalcinuria
without hypercalcemia; one-fifth of patients with this type of stone have hyperuricosuria.
Cholesterol (choice B) is found in gall stones, not kidney stones.
Cystine stones (choice C) are rare; they may be seen in patients with cystinuria, an autosomal
recessive disorder of amino acid metabolism.
Uric acid stones (choice E) are seen in gout, leukemia, and in patients with acidic urine.
A radiologist notes the presence of fine, radiographically dense crystals in the tissues of a knee
joint. This patient most likely has which of the following types of arthropathy?
A. Gonococcal arthritis
B. Gouty arthritis
C. Osteoarthritis
D. Pseudogout
E. Rheumatoid arthritis
Explanation:
The correct answer is D. Radiographically dense calcium pyrophosphate dihydrate (CPPD) crystals
are deposited in cartilage and joint soft tissues in pseudogout, which can involve the knees,
wrists, elbows, shoulders, or ankles. If the patient is asymptomatic because the deposition is
primarily within cartilage, the condition is sometimes called chondrocalcinosis. In its more
severe form, the joint involvement can clinically resemble rheumatoid arthritis.
Neither gonococcal arthritis (choice A), osteoarthritis (choice C), nor rheumatoid arthritis
(choice E) are associated with crystal formation.
The monosodium urate crystals deposited in joints in gouty arthritis (choice B) are not radioopaque.
In a hindsight study, the progeny of woman who took diethylstilbestrol had an increased risk for
developing which of the following cancers?
A. Bladder cancer
B. Endometrial carcinoma
C. Ovarian carcinoma
D. Renal cell carcinoma
E. Vaginal clear cell adenocarcinoma

121
Explanation:
The correct answer is E. Diethylstilbestrol, or DES, is an estrogen analog that was prescribed
in the 1950s-1970s for prevention of spontaneous abortion. The progeny of woman taking DES were
often diagnosed with clear cell vaginal or cervical adenocarcinoma, which is rarely seen in
unexposed women.
Bladder cancer (choice A) is often seen in individuals exposed to beta-naphthalene dyes or
phenacetin.
Endometrial carcinoma (choice B) is seen in individuals on long-term estrogen therapy, but was
not seen in the progeny of DES-treated women.
Ovarian carcinoma (choice C) is increased in nulliparous women, and in those with a family
history of the disorder.
Renal cell carcinoma (choice D) has been linked to such epidemiologic factors as smoking, and
the incidence is increased in patients with von Hippel-Lindau syndrome.
A 45-year-old man presents to the emergency department with severe headache and vomiting. A CT scan
shows a well-circumscribed cystic lesion within the 3rd ventricle; there is no calcium
deposition. The cyst is surgically removed. On histologic examination, the wall of the cyst
consists of a single layer of mucin-producing columnar epithelium with a ciliated apical
surface. Which of the following is the most likely diagnosis?
A. Colloid cyst
B. Craniopharyngioma
C. Cysticercosis
D. Echinococcus cyst
E. Pilocytic astrocytoma
Explanation:
The correct answer is A. This patient has symptoms of increased intracranial pressure, ie,
nausea, vomiting, and headache. This pressure is due to the location of the mass in the 3rd
ventricle, which is blocking the flow of cerebrospinal fluid from the lateral ventricles into
the 3rd ventricle through the foramen of Monro. All cysts and tumors listed here may be found
in the 3rd ventricle, but the histologic features are consistent with a colloid cyst. Colloid
cysts are thought to derive from embryologically misplaced endodermal epithelium, which
explains the presence of mucin-producing ciliated cells similar to respiratory epithelium. The
cyst contains mucinous fluid. It usually presents with these type of symptoms in adults,
especially middle-aged patients.
Craniopharyngioma (choice B) is a neoplasm arising in the suprasellar region of children and
young adults. It derives from misplaced odontogenic epithelium; indeed, the tumor is
histologically identical to the most common tumor of teeth, adamantinoma. Craniopharyngiomas
contain abundant calcium deposits. Although histologically benign, these tumors frequently
recur since complete surgical excision is rarely feasible.
A cysticercus is a cyst that develops following ingestion of Tenia solium eggs in undercooked
pork. The brain is one of the preferred sites of cysticercosis (choice C), and cysticerci may
develop within the brain (intraparenchymal), within the ventricles, or in the subarachnoid

122
space. The lining of a cysticercus, however, is entirely different from a colloid cyst: it
consists of three cell layers, the most superficial of which resembles dome-shaped urothelial
cells.
An Echinococcus cyst (choice D) is caused by infestation from another cestode parasite,
Echinococcus granulosus, which is acquired from dog feces. The defining feature of an
Echinococcus cyst is a tiny, hooklike projection called a scolex. Scolices detach from the
wall, accumulating within the cyst and producing a sandlike sediment. Liver and lungs are the
usual locations of echinococcosis.
Pilocytic astrocytoma (choice E) is a tumor of astrocytic origin that affects children and
young adults. It usually develops either in the cerebellar hemispheres or in the hypothalamic
region. In the latter location, it may grow within the 3rd ventricle. Pilocytic astrocytomas
often have a cystic structure. Histologically, the tumor consists of elongated astrocytes
within an abundant fibrillary background and numerous Rosenthal fibers. The wall of the cystic
component is composed of glial (astrocytic) cells, usually of a reactive nature.
A 70-year-old man has smoked for years despite a chronic productive cough. One day, he notices blood in
his sputum and goes to see his physician. A chest x-ray reveals a mass in one lung, which is
biopsied during a fiberoptic bronchoscopy. Assuming that this patient's cancer is etiologically
related to his smoking, which of the following diagnoses are most likely to be returned by the
pathology laboratory?
A. Adenocarcinoma or bronchioloalveolar carcinoma
B. Adenocarcinoma or small cell carcinoma
C. Bronchioloalveolar carcinoma or small cell carcinoma
D. Bronchioloalveolar carcinoma or squamous cell carcinoma
E. Small cell carcinoma or squamous cell carcinoma
Explanation:
The correct answer is E. Smoking does not increase the risk of all types of lung cancers to the
same degree. Oat (small) cell carcinoma has a very strong association with smoking, with only
1% of cases occurring in nonsmokers. Squamous cell carcinoma is also strongly associated with
smoking, because smoking predisposes for squamous metaplasia, a precancerous condition. The
association of smoking with bronchogenic adenocarcinoma and with bronchioloalveolar carcinoma
is much weaker.
A 46-year-old female with rheumatoid arthritis develops progressively worsening renal failure. She
undergoes diagnostic renal biopsy, revealing thickening of the mesangial matrix and widened
capillary basement membranes due to deposition of an amorphous, eosinophilic material that
stains with Congo red. The material is shown to be composed of AA (amyloid-associated) protein
fibrils. In which of the following locations is this protein synthesized?
A. Bone marrow
B. Brain
C. Liver
D. Synovium

123
E. Thyroid
Explanation:
The correct answer is C. AA protein represents an enzymatically degraded form of SAA (serum
amyloid-associated) protein, which is synthesized in the liver. SAA is produced in response to
cytokines that are released in a number of inflammatory conditions, including chronic
infections, connective tissue disorders, and inflammatory bowel disease. It is not known why
only a minority of people with these conditions develop amyloidosis.
Bone marrow (choice A) and lymph nodes are the source of the AL (amyloid light chain) proteins,
which cause amyloidosis in plasma cell and B-cell proliferations (e.g., multiple myeloma). This
is the most common form of amyloidosis in the U.S. today.
Beta 2-amyloid protein (A&beta;2) is found in the brain (choice B) and forms the core of the
plaques in Alzheimer's disease.
The synovium (choice D) may be the site of chronic inflammation and pannus formation in
rheumatoid arthritis, but amyloid is not synthesized here.
In medullary carcinoma, excess calcitonin production leads to heavy amyloid deposition within
the thyroid gland (choice E), but this amyloid is not derived from AA protein.
A 62-year-old woman in the hospital for a hip fracture develops a deep venous thrombosis that embolizes.
The embolus lodges at the bifurcation of the pulmonary trunk, nearly completely occluding the
vessel. Which part of the heart would be most significantly and immediately affected by this
event?
A. Left main coronary artery
B. Left ventricle
C. Right atrium
D. Right main coronary artery
E. Right ventricle
Explanation:
The correct answer is E. A massive pulmonary embolus, such as this patient sustained, can
interrupt pulmonary blood flow, producing acute cor pulmonale with abruptly developing right
ventricular dilatation. Other parts of the heart are secondarily affected somewhat later.
Acute cor pulmonale is a surgical emergency requiring immediate correction of the underlying
problem, which is usually a pulmonary embolus lodged early in the pulmonary circulation. Acute
cor pulmonale is less common than chronic cor pulmonale, which is seen as a complication of
many chronic lung diseases.
The left main coronary artery (choice A) and right main coronary artery (choice D) would be
affected secondarily to the reduced blood flow to the left heart and aorta, from which the
coronary arteries arise.
The left ventricle (choice B) would be affected secondarily by reduced blood flow from the
pulmonary veins to the left atrium.
The right atrium (choice C) would be affected after the right ventricle since it is farther

124
from the circulatory block.
A 1-week-old, bottle-fed, low-birth-weight neonate develops severe abdominal pain with bloody diarrhea.
Several hours later, the neonate undergoes emergency surgery. A portion of small intestine is
resected, and pathologic examination demonstrates a perforation. Which of the following would
most likely be seen on microscopic examination of the involved bowel?
A. Inflammatory polyps
B. Multiple diverticula
C. Neoplastic polyps
D. Thickened collagenous band
E. Transmural necrosis
Explanation:
The correct answer is E. The disease is necrotizing enterocolitis, which is a common cause of
gastrointestinal emergency in premature and low-birth-weight infants. Typically, the infants
are bottle-fed and develop severe abdominal distress in the first week of life. Contributing
factors include intestinal ischemia, poor neonatal immune response, and microbial agents. Both
the small and large bowel may be affected. Necrotizing enterocolitis may be complicated by
intestinal gangrene, gastrointestinal bleeding, intestinal perforation, and sepsis. Survivors
of severe necrotizing enterocolitis may have had significant lengths of bowel surgically
removed, and later suffer from malabsorption and stricture formation. Histologically, the
appearance varies with disease stage, but typically shows varying degrees of necrosis
(transmural if perforation has occurred), inflammation, hemorrhage, and edema. A pseudomembrane
composed of coagulated fibrin, neutrophils, and cellular debris may overlie the mucosa.
Inflammatory polyps (choice A) can be seen following reepithelialization of ulcers, typically
in ulcerative colitis, which would not be seen in the first week of life.
Multiple diverticula (choice B) are seen in diverticulosis, which does not usually develop
before middle age.
Neoplastic (precancerous) polyps (choice C), even in familial syndromes with a high colonic
cancer rate, would not be expected to cause an acute abdominal emergency in a neonate.
A thickened collagenous band (choice D) between surface epithelial cells and the lamina propria
is seen with episodic watery diarrhea, usually in adults.
A 20-year-old man with new onset of seizures and no history of hypertension is evaluated with a
contrast-enhanced CT scan of the head, which demonstrates a mixed parenchymal and subarachnoid
hemorrhage. The parenchymal hemorrhage is centered over one cerebral hemisphere. Which of the
following is the most likely source of the hemorrhage?
A. Arteriovenous malformation
B. Berry aneurysm
C. Bridging vein
D. Charcot-Bouchard aneurysm

125
E. Middle meningeal artery
Explanation:
The correct answer is A. Arteriovenous malformations are composed of complex tangles of
congenitally malformed vessels that typically involve the superficial or deep cerebral
hemispheres. There is a slight male predominance, and bleeding typically occurs in adolescence
or young adulthood. Symptoms may be those of subarachnoid hemorrhage (headache, increased
intracranial pressure) and/or seizures. Surgical resection is usually required for therapy.
Berry aneurysms (choice B) can produce both subarachnoid and parenchymal hemorrhage, but are
usually centered near the base of the brain.
Bleeding from bridging veins (choice C) causes subdural hematoma.
Charcot-Bouchard aneurysms (choice D) are small, intraparenchymal aneurysms that are related to
hypertension.
Rupture of the middle meningeal artery (choice E) causes epidural hematoma.
A 78-year-old woman has multiple long-standing lesions on her face and back. These well-circumscribed
lesions are tan to brownish, slightly raised with a rough surface, and typically 0.5 to 1.5 cm
in diameter. The clinician examining the patient is able to "peel away" parts of the lesion
with the dull side of a scalpel blade. Which of the following diagnoses is most likely?
A. Eczema
B. Melanoma
C. Psoriasis
D. Seborrheic keratoses
E. Verruca vulgaris
Explanation:
The correct answer is D. Seborrheic keratoses, as described in the question stem, are very
common lesions of the skin of middle-aged and older individuals. These benign growths
histologically show hyperplasia of the epidermis. While cosmetically disturbing to some
patients, the primary medical concern is that, occasionally, seborrheic keratoses may mimic the
clinical appearance of basal cell carcinomas or squamous cell carcinomas. Consequently, biopsy
should be performed on atypical appearing or rapidly changing "seborrheic keratoses" to exclude
the presence of cancer.
Chronic eczema (choice A) produces dry, thick, and sometimes discolored skin.
Melanomas (choice B) characteristically look like dark moles with irregular margins and
variations in the degree of pigmentation.
Psoriasis (choice C) produces erythematous plaques with a silvery scale.
Verruca vulgaris (choice E), the common wart, produces verrucous papules that are most commonly
found on the hands. The face and back would be unusual sites.
A 13-year-old male presents to the emergency room with a deep skin abrasion on his knee. He states that

126
it has not stopped bleeding since it happened during recess approximately 20-30 minutes ago.
Physical examination reveals a well developed, well nourished adolescent. There are multiple
purpura over his legs and arms, and a few scattered petechiae on his chest and gums. His
bleeding time is 22 minutes, platelets = 300,000/mm3, hemoglobin = 11g/dL. A trial of
cryoprecipitate transfusion does not improve his bleeding time. A normal platelet transfusion
does improve bleeding time. Which of the following is the correct diagnosis?
A. Bernard-Soulier syndrome
B. Henoch-Schnlein purpura
C. Idiopathic thrombocytopenic purpura
D. Thrombotic thrombocytopenic purpura
E. Von Willebrand's disease
Explanation:
The correct answer is A. Bernard-Soulier syndrome is an autosomal recessive disease of platelet
adhesion which causes prolonged bleeding times in the presence of normal platelet counts. These
patients' platelets cannot bind to subendothelial collagen properly because of a deficiency or
dysfunction of the glycoprotein Ib-IX complex. Clinically the patients have impaired hemostasis
and recurrent severe mucosal hemorrhage. The only treatment for an acute episode is a
transfusion of normal platelets. This patient has a slightly decreased hemoglobin due to blood
loss.
Henoch-Schonlein purpura (choice B) is a self-limited autoimmune vasculitis that affects
children and young adults, usually following an upper respiratory infection. Affected
individuals develop purpuric rashes on the extensor surfaces of their arms, legs, and buttocks.
They also have abdominal pain and hematuria from glomerulonephritis. Despite the tendency
toward hemorrhage, the bleeding times and platelet count would be normal.
Idiopathic thrombocytopenic purpura (choice C) causes an increase in the bleeding time, but as
the name implies, platelet counts are decreased. There is bleeding from small vessels,
especially of the skin, gastrointestinal tract and genitourinary tract. Purpura and petechiae
frequently develop. It is considered a self-limited autoimmune disorder, typically affecting
children after a recent viral infection.
Thrombotic thrombocytopenic purpura (choice D) is characterized by an increased bleeding time,
but a decreased platelet count. It is a rare disorder of unknown etiology, thought to be
initiated by endothelial injury, which releases certain procoagulant materials into the
circulation, causing platelet aggregation. It causes purpura, fever, renal failure,
microangiopathic hemolytic anemia and microthrombi, generally in young women. In this disorder,
platelet transfusion is actually contraindicated, as it can precipitate thrombosis.
Von Willebrand's disease (choice E) causes increased bleeding times with normal platelet
counts. It is the most common inherited bleeding disorder, caused by a defect in von Willebrand
factor, which aids the binding of platelets to collagen. Even though the platelets themselves
are normal, binding is impaired, thus a platelet transfusion would not correct the problem.
Cryoprecipitate, a plasma fraction rich in von Willebrand factor, would help in the case of von
Willebrand's disease, but would not help with Bernard-Soulier syndrome.
Which of the following conditions may lead to development of an exudate within the pleural cavity?
A. Bacterial pleuritis

127

B. Cirrhosis of the liver


C. Congestive heart failure
D. Nephrotic syndrome
E. Protein-losing enteropathy
Explanation:
The correct answer is A. An exudate results from leakage of protein-rich fluid from the plasma
into the interstitium. It is usually the result of increased vascular permeability caused by
inflammation. Exudates also contain numerous acute or chronic inflammatory cells, depending on
the inciting event. Of the above choices, only bacterial pleuritis would produce an exudate. If
pleuritis is caused by pyogenic organisms, the exudate is purulent (neutrophil-rich). If
pleural inflammation is due to mycobacterial infection or neoplastic infiltration, the
resulting exudate will contain chronic inflammatory cells.
In contrast, a transudate contains less protein and few inflammatory cells. There are two main
mechanisms of transudate formation: 1) decreased oncotic pressure, such as that which occurs in
cirrhosis of the liver, nephrotic syndrome, and protein-losing enteropathy (choices B, D, and
E); and 2) increased hydrostatic pressure, which may result from congestive heart failure
(choice C).
A 45-year-old man presents to a clinician because of chronic fatigue. He states that he has stopped
exercising, and feels tired after mild activity. A complete blood count reveals a hematocrit of
31, a white count of 1,950, and a platelet count of 90,000. Which of the following radiologic
findings would most likely lead to the correct diagnosis?
A. Bony overgrowth
B. Generalized decrease in bone mass
C. Large calcified growth on the tibia
D. Multiple exostoses
E. Single "punched out" lesion of upper femur
Explanation:
The correct answer is A. Trilineage failure of blood cell production, producing pancytopenia,
suggests a generalized decrease in marrow cavity size, which can be due to bony overgrowth in
processes such as osteopetrosis. The anemia observed in these patients is often very refractory
to treatment.
Osteomalacia produces a generalized decrease in bone mass (choice B), and is not associated
with pancytopenia.
A large calcified growth on the tibia (choice C) suggests a benign or malignant tumor of bone,
which would not usually replace enough marrow to cause anemia.
Multiple exostoses (choice D) would affect the outer surface, not the marrow cavity of the
bone.

128
A single &ldquo;punched out&rdquo; bony lesion (choice E) located in the upper femur suggests
monostotic fibrous dysplasia, an asymptomatic, benign, bony lesion.
A young woman presents to her family physician with complaints of numbness, tingling, and burning of her
fingers. She states that these symptoms are typically accompanied by "blanching" of her
fingertips until they are nearly blue, followed by reddening as the episode resolves. The
condition described is the initial presenting complaint in over 70% of patients with which of
the following diseases?
A. Colonic polyposis
B. Congestive heart failure
C. Goodpasture's syndrome
D. Rheumatoid arthritis
E. Scleroderma
Explanation:
The correct answer is E. The finger color changes and pain are manifestations of Raynaud's
phenomenon, which is often idiopathic, but can be related to many connective tissue diseases,
notably progressive systemic sclerosis, also known as scleroderma (the initial complaint in
over 70% of patients).
Colonic polyposis (choice A) can be related to mucocutaneous pigmentation (Peutz-Jeghers
syndrome), and sebaceous cysts and other soft-tissue tumors of skin (Gardner's syndrome).
Congestive heart failure (choice B) can cause edema of subcutaneous tissues, but is not a cause
of Raynaud's phenomenon.
Goodpasture's syndrome (choice C) causes pulmonary hemorrhage and renal failure.
Raynaud's phenomenon can be seen in rheumatoid arthritis (choice D), but is not associated
nearly as frequently with this condition as with scleroderma.
A 25-year-old woman presents to her physician with complaints of easy bruising and excessive bleeding.
Physical examination demonstrates hepatosplenomegaly. A blood smear reveals pancytopenia and a
bone marrow biopsy demonstrates markedly enlarged cells containing a fine fibrillar material
resembling tissue paper. Leukocyte enzymatic studies demonstrate a deficiency of beta-Dglucosidase activity. Which of the following substances is most likely to accumulate in the bone
marrow cells?
A. Galactose
B. Glucosylceramide
C. Glycogen
D. Homocysteine
E. Mineralocorticoids
Explanation:

129
The correct answer is B. The disease is Gaucher disease, which is a glycolipid storage disease
in which glucosylceramide accumulates in spleen, liver, and bone marrow. The defective enzyme,
a lysosomal hydrolase known as acid beta-D-glucosidase, can be assayed in leukocytes separated
from the blood by centrifugation. Interestingly, the defective enzyme can now be supplied
intravenously, and strategies for transplantation of the cloned replacement gene are now being
developed.
Galactose (choice A) is a sugar. Deficiency of beta-D-glucosidase would not lead to
accumulation of galactose.
Glycogen (choice C) is a storage polymer of glucose. It accumulates in various glycogen storage
diseases, but has a different appearance than the glycolipid in Gaucher disease.
Homocysteine (choice D) is an amino acid. It does not accumulate in Gaucher disease.
Mineralocorticoids (choice E) are lipids, but do not accumulate in bone marrow.
A pathologist is examining a poorly differentiated tumor that he suspects may be derived from muscle
tissue. Immunohistochemical stains for which of the following would be expected to yield
positive results if he is correct?
A. Cytokeratin
B. Desmin
C. Glial fibrillary acidic proteins
D. Neurofilaments
E. Vimentin
Explanation:
The correct answer is B. Muscle cells contain desmin, which is a component of the intermediate
filaments found in these cells.
Epithelial cells contain cytokeratin (choice A).
Neuroglia contain glial fibrillary acidic proteins (choice C).
Neurons contain neurofilaments (choice D).
Connective tissue contains vimentin (choice E).
A very ill 3-year-old child is brought into the emergency room with a fever. On physical examination,
the child has large cervical lymph nodes and a desquamating skin rash that involves the palms,
soles, and mouth. This child should be monitored for the development of which of the following
conditions?
A. Abdominal aortic aneurysm
B. Aneurysm of the aortic root
C. Berry aneurysm
D. Coronary artery aneurysm

130

E. Dissecting aneurysm
Explanation:
The correct answer is D. The child has Kawasaki's syndrome (mucocutaneous lymph node syndrome),
which is the leading cause of acquired heart disease in children in the United States. In this
disorder, small, medium, and large arteries are affected, with transmural inflammation and
variable necrosis. About 20% of affected children have damage to the coronary vessels; some
develop coronary artery aneurysms. In 1-2% of cases, sudden death may occur from aneurysm
rupture or thrombosis producing infarction.
Abdominal aortic aneurysms (choice A) are associated with atherosclerosis.
Aneurysms of aortic root (choice B) are usually associated with syphilis.
Berry aneurysms (choice C) are caused by congenital defects in the vessel wall, and are
associated with polycystic kidney disease.
Dissecting aneurysms (choice E) are associated with hypertension or cystic medial degeneration
(seen in Marfan's syndrome).
A 35-year-old woman consults her gynecologist because she has a blood-tinged discharge from one nipple.
Physical examination demonstrates a small, palpable, subareolar tumor. The tumor is surgically
resected and then pathologically examined. Which of the following features is most helpful in
establishing its benign, rather than malignant, character?
A. Abnormal mitotic figures
B. Intraductal location of tumor
C. Presence of both epithelial and myoepithelial cells
D. Presence of papillary fronds
E. Small size of tumor
Explanation:
The correct answer is C. Intraductal papillomas are small (usually less than 1 cm), benign
neoplastic papillary tumors found within the larger ducts of the breast. They are typically
located near the nipple under the areola. They can be frightening to the patient because they
may produce a serous or bloody nipple discharge. They have a malignant counterpart, papillary
ductal carcinoma of the breast, so it is important to be able to distinguish the benign from
the malignant lesion. Features favoring the diagnosis of ductal carcinoma, rather than benign
intraductal papilloma, include severe cytologic atypia, abnormal mitotic figures, and a
cribriform growth pattern. Features that are specifically reassuring are the consistent
presence of a fibrovascular core and both epithelial and myoepithelial cells in the papillary
fronds. The myoepithelial cells are small cells with a contractile function usually found
around breast ducts.
Abnormal mitotic figures (choice A) are a feature of malignancy.
Both benign and malignant papillary tumors can be located within ducts (choice B).
Both benign and malignant papillary tumors can have papillary fronds (choice D).

131

Both benign and malignant papillary tumors can be small (choice E).
A diabetic patient complains to a physician of a sensation of walking on pebbles with bare feet.
Physical examination demonstrates "clawing" of the toes with flexion of the interphalangeal
joints and extension of the metatarsophalangeal joints. Atrophy of which of the following
muscles is most likely responsible for the observed changes?
A. Flexor digitorum longus
B. Lumbricals and interossei
C. Peroneus longus
D. Tibialis anterior
E. Tibialis posterior
Explanation:
The correct answer is B. The condition described in the question stem is very common among
diabetics, and is due to atrophy of lumbricals and interosseus muscles secondary to diabetic
neuropathy. Another finding that may be seen is the presence of corns and callosities on the
dorsal surface of the feet overlying the protuberant interphalangeal joints. All of these
degenerative changes add to the diabetic's foot disease, predisposing for sores that heal
poorly in the poorly vascularized diabetic foot, often leading to gangrene, requiring
amputation of the distal foot.
Flexor digitorum longus (choice A) is an extrinsic muscle of the foot that flexes the distal
phalanges of the lateral four toes and assists in plantar flexion of the foot.
Peroneus longus (choice C) is an extrinsic muscle of the foot that plantar flexes and everts
the foot.
Tibialis anterior (choice D) is an extrinsic muscle of the foot that dorsiflexes and inverts
the foot.
Tibialis posterior (choice E) is an extrinsic muscle of the foot that plantar flexes and
inverts the foot.
A 39-year-old male with a 20-pack-year history of cigarette smoking presents with fever, chills, and
painful urination. On physical examination, his prostate is boggy, and exquisitely tender on
palpation. Urinalysis reveals white blood cells and gram-negative rods. This man's symptoms are
most likely due to which of the following?
A. Acute bacterial prostatitis
B. Chronic abacterial prostatitis
C. Chronic bacterial prostatitis
D. Nodular hyperplasia
E. Prostate cancer
Explanation:

132

The correct answer is A. Most prostatic conditions are painless; acute bacterial prostatitis
stands out as producing a very tender prostate. The etiologic agents are usually the same as
those that produce urinary tract infection, and include E. coli, Klebsiella, Proteus,
Pseudomonas, Enterobacter, Serratia, Enterococcus, and Staphylococcus.
Both chronic abacterial prostatitis (choice B) and chronic bacterial prostatitis (choice C) may
be asymptomatic, or either may cause low back pain, perineal tenderness, or dysuria. Suspected
pathogens in this form of prostatitis include Chlamydia and Ureaplasma, but the causative agent
is not known with certainty.
Nodular hyperplasia (choice D) produces difficulty in urination.
Prostate cancer (choice E) produces a firm mass that is generally palpable on rectal
examination.
A family suspected of child abuse because one of their children has had multiple fractures consults an
orthopedic specialist at a children's hospital. The specialist examines the child, who is now 5
years old, and notes that the child has blue-tinged sclera, hearing loss, and small, slightly
blue, misshapen teeth. Radiologic studies confirm the presence of numerous fractures of various
ages. No significant degree of bruising is seen over sites of recent fracture. The disease this
child most likely has is related to abnormal metabolism involving which of the following
substances?
A. Collagen
B. Glycogen
C. Mucopolysaccharides
D. Purines
E. Tyrosine
Explanation:
The correct answer is A. The suspected disease is osteogenesis imperfecta, which is a rare
genetic disorder that occurs in both recessive and dominant forms. The clinical presentation,
depending on the specific form, varies from death in utero, to that described in the question
stem, to very mild disease with only a modest increase in bone fragility. The different types
all have defects in the synthesis of type I collagen, often with insufficient or abnormal
pro-&alpha;1(1) or pro-&alpha;2(1) chains. These deficits produce an unstable collagen triple
helix that is not as strong as normal collagen.
Defective glycogen (choice B) metabolism is associated with the various glycogen storage
diseases, such as von Gierke disease and Pompe disease. These diseases tend to present with
profound hypoglycemia, hepatomegaly, or muscle weakness.
Defective mucopolysaccharide (choice C) metabolism is associated with the
mucopolysaccharidoses, such as Hurler and Hunter syndromes. These diseases tend to present with
abnormal facies ("gargoylism"), deformed ("gibbus") back, claw hand, and stiff joints.
Abnormalities of purine metabolism (choice D) are present in gout, which presents with joint
inflammation and often involves the great toe.
Abnormalities of tyrosine metabolism (choice E) are associated with phenylketonuria (pale hair

133
and skin, mental retardation, musty smelling urine), albinism (pale hair, skin, increased skin
cancer), cretinism (decreased T3 and T4), tyrosinosis (liver and kidney disease), and
alkaptonuria (chronic arthritis and urine that turns black upon standing).
A 53-year-old man develops acute, excruciating chest pain that radiates to his back. En route to the
emergency room, he becomes unresponsive, and is pulseless on arrival. Resuscitation attempts
are unsuccessful. Autopsy reveals massive hemoperitoneum due to a ruptured aortic dissection.
There is a jagged intimal tear in the ascending aorta, with a dissecting hematoma in the media,
extending from the aortic valve to the renal arteries. Which feature of this scenario most
strongly suggests hypertension as the cause of the aortic dissection?
A. Adventitial tear above renal arteries
B. Dissection through media
C. Involvement of major aortic branches
D. Origin at ascending aorta
E. Rapid exsanguination
Explanation:
The correct answer is D. The two most common causes of aortic dissection are hypertension and
atherosclerosis. An important distinction between the two is that hypertensive dissections
generally originate in the ascending aorta, at an intimal surface free of atherosclerosis.
Dissection secondary to atherosclerosis is typically the consequence of a ruptured aortic
aneurysm, which originates in the abdominal aorta at the iliac bifurcation. Dissections due to
both hypertension and atherosclerosis generally course through the wall within the media
(choice B). They both can involve the entire length of the aorta, and may rupture anywhere
along its course (choices A and C). Well recognized sequelae of dissections include rupture
through the adventitia, compromise of major arterial branches or the aortic valve, cardiac
tamponade, and rapid exsanguination (choice E).
A leukemic patient develops disseminated intravascular coagulation. Examination of the marrow reveals
hypergranular promyelocytes, some of which contain multiple Auer rods. The diagnosis of acute
promyelocytic leukemia is made. Which of the following translocations is associated with the
development of this disorder?
A. t(4;11)
B. t(6;9)
C. t(8;14)
D. t(8;21)
E. t(15;17)
Explanation:
The correct answer is E. Acute promyelocytic leukemia (M3 by the FAB classification) is
associated with a t(15;17) (q22;q11) translocation. Disseminated intravascular coagulation can
occur in this disorder due to the release of procoagulant substances from the leukemic cells,
especially during treatment.

134

The t(4;11)(q21;q23) translocation (choice A) is associated with acute lymphocytic leukemia


(ALL) and undifferentiated leukemia.
The t(6;9)(p23;q34) translocation (choice B) is found in subtypes of AML with basophilia (M1,
M2, M4).
Burkitt's leukemia, which is related to Burkitt's lymphoma, is associated with t(8;14) (q
24;q32) (choice C).
The t(8;21) (q22;q22) translocation (choice D) is seen in M2 leukemia, also known as acute
myeloid leukemia (AML) with maturation, and some M4 (AML with granulocytic and monocytic
maturation).
A patient presents to a dermatologist because of skin changes. The skin is hyperpigmented, thickened,
and feels velvety. Multiple skin tags are present. The changes are worst in the axillae, groin,
and anogenital area, but are very widespread. This patient should be specifically evaluated for
which of the following diseases?
A. Hepatic cirrhosis
B. Lung cancer
C. Polycystic renal disease
D. Systemic lupus erythematosus
E. Ulcerative colitis
Explanation:
The correct answer is B. The condition described is acanthosis nigricans. Mild forms are common
and may be associated with obesity and endocrine abnormalities. More extensive forms, such as
in this patient, may be associated with malignant disease: usually an adenocarcinoma (often
lung), less commonly a lymphoma. The pattern is important to recognize because the eruption may
precede other symptoms of the malignancy by several years.
A 25-year-old man with infertility is diagnosed with Kartagener syndrome. He has also been particularly
susceptible to recurrent pulmonary infections and bronchiectasis. Which of the following
cellular functions is most likely disrupted in this patient?
A. Chloride transport
B. Formation of phagolysosomes
C. Motility of cilia
D. Oxidative burst
E. Synthesis of IgA
Explanation:
The correct answer is C. To answer this question, you need not be acquainted with Kartagener
syndrome, a rare autosomal recessive condition caused by mutations of the gene encoding the
protein dynein. The information provided in the history is sufficient to find the right choice.

135
First, dynein is a protein that forms the side arms of microtubule doublets that allow motility
of cilia. Thus, a defect in this protein would affect ciliated cells such as spermatozoa and
respiratory epithelium. Consequently, poor sperm motility results in infertility, whereas
deficient mucociliary function in the respiratory system leads to defective bacterial clearance
and recurrent pulmonary infections. Bronchiectasis is a frequent complication, since recurrent
bouts of lung infections cause destruction and subsequent dilatation of the bronchial walls.
Chloride transport (choice A) is defective in cystic fibrosis, an inherited autosomal recessive
disease caused by mutations in the gene coding for a chloride channel protein. Respiratory
pathology is frequent and usually severe in cystic fibrosis, consisting of recurrent infections
and, consequently, bronchiectasis. Ninety-five percent of male patients are infertile as well
because of obstruction of the vas deferens and resultant azoospermia. Cystic fibrosis and
Kartagener syndromes have clinical similarities but entirely different etiologies.
Formation of phagolysosomes (choice B) is involved in the breakdown of phagocytosed material,
both extracellular (heterophagy) and intracellular (autophagy). Vacuoles containing material to
be digested fuse with lysosomes that form phagolysosomes. An example of deficient formation of
phagolysosomes is Chediak-Higashi syndrome, a rare hereditary condition characterized by
neutropenia and deficient microbial killing. In this disease, fusion of lysosomes with
phagocytic vacuoles is impaired, leading to defective microbial killing and recurrent
infections.
The oxidative burst (choice D) refers to the production of reactive oxygen species following
rapid activation of NADPH oxidase. NADPH is oxidized, and, in the process, oxygen is reduced to
superoxide anion. Superoxide is then converted into H2O2, which is involved in bacterial
killing. A defect in the oxidative burst causes chronic granulomatous disease, characterized by
susceptibility to recurrent bacterial infections.
Synthesis of IgA (choice E) is essential in immunologic defense at mucosal barriers, such as
respiratory, gastrointestinal, and urogenital tracts. Defective IgA synthesis occurs in
isolated IgA deficiency, a common disorder that renders patients susceptible to sinopulmonary
and gastrointestinal infections.
A 72-year-old man with a significant smoking history presents to the emergency room with complaints of
dyspnea and truncal, arm, and facial swelling for one week. Physical examination is remarkable
for facial erythema and facial, truncal, and arm edema with prominence of thoracic and neck
veins. On chest x-ray, there is a mass in the right mediastinum with adenopathy. Which of the
following is the most likely diagnosis?
A. Adenocarcinoma
B. Hodgkin's lymphoma
C. Large cell carcinoma
D. Non-small cell carcinoma
E. Small cell carcinoma
Explanation:
The correct answer is E. Superior vena cava (SVC) syndrome is characterized by obstruction of
venous return from the head, neck, and upper extremities. Over 85% of cases of SVC syndrome are
related to malignancy. Bronchogenic carcinomas (most commonly small cell cancer and squamous
cell cancer) account for over 80% of these cases. Among bronchogenic carcinomas, the most
common causes of SVC syndrome (in order of frequency) are small-cell carcinoma, epidermoid

136
carcinoma, adenocarcinoma (choice A), and large-cell carcinoma (choice C). Lymphomas such as
Hodgkin's disease (choice B) and non-Hodgkin's lymphoma are uncommon causes of SVC syndrome.
Rare tumors associated with SVC syndrome include primary leiomyosarcomas and plasmacytomas.
Infectious etiologies include tuberculosis, syphilis, and histoplasmosis. SVC syndrome can also
occur as a result of an enlarged goiter, and from thrombus formation caused by indwelling
intravenous lines or pacemaker wires.
Non-small cell carcinoma (choice D) is not commonly associated with SVC syndrome.
A 35-year-old man develops oliguria, peripheral edema, and shortness of breath over a two week period.
Serum chemistries show markedly elevated BUN and creatinine. Renal biopsy shows many glomerular
crescents, and fluorescent antibody studies demonstrate linear deposits of IgG along the
glomerular basement membrane. This patient is also at increased risk for developing
A. bladder carcinoma
B. meningioma
C. pulmonary hemorrhage
D. rheumatoid arthritis
E. testicular carcinoma
Explanation:
The correct answer is C. The clinical scenario is that of rapidly progressive
glomerulonephritis. Linear deposits of IgG along the basement membrane make the diagnosis of
anti-glomerular basement membrane disease, which may either occur as an isolated finding or as
part of Goodpasture's syndrome, which also features prominent pulmonary hemorrhage as a result
of antibody attack on the alveolar capillary basement membrane. Formerly, the prognosis of
patients with these conditions was dismal (most dying within 6 months of either renal failure
or pulmonary hemorrhage), but the patients can now often be successfully treated with
aggressive management including plasma exchange, renal dialysis, and high-dose
immunosuppression.
Predisposing factors for bladder carcinoma (choice A) include industrial chemical exposure,
cigarette smoking, and infection with Schistosoma haematobium. There is no link between bladder
carcinoma and glomerulonephritis.
Predisposing factors for meningiomas (choice B) include von Recklinghausen's neurofibromatosis
and probably small, inconsequential trauma to the meninges. There is no link between
meningiomas and glomerulonephritis.
Glomerulonephritis was previously considered to be almost nonexistent in rheumatoid arthritis
(choice D); it is now acknowledged that rheumatoid arthritis may occasionally be associated
with mesangial proliferation, membranous nephropathy, and rarely, rapidly progressive
glomerulonephritis.
Testicular carcinoma (choice E) is unrelated to glomerulonephritis. Predisposing factors for
testicular carcinoma include a failure of testicular descent.
A 22-year-old black female with sickle cell disease presents to her physician with intermittent right
upper quadrant abdominal pain. Ultrasound studies demonstrate multiple shadows within the gall
bladder. Which of the following is the most likely composition of these structures?

137
A. Calcium bilirubinate
B. Cholesterol
C. Cystine
D. Struvite
E. Uric acid
Explanation:
The correct answer is A. Pigment bile stones, derived from degradation of heme, are seen in
patients with chronic hemolytic disorders (such as this patient's sickle cell disease),
alcoholic cirrhosis, advanced age, and biliary tract infection. The stones consist largely of
calcium salts of bilirubin.
Cholesterol (choice B) gallstones are associated with obesity, high estrogen states,
multiparity, Crohn's disease, rapid weight loss, clofibrate therapy, and Native American
origin.
Cystine (choice C), struvite (choice D), and uric acid (choice E) stones are found in the
urinary tract, not the gall bladder.
A 15-year-old girl is evaluated for failure to begin menstruation. Physical examination demonstrates
short stature and a webbed neck. Chromosomal analysis demonstrates a lack of one X chromosome.
This patient should be specifically evaluated for which of the following cardiovascular
anomalies?
A. Coarctation of the aorta
B. Dextrocardia
C. Ostium primum septal defect
D. Pulmonary stenosis
E. Tetralogy of Fallot
Explanation:
The correct answer is A. The girl has Turner syndrome, which is specifically associated with
coarctation of the aorta. Even when coarctation has not been diagnosed in early childhood and
the patient appears to be doing well, surgical correction of any significant degree of
coarctation is recommended. Uncorrected coarctation can cause death after about age 40 due to a
variety of causes, including congestive heart failure, infective aortitis (analogous to
infective endocarditis), and hypertension-induced intracranial hemorrhage or rupture of the
damaged (pre-coarctation) aorta.
Associate dextrocardia (choice B) with Kartagener syndrome.
Associate ostium primum septal defect (choice C) with Down syndrome.
Pulmonary stenosis (choice D) and tetralogy of Fallot (choice E) are not specifically
associated with Turner syndrome.

138
A 66-year-old man with a long history of cigarette smoking and alcohol abuse has developed a protruding,
centrally ulcerated mass in his mouth, and he is concerned he has cancer. Which of the
following locations is most commonly the primary site of oral squamous cell carcinoma?
A. Base of tongue
B. Buccal mucosa
C. Floor of mouth
D. Palate
E. Tip of tongue
Explanation:
The correct answer is C. Oral cancers are highly associated with alcohol and tobacco use, and
HPV-16 (human papilloma virus type 16, which is also found in cervical carcinomas) is found in
nearly half of all oral cancers. The pattern of tumor development probably reflects exposurerelated factors. Oral cancers develop in the following locations (in order of decreasing
frequency): (1) floor of the mouth (choice C); (2) tip of the tongue (choice E); (3) hard
palate (choice D); (4) base of the tongue (choice A); and (5) others (choice B).
A 35-year-old woman dies suddenly. Autopsy findings are within normal limits except for the heart,
which is shown in the accompanying photograph. Which of the following is the most likely diagnosis?
A. Acute endocarditis
B. Calcification of the mitral anulus
C. Marantic endocarditis
D. Mitral valve prolapse
E. Rheumatic valvular disease
Explanation:
The correct answer is D. The floppy valve seen is characteristic of mitral valve prolapse. In this
still idiopathic condition, myxomatous degeneration of the zona fibrosa of the valve structurally
weakens the valve leaflets. The disorder has a female predominance, and a midsystolic click is
typically heard on auscultation. Most individuals are asymptomatic through life, but complications
can include infective endocarditis, valvular insufficiency, arrhythmias, and, rarely, sudden death
(as in this patient).
In acute endocarditis (choice A) valves typically show large vegetations.
Calcification of the mitral anulus (choice B) would appear as calcified nodules around the ring
surrounding the valve.
Marantic endocarditis (choice C) causes small, sterile vegetations along the line of closure of
the valve.
Rheumatic valvular disease (choice E) causes scarring of the valves with thickened, blunted
leaflets.

139
A 55-year-old female presents for an annual exam. Her right breast is swollen, red, and tender. The
physician palpates a firm area in the breast and suspects inflammatory breast cancer. Which of
the following best describes the histological changes observed in this disorder?
A. Acute inflammation in breast carcinoma
B. Chronic inflammation in breast carcinoma
C. Dermal lymphatic invasion by cancer cells
D. Epidermal invasion by cancer cells
E. Fat necrosis in breast carcinoma
Explanation:
The correct answer is C. Inflammatory breast cancer is a pattern of invasive breast cancer in
which the neoplastic cells infiltrate widely through the breast tissue. The cancer involves
dermal lymphatics and therefore has a high incidence of systemic metastasis and a poor
prognosis. If the lymphatics become blocked, then the area of skin may develop lymphedema and
"peau d'orange," or orange peel appearance. The overlying skin in inflammatory breast cancer is
usually swollen, red, and tender.
Acute inflammation (choice A) is a rare finding in breast cancer and may be associated with
secondary infection or abscess.
Chronic inflammation in breast cancer (choice B) is a non-specific finding. In medullary breast
cancer, a type of invasive ductal carcinoma, there are a large number of lymphocytes around the
tumor and a desmoplastic reaction is often absent in the surrounding tissue. This type of
cancer carries a somewhat better prognosis.
Epidermal invasion by cancer cells (choice D) is a poor prognostic indicator. Intraepidermal
malignant cells are called Paget cells. Paget's disease of the nipple is a type of ductal
carcinoma that arises in large ducts and spreads intraepidermally to the skin of the nipple and
areola. There is usually an underlying ductal carcinoma.
Fat necrosis (choice E) is often seen following trauma to the breast, but is not specifically
associated with a particular type of breast cancer, although it may be confused with breast
cancer if areas of calcification are present.
An x-ray performed on a newborn infant shows enlargement of the left ventricle and left atrium as well
as dilatation of the aorta. Echocardiographic studies demonstrate volume-overloading of the
left ventricle. Cardiac auscultation reveals the presence of a continuous "machinery" murmur.
Which of the following is the most likely diagnosis?
A. Atrial septal defect
B. Patent ductus arteriosus
C. Pulmonic stenosis
D. Tetralogy of Fallot
E. Ventricular septal defect
Explanation:

140

The correct answer is B. Patent ductus arteriosus (PDA) is a congenital cardiac disorder in
which blood traveling in the aorta is shunted through the ductus arteriosus to the pulmonary
arteries. On x-ray, the left ventricle and left atrium may be enlarged, and pulmonary
hypertension may be observed. PDA is characterized by a continuous "machinery" murmur on
auscultation. If the ductus is widely patent, pulmonary hypertension may eventually develop,
and the initially left-to-right shunt is reversed, sending deoxygenated blood through the
descending aorta, and producing cyanosis (Eisenmenger syndrome). Since the deoxygenated blood
enters the descending aorta, the toes can be cyanotic, but the fingers are generally not.
In atrial septal defect (choice A), left-to-right shunting causes volume overloading of the
right ventricle, the increased flow across the pulmonic valve producing a midsystolic pulmonary
ejection murmur. The second heart sound is widely split. A diastolic murmur may also be heard,
reflecting increased flow from the right atrium into the right ventricle.
Pulmonic stenosis (choice C) typically produces a harsh systolic ejection murmur best heard at
the upper left sternal border, often preceded by a systolic ejection sound.
Tetralogy of Fallot (choice D) is a form of cyanotic congenital heart disease characterized by
ventricular septal defect, right ventricular outflow tract obstruction, an overriding aorta,
and right ventricular hypertrophy. The heart is often described as "boot-shaped" on chest xray.
A ventricular septal defect (choice E) would produce an initial left-to-right shunt,
characterized by a holosystolic murmur, and increased pulmonary vascularity on chest x-ray. A
mid-diastolic rumble may also be heard.
A patient with chronic pelvic pain undergoes a hysterectomy. The resected uterus is filled with nodules
composed of benign smooth muscle cells. Which of the following terms best describes these
nodules?
A. Angiosarcoma
B. Leiomyoma
C. Leiomyosarcoma
D. Rhabdomyoma
E. Rhabdomyosarcoma
Explanation:
The correct answer is B. The lesions are the very common leiomyomas of the uterus. The lesions,
if numerous or large, may cause chronic pelvic pain, and infertility or pregnancy complications
(if they intrude on the uterine cavity). Malignant transformation to leiomyosarcoma (choice C)
is rare (and some authors suggest it may not occur, with uterine leiomyosarcoma being a de novo
lesion).
Angiosarcomas (choice A) are malignant tumors of blood vessels.
Rhabdomyomas (choice D) and rhabdomyosarcomas (choice E) are benign and malignant tumors of
skeletal muscle, respectively.
A 35-year-old woman has had type 1 diabetes mellitus for 20 years. She is now developing advanced
disease with visual complaints, foot ulcers, and renal disease. Which of the following features

141
that might be seen on renal biopsy is most specific for diabetic glomerulosclerosis?
A. Mesangial IgA deposits
B. Necrotic epithelial cells in tubules
C. Nests of cells with abundant clear cytoplasm
D. Numerous neutrophils in tubules
E. Ovoid, periodic acid-Schiff (PAS)-positive, hyaline masses
Explanation:
The correct answer is E. The most specific lesion of diabetic glomerulosclerosis for the
purposes of the USMLE is the Kimmelstiel-Wilson nodule. These are ovoid, hyaline, PAS-positive
structures found in the mesangial core at the edge of the glomerulus. Although this lesion is
the most distinctive (pathognomonic) for diabetes, it is not seen in all renal biopsies from
diabetic patients. Other changes that may be present include glomerular capillary basement
membrane thickening, diffuse glomerulosclerosis, hyaline thickening of arteriolar walls,
tubular atrophy, interstitial fibrosis, and PAS-positive capsular drops in the parietal layer
of Bowman capsule.
Mesangial IgA deposits (choice A) are a feature of Berger disease (IgA nephropathy).
Necrotic epithelial cells in tubules (choice B) are a feature of acute tubular necrosis.
Nests of cells with abundant clear cytoplasm (choice C) are a feature of renal cell carcinoma.
Numerous neutrophils in tubules (choice D) are a feature of acute pyelonephritis.
A 45-year-old G2P2 presents with a chief complaint of pelvic pain that is worse immediately before and
during her period. A bimanual exam and Pap smear are performed and are unremarkable. An
ultrasound of her pelvic organs is performed and shows non-focal thickening of the myometrium
with unremarkable ovaries. Which of the following is most likely to be the cause of this
patient's complaint?
A. Adenomyosis
B. Endometrial polyps
C. Endometritis
D. Leiomyoma
E. Mittelschmerz
Explanation:
The correct answer is A. Adenomyosis is characterized by the presence of endometrial glands
within the myometrium of the uterus in addition to their normal location in the endometrium.
These glands undergo cyclic changes with the menstrual cycle in response to the same stimuli as
the normal endometrial glands. The cause of adenomyosis is not known but this condition may be
found in up to 20% of uteruses. Grossly, it may be inapparent if it is limited to a small
focus, or it may cause expansion of the uterine wall and have a glassy appearance.
Microscopically, the aberrant glands must be separated from the endometrium by 2-3 mm to be

142
diagnostic. Clinically pain is caused by the glands breaking down and bleeding within the
confines of the myometrium. The ultrasound shows non-circumscribed thickening.
Endometrial polyps (choice B), if symptomatic, could cause bleeding but not pain. The Pap smear
may show a few shed polyp cells, while an ultrasound would detect an intra-cavity polyp. It
could also be clinically silent. An endometrial curettage is needed for diagnosis.
Endometritis (choice C) is an infection of the endometrium that may present with pain or
bleeding. Diagnosis is made by endometrial biopsy. A Pap smear might not sample high enough to
detect infection in the uterine cavity. There is no uterine wall thickening and a bimanual exam
may elicit some pain.
Leiomyoma (choice D), a benign smooth muscle tumor of the myometrium also referred to as a
fibroid, may be asymptomatic even when it reaches a large size. Submucosal leiomyomas may cause
bleeding and infertility. Other problems include urinary frequency due to bladder pressure.
Pain is rare but may happen if the leiomyoma infarcts. Fibroids are usually detected with
bimanual exam if they are large or protuberant, and they are seen on ultrasound as wellcircumscribed masses.
Mittelschmerz (choice E) refers to mid-cycle ovulatory pain associated with normal physiologic
ovulation from the ovary. There are no ultrasound or clinical findings.
A 54-year-old woman presents to the emergency department after a fall. Skeletal roentgenograms show no
fractures. Serum chemistry studies reveal that her aspartate aminotransferase (AST) is markedly
elevated, while her alanine aminotransferase (ALT), gamma-glutamyl transpeptidase (GGT), and
alkaline phosphatase are all within normal limits. Disease of which of the following organs
would be most likely to cause this serum enzyme pattern?
A. Colon
B. Duodenum
C. Heart
D. Pancreas
E. Stomach
Explanation:
The correct answer is C. Myocardial infarction (MI) can cause AST elevation without
accompanying elevation of ALT or other liver enzymes. This is an important fact to remember
because it may be the first clue for heart disease in a patient who has an atypical
presentation of MI (as is common in women with MI). MI can be confirmed with measurement of the
MB fraction of creatine phosphokinase (CPK-MB).
Unfortunately, diseases of the tubular organs of the gastrointestinal tract, including colon
(choice A), duodenum (choice B), and stomach (choice E), do not produce distinctive serum
enzyme patterns.
Damage to the pancreas (choice D) is associated with elevated amylase levels.
An elderly woman living at a nursing home is brought to the emergency room in shock. The nursing home
staff is very upset and stresses that the patient was completely well yesterday, and had only
complained of "feeling a little ill" several hours before being brought in. Physical
examination reveals a tense abdomen with guarding. Which of the following is the most likely

143
etiology for this patient's condition?
A. Acute appendicitis
B. Acute cholecystitis with gall bladder rupture
C. Gastric rupture
D. Rupture of a diverticulum
E. Rupture of an ovarian cyst
Explanation:
The correct answer is D. Rupture of a colonic diverticulum, with resulting fecal peritonitis,
is a catastrophic complication of diverticulosis, particularly in the elderly, with a mortality
rate near 50%. The rupture frequently involves a diverticulum that is not inflamed (or only
minimally inflamed) and consequently the rupture may be inapparent, at least initially. Shock
secondary to septicemia develops rapidly, however. Vigorous resuscitation may be required to
stabilize the patient sufficiently for emergency surgery to resect the distal colon and form a
colostomy (Hartmann's operation).
Acute appendicitis (choice A), acute cholecystitis with gall bladder rupture (choice B),
gastric rupture (choice C), and rupture of an ovarian cyst (choice E) are all uncommon in this
age group.
A 10-week-old apparently healthy infant is laid down for a nap. The mother sits in a rocking chair
nearby reading. At one point, she hears the baby make a single small cry, but she keeps reading
because the baby quiets quickly. Later, she gets up to check on the child, whom she finds
dead.Careful autopsy would be most likely to reveal which of the following cardiac findings?
A. Endocarditis
B. Failure of development of the endocardial cushion
C. Large interventricular septal defect
D. Mitral valve stenosis
E. Right ventricular hypertrophy
Explanation:
The correct answer is E. The baby is probably a victim of sudden infant death syndrome (SIDS).
The etiology remains unknown. Rare cases have been witnessed, and some of the SIDS babies give
a single small cry, as described in the question stem. Some of these babies were premature at
birth and some have upper respiratory infections in the preceding few days before death.
Findings at autopsy are subtle and may be absent. The cardiovascular system may show right
ventricular hypertrophy, which is possibly secondary to smooth muscle hypertrophy in small
pulmonary arteries, and possibly cardiac conduction system abnormalities. Other features
include brainstem gliosis (suggesting chronic hypoxia), extramedullary hematopoiesis, and
retained periadrenal brown fat.
A baby with endocarditis (choice A) would be very obviously ill.
A serious congenital heart defect, such as failure of development of the endocardial cushion

144
(choice B), large interventricular septal defect (choice C), or a mitral value stenosis (choice
D) would have been picked up in the comprehensive physical examination at the child's birth.
A woman with swelling of the oral mucosa and dry mouth is found to have intense destructive inflammation
of the salivary glands and antibodies against the ribonucleoprotein La. Which of the following
clinical findings would most likely be associated with this syndrome?
A. Conjunctivitis
B. Goiter
C. Hemolytic anemia
D. Proximal muscle weakness
E. Urethritis
Explanation:
The correct answer is A. The patient has Sjgren's syndrome, an autoimmune disease
characterized by dry eyes (keratoconjunctivitis) and a dry mouth (xerostomia) due to
destruction of the lacrimal and salivary glands. Sjgren's syndrome is also characterized by
autoantibody production. The most diagnostic autoantibodies are those against
ribonucleoproteins Ro (SS-A) and La (SS-B), although coexisting rheumatoid factor and lupus
antibodies are not uncommon.
Goiters (choice B) are not typical of Sjgren's syndrome. Although autoimmune thyroiditis is
associated with Sjgren's syndrome, ocular involvement is much more characteristic than thyroid
involvement.
Hemolytic anemia (choice C) is not characteristic of Sjgren's syndrome. Primary
autoantibodies, drugs, and systemic lupus erythematosus may be associated with hemolytic
anemia, but the findings of anti-La and inflammation of the salivary glands indicate that this
patient has Sjgren's syndrome.
Proximal muscle weakness (choice D), in association with autoantibodies, is expected in
polymyositis or dermatomyositis. Although polymyositis may occur in association with Sjgren's
syndrome, keratoconjunctivitis would be much more common than muscle weakness.
Urethritis (choice E) in autoimmune disease is typical of Reiter's syndrome, not Sjgren's
syndrome.
A 3-year-old child develops headaches and is brought to the family doctor. Funduscopic examination
reveals papilledema; one retina also shows a very vascular tumor. CT of the head demonstrates a
cystic tumor of the cerebellum. This child has a high likelihood of later developing which of
the following?
A. Berry aneurysm of the basilar system
B. Bilateral renal cell carcinoma
C. Cancer of a peripheral nerve
D. Choreiform movements related to decreased GABA and acetylcholine
E. Serum cholesterol of greater than 700 mg/dL

145

Explanation:
The correct answer is B. The disease is von Hippel-Landau disease, which is associated with a
deletion involving the VHL gene on chromosome 3 (3p). Affected individuals develop vascular
tumors (hemangioblastomas) of the retina, cerebellum, and/or medulla. Roughly half of the
affected individuals later develop multiple, bilateral renal cell carcinomas.
Berry aneurysms (choice A) are unrelated to hemangioblastomas, but are instead associated with
adult polycystic disease.
Peripheral nerve cancers (choice C) are a feature of von Recklinghausen's disease
(neurofibromatosis type I).
Choreiform movements, related to decreased GABA and acetylcholine (choice D), are a feature of
Huntington's disease.
Extremely high serum cholesterol (choice E) suggests the homozygous form of familial
hypercholesterolemia.
A 2-year-old child with leukemia develops nephrotic syndrome. Light microscopic studies are normal.
Electron microscopic studies demonstrate fusion of epithelial foot processes. The current
hypothesis for the pathogenesis of this change is that it is secondary to which of the
following?
A. Consumption of complement factors
B. IgG directed against basement membrane
C. Immune complex deposition
D. Lymphokine production by T cells
E. Mesangial IgA deposition
Explanation:
The correct answer is D. The child is suffering from minimal change or nil disease (lipoid
nephrosis), which has a peak incidence at 2-3 years of age. Minimal change disease can be
associated with food allergies, medications, or hematologic malignancies, or it can occur
idiopathically. The pathology does not appear to involve complement, immunoglobulins, or immune
complex deposition. Rather, an altered cell-mediated immunologic response with abnormal
secretion of lymphokines by T cells is thought to reduce the production of anions in the
glomerular basement membrane, thereby increasing the glomerular permeability to plasma albumin
through a reduction of electrostatic repulsion. The loss of anionic charges is also thought to
favor foot process fusion. Some authors have noted that other conditions associated with T-cell
abnormalities, such as Hodgkin's disease and T-cell lymphoma, are sometimes associated with
minimal change disease.
Consumption of complement factors (choice A) is observed in many conditions in which complement
activation occurs, for example, membranoproliferative glomerulonephritis.
IgG directed against renal and pulmonary basement membranes (choice B) is found in
Goodpasture's syndrome, a cause of rapidly progressive glomerulonephritis and hemoptysis.
Immune complex deposition (choice C) is associated with type III hypersensitivity reactions,

146
including postinfectious glomerulonephritis, lupus nephritis, Henoch-Schnlein purpura,
cryoglobulinemia, and bacterial endocarditis.
Mesangial IgA deposition (choice E) is associated with Berger's disease, or IgA nephropathy, a
cause of glomerulonephritis.
A 65-year-old man presents to a physician because of a palpable mass immediately below the left
clavicle. Biopsy of the mass demonstrates metastatic adenocarcinoma in a lymph node. Which of
the following organs should be most strongly suspected as containing the primary tumor?
A. Bladder
B. Large bowel
C. Liver
D. Pancreas
E. Stomach
Explanation:
The correct answer is E. The supraclavicular nodes (Virchow's node) can be involved early in
mediastinal and neck cancers. A palpable mass in the left node can also be the presenting
finding of gastric carcinoma, which is worth remembering when you cannot find the primary in
the lungs or neck.
Bladder cancer (choice A) typically presents with hematuria or urinary symptoms.
Colon cancer (choice B) typically presents with blood in stool or changes in bowel habits.
Primary liver cancer (choice C) and pancreatic cancer (choice D) unfortunately tend to be
clinically silent until well advanced.
A one month-old baby develops vomiting of increasing severity after feeding, eventually developing
"projectile" vomiting. The vomitus contains milk and mucus, but not bile. The baby also fails to
gain weight and becomes constipated. Physical examination performed after feeding demonstrates
visible waves of peristalsis travelling from left to right in the epigastrium. An olive-sized
mass can be felt lying deep to the edge of the right rectus abdominis when the stomach is empty.
Which of the following techniques would be most useful in correcting the baby's problem?
A. Barium enema
B. Duodenoduodenostomy
C. Gastric resection
D. Pyloromyotomy
E. Surgical reduction of volvulus
Explanation:
The correct answer is D. The history is classic for "congenital" pyloric stenosis, which
typically presents from 3 to 6 weeks of life. The mass felt is the hypertrophied pylorus. This
problem can be easily surgically corrected with pyloromyotomy, in which the pyloric muscle is

147
partially cut, relieving the obstruction.
Barium enema (choice A) is occasionally effective in reducing childhood intussusception, but
would not be of value in pyloric stenosis.
Duodenoduodenostomy (choice B) is used to correct congenital duodenal obstruction, but is not
required for congenital pyloric stenosis.
Gastric resection (choice C) would be completely unnecessary in this case.
Volvulus (choice E) usually involves the small intestine and can produce an "acute abdomen"
secondary to infarction of the bowel.
A 14-year-old boy presents with a 1-month history of knee pain and a 6-pound weight loss. He is pale and
afebrile. An x-ray reveals a densely sclerotic lesion in the distal femur extending from the
growth plate into the diaphysis. The periosteum is lifted, forming an angle with the cortex. The
surrounding soft tissue resembles a "sunburst" on the radiograph. Which of the following is the
most likely diagnosis?
A. Nonossifying fibroma
B. Osteochondroma
C. Osteomyelitis
D. Osteosarcoma
E. Paget's disease
Explanation:
The correct answer is D. This patient has osteosarcoma, a malignant bone tumor that produces
osteoid and bone. Prognosis is poor. X-ray reveals bone destruction, soft tissue with
"sunburst" appearance, and Codman's triangle (periosteal elevation that forms an angle with the
cortex of the bone)&mdash;all classic clues to the diagnosis. Other hints were the patient's
weight loss and pallor, which should have raised your suspicion that a malignancy existed.
Choice D is the only malignant process among the answer choices. This tumor usually occurs in
the second or third decades of life. At the same time, it is the most common bone tumor in the
elderly and is often associated with Paget's disease (choice E). Histological findings classic
for osteosarcoma are anaplastic cells with osteoid (a pink amorphous material that is variably
mineralized).
Nonossifying fibroma (choice A) is also known as fibrous cortical defect. It is a common
developmental abnormality seen in the bones of the lower extremities of children. They are nonneoplastic lesions of bone cortex that are composed of fibrous connective tissue and that
usually resolve spontaneously. X-ray reveals irregular, well-demarcated radiolucent defects in
the bony cortex with an intact subperiosteal shell of bone. In the metaphysis, there are whorls
of connective tissue. These fibromas do not cross the epiphyses of bone, which distinguishes
them from giant cell tumors of bone.
Osteochondromas (choice B) are hereditary multiple exostoses (bony metaphyseal projections
capped with cartilage) that may be asymptomatic or may produce deformity and compromise the
blood supply of bone. Five percent of them progress to sarcomas. If a patient presents with
exostoses, sebaceous cysts, dermoid tumors, and colonic polyps, the likely diagnosis is
Gardner's syndrome.

148
Osteomyelitis (choice C) usually produces fever, localized pain, erythema, and swelling. The
patient in question is afebrile, which decreases the likelihood of this diagnosis. Though x-ray
may show periosteal elevation, more specific findings would be expected in a patient with this
condition, such as sequestrum (necrotic bone fragment), involucrum (new bone that surrounds the
area of inflammation), and Brodie's abscess (localized abscess formation in the bone).
Paget's disease (choice E), also known as osteitis deformans, is due to excessive bone
resorption with replacement by soft, poorly mineralized matrix (osteoid) in a disorganized
array. It generally affects the skull, pelvis, femur, and vertebrae. Skull involvement might
produce deafness by impinging on the cranial nerves. Malignant transformation to osteosarcoma
is seen in 1% of cases. X-ray reveals enlarged, radiolucent bones. Lab tests reveal extremely
elevated alkaline phosphatase. If you are given a patient over 40 with bone fracture, hearing
loss, and increased alkaline phosphatase, suspect Paget's disease. Note that this disease
rarely occurs in the young and could have been ruled out as a possible answer by virtue of the
patient's age.
A 38-year-old woman with multiple sclerosis (MS) has stable neurologic deficits resulting from old
demyelinated plaques. Which of the following histopathologic features would be prominent in
this patient's old plaques?
A. Complete loss of axons
B. Gliosis
C. Histiocytic infiltration
D. Lymphocytic infiltration
E. Myelin breakdown
Explanation:
The correct answer is B. Demyelinating plaques in MS mature through an orderly sequence of
events. Acute plaques show dense lymphohistiocytic infiltration and active digestion of myelin
byproducts. These features tend to disappear as plaques age. As inflammatory infiltration and
myelin breakdown abate, hyperplasia and hypertrophy of astrocytes transform the plaque into a
gliotic area, in which axons are relatively preserved but oligodendroglial cells are greatly
diminished. The term gliosis indicates proliferation of glial cells and is usually used as a
synonym of astrocytosis. Astrocytosis occurs as a nonspecific reaction to any damage to the
CNS, e.g., loss of neurons following hypoxic injury, inflammation due to infectious agents, and
neoplastic invasion. Hypertrophic astrocytes acquire abundant eosinophilic cytoplasm and are
known as gemistocytes.
Loss of axons occurs in MS plaques to some degree but is never complete (choice A). In fact,
axons can be remarkably spared, and loss of function in MS is thought to result from impaired
conduction along entirely denuded or incompletely remyelinated nerve fibers.
Histiocytic and lymphocytic infiltration (choices C and D) are prominent in the acute stage of
myelin destruction. Lymphocytes belong to both CD4 (helper lymphocytes) and CD8 (cytotoxic
lymphocytes) subgroups and are key players in MS immune-mediated pathogenesis. Histiocytes act
not only as scavengers that digest myelin debris, but also as important antigen-presenting
cells. As the acute stage of myelin destruction resolves, lymphocytes and histiocytes fade
away. Evidence of myelin breakdown (choice E), therefore, is minimal or absent in chronic
stages.
A patient who is being treated for leukemia develops unilateral flank pain. Radiologic studies

149
demonstrate a dilated renal pelvis and dilation of the upper one-third of the corresponding
ureter. A stone with which of the following compositions is most likely causing this patient's
problems?
A. Calcium salts
B. Cholesterol
C. Cystine
D. Struvite
E. Uric acid
Explanation:
The correct answer is E. Patients prone to develop uric acid stones include those with gout,
leukemia (particularly during chemotherapy, which releases large amounts of nucleic acids from
dying leukemia cells), and acidic urine.
Calcium-containing stones (choice A) are the most common kind in the general population.
Cholesterol stones (choice B) are found in the gall bladder.
Cystine stones (choice C) are a rare form of renal stone seen in patients with cystinuria.
Struvite (magnesium-ammonium phosphate) stones (choice D) are known for producing massive
staghorn stones that fill the renal pelvis.
A 5-year-old girl is thoroughly evaluated because of growth failure. The child has been complaining of
headaches, which are exacerbated when she tries to read. Funduscopic examination reveals
papilledema. CT scan demonstrates a mass involving the area above and within the sella turcica.
Surgical resection of the mass yields a multiloculated cystic and solid tumor containing dark
brown, oily fluid.This tumor is thought to arise from epithelial rests derived from which of the
following structures?
A. Hypothalamus
B. Pineal gland
C. Posterior pituitary gland
D. Rathke's pouch
E. Superior colliculus
Explanation:
The correct answer is D. The tumor described is a craniopharyngioma, which is one of the more
common brain tumors of children. These tumors arise from epithelial rests derived from Rathke's
pouch, which is an oral invagination that gives rise to the cells that form the anterior
pituitary gland. Histologically, craniopharyngiomas can resemble ameloblastomas, which are
tumors derived from dental epithelium.
Note that this question could also have been answered very simply by noting that the
hypothalamus, pineal gland, pituitary gland, and the superior colliculus are all adult

150
structures; only Rathke's pouch is an embryonic structure. Therefore, only Rathke's pouch could
be the source of epithelial rests, which are remnants of embryonic tissues that persist in the
adult.
While craniopharyngiomas often occur in close proximity to the hypothalamus (choice A) and
posterior portion of the pituitary gland (choice C), they do not arise from neural tissue.
The pineal gland (choice B) and superior colliculus (choice E) are found on the posterior
aspect of the brain stem.
A 15-year-old patient is taken to a physician because of severe episodic headaches, accompanied by
perspiration and palpitations. The patient is experiencing a headache at the time of the
examination and his blood pressure is 175/125 mm Hg with regular heart rate of 90. Treatment
with phenoxybenzamine relieves his symptoms. Which of the following studies would be most
helpful for establishing the likely diagnosis?
A. Serum albumin
B. Serum beta-hCG
C. Serum carcinoembryonic antigen (CEA)
D. Urinary Bence-Jones proteins
E. Urinary vanillylmandelic acid (VMA)
Explanation:
The correct answer is E. The suspected tumor is pheochromocytoma, 10% of which occur in
children. The symptoms (paroxysmal hypertension, palpitations, anxiety) are produced when the
tumor secretes epinephrine, norepinephrine, and other vasoactive amines into the circulation.
The diagnosis can be established with plasma catecholamine concentrations or concentrations of
the norepinephrine metabolite VMA in a 24-hour urine. The latter offers the advantage of
providing a longer time sample so that the intermittent secretion is more likely to be picked
up.
Serum albumin (choice A) can be low in liver and renal disease, but is unaffected in
pheochromocytoma.
beta-hCG (choice B) is a marker for choriocarcinoma and related lesions.
CEA (choice C) is a nonspecific marker that is positive in many cases of colorectal and
pancreatic cancer. Bence-Jones proteins (choice D) are a marker for multiple myeloma, representing
urinary excretion of myeloma light chains.
A 44-year-old man goes to his physician for an employment physical examination. The clinician notices
that the patient's fingers are clubbed. He also demonstrates tenderness over the distal ends of the
radius, ulna,and fibula. This patient should be explicitly evaluated to exclude which of the following?
A. Gastrointestinal cancer
B. Liver cancer
C. Lung cancer
D. Renal cancer

151

E. Testicular cancer
Explanation:
The correct answer is C. This patient has the finger clubbing and hypertrophic pulmonary
osteoarthropathy that can be associated with bronchogenic carcinoma (other than squamous cell
carcinoma), benign mesothelioma, and diaphragmatic neurilemmoma. X-ray of the bones generally
shows formation of new periosteal bone; arthritis may be present. The etiology of these changes
remains a mystery. An alert clinician may identify a cancer at an earlier, potentially curable
stage by investigating a possible paraneoplastic syndrome.
After four days, a neonate has not passed meconium, and begins vomiting. Physical examination reveals
abdominal distension. Which of the following is the correct diagnosis?
A. Hirschsprung's disease
B. Meckel's diverticulum
C. Omphalocele
D. Renal agenesis
E. Tracheoesophageal fistula
Explanation:
The correct answer is A. The neonate is suffering from Hirschsprung's disease, which is caused
by an absence of ganglion cells in both the submucosal and the inter-myenteric plexus of a
segment of bowel. The aganglionic bowel segment is narrowed because the lack of peristalsis
keeps stool from moving into that segment. The distal rectum is always involved, but the lesion
extends proximally anywhere from a few centimeters all the way to the small intestine. The
bowel proximal to the lesion is usually dilated. Treatment is by surgical removal of the
affected segment.
A Meckel's diverticulum (choice B) is a small, usually asymptomatic pouch located near the
ileocecal valve.
An omphalocele (choice C) would present with an obvious sac filled with intestines at the site
of a defect in the ventral abdominal wall.
Renal agenesis (choice D) would present with failure to pass urine.
Tracheoesophageal fistula (choice E) would present with aspiration during feeding.
A 15-year-old male experiences sporadic episodes of severe muscle weakness. He has noticed that the
episodes occur after severe exercise and after large meals rich in carbohydrates. The patient
has also had nighttime attacks, from which he awoke nearly paralyzed. He notes that similar
problems have occurred in his father and brother. Serum chemistries performed during one of the
patient's attacks would most likely demonstrate which of the following?
A. Decreased bicarbonate
B. Decreased calcium
C. Decreased potassium

152

D. Increased glucose
E. Increased sodium
Explanation:
The correct answer is C. This is a classic presentation of the rare condition known as
hypokalemic periodic paralysis. There is often a family history suggesting autosomal dominant
inheritance. Symptoms tend to appear late in the first decade or in the second decade of life
(when physical education instructors or other kids may criticize the child for being weak). In
addition to the triggers listed in the question stem, other precipitating factors include
tension, anxiety, and a habitual high-salt diet. The condition may be difficult to demonstrate,
since serum chemistries are normal between attacks. Muscle biopsy may demonstrate vacuolation
or damage to myofibrils. The pathophysiology of this condition was previously poorly
understood, but recent advances suggest that the primary defect is in calcium channels.
Treatment is with potassium supplementation during acute attacks, and prophylactic
acetazolamide, triamterene, amiloride or spironolactone.
Decreased serum bicarbonate (choice A) is a characteristic of metabolic acidosis.
Hypocalcemia (choice B) can cause muscle weakness (and more seriously, cardiac arrhythmias),
but this patient's history suggests a specific alternative etiology. Although calcium channels
may be involved in this disorder, the serum calcium level is generally normal.
Increased serum glucose (choice D) is seen in diabetes mellitus, not hypokalemic periodic
paralysis.
Hypernatremia (choice E) is usually caused by a failure to replace water lost from the body,
due to an inability to obtain or drink water, or rarely, by primary disorders of the thirst
mechanism or excessive ingestion of sodium.
A 50-year-old woman presents to her physician with fatigue and a 30 lb weight loss. A peripheral smear
and bone marrow biopsy demonstrate a leukemia composed of cells in the neutrophil, eosinophil,
and basophil lineages. No blast forms are seen. Which of the following is associated with this
leukemia?
A. bcl-2 activation
B. c-myc activation
C. t(8, 14)
D. t(9, 22)
E. t(14,18)
Explanation:
The correct answer is D. The disease is chronic myelogenous leukemia (CML), which usually
affects adults between 40 and 59 years of age. CML is associated with the Philadelphia
chromosome, which is actually a translocation involving chromosomes 9 and 22 that produces an
abl-bcr hybrid.
bcl-2 activation (choice A) and a t(14,18) (choice E) translocation are features of follicular
lymphomas.

153

c-myc activation (choice B) and a t(8,14) (choice C) translocation are features of Burkitt's
lymphoma.
Which of the following is the most frequent primary malignant tumor of the CNS?
A. Glioblastoma multiforme
B. Medulloblastoma
C. Meningioma
D. Oligodendroglioma
E. Pituitary adenoma
Explanation:
The correct answer is A. Glioblastoma multiforme (GBM) is the most frequent primary malignant
tumor of the CNS. It usually affects middle-aged or elderly patients and most frequently arises
in the cerebral white matter. GBM is a malignant astrocytoma; it is referred to as grade IV
astrocytoma in the WHO classification and grade 4 astrocytoma in the St. Anne-Mayo grading
system. This tumor is very aggressive; median survival is approximately 12-14 months following
optimal treatment, ie, a combination of surgery and radiation therapy.
Medulloblastoma (choice B), a much less frequent malignant tumor, affects children; it arises
in the posterior fossa in this population. The histogenetic origin of this tumor is uncertain,
although it is thought to develop from immature cell precursors that give rise to glial and
neuronal cells.
Meningioma (choice C) and pituitary adenoma (choice E) are frequent but benign brain tumors
that arise from meningothelial cells and endocrine cells of the adenohypophysis, respectively.
Oligodendrogliomas (choice D) constitute 5% of all primary brain tumors. Although most
oligodendrogliomas have benign histologic features (e.g., low mitotic rate and mild nuclear
atypia), their complete surgical excision is virtually impossible owing to a diffuse, poorly
circumscribed pattern of growth. This tumor allows a much longer survival (up to 7-10 years)
than GBM, but eventually transforms into a malignant glioma indistinguishable from GBM.

A homeless man is brought in to the emergency room by the police after they observe him staggering down
the street in a disoriented state. He appears malnourished and smells like wine. The edge of
the liver is difficult to palpate, but feels nodular to the examiner. Also noted on physical
examination are multiple bruises, and a new bruise is produced by the blood pressure cuff.
Ocular examination reveals horizontal nystagmus on lateral gaze, and bilateral lateral rectus
palsies. Deficiency of which of the following vitamins is most likely responsible for the
neurological problems observed in this patient?
A. Folate

154
B. Thiamine
C. Vitamin B12
D. Vitamin C
E. Vitamin K
Explanation:
The correct answer is B. This is an example of Wernicke-Korsakoff syndrome, which is caused by
thiamine deficiency, usually secondary to chronic alcohol abuse. The acute symptoms are
referred to as Wernicke's encephalopathy and include confusion, ataxia, vestibular dysfunction,
sluggish pupillary light reflexes, anisocoria, and oculomotor dysfunction (the lateral rectus
palsies are caused by abducens nerve lesions). If untreated, chronic symptoms including
anterograde amnesia and confabulation may occur (Korsakoff's syndrome).
Deficiency of folate (choice A) can cause megaloblastic anemia, but is not usually associated
with neurological problems, with the possible exception of neuropathies.
Lack of vitamin B12 (choice C) could cause megaloblastic anemia and spinal cord damage,
producing ataxia, but would not produce the cranial nerve deficits observed in this individual.
Coexistent vitamin C (choice D) deficiency may be present in alcoholics, and contributes to
easy bruising because of small vessel fragility caused by poor collagen formation around small
blood vessels.
Alcoholic patients with hepatic cirrhosis may have impaired ability to synthesize vitamin K
(choice E) dependent blood clotting factors, leading to increased bleeding and increased
prothrombin and partial thromboplastin times.
A 56-year-old male presents with complaints of gnawing pain in the midepigastrium, with occasional
radiation to the back. He also notes a 15 pound weight loss over the last 3 months. The
clinician suspects pancreatic carcinoma. Which of the following tumor markers would aid in
confirming this diagnosis?
A. AFP

155

B. CA-125
C. CEA
D. hCG
E. PSA
Explanation:
The correct answer is C. Tumor markers can be very helpful in narrowing the possible primary
sources for metastatic lesions. Carcinoembryonic antigen (CEA) can be seen in any tumor derived
from gut epithelium, notably colon cancer and pancreatic cancer. Tumor markers should not be
used as the primary tool for cancer diagnosis, but they have considerable utility in the
confirmation of the diagnosis, as well as for monitoring recurrence or response to therapy.
Alpha-fetoprotein (AFP; choice A) is seen in hepatocellular carcinoma, embryonal cell tumor of
the testis, and malignant teratoma.
CA-125 (choice B) is produced by ovarian cancer.
The beta subunit of human chorionic gonadotropin (hCG; choice D) is seen in choriocarcinoma,
hydatidiform mole, and germinoma.
Prostate-specific antigen (PSA; choice E) is seen in prostatic carcinoma.
You should also remember that serum elevations of many of these markers can also be seen in
some benign conditions of similar tissues. HCG, for example, is elevated normally in pregnancy.
A patient has a lymphoproliferative disorder containing mature B cells that mark with weak kappa light
chain expression on the plasma membrane, strong CD5, strong CD23, and weak CD22. What is the
most likely diagnosis?
A. Chronic lymphocytic leukemia
B. Hairy cell leukemia

156

C. Non-Hodgkin's lymphoma
D. Prolymphocytic leukemia
E. Szary syndrome
Explanation:
The correct answer is A. The immunophenotype of chronic lymphocytic leukemia is distinctive
among clonal B-cell disorders. There is usually strong expression of both CD5 and CD23, and
weak expression of surface immunoglobulin (either kappa or lambda chain) and CD22.
In contrast, other disorders that contain mature B cells, such as hairy cell leukemia (choice
B), non-Hodgkin's lymphoma (choice C) (including follicular and mantle cell lymphomas), and
prolymphocytic leukemia (choice D), typically have strong expression of surface immunoglobulin
and CD22. There is usually absent expression of CD5 (may be seen in prolymphocytic leukemia and
some non-Hodgkin's lymphomas) and CD23 (may be seen in some non-Hodgkin's lymphomas).
Szary syndrome (choice E) is a form of T-cell lymphoma/leukemia.
A 70-year-old man comes to clinical attention with progressive memory loss, urinary incontinence, and
gait instability. MRI of the brain shows dilatation of the ventricular cavities, while the
cerebral cortex appears normal. No infarcts are seen. Repeated lumbar punctures reveal
occasional increases in cerebrospinal fluid (CSF) pressure. A biopsy of the frontal cortex
demonstrates the absence of neurofibrillary tangles and senile plaques. Which of the following
is the most appropriate treatment?
A. Acetylcholinesterase inhibitors such as tacrine
B. Aspirin
C. L-DOPA
D. Ventricular shunt
E. Vitamin B1 (thiamine)

157

Explanation:
The correct answer is D. This is an "interdisciplinary" question, which begins with a clinical
scenario and ends by testing your knowledge of pathophysiology and pharmacology. The clinical
presentation has the classic triad of memory loss, urinary incontinence, and gait abnormalities
characteristic of normal pressure hydrocephalus (recently renamed intermittently raised
pressure hydrocephalus). This condition results from intermittent increases in CSF pressure,
leading to progressive damage to cerebral white matter and dilatation of the ventricles. No
significant cortical atrophy is present. The lack of Alzheimer-related changes in the biopsy
and the measurements of CSF pressure support the diagnosis. Relief of CSF pressure can be
obtained by placing a ventricular shunt, which usually improves clinical symptomatology.
Tacrine and other inhibitors of acetylcholinesterase (choice A) are used in the symptomatic
therapy of Alzheimer disease. The pathology of this disease includes degeneration of cerebral
cholinergic systems, such as the basal nucleus of Meynert, which project to the neocortex and
are involved in memory and learning. Acetylcholinesterase degrades acetylcholine after release
from presynaptic boutons. Acetylcholinesterase inhibitors therefore enhance the concentration
of acetylcholine and amplify cholinergic neurotransmission.
Aspirin (choice B) is used to prevent aggregation and adhesion of platelets in patients with
risk factors for cerebrovascular disease. Prophylactic aspirin treatment lowers the incidence
of transient ischemic attacks and brain infarcts.
L-DOPA (choice C) is a precursor of dopamine, the neurotransmitter of the nigrostriatal neurons
that degenerate in Parkinson disease. L-DOPA, especially in combination with a peripheral
decarboxylase inhibitor, is therefore used in the treatment of this disorder.
Vitamin B1(choice E) is used in the treatment of Wernicke encephalopathy, which is caused by
thiamine deficiency. If untreated, Wernicke encephalopathy progresses to Korsakoff syndrome,
characterized by memory loss, confusion, and confabulation.
An immigrant from Brazil presents with difficulty swallowing, and is referred to a gastroenterologist
for evaluation of his dysphagia. Special studies demonstrate massive dilatation of the
esophagus. Which of the following other organs may also be seriously affected by the patient's
disease?

158
A. Bladder
B. Brain
C. Heart
D. Lungs
E. Small intestine
Explanation:
The correct answer is C. The disease is Chagas' disease, which is due to the intracellular
protozoan parasite, Trypanosoma cruzi. Chagas' disease is an important cause of cardiac failure
in some Latin American countries, and Brazilian cases can be associated with megaesophagus and
megacolon.
Involvement of other organs including bladder (choice A), brain (choice B), lungs (choice D)
and small intestine (choice E) is usually not clinically significant.
A 65-year-old man develops increased frequency of urination, nocturia, dysuria, and difficulty starting
and stopping the flow of urine. Serum prostate-specific antigen (PSA) is within normal limits,
and prostate biopsy demonstrates benign tissue. In which of the following sites is the patient's
lesion most likely located?
A. Bladder neck
B. Penile urethra
C. Periurethral prostate
D. Posterior prostate
E. Ureters
Explanation:

159
The correct answer is C. The man most likely has benign prostatic hyperplasia, which is thought
to be related to the growth-promoting effects of dihydrotestosterone (DHT). This is possibly
coupled with an age-related increase in estradiol, which may potentiate the action of DHT on
the prostate. The growth characteristically involves the periurethral prostate. In contrast,
prostatic carcinoma (which, if extensive, would elevate the PSA) characteristically involves
the posterior lobe of the prostate (choice D).
Stones and tumors can also involve the bladder neck (choice A), causing difficulty in
urination, but these are less common than benign prostatic hypertrophy.
Lesions of the penile urethra (choice B) can cause dysuria, but do not usually cause difficulty
stopping and starting urine flow.
Lesions of the ureters (choice E) can cause renal failure, but would not cause difficulties
with urination.
An elderly patient dies with chronic dementia. At autopsy, the brain shows diffuse cortical atrophy with
relative sparing of primary motor and sensory areas. Which of the following would most likely
be a prominent feature on microscopic examination of her brain tissue?
A. Central chromatolysis
B. Lewy bodies
C. Gliosis of the caudate nucleus
D. Loss of pigmented neurons
E. Neurofibrillary tangles
Explanation:
The correct answer is E. The clinical history and gross pathology suggest Alzheimer's disease.
Neurofibrillary tangles and senile plaques are found primarily in higher order association
cortex. Other microscopic features of Alzheimer's disease include granulovacuolar degeneration
and, sometimes, Hirano bodies.

160
Central chromatolysis (choice A) is a loss of histological staining seen in lower motor neurons
whose axons have been damaged, for example by injury to the ventral roots.
Gliosis of the caudate nucleus (choice C) is a feature of Huntington's disease, and certain
other degenerative diseases of the basal ganglia.
Lewy bodies (choice B) (eosinophilic intracytoplasmic spherules) and loss of pigmented neurons
(choice D) in the substantia nigra are features of Parkinson's disease.
A patient has severe arthritis involving the lower back. Before making a diagnosis of ankylosing
spondylitis, the patient should be questioned about which of the following diseases?
A. Carcinoid syndrome
B. Celiac disease
C. Crohn's disease
D. Peptic ulcer
E. Whipple's disease
Explanation:
The correct answer is C. 10-20% of patients with Crohn's disease and ulcerative colitis
develop an arthritis that resembles ankylosing spondylitis. Similar arthropathies are seen in
psoriasis, Reiter's syndrome (arthritis, urethritis, conjunctivitis, and rash following
chlamydial infection, and related syndromes seen following Shigella, Salmonella, or Yersinia
enterocolitis). The other answers are distracters.
A child develops a tumor of the cerebellum. Biopsy reveals evidence of both neuronal and glial
differentiation. Which of the following is the most likely diagnosis?
A. Astrocytoma
B. Glioblastoma multiforme

161
C. Medulloblastoma
D. Meningioma
E. Oligodendroglioma
Explanation:
The correct answer is C. Medulloblastoma is the only tumor on the list characterized by both
neuronal and glial differentiation. This tumor tends to form gray-white masses in the
cerebellum of children and in the cerebral hemispheres of adults. Histologically, the tumor
forms sheets of densely packed, poorly differentiated small cells. Occasional cells show
spindle cell morphology and stain for the glial marker glial fibrillary acidic protein (GFAP).
Other occasional cells show neuronal differentiation and form rosettes. The tumor has a 50% 10year survival rate when treated with combined surgery and radiation.
Astrocytoma (choice A), glioblastoma multiforme (choice B), and oligodendroglioma (choice C)
are pure glial tumors.
Meningioma (choice D) is a tumor of the meninges.
A concerned mother brings her 6-year-old son to the pediatrician because he has developed a "puffy
face." On examination, the child has a blood pressure of 90/60, marked periorbital edema, and
pitting edema of both the hands and feet. His cardiac exam is unremarkable and he has no
splenomegaly or signs of liver disease. Laboratory values are notable for decreased serum
albumin and increased total cholesterol. A urinalysis reveals 4+ proteinuria but no red blood
cells, white blood cells, or casts. The most likely cause of this child's disease is
A. Goodpasture's syndrome
B. minimal change nephropathy
C. poststreptococcal glomerulonephritis
D. systemic lupus erythematosus
E. Wegener's granulomatosis

162

Explanation:
The correct answer is B. This boy has nephrotic syndrome: edema, heavy proteinuria with a
benign urinary sediment, hypoalbuminemia, and hypercholesterolemia. Of the choices listed, only
minimal change nephropathy is a recognized cause of nephrotic syndrome. It is the most common
cause of nephrotic syndrome in children.
All the other choices are not consistent with the clinical history and are causes of nephritis,
not nephrosis. Remember that nephritic patients present with hypertension, moderate
proteinuria, hematuria, and an active urinary sediment with red blood cell casts.
A 30 year-old man comes to medical attention because of fevers and night sweats. CT scan reveals masses
involving the mediastinal nodes, abdominal nodes, and liver. Biopsy of one of the nodes
demonstrates a nodular lymphoma with nodular areas composed predominately of small (6-12
micrometer) lymphocytes with scanty cytoplasm and irregular, twisted, and indented nuclei. This
lymphoma is most closely linked to a translocation involving which of the following?
A. Immunoglobulin heavy chain and bcl-1
B. Immunoglobulin heavy chain and bcl-2
C. Immunoglobulin heavy chain and myc
D. Kappa immunoglobulin light chain and myc
E. Lambda immunoglobulin light chain and myc
Explanation:
The correct answer is B. The lymphoma described in the question stem is the follicular type of
small cleaved cell (B cell) lymphoma, and its characteristic translocation is t(14;18)
(q32;q21). The site on chromosome 14 involves the heavy chain of immunoglobulins (IgH), while
that on chromosome 18 involves the bcl-2 oncogene. The translocation produces a hybrid
bcl-2/immunoglobulin heavy-chain transcript.
A t(11,14)(q13;q32) translocation involving bcl-1 and IgH (choice A) is seen in small non-

163
cleaved lymphocytic lymphoma, myeloma, and some mantle cell lymphomas.
Burkitt's lymphoma is usually associated with t(8;14) (myc and IgH; choice C), but sometimes
with t(2;8) (kappa immunoglobulin light chain and myc; choice D) or t(8;22) (lambda
immunoglobulin light chain and myc; choice E).
A baby is born with a testicular mass. Histologic sections made of the homogeneous yellow-white mass
after its removal demonstrate epithelial-lined spaces that have flattened-to-cuboidal epithelial
cells with vacuolated cytoplasm containing eosinophilic, hyaline-like globules. Scattered
structures resembling primitive glomeruli (endodermal sinuses) are also seen. If appropriate
immunohistochemical stains are performed, the eosinophilic cytoplasmic globules would most
likely contain which of the following?
A. Alpha-fetoprotein
B. Estrogen receptors
C. Human chorionic gonadotropin
D. Human papilloma virus
E. Melanin
Explanation:
The correct answer is A. The malignant tumor is a yolk sac tumor, also known as infantile
embryonal carcinoma and endodermal sinus tumor. The biggest tip-off in the question stem is the
presence of endodermal sinuses that resemble primitive glomeruli. The cytoplasmic globules
described contain alpha-fetoprotein, indicating yolk cell differentiation, and alpha-1antitrypsin. Alpha-fetoprotein can also be used as a serum marker for recurrent disease. Yolk
sac tumors occur in pure form in infants and children and may be part of mixed tumors in
adults.
Estrogen receptors (choice B) are important markers in breast cancer because they predict tumor
response to hormonal manipulation.
Human chorionic gonadotropin (choice C) is found in the syncytial cells of embryonal carcinoma.

164

Human papilloma virus (choice D) can be found in condylomas, cervical cancer, penile cancer,
laryngeal polyps, and warts.
Melanin (choice E) can be found in melanomas.
Physical examination of a 45-year-old diabetic patient demonstrates a pulsatile abdominal mass.
Radiographic studies demonstrate a 10 cm diameter aneurysm of the abdominal aorta with foci of
calcification in the walls. Which of the following is the most likely etiology for this
aneurysm?
A. Atherosclerosis
B. Congenital weakness
C. Cystic medial necrosis
D. Syphilis
E. Vasculitis
Explanation:
The correct answer is A. This is a typical presentation of an abdominal aortic aneurysm, which
is almost always due to severe atherosclerosis. The foci of calcification described occur
within the atherosclerotic plaques, and indicate severe atherosclerotic disease.
Congenital weakness of vessels (choice B) can produce berry aneurysms, especially in cerebral
vessels in the circle of Willis.
Cystic medial necrosis (choice C) can produce dissecting aneurysms, especially in Marfan's
syndrome.
Syphilitic aneurysms (choice D) typically involve the aortic root as it leaves the heart.
Vasculitis (choice E) can produce aneurysms in small arteries.

165
A nontender, enlarged lymph node develops in the neck of a 17-year-old girl. On biopsy, the lymph node
shows a diffuse effacement of its architecture, with presence of fibrous bands creating a
nodular pattern. Numerous lacunar cells and scattered Reed-Sternberg cells are present among a
mixed population of lymphocytes, eosinophils, macrophages, and plasma cells. Which of the
following is the most likely diagnosis?
A. Diffuse large cell lymphoma
B. Hodgkin disease, lymphocyte depletion
C. Hodgkin disease, lymphocyte predominance
D. Hodgkin disease, mixed cellularity
E. Hodgkin disease, nodular sclerosis
Explanation:
The correct answer is E. The presence of Reed-Sternberg (RS) cells is pathognomonic of Hodgkin
disease (HD). After long debate, RS cells have been recognized as the true neoplastic elements
of HD. A classic RS cell is a giant, usually binucleated cell, with large "inclusion-like"
nucleoli. The nuclei are similar to each other, giving the impression of mirror images. Classic
RS cells and lacunar cells are probably transformed B lymphocytes and are positive for CD15 and
CD30. Variants of RS cells are found in different subtypes of HD. Lacunar cells are
mononucleated and surrounded by a pericellular halo. The lymphohistiocytic (L+H) cell is a
variant of RS cell with a characteristic convoluted nucleus resembling a popcorn kernel
("popcorn cell"). The nodular sclerosis subtype affects adolescents and young adults.
Histologically, it is characterized by the lacunar cell variant and the presence of broad bands
of fibrous tissue that divide the lymph node into nodules. Nodular sclerosis HD has an
excellent prognosis.
Lymphocyte depletion HD (choice B) is rare and controversial. Most cases are large cell
lymphomas, affecting older males with disseminated disease.
Lymphocyte predominance HD (choice C) accounts for 6% of all cases. This subtype is
characterized by the L+H variant of the RS cell. The cells are CD15- and CD30-, but CD20+. This
lymphoma is of follicular B-cell origin and mostly affects young males with cervical

166
lymphadenopathy.
The mixed cellularity (choice D) form of HD is characterized by frequent RS cells. There is a
biphasic incidence curve, with peaks affecting young adults and those older than 55. This
lymphoma presents at advanced stages.
Diffuse large cell lymphoma (choice A) is an aggressive B-cell lymphoma that represents 20% of
non-Hodgkin lymphomas. It affects older people. Neoplastic lymphocytes are large, with
prominent nucleoli. The histopathologic and clinical features of this case clearly rule out
this form of lymphoma.
A young adult has progressive intellectual deterioration, weakness, ataxia, and seizures. Reference
laboratory tests demonstrate an abnormality of an important mitochondrial enzyme. From which of
the following diseases is this person most likely suffering?
A. Adrenoleukodystrophy
B. Central pontine myelinolysis
C. Krabbe's disease
D. Leigh's disease
E. Metachromatic leukodystrophy
Explanation:
The correct answer is D. Leigh's disease, also known as subacute necrotizing
encephalomyelopathy, is a very rare disease that most physicians will never encounter in real
life. However, it may show up on the USMLE because it is one of the few examples of diseases
involving defects in the mitochondrial DNA rather than the chromosomal DNA. The specific
defective enzyme, in at least some cases, is cytochrome oxidase, one of the components of the
electron transport system. Muscle and brain are particularly affected. Patients have the
presentation described in the question and typically die within several years.
Adrenoleukodystrophy (choice A) is an X-linked disease with abnormal lipid metabolism leading
to demyelination in the CNS.

167

Central pontine myelinolysis (choice B) is characterized by demyelination of the pons


(especially the basis pontis) seen after overlying rapid correction of hyponatremia.
Krabbe's disease (choice C) is an autosomal recessive accumulation of cerebrosides in
histiocytes in the CNS.
Metachromatic leukodystrophy (choice E) is an autosomal recessive accumulation of sphingolipids
in the CNS and elsewhere.
A 6-month-old child with sickle cell anemia has a chronically enlarged spleen. By 5 years of age, the
child's spleen is no longer palpable. The decrease in size is most likely related to which of
the following?
A. Chronic infection
B. Hodgkin's lymphoma
C. Infarctions
D. Non-Hodgkin's lymphoma
E. Portal hypertension
Explanation:
The correct answer is C. Early in sickle cell anemia, affected children usually have an
enlarged spleen. The spleen enlarges because the children are in a state of hypersplenism, in
which the spleen vigorously phagocytizes any abnormally shaped erythrocytes. During the first
decade of life, sickling within the spleen occludes its blood flow, causing repeated small
infarctions, leading to "autosplenectomy." The autosplenectomy is actually helpful to the
patient hematologically, as it may ameliorate the chronic anemia. Unfortunately, the lack of a
functional spleen renders the patients susceptible to infections with Salmonella and
encapsulated organisms such as Streptococcus pneumoniae and Haemophilus influenzae.
Patients with sickle cell anemia are not particularly vulnerable to chronic infection (choice
A), although acute infections can trigger painful crises.

168

Patients with sickle cell anemia do not have an increased incidence of either Hodgkin's disease
(choice B) or non-Hodgkin's lymphoma (choice D).
Portal hypertension (choice E) causes an enlarged, rather than a small, spleen.
A 2-week-old child shows difficulty feeding, somnolence, failure to thrive, and constipation. An alert
pediatrician orders blood studies that demonstrate low T4 and high TSH. If appropriate therapy
is not promptly instituted, which of the following complications would most likely occur?
A. Bronchiectasis
B. Gigantism
C. Hepatic cirrhosis
D. Mental retardation
E. Renal failure
Explanation:
The correct answer is D. The disease is infantile hypothyroidism, which can produce the
syndrome of cretinism if left untreated. Early manifestations include those listed in the
question stem. Later manifestations of untreated disease include impaired skeletal growth,
protruding abdomen, dry skin, and, most seriously, neurologic problems that typically include
deaf-mutism, spasticity, and profound mental retardation. Many hospitals routinely screen
neonates for hypothyroidism and institute appropriate thyroid hormone replacement therapy
immediately if necessary, thereby preventing the many problems these children would otherwise
have.
Associate bronchiectasis (choice A) with the inherited childhood disease cystic fibrosis.
Associate gigantism (choice B) with pituitary tumors, which can be congenital.
Associate hepatic cirrhosis in childhood (choice C) with the inherited disease alpha-1antitrypsin deficiency.

169

Associate renal failure in infancy (choice E) with congenital anomalies and renal cystic
diseases.
Physical examination of a perimenopausal woman demonstrates bony swellings in the distal interpharyngeal
joints; no inflammation is apparent. The proximal interphalangeal joints, hands, and wrists are
not involved, and the patient is asymptomatic. Which of the following processes is
etiologically related to these findings?
A. Autoantibody formation
B. Bacterial infection
C. Crystal deposition
D. Joint trauma
E. Viral infection
Explanation:
The correct answer is D. The disease is osteoarthritis, and the bony swellings are called
"Heberden's nodes", which may or may not be symptomatic. Osteoarthritis is thought to be
related to repetitive joint trauma.
Autoantibodies (choice A) are important in rheumatoid arthritis.
Bacterial infections (choice B) cause septic arthritis.
Crystal deposition (choice C) is important in gout and pseudogout.
Viral infections (choice E) can cause transient arthralgias and arthritis.
Physical examination of a two-year-old child reveals a large abdominal mass. CT studies show that the
mass arises in the kidney. Which of the following is most likely a distinctive feature of this
lesion?

170
A. Abundant clear cells
B. Embryonic glomerular structures
C. Eosinophilic cells packed with mitochondria
D. Hamartomatous blood vessels
E. Necrotic bacterial forms in macrophages
Explanation:
The correct answer is B. A two-year-old with a tumor of the kidney most likely has Wilms tumor.
This aggressive lesion (now with a 90% survival rate thanks to modern management) is
microscopically interesting because it contains embryonic glomerular and tubular structures
embedded in a spindle cell stroma which may also contain smooth muscle, striated muscle, bone,
cartilage, fat, and fibrous tissue.
Abundant clear cells (choice A) are a feature of renal cell carcinoma.
Eosinophilic cells packed with mitochondria (choice C) suggests oncocytoma.
Hamartomatous blood vessels (choice D) suggest angiomyolipoma.
Necrotic bacterial forms in macrophages (choice E) suggest malakoplakia, which can cause mass
lesions in the kidney without being a true tumor.
A 51-year-old alcoholic male suddenly starts vomiting blood and eventually loses consciousness; his wife
finds him laying on the bathroom floor. He has no prior history of hematemesis and had not been
vomiting prior to the appearance of the blood. Which of the following conditions is the most
likely cause of this man's presentation?
A. Esophageal varices
B. Mallory-Weiss tear
C. Plummer-Vinson syndrome

171

D. Schatzki ring
E. Zenker's diverticulum
Explanation:
The correct answer is A. This is a classic presentation of life-threatening bleeding from
esophageal varices (tortuous, dilated, submucosal esophageal vessels). Sclerotherapy of the
vessels and pressure on the bleeding site with an "esophageal balloon" may temporarily control
the problem, but, unfortunately, bleeding often recurs and exsanguination is a frequent cause
of death in these patients.
In contrast, Mallory-Weiss tears (choice B) occur as a complication of repeated vomiting and do
not often result in this amount of blood loss.
The other esophageal conditions listed do not usually cause hematemesis.
Plummer-Vinson syndrome (choice C) consists of hypochromic, microcytic anemia, atrophic
glossitis, and esophageal webs.
Schatzki rings (choice D) are benign mucosal rings found at the squamocolumnar junction of the
esophagus, below the aortic arch.
Zenker's diverticulum (choice E) is an esophageal evagination at the junction of the pharynx
and esophagus.
A patient with long-standing, moderately severe anemia dies in an automobile accident. He was driving,
and an autopsy is performed to determine the cause of death. Which of the following cardiac
changes will most likely be seen when the heart is examined?
A. Endocarditis
B. Fatty change of myocytes
C. Fibrinous pericarditis

172
D. Fibrosis of valve leaflets
E. Frank myocardial infarction
Explanation:
The correct answer is B. Chronic anemia is not simply a disease of the blood. Other changes
that can occur with long-standing, moderately severe anemia include skin atrophy and
koilonychia (spoon-shaped nails, particularly in iron-deficiency anemia). Fatty change of
cardiac myocytes, renal proximal convoluted tubule cells, and centrilobular hepatic cells, and
degeneration of neurons of the cerebral cortex and basal ganglia are also seen. With time, some
of these cells may die, producing a patchy cell loss.
Endocarditis (choice A) usually has an infectious etiology.
Fibrinous pericarditis (choice C) typically is seen following myocardial infarction.
Fibrosis of valve leaflets (choice D) can be seen with aging or as a complication of infective
endocarditis.
Frank myocardial infarction (choice E) can occur in very severe anemia, particularly if
superimposed by an additional hypoxic insult, such as hemorrhage, shock, or pneumonia, but is
not typical of long-standing moderately severe anemia.
A leukemia patient who has undergone multiple courses of chemotherapy develops herpes simplex
encephalitis. Which of the following would you expect a CT scan of the patient's brain to show?
A. Generalized volume loss
B. Volume loss selectively in the basal ganglia
C. Volume loss selectively in the brainstem
D. Volume loss selectively in the cerebellum
E. Volume loss selectively in the temporal and frontal lobes

173
Explanation:
The correct answer is E. Herpes simplex can cause a necrotizing, hemorrhagic acute encephalitis
that may rapidly produce death. The encephalitis characteristically involves the lower portions
of the cerebral cortex, notably the temporal lobes and the base of the frontal lobes, possibly
because the infection spreads from the oropharynx.
A 32-year-old woman complains of a sore throat, increasing fatigue, and shortness of breath for the past
3 days. On physical examination, her temperature is 101.5 F and her heart rate is 100, with no
murmurs. A CBC with reticulocyte count shows 1500 white blood cells/mm3. The platelet count is
12 x 109/L. Bone marrow aspiration and biopsy demonstrates cellularity less than 25%, and the
corrected reticulocyte count is less than 1%. Which of the following is the most likely
diagnosis?
A. Anemia of chronic disease
B. Aplastic anemia
C. Myelodysplasia with myelofibrosis
D. Myelophthisic anemia
E. Pure red cell aplasia
Explanation:
The correct answer is B. Aplastic anemia is characterized by pancytopenia. The bone marrow
biopsy typically reveals a normal architecture with a decrease in cellularity to levels less
than 25% of normal. The corrected reticulocyte count (i.e., reticulocyte count patient's
hematocrit/45) is often 1%. Platelet counts range from 20 to 60 109/L, and the absolute
neutrophil counts are often less than 0.5 109/L. Aplastic anemia may be either hereditary
(e.g., Fanconi's anemia) or acquired. Certain pharmacologic agents, such as busulfan and
vincristine, often produce severe marrow hypoplasia or aplasia. Chloramphenicol, some
anticonvulsant drugs, phenylbutazone, and a variety of other agents may produce aplastic anemia
in an idiosyncratic manner. Aplastic anemias have also been linked to exposure to environmental
toxins, such as benzene and insecticides, and are often found to be preceded by viral
infections from cytomegalovirus, parvovirus, and hepatitis. The fever and sore throat in the

174
patient described in the question are consistent with cytomegalovirus infection.
Anemia of chronic disease (choice A) produces an isolated deficit of red cell production and
may resemble iron deficiency.
The myelodysplastic syndromes (e.g., myelodysplasia with myelofibrosis, choice C) are
characterized by replacement of the bone marrow with abnormal (dysplastic) stem cells and
ineffective hematopoiesis.
Myelophthisic anemia (choice D) is caused by infiltration of the bone marrow by expansile
lesions (e.g., metastatic carcinoma) that distort the marrow architecture and generally depress
the production of all three types of formed blood elements.
Pure red cell aplasia (choice E) would produce a selective deficit of the erythrocytic lineage,
not all three lineages.
A concerned couple bring in a 2-year-old boy with gonadal dysgenesis to a clinic for a follow-up visit.
The physician notices a large abdominal mass during his physical examination. Which of the
following disorders does the patient most likely have?
A. Renal cell carcinoma
B. Renal hamartoma
C. Squamous cell carcinoma of the bladder
D. Transitional cell carcinoma of the bladder
E. Wilms' tumor
Explanation:
The correct answer is E. The only childhood malignancy listed is Wilms' tumor, which commonly
presents in a toddler as a large abdominal mass. There is now a 90% survival rate for this
tumor with combined therapy with surgery, chemotherapy, and radiotherapy.
Renal cell carcinoma (choice A), squamous cell carcinoma of the bladder (choice C), and

175
transitional cell carcinoma of the bladder (choice D) are malignant tumors of adults.
Renal hamartoma (fibroma; choice B) causes a small, gray, benign module in the renal pyramids
and is usually only identified as an incidental finding at autopsy.
A 23-year-old African-American woman presents in the emergency room with severe abdominal pain and
evidence of dehydration. Her past medical history is significant for numerous pulmonary
infections and effusions of the knees. This patient's erythrocytes would be expected to exhibit
A. an absence of central pallor
B. deletion of all four alpha Hb genes
C. denaturation during deoxygenation
D. Heinz bodies on RBC staining
Explanation:
The correct answer is C. Whenever the Step 1 exam specifies a patient's ethnic background,
there is a good chance that it provides an important clue to the question. The patient
described is an African-American woman with symptoms and history suggestive of sickle cell
anemia. The woman is experiencing the most frequent type of vaso-occlusive crisis in sicklers
(infarctive). Crises may be precipitated by dehydration or infection. Hb electrophoresis is
used to diagnose the illness, which is caused by an amino acid substitution of valine for
glutamate in position 6 of the beta Hb chain. The resulting abnormal hemoglobin (HbS) tends to
polymerize during low oxygen states, causing the red blood cells to assume a sickle shape.
Absence of central pallor (choice A) may be observed in patients with hereditary spherocytosis
(usually autosomal dominant) resulting from abnormal red cell membrane proteins (e.g.,
spectrin, ankyrin). Spherocytes will exhibit increased osmotic fragility because they have
decreased surface area per unit volume.
Deletion of all four alpha genes (choice B) occurs in the most severe form of alpha thalassemia
and is incompatible with life. It can result in hydrops fetalis and intrauterine death.
Heinz bodies (choice D) are clumps of Hb-degradation products seen when RBCs are stained with

176
methylene blue. They are found in the peripheral blood smears of people with G6PD deficiency,
especially common in males of Mediterranean or West African descent. Hemolysis occurs in
response to oxidant stress that may be caused by drugs (e.g., antimalarials, sulfonamides, or
nitrofurantoin) or the ingestion of fava beans.
A woman presents to a dermatologist because she has lost almost all the hair on her body, including
scalp hair, eye brows, eye lashes, arm pit and groin hair, and the fine hairs on her body and
extremities. She most likely has a variant of which of the following?
A. Alopecia areata
B. Androgenic alopecia
C. Chronic cutaneous lupus erythematosus
D. Lichen planopilaris
E. Trichotillomania
Explanation:
The correct answer is A. Alopecia areata is caused by an autoimmune attack on hair follicles.
It has a wide range of clinical severity, with most cases involving a localized patch of hair
(which regrows within 1 year in half of the patients). The hair that does regrow may be gray or
depigmented. More severe cases can involve the entire scalp (alopecia totalis) or, as in this
patient, the entire body surface (alopecia universalis). These more severe cases are less
likely to resolve adequately. Treatment of alopecia areata is often unsuccessful, but topical
steroids are typically tried.
Androgenic alopecia (choice B) is common male pattern baldness.
Chronic cutaneous lupus erythematosus (choice C) can produce localized baldness.
Lichen planopilaris (choice D) can produce localized baldness.
Trichotillomania (choice E), also called traumatic alopecia, is alopecia due to trauma, such as
hair pulling or tight braids.

177

A 68-year-old female undergoes surgery to remove an abdominal tumor. Pathological examination shows that
the mass contains a large number of blood vessels. An increase in which of the following is the
most likely stimulus for the proliferation of these vessels?
A. Angiostatin
B. Growth hormone
C. Thrombospondin
D. Tissue oxygen partial pressure
E. Vascular endothelial growth factor
Explanation:
The correct answer is E. Vascular endothelial growth factor (VEGF) is a heparin-binding
glycoprotein that increases endothelial cell proliferation in vitro and increases capillary
growth (i.e., angiogenesis) in vivo. Unlike most other growth factors, VEGF has unique target
cell specificity for vascular endothelial cells. VEGF is overexpressed in solid tumors as well
as in ischemic areas of the heart and retina. VEGF levels are also reversibly increased in a
variety of normal and transformed cells exposed to a hypoxic environment. These characteristics
of VEGF make it an ideal candidate as a regulator of angiogenesis in physiological and
pathophysiological situations in which vessel growth is preceded by deficient perfusion of the
tissues.
Angiostatin (choice A) is an anti-angiogenic factor expressed by tumors that tends to inhibit
angiogenesis.
Growth hormone (choice B) has a general effect that causes growth of almost all tissues of the
body, but does not appear to stimulate angiogenesis to a significant extent in solid tumors.
Thrombospondin (choice C) is a multifunctional glycoprotein that interferes with tumor growth,
angiogenesis, and metastasis.
An increase in tissue oxygen partial pressure (choice D) is unlikely to be the stimulus for new

178
vessel growth because (1) solid tumors are invariably hypoxic or ischemic, i.e., the partial
pressure of oxygen is low; and (2) an increase in the partial pressure of oxygen in a tumor
would tend to decrease the expression of VEGF, thereby decreasing the amount of angiogenesis in
the tissues.
A neuropathologist is performing an autopsy on a 65-year-old man who had resting tremor, rigidity, and
akinesia. Histologic sections of the substantia nigra reveal neurons containing round
eosinophilic inclusions. Such inclusions most likely represent
A. granulovacuolar degeneration
B. Hirano bodies
C. Lafora bodies
D. Lewy bodies
E. Marinesco bodies
F. Negri bodies
G. neurofibrillary tangles
H. Pick bodies
Explanation:
The correct answer is D. This item lists a number of characteristic inclusions found in
pathologic conditions of the CNS. The trick here is to first identify the classic triad of
Parkinson disease (namely, tremors, rigidity, and akinesia), then recognize its typical
histopathologic hallmark: round, eosinophilic, intracytoplasmic inclusions seen in neurons of
substantia nigra. Lewy bodies are filamentous inclusions that appear brightly eosinophilic on
hematoxylin and eosin stain and accumulate in the cytoplasm of dopaminergic neurons in the
substantia nigra in Parkinson disease. All the other inclusions listed in the answer choices
have different pathologic significance.
Granulovacuolar degeneration (choice A) and Hirano bodies (choice B) are intracytoplasmic

179
inclusions found in hippocampal pyramidal neurons of Alzheimer patients and intellectually
intact elderly people.
Lafora bodies (choice C) contain polyglucosans and are found in neurons, glial cells, sweat
glands, liver, and skeletal muscles in patients with the rare Lafora disease.
Marinesco bodies (choice E) are also present in the dopaminergic neurons of substantia nigra.
However, they are intranuclear and are not associated with Parkinson disease. These inclusions
are found occasionally in normal brains, and their significance is unknown.
Negri bodies (choice F), which develop in the brains of patients with rabies, are filled with
viral particles and are frequently seen in Purkinje neurons (cerebellar cortex) and hippocampal
pyramidal neurons.
Neurofibrillary tangles (choice G) are intracytoplasmic filamentous inclusions found in
Alzheimer disease and, to a lesser extent, in normal aging brains.
Pick bodies (choice H) are characteristic of Pick disease, a form of frontotemporal dementia.
These are present in cortical neurons and appear as round intracytoplasmic inclusions that
stain dark with silver impregnation.
A 12-year-old girl with history of a streptococcal sore throat several weeks ago presents to her doctor
with an erythematous macular skin rash in a "bathing suit" distribution. Which one of the
following signs would be necessary to make the diagnosis of rheumatic fever?
A. Elevated erythrocyte sedimentation rate
B. Leukocytosis
C. Migratory polyarthritis
D. Prolonged PR interval on EKG
E. Temperature greater than 100.4 F
Explanation:

180
The correct answer is C. This patient's macular skin rash in a "bathing suit" distribution is
also known as erythema marginatum, one of the five major Jones criteria for diagnosing
rheumatic fever. The other four major criteria are migratory polyarthritis, Sydenham's chorea,
subcutaneous nodules, and pancarditis. In context of prior streptococcal infection, the
presence of two of five of the major criteria or one major plus two minor (choices A, B, D, E)
are sufficient to establish the diagnosis. Other minor criteria include previous rheumatic
fever and elevated C-reactive protein. A mnemonic for the major criteria is "CANCER" (not
pleasant, but it's easy to remember): Chorea, Arthritis, Nodules, Carditis, Erythema
marginatum, Rheumatic fever diagnosis.
A newborn baby who was apparently healthy at birth develops aspiration pneumonia in the first two days
of life. All attempts to feed the infant cause it to cough and choke. Which of the following
abnormalities is the most likely cause of the infant's difficulties?
A. Bronchogenic cysts
B. Congenital pulmonary cysts
C. Posterior deviation of the tracheoesophageal septum
D. Pulmonary immaturity
E. Pulmonary sequestration
Explanation:
The correct answer is C. The infant probably has esophageal atresia, which is typically caused
by posterior deviation of the tracheoesophageal septum. Attempts at feeding cause fluid to
spill into the trachea, and secondarily cause aspiration pneumonia. Emergent surgical
correction is usually required.
Bronchogenic cysts (choice A) are centrally located cysts that are often asymptomatic and may
be associated with cysts of other organs.
Congenital pulmonary cysts (choice B) are often multiple and located in the lung periphery
without connection to the bronchi; they are vulnerable to infection and rupture complicated by
pneumothorax and/or hemoptysis.

181

Pulmonary immaturity (choice D) produces progressive difficulty in breathing beginning in the


first few hours of life.
Pulmonary sequestration (choice E) represents extrapulmonary lung tissue supplied by systemic
blood vessels rather than by pulmonary arteries.
An 8-year-old male with oliguria is sent by his pediatrician to the radiologist for a renal ultrasound.
The ultrasound shows large, smooth kidneys with radial fusiform cysts in the cortex and
medulla. Neither parent has a history of renal disease, but the father recalls a family history
of "kidney problems." Which of the following is the correct diagnosis?
A. Glomerulocystic disease
B. Infantile polycystic disease
C. Medullary cystic disease
D. Segmental dysgenesis
E. Uremic medullary cystic disease
Explanation:
The correct answer is B. A disorder occurring in offspring of unaffected parents (in the
context of a family history of the disease) suggests autosomal recessive inheritance. Infantile
polycystic disease is the only disease listed in the choices with autosomal recessive
inheritance. This disease affects both infants and children and is usually bilateral. The cysts
are fluid-filled and lined by cuboidal epithelium. It is thought to result from failure of
communication between the nephron and pericaliceal system during development, leading to renal
failure.
Glomerulocystic disease (choice A) is a rare disease without a recognized hereditary component
that causes cortical cysts of the kidneys bilaterally, eventually producing renal failure.
Grossly, the kidneys are enlarged with 1- to 8-mm cortical cysts that develop from dilated
Bowman's spaces in the glomeruli. Newborns, infants, and children are affected.

182
Medullary cystic disease (choice C), or medullary sponge kidney, can affect all ages, but
typically affects adults. It is not thought to be inherited and the cause is not known. The
cysts are located at the tips of the papillae. It is usually asymptomatic, but can lead to
defective sodium reabsorption in the tubule.
Segmental dysgenesis (choice D) is not inherited and affects all ages. The kidneys show
irregular cysts with scarring. Symptoms depend upon the severity of disease. Patients may be
asymptomatic if one kidney is spared.
Uremic medullary cystic disease (choice E) is a rare disease of children and young adults that
causes multiple cysts of the medulla. There is also cortical tubular atrophy and intestinal
fibrosis. Pathogenesis is unknown. Chronic renal failure occurs and mortality is high.
X-ray of the leg of a 75-year-old man demonstrates linear calcifications along the distal course of the
femoral artery. Angiography shows no change in the luminal diameter of the artery. Which of the
following is the most likely diagnosis?
A. Atheromatous plaques
B. Hyaline arteriolosclerosis
C. Hyperplastic arteriolosclerosis
D. Hypersensitivity angiitis
E. Mnckeberg's arteriosclerosis
Explanation:
The correct answer is E. Mnckeberg's arteriosclerosis, also called medial calcific sclerosis,
is a disease of the elderly in which ring-like calcifications form in the media of medium-sized
to small muscular arteries. The process is not a vasculitis, as no inflammation occurs. It is
clinically insignificant because no change in vessel luminal diameter occurs.
Atheromatous plaques (choice A) might be calcified, but would narrow the vessel lumen.
Hyaline arteriolosclerosis (choice B) and hyperplastic arteriolosclerosis (choice C) typically

183
do not calcify and involve arterioles, rather than arteries.
Hypersensitivity angiitis (choice D) does not calcify.
A patient experiences intermittent, severe pain in the right upper quadrant of the abdomen, especially
after fatty meals. Ultrasound demonstrates multiple small opacities in the gall bladder that
change with the patient's position. Which of the following is a risk factor for this patient's
disorder?
A. 20 years of age
B. Male sex
C. Native American origin
D. Nulliparity
E. Thin build
Explanation:
The correct answer is C. The patient has gallstones (cholelithiasis). A useful mnemonic to
remember the risk factors for gallstones is the 5 F's: female, fat, fertile, forty, and
flatulent. Gallstones are usually composed predominantly of cholesterol with lesser amounts of
other components such as calcium salts, bile acids, and bile pigments. Exceptions to this rule
are nearly pure cholesterol stones and pigment (calcium bilirubinate) stones. The incidence of
mixed stones and cholesterol stones is also increased in association with Crohn's disease,
cystic fibrosis, clofibrate therapy, estrogen therapy, rapid weight loss, and Native American
origin. Risk factors for pigment stones include chronic hemolysis, alcoholic cirrhosis, and
biliary infection.
The typical age for gallstones is Forty years or older, not 20 years (choice A).
Female, not male (choice B) sex predisposes the individual for gallstones.
Many patients with gallstones are multiparous (Fertile), not nulliparous (choice D).

184
Gallstones are associated with obesity (Fat), not a thin build (choice E).
A 35-year-old woman presents to a gastroenterologist with a chief complaint of difficulty swallowing
solids and some liquids for the past several years. After detailed evaluation, the physician
suggests the possibility that the patient's esophageal condition is autoimmune in nature.Which
of the following findings on physical examination would help confirm the likely diagnosis?
A. Butterfly rash
B. Dry mouth
C. Enlarged thyroid
D. Thickened skin
E. Tumor-like nodules on tongue
Explanation:
The correct answer is D. The autoimmune disease to specifically associate with esophageal
dysfunction is scleroderma (progressive systemic sclerosis), which also causes thickening of
the dermis because of extensive collagen deposition. A similar type of fibrosis can involve the
gastrointestinal tract, particularly the distal two-thirds of the esophagus. The term "rubberhose" inflexibility is sometimes applied to an affected esophagus. Microscopically, excessive
collagenization of the lamina propria and submucosa is seen. The overlying mucosa may be
thinned and ulcerated.
Butterfly rash (choice A) is associated with systemic lupus erythematosus, which is usually not
accompanied by esophageal dysfunction.
Dry mouth (choice B) is associated with Sjgren syndrome. While this could potentially affect
swallowing of dry food, the patient had problems with a wider range of foods.
The thyroid gland is enlarged (choice C) in the autoimmune disorders known as Graves disease
and Hashimoto disease.
Tumor-like submucosal nodules on the tongue (choice E) are seen in some cases of amyloidosis.

185
This disorder can also affect the gastrointestinal tract, but more commonly involves the
intestines, rather than the esophagus.
A 67-year-old man is evaluated for persistent shooting pains, lower limb ataxia, and bladder
dysfunction. Physical examination demonstrates small irregular pupils that constrict with
accommodation but not in response to light. A VDRL test is positive. A CT scan of the spinal
cord would most likely demonstrate atrophy of which of the following structures?
A. Dorsal column
B. Dorsal horn
C. Lateral column
D. Ventral column
E. Ventral horn
Explanation:
The correct answer is A. The patient has tabes dorsalis, which is a form of neurosyphilis seen
10 to 25 years or longer after primary disease. The pupils described are Argyll Robertson
pupils, and are considered diagnostic for neurosyphilis. Characteristically, the dorsal
columns, which contain the ascending tracts for sensory information, become atrophic, probably
as a result of damage to the dorsal root ganglion cells.
The dorsal gray horn (choice B) contains neurons that respond to sensory input.
The lateral column (choice C) contains both descending (e.g., lateral corticospinal,
rubrospinal) and ascending (e.g., spinocerebellar, spinothalamic) tracts.
The ventral column (choice D) contains both descending (e.g., anterior corticospinal,
tectospinal) and ascending (e.g., spinothalamic) tracts.
The ventral horn (choice E) contains lower motor neurons.
A 68-year-old man presents to his physician with complaints of fatigue and a dragging sensation in his

186
abdomen. Physical examination is remarkable for massive splenomegaly without lymphadenopathy.
The patient's hematocrit is 30%, his platelet count is 80,000/mm3, and his leukocyte count is
2500/mm3. Bone marrow biopsy shows "fried egg" cells expressing the pan B-cell markers CD19 and
CD20, and the monocyte marker CD11c. Positive staining for which of the following substances
would most likely verify the likely diagnosis?
A. CD4
B. GpIIb/IIIa
C. Myeloperoxidase
D. Nonspecific esterase
E. Tartrate-resistant acid phosphatase
Explanation:
The correct answer is E. Hairy cell leukemia is characterized by pancytopenia and splenomegaly.
Lymphadenopathy is unusual. The proliferating cells express the pan B-cell markers (CD19 and
CD20), the monocyte marker (CD11c), the interleukin-2 receptor (CD25), and plasma cellassociated antigen-1 (PCA-1). Histologically, the cells display fine, hairlike projections when
viewed with phase-contrast microscopy. A "fried-egg" appearance on stained bone marrow biopsy
arises because of fixation artifacts that occur as a result of the hairlike projections on many
of the cells. Tartrate-resistant acid phosphatase is virtually diagnostic of hairy cell
leukemia.
CD4 (choice A) is a cluster designation antigen that is displayed on the surfaces of T cells
and maturing thymocytes.
GpIIb/IIIa (choice B) is a platelet membrane receptor for fibrinogen, fibronectin, and von
Willebrand's factor.
Myeloperoxidase (choice C) is a marker for late myeloblasts and mature granulocytes. Monocytes
also stain to some extent.
Diffuse nonspecific esterase (choice D) staining is used to identify cells in the monocytic

187
lineage. Punctate nonspecific esterase staining is observed in T cells.
A 19-year-old sexually active female presents to urgent care with abdominal pain and nausea. Her last
menstrual period began seven days ago. Physical examination is remarkable for fever and lower
abdominal pain. Pelvic examination reveals a mucopurulent discharge, adnexal tenderness, and
tenderness with cervical motion. A pregnancy test is negative; the patient's white count is
elevated. Which of the following complications is associated with this disorder?
A. Ascites
B. Cervical carcinoma
C. Exsanguination
D. Infertility
E. Pseudomyxoma peritonei
Explanation:
The correct answer is D. The young woman is suffering from pelvic inflammatory disease (PID).
This disorder is classically caused by ascending infection with Neisseria gonorrhoeae or
Chlamydia trachomatis. Symptoms include a mucopurulent cervicitis (often producing a vaginal
discharge), abdominal pain, or abnormal vaginal bleeding. Fever, an elevated white count, and
an elevated erythrocyte sedimentation rate may also be seen. Therapy consists of antibiotics
with activity against N. gonorrhoeae, C. trachomatis, vaginal anaerobes and facultative gramnegative rods (e.g., cefoxitin plus doxycycline or clindamycin plus gentamicin). Infertility is
a feared complication of tubal scarring and adhesions following salpingitis in PID. Bacteremia,
peritonitis, and intestinal obstruction may also complicate this disorder.
Abdominal enlargement due to ascites (choice A) is observed in advanced cases of ovarian
carcinoma.
Cervical carcinoma (choice B) is related to infection with certain subtypes of human papilloma
viruses.
Exsanguination (choice C) is a feared complication of a ruptured ectopic pregnancy.

188

Pseudomyxoma peritonei (choice E) is a very serious complication of mucinous tumors of the


ovary or appendix.
A patient with large, penetrating vegetations on his mitral and aortic valves develops severe headaches.
Funduscopic examination reveals papilledema. CT scan of the brain demonstrates a ring-enhancing
lesion. Which of the following organisms is the most likely cause of the patient's disorder?
A. Herpesvirus
B. Mycobacterium tuberculosis
C. Staphylococcus aureus
D. Streptococcus pneumoniae
E. Treponema pallidum
Explanation:
The correct answer is C. The cardiac lesions described are characteristic of acute bacterial
endocarditis. The fact that the valvular vegetations are large and penetrating strongly
suggests that they are due to a virulent, pyogenic pathogen. Among the answer choices,
Staphylococcus aureus best fits this description, and is in fact the most common cause of acute
bacterial endocarditis. Brain abscess, which produces ring-enhancing radiologic lesions, is a
known complication of bacterial endocarditis, occurring when the vegetations fragment and
release septic emboli.
Herpesvirus (choice A) can cause encephalitis, but is not a cause of vegetative endocarditis.
Mycobacterium tuberculosis(choice B) can cause brain masses, but does not usually cause
endocarditis.
Streptococcus pneumoniae(choice D) can cause acute pyogenic meningitis, but does not usually
cause endocarditis.
Treponema pallidum(choice E), the causative agent of syphilis, can cause aortic aneurysms and a

189
variety of neurologic problems, but does not usually cause endocarditis or brain abscess.
A 5-year-old boy who has been diagnosed with a mucopolysaccharidosis is given an IQ test and found to
have nearly normal intellectual functioning. Which of the following are possible diagnoses for
this patient?
A. Hunter syndrome or Hurler syndrome
B. Hunter syndrome or Scheie syndrome
C. Hurler syndrome or Sanfilippo syndrome
D. Morquio syndrome or Sanfilippo syndrome
E. Morquio syndrome or Scheie syndrome
Explanation:
The correct answer is E. All of the diseases listed are mucopolysaccharidoses, due to defective
degradation of mucopolysaccharides. Features typical of mucopolysaccharidoses include
hepatomegaly, skeletal abnormalities, and, often, mental retardation. The two exceptions to the
mental retardation are the milder conditions, Morquio syndrome and Scheie syndrome.
Autopsy of a hypertensive patient reveals a massive intracerebral hemorrhage filling the ventricles. A
thorough search for the source of the hemorrhage is likely to demonstrate that the bleeding
began in which of the following sites?
A. Basal ganglia and internal capsule
B. Central white matter
C. Cerebellum and medulla
D. Pons
E. Thalamus and hypothalamus

190
Explanation:
The correct answer is A. Intraparenchymal hemorrhage is typically due to hypertension, and is
the most common cause of death from stroke. Such hemorrhages can occur at all of the sites
listed in the answer choices, but roughly half of such cases involve the basal ganglia and
internal capsule. Hypertensive hemorrhages are often massive and frequently dissect through the
brain parenchyma into the ventricular system. There is frequently nothing a physician can do to
prevent a fatal outcome in patients with massive intracerebral hemorrhage.
The other sites listed (choices B, C, D, and E) are important sites of intraparenchymal
hemorrhage, but they are less frequently involved than the basal ganglia and the closely
related internal capsule.
A 30-year-old woman undergoes her yearly gynecologic examination and is found to have microscopic
hematuria. The hematuria is again documented on several more occasions when she is seen for other
reasons. Because no etiology is obvious, renal biopsy is eventually performed; a photomicrograph
of a hematoxylin and eosin-stained section is shown above. Immunofluorescent staining of
histologic sections similar to the one shown demonstrates IgG, IgM, and complement in the
glomerular mesangium. Which of the following is the most likely diagnosis?
A. Amyloidosis
B. Goodpasture syndrome
C. Henoch-Schnlein purpura
D. Lipoid nephrosis
E. Systemic lupus erythematosus
Explanation:
The correct answer is E. Systemic lupus erythematosus can produce a wide range of different
glomerular lesions; in some cases, the presenting symptoms may be associated with renal disease.
The pattern illustrated in the photomicrograph is mesangial expansion with cellular proliferation
and occurs in approximately 25% of lupus patients. Granular mesangial deposits of immunoglobulin
and complement are characteristically present and probably reflect the immune complex basis of the

191
problem.
Amyloidosis (choice A) is not usually associated with immune complex deposition.
Goodpasture syndrome (choice B) is characterized by linear IgG deposition along basement
membranes.
Henoch-Schnlein purpura (choice C) characteristically shows IgA deposits.
The glomeruli in lipoid nephrosis (choice D), also known as minimal change disease, appear normal
or very nearly normal.
A 66-year-old man develops low back pain, and radiologic investigations reveal multiple osteoblastic
lesions of the lumbar vertebrae. A hard, irregular nodule on the posterior surface of the
prostate is appreciated on rectal examination. Plasma levels of prostatic specific antigen (PSA)
are markedly increased. Which of the following is the most likely diagnosis?
A. Chronic prostatitis
B. Multiple myeloma
C. Prostatic adenocarcinoma
D. Prostatic hyperplasia
E. Prostatic intraepithelial neoplasia (PIN)
Explanation:
The correct answer is C. This is the typical clinical presentation of carcinoma of the
prostate, which is probably the most frequent cancer in males (although lung cancer is the most
frequent neoplastic cause of death). Autopsy studies show that small foci of adenocarcinoma are
found in the prostate of up to 70% of men older than 80. Prostatic specific antigen (PSA) is
expressed by both normal prostatic tissue and adenocarcinoma of the prostate. A rise in PSA
plasma levels, therefore, simply results from any increase in the mass of prostatic tissue and
cannot be used to differentiate between cancer and hyperplasia. Cancer arises more frequently
in the peripheral zone of the gland, producing firm nodules on the posterior surface that can

192
be detected on rectal examination. Metastasis to the bone can be osteoblastic (with formation
of new bone) or osteolytic (with destruction of bone). Osteoblastic metastases in the vertebral
column are characteristic of prostatic adenocarcinoma.
Chronic prostatitis (choice A) presents with dysuria and perineal discomfort. The diagnosis
relies on finding at least 10 leukocytes per high-power field on microscopic examination of
expressed prostatic secretions. Chronic prostatitis is further classified into bacterial
prostatitis if cultures are positive for bacteria, or abacterial prostatitis if cultures are
negative. Increased plasma PSA is frequently seen.
Multiple myeloma (choice B) is a neoplasm of plasma cells that frequently manifests with
osteolytic (not osteoblastic) lesions in the vertebral column as well as other bones, such as
the cranial vault and ribs.
Prostatic hyperplasia (choice D) is an extremely common disorder in middle-aged and elderly
men. It involves the periurethral zone of the gland, leading to compression of the urethra,
difficulty in urination, urinary retention, and recurrent cystitis. PSA levels are increased in
a manner roughly proportional to the degree of prostatic enlargement.
Prostatic intraepithelial neoplasia (PIN; choice E) is the precursor of invasive prostatic
adenocarcinoma and consists of atypical proliferation of epithelial cells confined within the
basement membrane of prostatic acini. In one third of cases, PIN progresses to invasive cancer
within 10 years. By definition, PIN is not associated with metastasis. Other examples of cancer
precursors are CIN (cervical intraepithelial neoplasia) and VIN (vulvar intraepithelial
neoplasia).
A 40-year-old, moderately obese man presents with a persistent productive cough. The cough has been
present for several weeks, but recently the man noted that his sputum has assumed a greenish
color. Further questioning elicits a history of productive cough, usually in the winter months,
over the past several years. He has smoked two packs of cigarettes per day since he was 16
years old. On examination, the man is febrile (100 F), and coarse rhonchi and wheezes can be
appreciated bilaterally. Which of the following is the most likely diagnosis?
A. Bronchogenic carcinoma
B. Chronic bronchitis with superimposed infection

193
C. Cystic fibrosis
D. Emphysema
E. Pulmonary tuberculosis
Explanation:
The correct answer is B. The diagnosis of chronic bronchitis requires the presence of chronic
productive cough over at least 3 months of the year for 2 successive years. The symptoms in
this patient (green productive sputum, fever) suggest that he has an acute infection
superimposed on chronic bronchitis (history of cigarette smoking, history of excessive mucus
production over many years). Histologically, the mucus-producing glands in the bronchi would
show hyperplasia and hypertrophy and extend to a greater depth in the bronchial wall, resulting
in a higher Reid index (ratio of thickness of mucus gland to thickness of bronchial wall).
Although the patient is a smoker and is at increased risk for bronchogenic carcinoma (choice
A), this is unlikely to be the cause of his symptoms. Hemoptysis and weight loss might also be
present if he had cancer.
Cystic fibrosis (choice C) presents earlier in life and may be associated with severe
production of mucus, especially if bronchiectasis supervenes. The age of the patient and the
relative late onset of disease preclude this diagnosis.
This patient may also have emphysema (choice D), as chronic bronchitis and emphysema are often
coexistent. However, although pure emphysema might cause dyspnea, it would not be associated
with a fever or a productive cough.
Pulmonary tuberculosis (choice E) would typically present with a history of hemoptysis rather
than abundant green sputum. Weight loss, night anorexia, malaise, and weakness may also be
present.
A 55-year-old man with a history of recurrent calcium-containing renal stones presents to the emergency
room with excruciating flank pain and blood in the urine. This patient is likely to have which
of the following underlying disorders?
A. Anemia of chronic disease

194

B. Chronic Proteus infection


C. Factor VIII deficiency
D. Hyperaldosteronism
E. Hyperparathyroidism
Explanation:
The correct answer is E. This patient is experiencing the very painful passage of a renal
stone, which is often accompanied by hematuria. His history of recurrent urolithiasis with
calcium-containing stones implies a disorder in the regulation of calcium concentration.
Hyperparathyroidism is associated with increased parathormone (PTH) levels, which can produce
hypercalcemia, hypercalciuria, and, ultimately, renal stones.
Anemia of chronic disease (choice A) does not produce calcium stones. It is an attractive
distracter because the patient presents with a chronic condition and hematuria. Note that
urinary blood loss is not usually significant enough to produce an anemic state.
Chronic Proteus infection (choice B) would produce struvite (magnesium-ammonium phosphate), not
calcium stones. Staghorn calculi are also seen.
Factor VIII deficiency (choice C) occurs in hemophilia, a hereditary clotting disorder. It is
not associated with calcium stones.
Hyperaldosteronism (choice D) results in potassium depletion, sodium retention, and
hypertension. Primary hyperaldosteronism (Conn's syndrome) is associated with adrenocortical
adenomas in 90% of patients and is characterized by decreased renin. Secondary
hyperaldosteronism results from excessive stimulation by angiotensin II that is caused by
excess renin production (plasma renin-angiotensin levels are high). Neither condition is
associated with renal stones.
A 36-year-old gravid female notes vaginal bleeding. Ultrasound reveals small grape-like cystic
structures without evidence of a developing embryo. A diagnosis of complete hydatidiform mole
is made at the hospital. Further analysis is most likely to reveal that

195

A. hCG levels are markedly decreased


B. serum levels of alpha fetoprotein are elevated
C. the genotype of the mole is 46, XX and is completely paternal in origin
D. the genotype of the mole is triploid
E. two or more sperm fertilized the ovum
Explanation:
The correct answer is C. A complete hydatidiform mole is characterized by elevated hCG and
grape-like cystic structures filling the uterus with no detectable embryo on ultrasound. The
genotype of a complete hydatidiform mole is purely paternal, caused by fertilization of an egg
that has lost its chromosomes. Hydatidiform mole is associated with increasing maternal age,
and may be a precursor to choriocarcinoma.
hCG levels are increased relative to normal values for dates, rather than decreased, in a molar
pregnancy (choice A).
Alpha fetoprotein (AFP; choice B) is a marker for endodermal yolk sac tumors, embryonal tumors
in men, and hepatocellular carcinoma. It is made by the fetus. In a complete hydatidiform mole,
AFP is not detectable since there is no fetus.
Triploidy and even tetraploidy are characteristics of partial moles (choice D). Partial moles
are thought to be due to fertilization of an egg with two different sperm, one with an X and
one with a Y chromosome, typically leading to triploidy.
Two or more sperm may fertilize an ovum, leading to a triploid fetus and partial mole (choice
E).
A severely ill hospitalized patient develops a coagulopathy characterized by prolongation of the
prothrombin time (PT) and partial thromboplastin time (PTT) with a reduced platelet count.
Several hours later, acute anuria and uremia develop. The patient's blood pressure is initially
high normal, but then later falls as he is dying. Which of the following would most likely be

196
the major finding seen at autopsy?
A. Chronic cortical inflammation with tubular atrophy and interstitial fibrosis
B. Diffuse cortical necrosis
C. Patchy papillary necrosis
D. Small uric acid stones in the renal pelvis
E. Staghorn calculi
Explanation:
The correct answer is B. Prolongation of the PT and PTT with reduced platelets suggest
disseminated intravascular coagulation (DIC). DIC may be secondary to liberation of tissue
factors due to obstetric catastrophes, neoplasms, hemolysis, fat embolism, severe trauma,
endothelial damage, and infections. In DIC, widespread clotting and fibrinolysis occur
simultaneously throughout the body. Severe DIC can produce thrombosis in so many renal vessels
as to cause diffuse cortical necrosis.
Choice A describes chronic interstitial nephritis, which is by definition is a chronic process
that would not develop over hours.
Patchy papillary necrosis (choice C) is seen in sickle cell renal disease, not DIC.
Uric acid stones (choice D) can be associated with gouty nephropathy and staghorn calculi
(choice E) occur with Proteus infections. In either case, the stones develop slowly, and are
not associated with DIC.
A pediatrician discovers a large mass in the abdomen of a 3-year-old child. Ultrasound examination
demonstrates that the mass appears to arise from the right kidney. Which of the following tumors
is most likely present?
A. Cortical adenoma
B. Hemangioma

197

C. Nephroblastoma
D. Oncocytoma
E. Renal cell carcinoma
Explanation:
The correct answer is C. Nephroblastoma (Wilms tumor) characteristically affects children
between 2 and 4 years of age and can form large spherical masses composed of variegated tissues
including primitive renal epithelial elements, a sarcomatous-appearing stroma, abortive
glomeruli and tubules, and heterologous tissues such as muscle, cartilage, fat, and bone.
Cortical adenoma (choice A) is a small (under 2 cm) benign tumor that is usually found
incidentally at autopsy.
Hemangiomas (choice B) can occur in the kidney, but would not usually produce a large mass.
Oncocytomas (choice D) are benign tumors that can be large but do not usually affect young
children.
Renal cell carcinoma (choice E) is usually a disease of older adults.
A 58-year-old Japanese male with a 15-pound weight loss and guaiac-positive stools is found to have an
ulcerated gastric tumor on endoscopy. Which of the following mechanisms is thought to account
for the increased incidence of gastric carcinoma in the Japanese population?
A. Food preservatives
B. Helicobacter species
C. Inadequate screening
D. Menetrier's disease
E. Schistosoma species

198

Explanation:
The correct answer is A. The diet of the Japanese, with its increased concentration of
polycyclic hydrocarbons present in smoked fish, has been strongly implicated in the
pathogenesis of gastric adenocarcinoma. Nitrites are also potential carcinogens present in
preserved foods, and have been shown to cause gastric cancer in laboratory animals.
Helicobacter pylori(choice B) in the antral mucosa is highly associated with peptic ulcer
disease, gastric lymphoma, and gastric adenocarcinoma. There is no evidence, however, that this
bacterium is more prevalent in the Japanese population than in the American population.
The Japanese are routinely screened for gastric adenocarcinoma (compare with choice C), and as
such are more frequently treated when the tumor is at an earlier stage. Screening in the United
States is not routinely performed.
Menetrier's disease (or hypertrophic gastritis; choice D) is a condition characterized by
markedly thickened gastric rugal folds, accompanied by hyperplasia of mucous glands and
protein-losing enteropathy. Menetrier's disease is not associated with gastric carcinoma.
Schistosomiasis (choice E) may injure the liver, bladder, or intestines, depending upon the
species, but it is not a risk factor for gastric carcinoma. Schistosoma haematobium infection
is a risk factor for squamous cell carcinoma of the bladder.
A 26-year-old man presents to his physician with a chronic cough. The man is a smoker, and states that
he also gets frequent headaches and aches in his legs when he exercises. Chest x-ray
demonstrates notching of his ribs. Which of the following undiagnosed congenital defects may be
responsible for these findings?
A. Coarctation of the aorta
B. Eisenmenger's syndrome
C. Tetralogy of Fallot
D. Transposition of great vessels

199
E. Ventricular septal defect
Explanation:
The correct answer is A. Coarctation of the aorta occurs in two patterns. In the infantile
type, the stenosis is proximal to the insertion of the ductus arteriosus (preductal); this
pattern is associated with Turner's syndrome. In the adult form, the stenosis is distal to the
ductus arteriosus (postductal) and is associated with notching of the ribs (secondary to
continued pressure from the aorta on them), hypertension in the upper extremities, and weak
pulses in the lower extremities. Headache, cold extremities, and lower extremity claudication
with exercise are typical if the patient is symptomatic (many adults with mild distal
coarctation may remain asymptomatic for years). Upper extremity hypertension with weak pulses
in the lower extremities, and a midsystolic (or continuous) murmur over the chest or back may
be the only obvious signs in some. Note that the chronic cough is probably related to the man's
smoking, and is not caused by the coarctation.
Eisenmenger's syndrome (choice B) is a shift from a left-to-right shunt to a right-to-left
shunt secondary to pulmonary hypertension.
Tetralogy of Fallot (choice C) and transposition of great arteries (choice D) cause cyanosis
and are usually diagnosed in infancy.
Ventricular septal defect (choice E) might remain undiagnosed until adulthood, but would not
cause notching of the ribs.
A newborn child is noted to have a bulky abdominal tumor. CT scans reveal that the mass involves the
right abdomen and retroperitoneum. The tumor is resected, revealing ganglion cells and
primitive, small, round cells occasionally organized in rosettes, embedded in a fibrillary pink
matrix. Special studies confirm the likely diagnosis. Which of the following features of this
tumor is associated with a poorer prognosis?
A. Amplification of N-myc gene
B. Cellular aneuploidy
C. Diagnosis at birth

200
D. Numerous ganglion cells
E. Tumor spread into right kidney
Explanation:
The correct answer is A. Neuroblastoma is a childhood tumor of the sympathetic nerves, which
most commonly arises along the paravertebral chain or within the adrenal medulla. Neuroblastoma
has two characteristic genetic markers: a 1p deletion and an N-myc translocation. N-myc is an
oncogene that resides on chromosome 2p, and is amplified up to 300 times in neuroblastoma.
Amplification of this gene is associated with a poorer prognosis.
Cellular aneuploidy (choice B) and hyperdiploidy reflect a better prognosis for neuroblastoma,
whereas diploid and tetraploid tumors have an intermediate or poor prognosis.
Tumor presentation at a younger age predicts a better outcome for neuroblastoma. Children under
one year of age (choice C) have an excellent prognosis, regardless of tumor stage.
Ganglion cells (choice D) within the tumor are a reflection of focal differentiation of the
neuroblastoma into ganglioneuroma. Better differentiated tumors have a more favorable outcome.
In general, tumor stage is not very predictive of neuroblastoma survival. Nonetheless, higher
grades of tumor (III and IV) extend across the vertebral midline and into distal viscera or
nodes (choice E).

A 35-year-old man with a history of rhinitis and asthma presents to his physician with complaints of
intermittent severe abdominal pain and a chronic maculopapular rash. Peripheral blood smear
demonstrates a marked eosinophilia. Biopsy of a skin lesion demonstrates necrotizing vasculitis
with large numbers of eosinophils. Which of the following diagnoses is most likely?
A. Churg-Strauss syndrome
B. Leukocytoclastic angiitis
C. Mnckeberg's arteriosclerosis

201
D. Temporal arteritis
E. Wegener's granulomatosis
Explanation:
The correct answer is A. The most likely condition is Churg-Strauss syndrome, also known as
allergic granulomatosis and angiitis. This variant of polyarteritis nodosa is clinically
associated with asthma and eosinophilia. The vascular lesions are those described in the
question stem. Granulomas are present in many, but not all, cases. Pulmonary involvement may be
prominent, but this is not always the case. In this case, the patient's abdominal pain is
related to GI vasculitis, and his skin rash is related to dermal vasculitis. Churg-Strauss
syndrome should be suspected in any patient with vasculitic symptoms, eosinophilia, and asthma.
In leukocytoclastic angiitis (choice B), the dominant inflammatory cells are neutrophils.
In Mnckeberg's arteriosclerosis (choice C), there is vessel calcification.
In temporal arteritis (choice D), which especially affects the cranial vessels, giant cells are
found.
Wegener's granulomatosis (choice E) affects the respiratory and renal systems.
An 11-year-old boy complains of pain in his left leg with no history of prior trauma. The pain is not
relieved by acetaminophen and slowly progresses. Radiographic examination shows a femoral tumor
with an "onion-skin" pattern of circumferential reactive periosteal bone. Examination of biopsy
material stained with hematoxylin and eosin reveals small uniform blue round cells. On the basis
of this information, which of the following is the most likely genetic mutation?
A. del 1p
B. t(2,13)
C. t(11,22)
D. t(12,22)

202
E. t(X,18)
Explanation:
The correct answer is C. This is the chromosomal mutation for Ewing sarcoma. This case
demonstrates the typical presentation of Ewing sarcoma, occurring mainly in the long bones of
children and adolescents. It presents with pain and has a unique onion-skin effect on x-ray.
Histologically, it is known as a small, round, blue-cell tumor. A number of different tumors
look similar, and molecular methods are increasingly used for diagnosis.
del 1p (choice A) is associated with leiomyosarcoma, which is mainly found in the extremities,
subcutis, and retroperitoneum. It is located outside of bone in the soft tissues and
histologically is composed of bundles of spindle cells.
t(2,13) (choice B) is associated with alveolar rhabdomyosarcoma, a tumor of young persons that
can affect the extremities. It is composed of small round blue cells that form small nests or
alveoli. The radiographic appearance shows a soft tissue mass rather than a bony lesion. This
is an aggressive malignancy.
t(12,22) (choice D) is associated with clear cell sarcoma, also known as malignant melanoma of
soft parts. It appears like a malignant melanoma, which arises in the soft tissue rather than
the skin. It usually is found on the tendons of extremities of young patients. Histologically,
the cells can have pigment or clear cytoplasm.
t(X,18) (choice E) is associated with synovial sarcoma, which is a tumor arising around a joint
space, usually in a young person. The knee and shoulder are the most common location.
Histologically, the cells can be spindled or plump, pink, and epithelioid.
A terminally ill HIV infected patient develops focal neurologic signs, dementia, and coma. Amoebic
parasites are demonstrated in CSF. Which of the following organisms is most likely to be the
causative agent?
A. Acanthamoeba sp.
B. Entamoeba histolytica
C. Giardia lamblia

203

D. Naegleria fowleri
E. Trichomonas vaginalis
Explanation:
The correct answer is A. Two types of free-living amoeba can infect the brain and meninges:
Naegleria fowleri and Acanthamoeba species. The former affects healthy adolescent or adult
divers, while the latter causes infection in patients with immunosuppression because of
diabetes, alcoholism, cancer, or HIV infection. The brain infection characteristically has a
prominent perivascular character, which causes a multifocal hemorrhagic necrotizing
meningoencephalitis. Skin ulcers, nasal infection, or pneumonia may also be present. It is
thought that the organisms may release a toxin causing host tissue necrosis. Systemic
antifungal agents (e.g., amphotericin) have some activity against this organism, but most cases
are fatal.
Entamoeba histolytica(choice B) causes amoebic dysentery and liver abscess.
Giardia lamblia(choice C) is a flagellate, rather than an amoeba, and causes diarrhea.
Naegleria fowleri(choice D) is an amoebic cause of meningoencephalitis in previously healthy
swimmers and divers.
Trichomonas vaginalis(choice E) is a flagellate, rather than an amoeba, and causes vaginitis.
A 46-year-old woman presents with insidious onset of shortness of breath, chest pain, and fatigue. Chest
x-ray films reveal bilateral pulmonary infiltrates and enlarged hilar lymph nodes. A biopsy of
one of these lesions shows non-necrotizing granulomas. Special stains for fungi and
mycobacteria are negative. The patient works as a secretary and has no history of occupational
exposure to airborne minerals or organic dusts. Which of the following is the most likely
diagnosis?
A. Asbestosis
B. Berylliosis

204
C. Byssinosis
D. Sarcoidosis
E. Tuberculosis
Explanation:
The correct answer is D. The diagnosis of sarcoidosis is usually made by exclusion. This
disease is characterized by non-necrotizing granulomas developing most frequently in the lungs,
lymph nodes, retina, heart, spleen, skin, and liver. Non-necrotizing granulomas may be seen in
a number of other conditions, however, such as infections and certain forms of pneumoconiosis,
which must be ruled out before making a diagnosis of sarcoidosis. The etiopathogenesis of
sarcoidosis is obscure. Cell-mediated mechanisms are thought to be involved in an immune
response to as yet unidentified antigens. About two-thirds of patients with sarcoidosis recover
without residual functional deficits, 20% have permanent pulmonary or visual damage, and 10%
die of pulmonary or cardiac involvement.
Asbestosis, berylliosis, and byssinosis are all lung diseases caused by inhalation of airborne
dusts.
Asbestosis (choice A) is due to chronic inhalation of asbestos fibers, which may affect workers
involved in installation and removal of insulation. Asbestosis leads to diffuse fibrosis of the
lungs. In this case, granulomas are absent or scanty; the most characteristic sign of asbestos
exposure is the asbestos body, which is an elongated beaded rod composed of asbestos fibers
coated by proteins.
Chronic exposure to beryllium in mining and fabrication leads to berylliosis (choice B), a
granulomatous condition of the lungs indistinguishable morphologically from sarcoidosis.
Clinical history is therefore essential in excluding this condition.
Byssinosis (choice C) results from occupational exposure to cotton fibers. This condition
belongs to a diversified category of diseases mediated by a hypersensitivity reaction against
inhaled organic dusts. Farmer's lung, pigeon breeder's lung, air-conditioner lung, and
byssinosis are typical examples of these conditions, which lead to a granulomatous reaction
that may progress to diffuse pulmonary fibrosis. Again, clinical history is crucial in the
identification of this etiology.

205

Tuberculosis (choice E) is usually associated with necrotizing granulomas. The type of necrosis
is also described as caseating because the necrotic material resembles cheese on gross
examination. The absence of acid-fast organisms in the biopsy material helps rule out this
diagnosis.
Radiographic studies of a 2-year-old child brought to an emergency room reveal a new fracture of the
humerus and evidence of multiple old fractures in ribs and long bones of the extremities. A
social worker wants to initiate prosecution of the parents for child abuse, but an alert
emergency room physician notices that, despite the broken arm, the toddler shows minimal
bruising. A very careful, directed, physical examination reveals that the toddler has "peculiar
teeth," a blue tinge to the sclera, and unusually mobile joints. The disease that the physician
suspects the child has is characterized by an abnormality of which of the following biochemical
functions?
A. Collagen type I synthesis
B. Collagen type II synthesis
C. Collagen type III synthesis
D. Collagen type IV synthesis
E. Collagen type V synthesis
Explanation:
The correct answer is A. The child has the most common variant (type I) of osteogenesis
imperfecta, which is an autosomal dominant genetic defect in the synthesis of type I collagen,
due to decreased synthesis of the procollagen alpha1(1) amino acid chain. This defect (unlike
that of the perinatal, lethal, type II form of osteogenesis imperfecta) is compatible with
survival, but does cause skeletal fragility, dentinogenesis imperfecta (abnormal teeth), blue
sclera, joint laxity, and hearing impairment. Unfortunately, a number of families with this
defect have had their children removed because of "abuse," only to find that the broken bones
continue in the new environment. Type I collagen is found in skin, bone, tendons, and most
other organs.

206
Type II collagen (choice B) is found in cartilage and vitreous humor.
Type III collagen (choice C) is found in blood vessels, uterus, and skin.
Type IV collagen (choice D) makes basement membranes.
Type V collagen (choice E) is a minor component of interstitial tissues and blood vessels.
There are also type VI-XI collagens, which are minor constituents of various tissues.
A 57-year-old man is seen in clinic for a chronic cough. Chest x-ray demonstrates a lung mass;
bronchoscopy with bronchial lavage and biopsy reveals a tumor composed of small neoplastic
cells with prominent crush artifact that do not stain with immunohistochemical stains for
lymphocyte common antigen (LCA). Which of the following oncogenes has been associated with this
tumor?
A. bcl-2
B. erb-2
C. L-myc
D. N-myc
E. ret
Explanation:
The correct answer is C. The presence of a lung mass suggests carcinoma. The small size of the
neoplastic cells suggests that the tumor may be small cell carcinoma (oat cell carcinoma). The
other possibility on morphologic grounds is lymphoma, but lymphoma can be excluded by the
negative LCA study. The oncogene associated with small cell carcinoma of the lung is L-myc.
bcl-2 (choice A) is associated with follicular and undifferentiated lymphomas.
erb-2 (choice B) is associated with carcinomas of the breast, ovary, and stomach.
N-myc(choice D) is associated with neuroblastoma.

207

ret(choice E) is associated with multiple endocrine neoplasia, types II and III.


A patient with severe arthritis suddenly becomes unable to move his arms or legs. He is admitted to the
neurology floor of the hospital, but his quadriplegia does not improve with time. If the
quadriplegia is due to his arthritis, which of the following types of arthritis does this
patient most likely have?
A. Ankylosing spondylitis
B. Gouty arthritis
C. Osteoarthritis
D. Rheumatoid arthritis
E. Septic arthritis
Explanation:
The correct answer is D. Involvement of the cervical spine occurs in almost 80% of patients
with rheumatoid arthritis, and involves the atlanto-axial joint in up to 25% of patients
hospitalized for rheumatoid arthritis. Fortunately, large degrees of subluxation of the joint
are rare, but if present, can be associated with quadriplegia, and even sudden death if the
odontoid peg separates from the arch of the atlas and compresses the spinal cord.
Serious complications of ankylosing spondylitis (choice A) include atrioventricular block,
bladder and bowel dysfunction, uveitis, pulmonary fibrosis, psoriasis, and inflammatory bowel
disease.
Serious complications of gout (choice B) include renal impairment.
Serious complications of osteoarthritis (choice C) are uncommon, but the local manifestations
can be crippling.
Serious complications of septic arthritis (choice E) include joint destruction, osteomyelitis,
and systemic infection.

208

A 72-year-old black woman has the abrupt onset of right-sided weakness affecting the face and arms. MRI
is consistent with an infarct in the territory of the middle cerebral artery. Pathologic
examination of the patient's brain would likely show
A. caseous necrosis
B. coagulative necrosis
C. enzymatic fat necrosis
D. gangrenous necrosis
E. liquefactive necrosis
Explanation:
The correct answer is E. Liquefactive necrosis occurs in brain or other neural tissues and in
pancreatic tissue. In this type of necrosis, the tissue appears liquefied under the microscope,
without preservation of cell outlines. Liquefactive necrosis can also be seen in some bacterial
infections, especially those caused by pyogenic Staphylococci, Streptococci, or certain
coliform bacteria.
Caseous necrosis (choice A) is generally an indication of infection by Mycobacterium
tuberculosis. The term caseous refers to the appearance of the tissue, i.e., soft, white
necrotic areas that have a cheese-like appearance. Microscopically, the necrotic areas are
lightly eosinophilic (stain light pink), with little or no discernible cellular detail. The
eosinophilia reflects staining of residual cellular proteins.
Coagulative necrosis (choice B) is a more common type of necrosis, characteristic of anoxic
injury and most infarcts. Cellular outlines are preserved, but proteins are denatured, and the
cells stain in an eosinophilic manner.
Enzymatic fat necrosis (choice C) is seen primarily with pancreatic injury when pancreatic
lipases are released and digest fat to form free fatty acids. These fatty acids complex with
calcium, resulting in the production of calcium soaps (saponification) in the pancreatic tissue
or in extrapancreatic fatty tissues (eg, omentum).

209

In gangrene (choice D), bacterial infection is superimposed on a background of massive necrosis


and putrefaction.
A 58-year-old male alcoholic with chronic pancreatitis develops a palpable abdominal mass. Ultrasound
reveals a 9-cm cystic lesion adjacent to the pancreas. An important complication that might
occur if this cyst ruptured would be
A. anaphylactic shock
B. carcinomatosis
C. disseminated infection
D. hypoglycemia
E. intestinal hemorrhage
Explanation:
The correct answer is E. The patient most likely has a pancreatic pseudocyst, which is a
complication of pancreatitis. Pancreatic pseudocyst is not a true cyst; it is lined by
granulation tissue and collagen. It contains pancreatic juices and lysed blood, so rupture
would spill the active digestive enzymes onto the adjacent viscera, particularly the stomach,
small intestine, and transverse colon. Digestive action produces potentially severe
gastrointestinal hemorrhage.
Anaphylactic shock (choice A) results from massive activation of the IgE-mediated branch of the
immune system. Pancreatic secretions do not elicit an IgE response. The classic abdominal cyst
that ruptures, producing anaphylactic shock, is a hydatid cyst.
Carcinomatosis (choice B) is widespread serosal spread of a carcinoma, typically due to tumor
spillage into a body cavity. Although this may occur with pancreatic mucinous
cystadenocarcinoma, this disease is far less likely to occur than is pancreatic pseudocyst in a
patient with chronic pancreatitis.
Pancreatic pseudocyst is not an infective disease. Although septic abscesses do occur in the

210
abdomen, and may even complicate a pancreatic pseudocyst, the danger of rupture is more
associated with tissue destruction by pancreatic enzymes than with infection (choice C).
Hypoglycemia (choice D) would occur if the pancreatic pseudocyst contained an insulin-like
compound; however, pancreatic pseudocysts contain exocrine pancreatic enzymes, not hormones
from the endocrine pancreas.
A 35-year-old patient presents to his clinician because of a painful great toe. Physical examination
demonstrates that the toe is exquisitely painful, inflamed, and warm. On questioning, the
patient reveals that this has happened before, generally occurring the morning after a previous
evening's "partying." Which of the following would most likely be demonstrated by aspiration of
joint fluid?
A. Giant cells and acid-fast positive rods
B. Needle-like crystals that are strongly negatively birefringent
C. Numerous neutrophils and gram-negative rods
D. Numerous neutrophils and intracellular gram-negative cocci
E. Roughly cuboidal crystals that are weakly positively birefringent
Explanation:
The correct answer is B. The disease is acute gout, which is characterized by strongly
negatively birefringent needle-like crystals of uric acid in the joint fluid. Colchicine is
able to interrupt these attacks of acute gout.
Tubercular arthritis causes granuloma formation with giant cells and acid-fast mycobacteria
(choice A).
Infectious arthritis due to Salmonella (sickle cell patients) or Haemophilus (children) would
be associated with neutrophils and gram-negative rods (choice C) in joint fluid.
Infectious arthritis due to N. gonorrhoeae would be associated with neutrophils and
intracellular gram-negative cocci (choice D).

211

Calcium pyrophosphate (pseudogout) produces roughly cuboidal crystals that are weakly
positively birefringent (choice E).
In which of the following respects do a seminoma involving the testis and a germinoma involving the
ovary differ most significantly?
A. Most common age of presentation
B. Number of mitoses
C. Potential to contain foci of more aggressive tumors
D. Radiosensitivity
E. Ultrastructural appearance
Explanation:
The correct answer is A. Seminomas and dysgerminomas are very similar tumors but differ in two
significant respects: the most common age of presentation in men is in the fourth decade, while
in women, it is in the third decade. Also, seminomas are relatively common in men (30% of
testicular germ cell tumors), while dysgerminomas are rare in women (1% of ovarian tumors).
Both of these tumors are composed of sheets of uniform polyhedral cells with intervening
fibrous septa of connective tissue, lymphocytes, and multinucleated giant cells. The number of
mitoses (choice B) per high-power field and ultrastructural appearance (choice E) do not differ
greatly between the two tumors.
These tumors in pure form are very radiosensitive (90% 5-year survival; choice D), but can be
much more aggressive (choice C) if foci of other germ cell tumors (notably embryonal carcinoma,
choriocarcinoma, and yolk sac tumors) are present.
A stenotic valve is removed from a 70-year-old man. The value demonstrates hard nodular masses heaped up
within the sinuses of Valsalva. On microscopic section, the acellular masses stain darkly blue
with hematoxylin and eosin. Which of the following is the most significant constituent of the
masses?

212

A. Calcium oxalates
B. Calcium phosphates
C. Complexed iron
D. Magnesium phosphates
E. Uric acid
Explanation:
The correct answer is B. The valvular disease is calcific aortic stenosis, a disease of the
elderly in which nodules of calcium salts, notably calcium phosphate salts, form as a result of
dystrophic calcification. Dystrophic calcification can also be seen at sites of previous
necrosis of many etiologies (coagulative, caseous, liquefactive, or enzymatic fat necrosis) and
in advanced atherosclerosis.
Calcium oxalates (choice A) are found in some kidney stones.
Complexed iron (choice C) is seen in hemosiderin depositions, often at sites where there has
been previous hemorrhage or if there is a systemic overload of iron.
Magnesium phosphates (choice D) are found in some kidney stones.
Uric acid (choice E) and urate deposits can be seen in gout and some kidney stones.
An elderly couple living in a very cold apartment turn on the oven, open the oven door, and go to sleep.
The next morning, the neighbors find the couple dead. The direct mechanism by which death was
produced most likely involves which of the following?
A. Damage to the plasmalemma
B. Decreased oxygen-carrying capacity of blood
C. Increased calcium transport into mitochondria

213

D. Poisoning of oxidative phosphorylation


E. Rupture of lysosomes
Explanation:
The correct answer is B. The scenario described in the question stem is unfortunately not at
all uncommon. The open oven door is a tip-off that carbon monoxide was involved. Carbon
monoxide has a very high affinity for hemoglobin, and binds, nearly irreversibly, to it in such
a manner that oxygen cannot bind, drastically decreasing the oxygen-carrying capacity of the
blood. Carbon monoxide also causes the oxygen-hemoglobin dissociation curve to shift to the
left, making oxygen more difficult to unload. Traditionally, patients with carbon monoxide
poisoning have been described as having "cherry red" blood and, consequently, skin, but this
change is somewhat unreliable in real life.
Choices A, C, and E list secondary changes that are commonly observed in injured cells, no
matter what the cause of the injury.
Cyanide acts by poisoning oxidative phosphorylation (choice D).
A 52-year-old alcoholic male develops chronic severe upper abdominal pain and maldigestion. Ultrasound
studies demonstrate pancreatic calcifications. Which of the following disorders will this
patient most likely develop?
A. Diabetes mellitus
B. Hyperthyroidism
C. Hypothyroidism
D. Pheochromocytoma
E. Zollinger-Ellison syndrome
Explanation:

214
The correct answer is A. Pancreatic calcifications constitute strong radiologic evidence of
chronic pancreatitis. Up to 30% of patients with chronic pancreatitis eventually develop
secondary diabetes mellitus as a consequence of destruction of islets in addition to pancreatic
ducts. This form of diabetes mellitus may also eventually cause small vessel damage and
blindness.
Neither hyperthyroidism (choice B) nor hypothyroidism (choice C) are related to chronic
pancreatitis.
Pheochromocytoma (choice D) is unrelated to chronic pancreatitis.
Chronic pancreatitis does predispose for pancreatic carcinoma, but not for the gastrinomas that
cause Zollinger-Ellison syndrome (choice E).
A gastric biopsy is performed on a patient with suspected graft-versus-host disease following bone
marrow transplantation. The biopsy demonstrates many isolated dying epithelial cells in crypts
showing fragmented, hyperchromatic nuclei and small discrete blebs containing both cytoplasm and
nuclear fragments. The biopsy demonstrates which of the following?
A. Apoptosis
B. Caseous neurosis
C. Coagulative necrosis
D. Gangrenous necrosis
E. Liquefactive necrosis
Explanation:
The correct answer is A. The changes described are those of apoptosis, which is a form of
programmed cell death. Apoptosis can be seen in a variety of settings. In this case, it is
occurring in the context of graft-versus-host disease, which is mediated by CD8+ and CD4+
cells. Apoptosis also occurs during embryogenesis, during hormone-dependent involution in the
adult (eg, during menstruation), during rapid proliferation of cell populations (eg, intestinal
crypt epithelia), and in the immune system (e.g., in developing thymus). It can also occur

215
after duct obstruction in organs such as pancreas and during some viral diseases. The hallmark
of this programmed method of cell death is the fragmentation of the cell with formation of
cytoplasmic blebs and apoptotic bodies that are phagocytized by other healthy cells.
Caseous necrosis (choice B), such as is seen in tuberculosis, shows necrotic, amorphous
granular debris surrounded by a granulomatous response.
Coagulative necrosis (choice C), such as is seen in myocardial infarction, shows coagulated,
anuclear cell "ghosts."
Gangrenous necrosis (choice D) is a commonly used surgical term (eg, in diabetic feet) that
does not have a precise pathologic counterpart.
Liquefactive necrosis (choice E) is usually seen in focal tissue-destructive bacterial
infection; a sterile liquefactive necrosis can also be observed in ischemic necrosis of the
brain.
A 60-year-old woman is seen at an emergency room after she fractures the neck of her right femur during
a minor fall. Radiologic studies demonstrate a generalized loss of bone mass. Exogenous therapy
with which of the following hormones would have been most likely to slow or prevent the
patient's bone disease?
A. Cortisol
B. Epinephrine
C. Estrogen
D. Thyroxine
E. Vasopressin
Explanation:
The correct answer is C. The disease is osteoporosis, and estrogen replacement in postmenopausal women appears to play an important role in preventing or limiting development of
osteoporosis in post-menopausal women.

216

Cortisol (choice A) excess, as in endogenous or exogenous Cushing's syndrome, is a contributing


cause of osteoporosis.
Epinephrine (choice B) levels appear to be unrelated to osteoporosis.
Thyroxine (choice D) excess (e.g., in thyrotoxicosis) may contribute to bone loss in some cases
of osteoporosis.
Vasopressin (choice E) levels appear to be unrelated to osteoporosis.
During a routine physical examination of a 74-year-old man, a physician palpates a large, pulsating mass
in the lower abdomen. To which of the following is this mass most likely etiologically related?
A. Atherosclerosis
B. Bacterial infection
C. Congenital anomaly
D. Cystic medial degeneration
E. Syphilis
Explanation:
The correct answer is A. The mass is an abdominal aortic aneurysm, typically found in older
men. Such aneurysms are almost always related to the formation of complicated atherosclerotic
plaques in the aorta. Associated coronary artery disease is commonplace.
Bacterial infection (choice B) can cause "mycotic" aneurysms; these usually involve smaller
vessels.
The small berry aneurysms that can involve the circle of Willis are congenital anomalies
(choice C).
Cystic medial degeneration (choice D) is related to the development of dissecting aneurysms

217
(actually dissecting hematomas).
Tertiary syphilis (choice E) typically causes aneurysms of the root and arch of the aorta,
rather than the descending aorta.
At autopsy, a patient who had died with acute anuria and uremia is found to have ischemic necrosis of
the cortex of both kidneys with relative sparing of the medulla. These pathological findings
are most likely related to which of the following underlying conditions?
A. Disseminated intravascular coagulation
B. Multiple myeloma
C. Polycystic kidney disease
D. Pyelonephritis
E. Sickle cell anemia
Explanation:
The correct answer is A. Diffuse cortical necrosis, as described in this patient, is usually
seen in the setting of disseminated intravascular coagulation, typically in the context of
overwhelming sepsis. It can also be seen following hypotension combined with vasoconstriction.
Multiple myeloma (choice B) is associated with renal deposition of amyloid protein and damage
to both glomeruli and tubules.
Adult polycystic kidney disease (choice C) would produce enlarged kidneys filled with cystic
masses.
Pyelonephritis (choice D) would produce inflammation, often most severe in the renal pelvis.
Sickle cell anemia (choice E) usually affects the medulla most severely, and can cause
papillary necrosis.
A 22-year-old female Asian immigrant presents with complaints of malaise, fever, arm pain, loss of

218
appetite, and visual problems. Her mother states that she fainted 1 week ago. The physician
cannot palpate the patient's lower extremity pulses and notes that the dial pulses are weak. The
erythrocyte sedimentation rate (ESR) is elevated. Which of the following is the most likely
diagnosis?
A. Buerger's disease
B. Kawasaki's disease
C. Takayasu's arteritis
D. Thrombophlebitis
E. Wegener's granulomatosus
Explanation:
The correct answer is C. Takayasu's arteritis is an uncommon disorder that is most prevalent in
young Asian women. In Takayasu's, an inflammatory process produces fibrous thickening of the
aortic arch, causing narrowing or near obliteration of the origins of arteries that branch from
the arch. Absent or diminished pulses are noted, especially in the upper extremities.
Involvement of the carotid circulation can produce ocular disturbances or other neurological
dysfunction (syncope or, less commonly, stroke).
Buerger's disease (thromboangiitis obliterans; choice A) is a disease of young-to middle-aged
adult (predominantly males) smokers. It is characterized by segmental thrombosis of arteries
and veins, often in extremities.
Kawasaki's disease (choice B) is usually seen in children, and is characterized by rash, fever,
conjunctivitis, and lymphadenopathy. Coronary vasculitis and aneurysms may occur.
Thrombophlebitis (choice D) usually involves the deep veins of the legs and often occurs in
association with predisposing factors such as immobilization, clotting disorders, heart
disease, cancer, pregnancy, or tissue injury.
Wegener's granulomatosis (choice E) generally affects patients over 50 with vasculitis,
glomerulonephritis, and multiple granulomata of the upper and lower respiratory tract.

219

A 28-year-old female with a several-year history of intermittent diarrhea and abdominal pain is seen for
inflammatory bowel disease. Endoscopic evaluation of her terminal ileum, colon, and rectum is
undertaken. Which of the following endoscopic observations is more indicative of Crohn's
disease than of ulcerative colitis?
A. Discontinuous mucosal involvement
B. Mucosal atrophy
C. Mucosal ulceration
D. Pseudopolyps
E. Rectal involvement
Explanation:
The correct answer is A. Crohn's disease is frequently associated with "skip lesions,"
discontinuous areas of active disease in the colon and small intestine with intervening
segments that appear normal. This is in marked contrast to ulcerative colitis, which most
commonly shows continuous mucosal involvement.
Both ulcerative colitis and Crohn's disease can show mucosal atrophy (choice B). Chronic
mucosal inflammation produces glandular atrophy, and a loss of mucosal folding.
Mucosal ulceration (choice C) is seen in both Crohn's disease and ulcerative colitis. The
ulcers of Crohn's disease are generally described as linear fissures, following the
longitudinal axis of the intestine. Ulcerative colitis typically produces broad, extensive
areas of ulceration.
Pseudopolyps (choice D) are most commonly associated with ulcerative colitis, and represent the
islands of spared mucosa between the broad ulcerations.
Rectal involvement (choice E) in inflammatory bowel disease is more typical of ulcerative
colitis than of Crohn's disease. Whereas ulcerative colitis is a "pancolitis," that is usually
most severe in the rectum and right colon, Crohn's disease is usually a disease of the small

220
intestine, and may involve the small intestine alone (40%) or both the small intestine and
colon (30%).
A renal biopsy is performed during a workup for renal insufficiency in a patient with a slightly reduced
renal size. The biopsy incidentally samples one of the arcuate arteries, which shows
reduplication of the elastic lamina and fibrosis of the media. These findings are most likely
associated with which of the following?
A. Amyloidosis
B. Escherichia coli enterotoxin
C. Hypertension
D. Lipoid nephrosis
E. Sickle cell disease
Explanation:
The correct answer is C. The lesion described is called fibroelastic hyperplasia, and it can
affect the media of larger interlobular and arcuate arteries. These changes are related to
benign nephrosclerosis, usually in hypertensive patients. Other changes seen in benign
nephrosclerosis include hyaline arteriolosclerosis and sometimes patchy ischemic atrophy.
Amyloidosis (choice A) can cause deposition of amyloid material (homogeneous eosinophilic
extracellular deposits) around small blood vessels.
Escherichia coli enterotoxin (choice B) has been implicated in childhood hemolytic uremic
syndrome with microangiopathic hemolytic anemia related to vascular intimal hyperplasia,
fibrinoid necrosis, and thrombi.
Lipoid nephrosis (minimal change disease, choice D) has a normal or abnormal appearance by
light microscopy.
Sickle cell disease (choice E) can cause focal occlusions of the vasa recta (the hypertonic,
hypoxic milieu of the renal medulla favors sickling), leading to patchy papillary necrosis,

221
proteinuria, and sometimes cortical scarring.
A 51-year-old alcoholic man is admitted to the hospital after profuse upper gastrointestinal bleeding.
His wife states that she heard vomiting in the bathroom, then went to check on her husband
after he didn't emerge for 10 minutes, and found him in a pool of blood, unconscious. Which of
the following is the most likely cause of this man's bleeding?
A. Barrett's esophagus
B. Helicobacter gastritis
C. Mallory-Weiss tear
D. Schatzki ring
E. Zenker's diverticulum
Explanation:
The correct answer is C. This is the classic presentation of a Mallory-Weiss tear, which is a
mucosal tear at the gastroesophageal junction secondary to recurrent vomiting (the stomach
temporarily evulses through the esophagus, tearing the esophagus). The result can be massive
hematemesis, but the lesions usually heal without problems if the patient does not die from
exsanguination.
Barrett's esophagus (choice A) is characterized by the replacement of normal esophageal
epithelium with gastric-type epithelium. Barrett's carries a high risk of adenocarcinoma, not
bleeding.
Helicobacter gastritis (choice B) does not usually cause profuse bleeding.
Schatzki rings (choice D) are benign mucosal rings at the squamocolumnar junction below the
aortic arch.
Zenker's diverticula (choice E) are esophageal evaginations at the junction of the pharynx and
esophagus. They are not typically associated with bleeding.

222
A patient presents to her hematologist with acute myeloblastic leukemia. A marrow smear contains many
promonocytes that stain positively for non-specific esterase. According to the French-AmericanBritish (FAB) classification, this leukemia would be best classified as which of the following?
A. M1
B. M2
C. M3
D. M5
E. M7
Explanation:
The correct answer is D. Acute myeloblastic leukemia is subclassified based on the degree of
differentiation of the tumor cells. The cells described in the question stem are those of acute
monocytic leukemia-M5 according to the FAB classification.
M1 (choice A) is acute myelocytic leukemia without differentiation. A substantial proportion of
cells are myeloperoxidase-positive; few cells mature beyond the myeloblast stage.
M2 (choice B) is acute myelocytic leukemia with differentiation. Myeloblasts and promyeloblasts
with a few Auer rods are typically seen.
M3 (choice C) is acute promyelocytic leukemia, characterized by promyelocytes with many Auer
rods.
M7 (choice E) is acute megakaryocytic leukemia. In this condition, blasts react with
antiplatelet antibodies.
Three days following a viral illness, a 3-year-old child develops severe vomiting and irritability,
followed by lethargy. The concerned parents take the child to the emergency room. Physical
examination is remarkable for hepatomegaly. The child is admitted to the hospital, but over the
next 24 hours, coma develops, and there is a marked elevation in serum ammonia levels. Electron
microscopy of brain or liver would most likely reveal profound alterations in which of the

223
following cellular organelles?
A. Centromeres
B. Cilia
C. Mitochondria
D. Peroxisomes
E. Ribosomes
Explanation:
The correct answer is C. The child has Reye's syndrome (fatty liver with encephalopathy), which
is an acquired mitochondrial abnormality (mechanism unknown) that can follow viral infections
in children under 15 years of age. The cause is unknown, and although aspirin use has since
been implicated, the syndrome may occur without exposure to salicylates. Reye's syndrome
usually follows an upper respiratory tract infection, especially influenza or chickenpox. The
mortality rate is near 50%, but if the child survives, full recovery is possible.
A 26-year-old female presents with a chief complaint of menstrual bleeding 10 days prior to the usual
onset of menses. She states that her last menstrual period was heavier that usual. The
physician examines her and performs a pregnancy test (which is negative) then concludes that
she is probably experiencing dysfunctional bleeding due to anovulation. If an endometrial
biopsy were performed, which of the following would be the most likely histologic appearance of
this type of endometrium?
A. Asynchronous secretory endometrium
B. Decidualized stroma with inactive glands
C. Early proliferative endometrium
D. Late secretory endometrium
E. Menstrual endometrium

224

F. Proliferative endometrium with stromal breakdown


Explanation:
The correct answer is F. The most common cause of dysfunctional uterine bleeding is
anovulation. This is caused by excessive and prolonged estrogen effect without the
postovulatory progesterone effect. This occurs most often around menarche and menopause, when
subtle hormonal imbalances commonly occur. An estrogenic stimulation causes the endometrial
glands to proliferate. Persistent proliferation without a progesterone phase will eventually
break down and bleed even though there is no secretory change present and the stroma is not
menstrual. That is why this is called "anovulation bleed," because it is a non-ovulatory bleed
that may seem like normal menstruation.
Asynchronous secretory endometrium (choice A) refers to secretory endometrium that has a
mismatch of 2 or more days between the glands and the stroma. For example, the glands may be at
day 17 while the stroma shows more maturity, corresponding to day 22. This is a type of
dysfunctional ovulatory bleeding and clinically presents with infertility. It is usually due to
an inadequate luteal phase because the corpus luteum is not producing enough progesterone even
though ovulation has occurred.
Decidualized stroma with inactive glands (choice B) is the common histologic appearance of
patients taking oral contraceptives. These patients have asynchronous glands and stroma. The
glands are usually not active and the stroma appears to be ready for implantation. This reverts
to normal with discontinuation of oral contraceptives.
Early proliferative endometrium (choice C) and late secretory endometrium (choice D) are
physiologic phases of the normal menstrual cycle.
Menstrual endometrium (choice E) histologically shows stromal and glandular breakdown with
bleeding following the full cycle. It follows the secretory phase and precedes the
proliferative phase.
A 27-year-old man with AIDS develops a reddish, slightly raised rash on his face, neck, and mouth,
consistent in appearance with Kaposi's sarcoma. Microscopically, the proliferating cells in this
malignancy most closely resemble which of the following?

225
A. Angiosarcoma
B. Carcinosarcoma
C. Lymphoma
D. Malignant fibrous histiocytoma
E. Melanoma
Explanation:
The correct answer is A. Kaposi's sarcoma is a spindle cell neoplasm that is highly associated
with AIDS and with the Herpes simplex virus type 8. The tumor has an appearance very similar to
that of angiosarcoma-proliferating stromal cells and endothelium creating vascular channels
that contain blood cells.
Carcinosarcoma (choice B) is a tumor that contains malignant epithelial cells and malignant
stromal cells. There is no epithelial element in Kaposi's sarcoma.
Although lymphoma (choice C) occurs with increased frequency in AIDS, it does not resemble
Kaposi's sarcoma. Lymphoma involves neoplastic lymphocytes, whereas Kaposi's sarcoma involves
neoplastic vascular structures.
Malignant fibrous histiocytoma (MFH); (choice D) is an extremely poorly differentiated
(anaplastic) stromal malignancy. MFH does not produce any recognizable mesenchymal structuresthus, the production of vascular structures by Kaposi's sarcoma differentiates the two tumors.
Melanoma (choice E) does produce colored skin lesions; however, the histological appearance of
the malignant melanocytes is quite unlike Kaposi's sarcoma. Melanoma in situ appears as small
nests of cells with large, red nucleoli in the dermis and epidermis; this lesion can progress
to a variety of forms, but none resemble Kaposi's sarcoma.
A 55-year-old man newly diagnosed with leukemia undergoes genetic studies that reveal a t(9;22)
translocation (the Philadelphia chromosome). Which of the following would a complete blood
count most likely show?

226
A. Increased lymphocyte count
B. Increased neutrophil count
C. Numerous lymphoblasts
D. Numerous myeloblasts
E. Pancytopenia
Explanation:
The correct answer is B. The Philadelphia chromosome is a hallmark of chronic myeloid leukemia
(CML). CML typically presents with markedly increased numbers of circulating neutrophils and
metamyelocytes, with lesser numbers of eosinophils and basophils and a small number of blasts.
The disease follows an indolent course, and usually progresses to an accelerated phase with
increased numbers of circulating blasts only after several years.
An increased lymphocyte count (choice A) would be seen in chronic lymphocytic leukemia (CLL),
another indolent leukemia.
Increased numbers of blasts (choices C and D) are seen in the late stages of CML and CLL, or in
the acute leukemias. The Philadelphia chromosome is occasionally associated with acute
lymphoblastic leukemia and acute myeloblastic leukemia, but these are diseases of children and
young adults.
Pancytopenia (choice E) is most typical of the myelodysplastic syndromes (MDS), in which
ineffective hematopoiesis of a pluripotent stem cell produces abnormal development of all cell
types. No specific chromosomal abnormality is associated with MDS.
A 35-year-old man presents to his doctor with loss of appetite, nausea and vomiting, and fatigue.
Laboratory examination confirms the diagnosis of hepatitis B, and the man becomes icteric 2
weeks later. This patient may also be particularly vulnerable to the development of which of
the following disorders?
A. Berry aneurysm

227
B. Coronary artery aneurysm
C. Dissecting aneurysm
D. Giant cell arteritis
E. Polyarteritis nodosa
Explanation:
The correct answer is E. Thirty percent of patients with polyarteritis nodosa have hepatitis B
antigenemia. Polyarteritis is a systemic necrotizing vasculitis that can be difficult to
diagnose, since the vascular involvement is typically widely scattered, and the specific
symptoms depend on the specific vessels (small- to medium-sized arteries) involved. Patients
typically present with low-grade fever, weakness, and weight loss. Abdominal pain, hematuria,
renal failure, hypertension, and leukocytosis may occur. The disease is frequently fatal if
untreated.
Berry aneurysms (choice A) are congenital saccular dilatations of vessels associated with adult
polycystic kidney disease.
Coronary artery aneurysms (choice B) are associated with Kawasaki's disease in children.
Dissecting aneurysms (choice C) are associated with cystic medial necrosis (seen in Marfan's
syndrome) and hypertension.
Giant cell arteritis (choice D) occurs in older adults; it is associated, in some cases, with
polymyalgia rheumatica.
A patient comes to medical attention because of a kidney stone. During the clinical evaluation, the
patient reveals that he has had a history of stomach ulcers. Which of the following diagnoses
should the physician consider?
A. Horner syndrome
B. Shy-Drager syndrome

228
C. Sipple syndrome
D. Turcot syndrome
E. Wermer syndrome
Explanation:
The correct answer is E. This patient may have Wermer syndrome (multiple endocrine neoplasia
type I; MEN type I). In this disorder: 1) parathyroid hyperplasia or adenomas can cause
hypercalcemia and kidney stones; 2) pancreatic neoplasms may secrete any of the islet hormones
or may secrete gastrin, producing Zollinger-Ellison syndrome with multiple peptic ulcers; and
3) pituitary adenomas may occur rarely.
Horner's syndrome (choice A) is characterized by ptosis, miosis, and in many cases,
hemianhidrosis. It is usually related to involvement of the cervical sympathetic plexus by
tumor (usually lung cancer).
In Shy-Drager syndrome (choice B), there is autonomic nervous system failure, leading to
orthostatic hypotension and parkinsonism.
Sipple syndrome (choice C) is MEN type II, characterized by medullary thyroid carcinoma,
pheochromocytoma, and parathyroid disease.
28

In the very rare Turcot syndrome (choice D), adenomas of the digestive tract and central

nervous system gliomas (astrocytoma and medulloblastoma) occur.


A 34-year-old man presents with weight loss, diarrhea, and flatulence. Jejunal biopsy demonstrates
marked atrophy of villi. This patient's condition may improve with removal of which of the
following from the diet?
A. Beef
B. Eggs
C. Potatoes

229
D. Tomatoes
E. Wheat
Explanation:
The correct answer is E. The patient probably has celiac sprue, which is caused by an allergic,
immunologic, or toxic reaction to the gliadin component of gluten (from wheat). The symptoms
and pathologic changes usually reverse with complete removal of gliadin from the diet.
Therapeutic failures are frequently due to hidden wheat in the diet. Patients with celiac
disease have an increased risk of developing gastrointestinal lymphoma.

A 45-year-old man develops a large meningioma that compresses the brain. A head CT scan demonstrates a
subfalcine herniation. This herniation would most likely damage which of the following structures?
A. Cerebellar tonsils
B. Cingulate gyrus
C. Medial temporal lobe
D. Medulla
E. Midbrain
Explanation:
The correct answer is B. Brain herniation can occur either with generalized increased intracranial pressure or
as a consequence of a mass lesion of the cranium. Subfalcine herniation occurs when part of one cerebral
hemisphere herniates under the falx membrane, which separates the two cerebral hemispheres along the
midline of the skull. The gyrus most affected is the cingulate gyrus, which runs along the medial aspect of the
cerebral hemisphere, just above the corpus callosum.
The cerebellar tonsils (choice A) can herniate into the foramen magnum, in a tonsillar herniation. The medulla
(choice D) can be compressed by the herniating cerebellar tonsils, producing damage to this structure.
The medial temporal lobe (choice C) can be damaged in uncal herniation.
The midbrain (choice E) can be damaged in the course of a uncal herniation.
A 25-year-old woman in her 22nd week of pregnancy develops hypertension and mild proteinuria. Due to the
baby's gestational age, her obstetrician chooses to carefully monitor the mother for any sign of developing
complications rather than to immediately deliver the baby. Which of the following complications account for the
most maternal deaths in preeclampsia?
A. Cerebral edema and laryngeal edema

230

B. Cerebral hemorrhage and adult respiratory distress syndrome


C. Convulsions and renal tubular necrosis
D. Hemolysis and hepatic infection
E. Hepatic rupture and renal cortical necrosis
Explanation:
The correct answer is B. The disease is preeclampsia, which may be complicated by a wide variety of serious
conditions. Historically, the appearance of convulsions defined the transition from preeclampsia to eclampsia;
however the concept of eclampsia is probably flawed because many other serious complications can occur
even in the absence of seizures. Statistically, the most common causes of maternal death in preeclampsia are
cerebral hemorrhage and pulmonary complications, notably adult respiratory distress syndrome.
Cerebral edema and laryngeal edema (choice A), hemolysis and hepatic infarction (choice D), and hepatic
rupture and renal cortical necrosis (choice E) can all be complications of preeclampsia, but are not the most
common causes of maternal mortality.
Convulsions and renal tubular necrosis (choice C) can be complications of preeclampsia, but convulsions
indicate that eclampsia has developed.
Other complications of preeclampsia not listed in the answer choices include retinal detachment, cortical
blindness, pulmonary edema, disseminated intravascular coagulation, low platelet counts, and in the fetus,
brain damage and death from asphyxia.

A 4-year-old girl is brought to the clinic by concerned parents. Physical examination is remarkable for extremity
edema. Urinalysis shows significant proteinuria, but no red cells or casts. Electron microscopy performed on a
kidney biopsy would be most likely to show which of the following findings?
A. Dense deposits
B. Fusion of epithelial foot processes
C. Mesangial deposits
D. Subendothelial deposits
E. Subepithelial spikes
Explanation:
The correct answer is B. The most frequent cause of nephrotic syndrome in children is minimal change disease
(lipoid nephrosis), which is characterized by an absence of findings by light microscopy, and fusion of epithelial
foot processes by electron microscopy.
Dense deposits (choice A) are seen in type II membranoproliferative glomerulonephritis.
Mesangial deposits (choice C) are a prominent feature of IgA nephropathy and are seen to a lesser degree in

231
several other glomerulonephritides.
Subendothelial deposits (choice D) are seen in type I membranoproliferative glomerulonephritis.
Subepithelial spikes (choice E ) are a feature of membranous glomerulonephritis.

A young man presents for an employment physical. He is very tall, has long fingers, and hyperflexible joints. He
states that he has always been called "double jointed". Which of the following disorders is associated with this
syndrome?
A. Dissecting aortic aneurysm
B. Hepatosplenomegaly
C. Polycystic kidneys
D. Progressive neurologic dysfunction
E. Retinoblastoma
Explanation:
The correct answer is A. The patient has Marfan syndrome, an autosomal dominant disorder caused by a defect
in the gene on chromosome 15 encoding fibrillin, a 350 kD glycoprotein. Fibrillin is a major component of
elastin-associated microfibrils, which are common in large blood vessels and the suspensory ligaments of the
lens. Abnormal fibrillin predisposes for cystic medial necrosis of the aorta, which may be complicated by aortic
dissection. Other features of the syndrome are subluxated lens of the eye, mitral valve prolapse, and a
shortened life span (often due to aortic rupture).

A 55-year-old patient presents with chronic cough. Chest x-ray demonstrates a coin lesion near the apex of the
lung. CT-guided biopsy of the lesion demonstrates a malignant tumor arising in an area of scarred lung. Which of
the following type of cancer is most likely to be present?
A. Adenocarcinoma
B. Bronchioloalveolar carcinoma
C. Large cell carcinoma
D. Oat cell carcinoma
E. Squamous cell carcinoma
Explanation:
The correct answer is A. Tumors that arise peripherally and can cause coin lesions include adenocarcinoma
(choice A), bronchioloalveolar carcinoma (choice B), and large cell carcinoma (choice C). Of these,
adenocarcinoma is the form specifically associated with lung scarring related to old granulomatous disease, old

232
chronic obstructive lung disease, or other old damage to the lungs.
Remember bronchioloalveolar carcinoma (choice B) as the one that is not associated with smoking.
Large cell carcinoma (choice C) is an aggressive, undifferentiated form of lung cancer.
Remember oat cell (small cell, choice D) carcinoma as the one that can commonly cause paraneoplastic
syndromes such as SIADH and Cushing's syndrome.
You should associate squamous cell carcinoma of the lung (choice E) with hypercalcemia.

A patient undergoes total thyroidectomy for a mass lesion of the thyroid. During the surgery, the surgeon notes
that the parathyroid glands appeared enlarged. The thyroid lesion shows neuroendocrine-type cells and amyloid
deposition. This patient's thyroid and parathyroid lesions may be related to which of the following oncogenes?
A. bcl-2
B. C-myc
C. erb-B2
D. L-myc
E. ret
Explanation:
The correct answer is E. The thyroid lesion is medullary carcinoma of the thyroid. The coexistence of
parathyroid hyperplasia suggests Sipple's syndrome (MEN II). These patients also tend to develop
pheochromocytoma. Both MEN II and MEN III are associated with the ret oncogene.
Associate bcl-2 (choice A) with follicular and undifferentiated lymphomas.
Associate C-myc(choice B) with Burkitt's lymphoma.
Associate erb-B2 (choice C) with breast, ovarian, and gastric carcinomas.
Associate L-myc(choice D) with small cell carcinoma of the lung.

A 2-year-old child develops a localized cutaneous infection with phage group II Staphylococcus aureus. A rash
resembling the rash of scarlet fever begins around the mouth, then spreads to the trunk and extremities. A few
bullae develop, which burst, leaving denuded skin. At what level in the skin did these bullae most likely develop?
A. Across the basal cells
B. Below the basement membrane
C. Between the basal cells and the basement membrane

233
D. High in the epidermis
E. Just above the basal cells
Explanation:
The correct answer is D. Bullous diseases of the skin are subdivided on the basis of the level at which the
cleavage for blister formation occurs. In general, the lower in the epidermis/dermis that the cleavage plane
occurs, the more dangerous and widespread the blistering. This is because blistering at lower levels,
particularly those that damage or remove the basal cell layer, permits loss of substantial amounts of fluid and
heals slowly (often with significant scarring). Scalded-skin syndrome (toxic epidermal necrolysis), as seen in this
patient, may follow staphylococcal (often phage group II) skin infection. This disorder fortunately involves the
very superficial squamous cells just beneath the granular layer. Consequently, the disease (which typically
produces bright red skin sloughing, predominately in intertriginous regions) usually resolves without sequelae
after antibiotic therapy. "Scalded-skin" may also be observed in association with drug-induced erythema
multiforme. Major inciting agents include phenylbutazones, sulfonamides, and barbiturates. This form usually
affects the mucosa (eyes, mouth) first, and is much more dangerous because the blistering is subepidermal,
and the entire overlying epidermis becomes necrotic.

Exploratory laparotomy in a patient with an acute abdomen demonstrates a several-foot long loop of small
intestine with a dark red-to-brown, edematous appearance. The lesion ends abruptly on both the distal and
proximal edges. Which of the following diagnoses is suggested by this appearance?
A. Abetalipoproteinemia
B. Celiac sprue
C. Ischemic bowel disease
D. Tuberculosis
E. Whipple's disease
Explanation:
The correct answer is C. The question describes the typical appearance of a gangrenous small intestine. This
life-threatening condition can be due to arterial thrombosis, embolic arterial occlusion, venous thrombosis, or
mechanical strangulation of vessels in twisted bowel loops. Treatment is surgical removal of the involved
segment.
Abetalipoproteinemia (choice A), celiac sprue (choice B), and Whipple's disease (choice E) involve the mucosa
and submucosa, and would not be obvious at laparotomy.
Tuberculosis (choice D) tends to produce mass lesions and strictures.

A 55-year-old woman presents to her physician after several episodes of syncope. Physical examination is
remarkable for a low-pitched "plopping" sound during mid-systole. Two-dimensional echocardiography
demonstrates a ball-valve type obstruction of the mitral valve. Which of the following would most likely be
observed if the cause of the obstruction were biopsied?

234

A. Benign glandular tissue


B. Densely packed smooth muscle
C. Densely packed striated muscle
D. Malignant glandular tissue
E. Scattered mesenchymal cells in a myxoid background
Explanation:
The correct answer is E. The most common primary cardiac tumor of adults is atrial myxoma, which typically
occurs as a single lesion in the left atrium that may intermittently obstruct the mitral valve. Histologically, these
tumors are composed of scattered mesenchymal cells in a prominent myxoid background.
Benign glandular tissue (choice A) suggests an adenoma, which is not usually found in the heart.
Densely packed smooth muscle (choice B) suggests a leiomyoma, which is not usually found in the heart.
Densely packed striated muscle (choice C) suggests rhabdomyoma, which is the most common primary cardiac
tumor in children, not adults.
Malignant glandular tissue (choice D) suggests carcinoma, which can be metastatic to the heart, but does not
usually cause a ball-valve obstruction.
A 65-year-old man develops oliguria and peripheral edema over a period of weeks. Urinalysis reveals hematuria
and proteinuria; examination of the urinary sediment reveals red cell casts. Radiologic and ultrasound studies fail
to demonstrate an obstructive lesion. Renal biopsy shows many glomerular crescents. This presentation is most
suggestive of which of the following conditions?
A. Anti-glomerular basement membrane disease
B. Diabetic nephropathy
C. Hypertensive nephropathy
D. Lupus nephritis
E. Minimal change disease
Explanation:
The correct answer is A. The two principal causes of rapidly progressive glomerulonephritis are anti-glomerular
basement membrane (including both Goodpasture's syndrome and isolated anti-glomerular basement disease)
and primary systemic vasculitis (including Wegener's granulomatosis, microscopic polyarteritis, idiopathic
rapidly progressive glomerulonephritis, Churg-Strauss syndrome, polyarteritis nodosa, giant-cell arteritis, and
Takayasu's arteritis). A very large variety of other systemic and primary glomerular disease may occasionally
cause rapidly progressive glomerulonephritis, but this is usually not the typical presentation for these diseases.

235
Diabetic nephropathy (choice B) typically begins with microalbuminuria and hypertension and progresses over a
10 to 20 year period to renal failure.
Hypertensive nephropathy (choice C) due to essential hypertension typically presents with slowly rising BUN
and creatinine; hypertensive nephropathy due to malignant hypertension presents with more rapidly rising BUN
and creatinine.
Lupus nephritis (choice D) can have many presentations, but the most typical is proteinuria, which may be
severe enough to cause nephrotic syndrome. Also, 90% of cases of systemic lupus erythematosus occur in
women, usually of child-bearing age.
Minimal change disease (choice E) typically presents with nephrotic syndrome and is not consistently
associated with recognizable glomerular changes by light microscopy. Podocyte foot-process fusion can be
seen by electron microscopy.

A 57-year-old female with megaloblastic anemia is found to be deficient in vitamin B12. Levels of all other
essential vitamins are within normal limits. Further investigations confirm that she has pernicious anemia. Which
of the following gastrointestinal diseases is most likely to be associated with this condition?
A. Angiodysplasia
B. Atrophic gastritis
C. Duodenal ulcer
D. Menetrier's disease
E. Ulcerative colitis
Explanation:
The correct answer is B. An important cause of vitamin B12 deficiency is pernicious anemia, an autoimmune
disease associated with atrophic gastritis. In atrophic gastritis, the gastric epithelium undergoes intestinal
metaplasia, thereby replacing gastric chief and parietal cells with goblet cells. The metaplastic epithelium
produces insufficient intrinsic factor to bind the dietary vitamin B12. Megaloblastic anemia develops over a
number of years.
Angiodysplasia (choice A) is a disease of intermittent lower intestinal bleeding due to rupture of mucosal and
submucosal venules and capillaries under normal colonic wall tension. Angiodysplasia may produce iron
deficiency anemia.
Duodenal ulcers (choice C) may also produce iron deficiency anemia due to chronic blood loss. They are
caused by damage to the duodenal epithelium by gastric acids and are associated with cirrhosis, COPD,
chronic renal failure, and hyperparathyroidism.
Menetrier's disease (choice D) is also known as hypertrophic gastropathy. It is an idiopathic condition typified by
thickened gastric rugae and hyperplastic mucosa, producing a thickened gastric wall, hypochlorhydria,
epigastric distress, and protein-losing gastroenteropathy.
Ulcerative colitis (choice E) is an inflammatory bowel disease almost entirely restricted to the large intestine. It
usually causes intestinal distress and diarrhea, and it may produce a malabsorption syndrome that includes
vitamin B12 deficiency.

236

A 50-year-old man presents at an emergency room because of several hours of progressively increasing chest
pain that no longer responds to sublingual nitroglycerin. This type of angina is thought to be due to which of the
following conditions?
A. Atherosclerosis alone
B. Coronary artery embolism
C. Coronary artery spasm
D. Thrombosis with or without underlying atherosclerosis
E. Vasculitis
Explanation:
The correct answer is D. The patient has unstable or crescendo angina, which is thought to be due to a slowly
developing thrombosis in a coronary artery branch. The thrombosis may or may not occur over an area of the
vessel involved by atherosclerotic plaque. Thrombolytic agents are particularly helpful in this type of patient.
Atherosclerosis alone (choice A) usually causes stable angina occurring with exertion.
Coronary artery embolism (choice B) is uncommon, but can occur if a plaque at the aortic orifice fragments and
is driven into a coronary artery.
Coronary artery spasm (choice C) is thought to cause angina at rest (Prinzmetal's angina). This type of angina
is typically severe, but does not have a crescendo pattern, and often occurs in younger patients.
Coronary artery vasculitis (choice E) is important in the pathogenesis of Kawasaki's disease.
A 3-year-old boy is brought to the emergency department after the acute onset of headache, vomiting, nuchal
rigidity, and impaired mental status. MRI reveals a posterior fossa tumor that fills the 4th ventricle. Surgery is
immediately started, and intraoperative consultation leads to a "frozen section" diagnosis of medulloblastoma.
Which of the following pathologic mechanisms most likely accounts for this child's clinical presentation?
A. Acute hemorrhage into the 4th ventricle
B. Alteration of medullary function
C. Increased intracranial pressure
D. Infiltration of the cerebellar vermis by the neoplasm
E. Spread of tumor to the subarachnoid space
Explanation:

237
The correct answer is C. Any tumor "filling the 4th ventricle" blocks the circulation of cerebrospinal fluid (CSF).
This blockage leads to increased intracranial pressure, which manifests with nausea, vomiting, headache,
nuchal rigidity, and mental status changes. If surgery is not performed promptly, cerebellar tonsillar herniation
and rapid death will ensue. In children, medulloblastoma and ependymoma are the most frequent neoplasms
presenting in this manner.
There is no evidence in this case suggesting that acute hemorrhage into the 4th ventricular cavity (choice A)
has occurred, nor is medulloblastoma typically associated with this complication. CNS tumors that frequently
bleed are metastases from melanoma, renal cell carcinoma, and choriocarcinoma.
Alterations in medullary function (choice B) lead to cardiorespiratory instability and may be caused by direct
tumor compression or infiltration of the medulla, neither of which is supported by MRI findings in this case.
Infiltration of the cerebellar vermis (choice D) is certainly seen in many cases of medulloblastoma, a tumor that
arises from this midline cerebellar structure. However, this would lead to truncal ataxia and gait instability, not
symptoms of increased intracranial pressure.
Medulloblastoma characteristically spreads to the subarachnoid space (choice E), from which the neoplasm
may metastasize to distant sites such as spinal cord. Plaques of medulloblastoma are often found on the
cerebellar surface, creating a characteristic sugar coating, but this would not cause any significant blockage of
CSF circulation.

Biopsy of a 4 mm rough, tan, and slightly raised skin lesion on the face of a 65-year-old man demonstrates
atypical basal cells with eosinophilic cytoplasm but persistent intercellular bridges. The stratum corneum is
thickened and parakeratosis is present; the remainder of the epidermis is thinned. Which of the following features
would probably additionally be seen in the dermis?
A. Benign nevus cells
B. Blue-gray elastic fibers
C. Large numbers of spindle-shaped fibroblasts
D. Malignant nevus cells
E. Touton giant cells
Explanation:
The correct answer is B. The lesion described is an actinic keratosis, which is a common premalignant lesion
caused by solar damage, which also characteristically damages the elastic fibers (changing their color in
stained tissue to blue-gray) of the superficial dermis.
Benign nevocellular nevus cells (choice A) are found in common moles (nevocellular nevi).
Large numbers of spindle-shaped fibroblasts (choice C) are found in dermatofibromas.
Malignant nevus cells (choice D) are found in melanoma.
Touton giant cells (choice E) are found in xanthomas.

238

Following a difficult delivery accompanied by significant maternal hemorrhage, a new mother develops a severe
retro-orbital hemorrhage with nausea and vomiting. The patient then develops fever and visual field loss, followed
by impairment of medial and downward gaze accompanied by diplopia and ptosis. CT scan demonstrates
hemorrhage in the pituitary gland with extension to the meninges. Immediate therapy with which of the following
hormone combinations should be instituted?
A. Glucocorticoids and estrogens
B. Glucocorticoids and thyroid hormone
C. Growth hormone and estrogens
D. Mineralocorticoids and growth hormone
E. Prolactin and mineralocorticoids
Explanation:
The correct answer is B. Pituitary apoplexy, which is a life-threatening infarction of the pituitary gland, may
result after obstetric hemorrhage (Sheehan's syndrome), with increased intracranial pressure, or during
systemic anticoagulation therapy. While the patient may have a generalized hypofunction of the pituitary in
these settings, the most important hormones to quickly replace are glucocorticoids (synthesized by the adrenal
cortex under pituitary ACTH control) and thyroid hormone (under pituitary TSH control), since deficiency of
these hormones produces life-threatening syndromes.
Growth hormone, estrogens, and prolactin (choices A, C, D, and E) do not require immediate replacement.
Mineralocorticoids (choices D and E) can be replaced if diabetes insipidus develops.

A 45-year-old man from southern China is diagnosed with a nasopharyngeal tumor. Histologically, this neoplasm
is composed of anaplastic cells immunoreactive for cytokeratin admixed with abundant lymphocytes. Which of the
following factors is most likely implicated in the pathogenesis of this neoplasm?
A. Cigarette smoking
B. Epstein-Barr virus infection
C. Ionizing radiation
D. Overexpression of the bcl-2 gene
E. Trisomy 8
Explanation:
The correct answer is B. The neoplasm described here is known as lymphoepithelioma, an inappropriate
designation that emphasizes the histologic features of a lymphocyte-rich squamous cell carcinoma. This tumor,
AKA nasopharyngeal carcinoma, is the most frequent childhood malignancy in certain African regions and one

239
of the most frequent adult tumors in southern China. Epstein-Barr virus genome has been identified in most of
these neoplasms.
Cigarette smoking (choice A) is a risk factor for a number of malignant neoplasms, such as lung cancer. In the
upper respiratory tract, cigarette smoking is a strong predisposing factor for laryngeal carcinoma, but it has not
been linked to the pathogenesis of nasopharyngeal carcinoma.
Radiation injury (choice C) is a well-known risk factor for many human neoplasms, such as leukemia and thyroid
cancer, but not for nasopharyngeal carcinoma. Although any cell type is susceptible to neoplastic
transformation by radiation, there is a sort of hierarchy of vulnerability. The most frequent radiation-induced
cancers are leukemias and thyroid carcinoma, followed by breast, lung, and salivary gland tumors. Skin, bone,
and the gastrointestinal tract are least vulnerable to radiation carcinogenesis.
Overexpression of the bcl-2 gene (choice D) is found in most B-cell follicular lymphomas. bcl-2 is a member of a
large family of genes that regulate apoptosis. bcl-2 inhibits apoptosis, so that its overexpression allows
neoplastic lymphocytes to survive for long periods and accumulate progressively in lymph nodes and bone
marrow.
Trisomy 8 (choice E) has been identified in some cases of olfactory neuroblastoma, a malignant tumor of small
round cells arising from the olfactory mucosa and resembling adrenal neuroblastoma.

A 38-year-old man has a cold, then recovers. A few days later, he notices a reddish-brown discoloration to his
morning urine. He presents to a physician, who elicits a recent history of fatigue, abdominal pain, and headaches.
Physical examination reveals slight jaundice. A complete blood count (CBC) reveals a hematocrit of 34, with a
leukocyte count of 3000/mm3 and a platelet count of 120,000/mm3. A peripheral smear shows mild macrocytosis
and polychromasia. The reticulocyte count is 5%. A urine sample appears yellow and clear. Which of the following
is the most likely diagnosis?
A. Aplastic anemia
B. Crigler-Najjar syndrome, type I
C. Dubin-Johnson syndrome
D. Hereditary spherocytosis
E. Paroxysmal nocturnal hemoglobinuria
Explanation:
The correct answer is E. The patient most likely has paroxysmal nocturnal hemoglobinuria (PNH), which can
cause a striking hemoglobinuria with red-brown to black urine. The condition usually presents in adulthood, the
incidence peaking in middle age. The hemoglobinuria, if present, is typically intermittent, although the classic
rhythmic nocturnal pattern is actually observed in only a minority of patients. Hemosiderinuria is usually present.
PNH is an acquired red cell defect caused by defective synthesis of a glycosylphosphatidylinositol (GPI) anchor,
which is associated with somatic mutations in the PIG-A gene (located on the X chromosome). Decreased GPI
anchor synthesis results in decreased expression of GPI-anchored proteins (GPI-AP), including
complement-regulating proteins, on the cell membrane. The deficiency of GPI-APs causes the red cells to
become extraordinarily sensitive to lysis by complement. Factors that may trigger hemolysis include infection
(even with the common cold), menstruation, surgical procedures, exposure to cold, vaccines, and other types of
stress. The patients are prone to venous thrombosis (thought to result from inappropriate platelet activation),
which may be the cause of the abdominal pain frequently experienced in this group. Thrombosis and

240
thromboembolism are the most frequent immediate causes of death in PNH patients, with the intrahepatic veins
being a favorite site for thrombosis (producing Budd-Chiari syndrome). Other complications include pigment
gallstones (secondary to chronic hemolysis) and granulocytopenia, leading to frequent infections. The definitive
test for PNH is Ham's test, in which serum is acidified to activate the complement pathway, then added to PNH
red cells, resulting in lysis.
In aplastic anemia (choice A), the reticulocyte count would not be elevated. Also, the slight degree of
pancytopenia observed in this man would tend to rule out marrow aplasia.
Type I Crigler-Najjar syndrome (choice B) is a cause of severe unconjugated hyperbilirubinemia because of a
defect in hepatic bilirubin conjugation. It is usually fatal in the first year of life.
Dubin-Johnson syndrome (choice C) is a usually asymptomatic cause of jaundice, but is not associated with
hemolytic anemia.
In hereditary spherocytosis (choice D), spherocytes would be seen on the peripheral smear, and splenomegaly
is common.

A 30-year-old patient presents to a clinician because of intermittent, severe headaches accompanied by


perspiration, palpitations, and pallor. Blood pressure on the initial examination was within normal limits, but, when
the patient came in later with a headache, it was 180/135 mm Hg. Urinary vanillylmandelic acid (VMA) levels were
elevated. Roughly, what percentage of the tumors causing this pattern is malignant?
A. 1%
B. 10%
C. 50%
D. 90%
E. 99%
Explanation:
The correct answer is B. The tumor is a pheochromocytoma that intermittently secretes epinephrine and other
vasoactive amines, producing episodes of elevated blood pressure accompanied by headache. This is the
tumor to associate with the rule of the 10's: 10% malignant, 10% bilateral, 10% extra-adrenal, 10% calcify, 10%
occur in kids, and 10% are familial. It is also a favorite target on examinations, although the incidence is quite
low.

A 25-year-old woman complains of abdominal pain of rapid onset in the right lower quadrant. She subsequently
undergoes surgery for suspected acute appendicitis. At surgery, however, a tubal pregnancy is discovered. The
most frequent predisposing factor for this condition is
A. endometriosis
B. an intrauterine device

241
C. leiomyomas of the uterus
D. pelvic inflammatory disease
E. previous surgery
Explanation:
The correct answer is D. The great majority of ectopic pregnancies (90%) occur in the fallopian tubes. The
other sites are the ovaries, abdominal cavity, and the intrauterine segment of the fallopian tubes. Any condition
that leads to anatomical abnormalities of the uterus and fallopian tubes may predispose to ectopic pregnancy.
The most frequent of such conditions is pelvic inflammatory disease (PID), which is usually associated with
salpingitis. PID is a common infectious condition most frequently caused by Neisseria gonorrhoeae and
Chlamydia trachomatis, both sexually transmitted. Other cases are due to a polymicrobial population, including
staphylococci, streptococci, coliform bacteria, and Clostridium perfringens, acquired during abortion or delivery.
PID leads to acute purulent salpingitis. If this is untreated or inadequately treated, it progresses to
salpingo-oophoritis, tubal abscesses, pyosalpinx, or hydrosalpinx. Even milder cases may cause adhesions
within the tube or between the tube and the ovary that interfere with implantation of the ovum and result in tubal
pregnancy. All the remaining conditions listed above may also predispose to ectopic pregnancy.
Endometriosis (choice A) refers to the presence of endometrium in abnormal locations, such as the ovary,
uterine ligaments, rectovaginal pouch, and pelvic peritoneum. It is an important clinical condition manifesting
with pain, dysmenorrhea, and infertility. Scarring at endometriotic sites may cause peritubal adhesions and
ectopic pregnancy.
An intrauterine device (choice B) may also increase the risk of ectopic pregnancy, but the mechanism of action
is not clear.
Leiomyomas of the uterus (choice C) are the most frequent benign tumor in women. They develop from the
smooth muscle as well-circumscribed nodules within the uterine wall (intramural), in a subserosal or submucosal
location. Leiomyomas may cause significant distortion of the uterine wall and interfere with implantation,
increasing the risk of ectopic pregnancy.
Previous surgery (choice E) of any kind may create scars and peritubal adhesions, thus predisposing to ectopic
pregnancy.
A 22-year-old male with fatigue, recurrent fever, and enlarged cervical lymph nodes has numerous atypical
lymphocytes in his peripheral blood smear. A biopsy from the patient's enlarged node shows expansion of
lymphoid follicles with preservation of the underlying architecture. Numerous atypical lymphocytes are present in
the paracortical areas. This patient most likely has
A. AIDS
B. Burkitt's lymphoma
C. Hodgkin's Disease
D. mononucleosis
E. non-Hodgkin's lymphoma
Explanation:

242

The correct answer is D. Infectious mononucleosis is a benign infection caused by the Epstein-Barr Virus (EBV),
a herpesvirus. Although B lymphocytes are infected by the virus, the characteristic atypical cells are activated
suppresser T cells - thus the paracortical location (T cell zone) in the lymph node. Lymph nodes in viral
infections show expansion of germinal centers without loss of normal architecture. All lymphomas, including
Burkitt's, Hodgkin's and non-Hodgkin's lymphomas, destroy the normal architecture of the lymph node.
Burkitt's lymphoma (choice B) produces a sea of monotonous, mitotically active cells in a "starry sky"
appearance.
Other non-Hodgkin's lymphomas (choice E) show either a nodular appearance or diffuse sheets of cells which
replace the germinal centers.
Hodgkin's lymphoma (choice C) also can show nodular or diffuse patterns, but is characterized by the presence
of Reed-Sternberg cells.
AIDS (choice A) is associated with a number of neoplastic and infectious processes that may alter the lymph
node structure. The characteristic lymph node changes in AIDS are progressive transformation of the germinal
centers, not paracortical hyperplasia.
A liver biopsy from a 54-year-old man shows many Mallory bodies. This finding is most suggestive of which of the
following diseases?
A. Alcohol abuse
B. Alpha1-antitrypsin deficiency
C. Hepatitis A
D. Hepatitis B
E. Wilson's disease
Explanation:
The correct answer is A. Mallory bodies are eosinophilic cytoplasmic inclusions ("alcoholic hyaline") that are
found in the largest numbers in alcoholic hepatitis. They were originally considered to be pathognomic of
alcohol abuse, but have since been found (in much smaller numbers) in many other liver conditions.
Alpha1-antitrypsin deficiency (choice B) involvement of the liver is characterized by periodic acid Schiff
(PAS)-positive cytoplasmic granules in hepatocytes.
Hepatitis A (choice C) and hepatitis B (choice D) infections are definitively established with serologic markers.
In Wilson's disease (choice E), there is excess copper deposition in the liver.

A 17-year-old boy has been taken to the emergency room three times for acute abdominal pain, but was
released each time without a definitive diagnosis. The frustrated emergency room physicians now consider him to
be "crazy," and have labeled him as having "irritable bowel syndrome." On the fourth occasion, he is brought in
passing stool-contaminated urine and a fistula is demonstrated between the small intestine and bladder. Which
of the following diseases would be most likely to cause this clinical scenario?

243

A. Celiac disease
B. Crohn's disease
C. Diverticulitis
D. Ulcerative colitis
E. Whipple's disease
Explanation:
The correct answer is B. Patients with gastrointestinal disease of a number of types have unfortunately had the
experience of "not being believed" when they complained to physicians about their problems. Both Crohn's
disease and ulcerative colitis (choice D) can present with abdominal pain, however the presence of a enteric
fistula favors the diagnosis of Crohn's. Fistulae can occur with Crohn's disease because the inflammatory
process involves the entire bowel wall, in contrast to the mucosal involvement in ulcerative colitis.
The distinctive feature of celiac disease (choice A) is malabsorption due to gluten sensitivity. The
malabsorption generally improves promptly with removal of gluten from the diet.
Diverticulitis (choice C) can present with abdominal pain and fever, but is usually a disease of older adults.
Whipple's disease (choice E) is a small intestinal malabsorption syndrome that has been related to microbial
infection (macrophages can be seen containing bacilliform bodies that have been identified as Tropheryma
whippellii ).

Before being approved by the FDA, a chemical is tested for carcinogenicity by examining its mutagenic effects on
bacterial cells in culture. Which of the following tests is used to make this determination?
A. Ames test
B. Nitroblue tetrazolium test
C. Watson-Schwartz test
D. Widal test
E. Woellner enzyme test
Explanation:
The correct answer is A. The test described is the Ames test, which measures damage to DNA and correlates
well with carcinogenicity in vitro. It is relatively inexpensive to perform, compared to other tests of
carcinogenicity, and is frequently used as a screening test for potential carcinogens.
The nitroblue tetrazolium test (choice B) is used to examine the ability of neutrophils to undergo a respiratory
burst, and is used in the diagnosis of hereditary immunodeficiencies.

244
The Watson-Schwartz test (choice C) detects porphobilinogen in urine, and is used in the diagnosis of
porphyrias.
The Widal test (choice D) is used to diagnose typhoid fever.
The Woellner enzyme test (choice E) detects heterophil antibodies in patients with Epstein-Barr virus infection,
such as infectious mononucleosis.

A 35-year-old sailor from a merchant ship that has been cruising the Caribbean presents to a clinician because of
painful, flocculent masses in his groin. Physical examination demonstrates multiple enlarged, abscessed lymph
nodes draining through the skin via indolent sinuses. The sailor describes previously having a small papular
lesion on his penis that spontaneously resolved. Which of the following is the most likely diagnosis?
A. Condyloma acuminatum
B. Granuloma inguinale
C. Herpes virus infection
D. Lymphogranuloma venereum
E. Syphilis
Explanation:
The correct answer is D. This is lymphogranuloma venereum. The enlarged, abscessed lymph nodes are
termed &ldquo;buboes&rdquo; (which can also occur in bubonic plague). The only other commonly discussed
sexually transmitted disease in which they can occur is chancroid. Histologically, the buboes of
lymphogranuloma venereum are enlarged lymph nodes with stellate abscesses. The primary lesion is usually a
self-healing papule or shallow ulcer. The causative organism is Chlamydia trachomatis.
Condyloma acuminatum (choice A) causes a papillary lesion and does not cause buboes.
Granuloma inguinale (choice B) causes a spreading ulcer and does not cause buboes.
Herpes virus infection (choice C) causes tiny vesicles and ulcers and does not cause buboes.
Syphilis (choice E) causes a painless nodule and does not cause buboes.

A 17-year-old woman notices that her urine becomes red after she is given sulfonamides for treatment of a
urinary tract infection. Both urine and serum test positive for free hemoglobin, and the urine red cell count is 1.2
million/mm3. A peripheral blood smear shows normocytic and normochromic red cells and a few "bite cells."
Deficiency of which of the following substances is most likely responsible for these symptoms?
A. Alpha-chain of hemoglobin
B. Beta-chain of hemoglobin

245

C. Glucose-6-phosphate dehydrogenase
D. Glycoprotein IIb/IIIa
E. Spectrin
Explanation:
The correct answer is C. The presence of free hemoglobin in the serum and urine, and "bite cells" due to splenic
removal of Heinz bodies (oxidized hemoglobin) all point to hemolysis. Hemolysis following oxidant injury by drugs
(sulfonamides, for example) or infection suggests glucose-6-phosphate dehydrogenase deficiency or the related
deficiencies of glutathione synthetase, pyruvate kinase, and hexokinase. These conditions are typically
asymptomatic between episodes of hemolysis.
Deficiencies of the alpha (choice A) and beta (choice B) chains of hemoglobin produce alpha and beta
thalassemia, respectively.
Deficiencies of glycoprotein IIb/IIIa (choice D) produce thrombasthenia, a platelet aggregation defect.
Deficiencies of spectrin (choice E) produce hereditary spherocytosis, characterized by hemolytic anemia and
splenomegaly.

A 27-year-old man presents to a physician because he was not able to feel a burn that he had sustained on the
stove burner. Neurologic studies demonstrate bilateral loss of pain and temperature sensation in the upper
extremities with preservation of touch sensation. No motor abnormalities are observed. This patient's condition
has been most frequently associated with which of the following?
A. Arnold-Chiari malformation
B. Broca's aphasia
C. Horner's syndrome
D. Tabes dorsalis
E. Wernicke's encephalopathy
Explanation:
The correct answer is A. The patient most probably has syringomyelia, in which softening and cavitation around
the central canal of the spinal cord damages crossing fibers of the spinothalamic tract with resulting bilateral
loss of pain and temperature (but not touch) sensation in the upper extremities. Many of the patients with
syringomyelia have Arnold-Chiari malformation, in which there is a congenital protrusion of the cerebellum and
medulla through the foramen magnum.
Broca's aphasia (choice B) is a nonfluent aphasia due to damage to the inferior frontal gyrus.
Horner's syndrome (choice C) is due to damage to the sympathetic innervation of the face, with resulting ptosis,
miosis, and anhidrosis.

246
Tabes dorsalis (choice D) is a degeneration of the dorsal columns and dorsal roots of the spinal cord due to
tertiary syphilis, producing impaired proprioception and locomotor ataxia.
Wernicke's encephalopathy (choice E) is a form of psychosis seen in alcoholics with vitamin B1 (thiamine)
deficiency.

A 5-year-old boy with a one month history of fevers and lassitude is found to have severe anemia, moderate
thrombocytopenia, and a white blood count of 12,000 cells per mm3. A bone marrow biopsy would most likely
reveal
A. acute lymphoblastic leukemia
B. acute myeloblastic leukemia
C. chronic lymphocytic leukemia
D. chronic myeloid leukemia
E. hairy cell leukemia
Explanation:
The correct answer is A. Acute lymphoblastic leukemia (ALL) is primarily a disease of children, with peak
incidence at 4 years of age. Approximately 80% of childhood leukemias are of the ALL type. Other features in
this scenario which further support an acute leukemia are fevers and anemia without marked elevations of the
white blood cell count.
Acute myeloblastic leukemia (AML; choice B) would present itself in a similar fashion - acute symptoms and
anemia with thrombocytopenia. Bone marrow biopsy is needed to definitively differentiate ALL and AML, but
AML represents only 20% of childhood leukemias. AML is primarily a disease of adolescents and young adults.
The chronic leukemias (choices C and D) are diseases of adulthood which present with nonspecific symptoms,
and are typically diagnosed when white counts are markedly elevated. Chronic myeloid leukemia (choice D), a
neoplasm of a pluripotent stem cell, also may present with thrombocytosis, rather than thrombocytopenia.
Hairy cell leukemia (choice E) is a relatively rare leukemia of older males. It infiltrates the spleen early in its
course and tends to present with pancytopenia due to bone marrow failure and splenic sequestration.

A lymph node biopsy from a patient with massively enlarged mediastinal nodes demonstrates multiple lacunar
cells. A few questionable classic Reed-Sternberg cells are found. This pattern is most consistent with which of the
following disorders?
A. Burkitt's lymphoma
B. Mixed cellularity Hodgkin's disease
C. Nodular sclerosing Hodgkin's disease
D. Normal lymph node

247

E. Predominately small cleaved cell lymphoma


Explanation:
The correct answer is C. Lacunar cells specifically suggest the nodular sclerosing variant of Hodgkin's disease.
Broad collagen bands should also be seen in the biopsy. The actual tumor cell in this lesion is the lacunar cell,
and classic Reed-Sternberg cells are difficult to find.
In Burkitt's lymphoma (choice A), sheets of small lymphocytes are interspersed with larger histiocytes,
producing a "starry sky" pattern.
In mixed cellularity Hodgkin's disease (choice B), classic Reed-Sternberg cells are plentiful.
Lacunar cells would not be seen in a normal lymph node (choice D) or in predominately small cleaved cell
lymphoma (choice E), a non-Hodgkin's lymphoma.

A 49-year-old African-American female with shortness of breath is found to have hilar lymphadenopathy on chest
x-ray. Biopsy of one of the lymph nodes reveals granulomas, and is highly suggestive of sarcoidosis. Which of
the following histological findings must have been present in the biopsy material to support the diagnosis of
granulomatous inflammation?
A. Asteroid bodies
B. Caseous necrosis
C. Epithelioid histiocytes
D. Fibroblast proliferation
E. Multinucleated giant cells
Explanation:
The correct answer is C. A granuloma is defined as a focus of chronic inflammatory reaction in which the
predominant cell type is the epithelioid macrophage or histiocyte. Epithelioid histiocytes are recognized on
standard H&E preparation by their pale pink cytoplasm and indistinct cell boundaries. Granulomas are
generally surrounded by a ring of lymphocytes and infrequent plasma cells. None of the other features are
required for the diagnosis of granulomas, although they may be present.
The granulomas of sarcoidosis usually contain many multinucleated giant cells, and frequently these giant cells
contain stellate inclusions called asteroid bodies. Neither asteroid bodies (choice A) nor giant cells (choice E)
are required to make the diagnosis of sarcoidosis, nor does their absence preclude the diagnosis.
Caseous necrosis (choice B) describes the gross appearance of cheesy, necrotic debris filling a nodule of
active tuberculosis, but also has come to describe its characteristic amorphous and eosinophilic appearance on
H&E sections. Caseous necrosis is suggestive of tuberculosis, not sarcoidosis.
Over time, fibroblasts proliferate (choice D) within the core of old, inactive granulomas, eradicating the
histiocytes and stereotypical appearance and producing a dense collagenous scar. This feature is not required
for the diagnosis of granulomas.

248

A patient's abdomen becomes distended with loculated masses of semi-translucent mucinous material produced
by a mucinous cystadenoma. Which of the following are the most likely sites for the primary tumor?
A. Colon or spleen
B. Liver or pancreas
C. Lung or bladder
D. Ovary or appendix
E. Prostate or gall bladder
Explanation:
The correct answer is D. This rare, but dramatic, condition is called pseudomyxoma peritonei, and is produced
when a malignant or benign (spread by rupture into the peritoneal cavity rather than true metastasis) mucus
producing tumor (mucinous cystadenoma or mucinous cystadenocarcinoma) produces gel-like mucus that fills
the peritoneal cavity. The usual sites of origin of these tumors are ovary and appendix. The condition, even
when benign, is difficult to treat because the mucus producing cells are spread all over the peritoneal lining,
and cannot be effectively removed. Home parenteral nutrition may be necessary as the gut (particularly
peristalsis) just does not function well in a sea of Jell-O that slowly becomes replaced with fibrous tissue bands.

A medical student examining a patient is startled when he cannot "find" the patient's heart during auscultation.
The patient laughs and tells him to "try the other side." Auscultation on the right side of the chest does
demonstrates an apparently normal heart beat. Further physical examination demonstrates that the liver edge
can be palpated on the left but not the right side of the abdomen. Questioning of the patient about his medical
history reveals a history of bronchiectasis and sinusitis. Which of the following should be suspected?
A. Down syndrome
B. Kartagener syndrome
C. Kawasaki disease
D. Marfan syndrome
E. Turner syndrome
Explanation:
The correct answer is B. Inversion of the heart, so that the morphologic left ventricle is in the right chest, is
called dextrocardia. Isolated dextrocardia is almost always associated with cardiac defects that may include
transposition of the atria and transposition of the great arteries. However, dextrocardia as part of situs inversus
totalis, with reversal of the thoracic and abdominal organs, is usually associated with a physiologically normal
heart. The cluster of situs inversus, sinusitis, and bronchiectasis is called Kartagener syndrome, which is
caused by defective ciliary function. Since migration of some embryonic cells during development appears to
be dependent on ciliary function, this is thought to explain the situs inversus.

249

You should associate Down syndrome (choice A) with an ostium primum type of atrial septal defect.
You should associate Kawasaki disease (choice C) with coronary artery aneurysms.
You should associate Marfan syndrome (choice D) with aortic dissection.
You should associate Turner syndrome (choice E) with coarctation of the aorta.

A 9-year-old boy complains of sudden severe pain in his left hip. There is no history of trauma. He says that he
had a "cold" for a few days. An x-ray film obtained at the local emergency department shows an "Erlenmeyer flask
shaped" distal femur. The medullary bone is hazy. There is osteosclerotic new bone formation, as well as areas of
corticomedullary osteonecrosis. Other members of his family have been known to have bone pain. Which of the
following findings would a biopsy of the marrow of the femur likely reveal?
A. Abnormal osteoclasts and mosaic lamellar bone
B. Accumulation of abnormal macrophages
C. Benign reactive bone around an osteoid nidus
D. Malignant sarcoma
E. Well-differentiated cartilage
Explanation:
The correct answer is B. The patient described above has Gaucher disease, a hereditary disease that affects
bones and other organs. Glucocerebrosides accumulate within macrophages in places such as bone marrow.
The failure of proper bone remodeling of the distal femur gives the characteristic "Erlenmeyer flask" shape on
x-ray. These patients suffer Gaucher crises from acute ischemia to the bone, usually in the pelvis and femoral
head. The pain is sudden, severe, and progressive, lasting 2 or more weeks before fading. The crisis usually
follows viral illness. Other findings include osteosclerotic bone and corticomedullary osteonecrosis.
Abnormal osteoclasts and mosaic lamellar bone (choice A) describe Paget disease of bone. This is a disease of
disordered bone remodeling that affects people older than 60. Although the etiology is unknown, it may be due
to viral effect on osteoclasts. The osteoclast activity is excessive, and increased absorption occurs. The
osteoclasts appear to have too many nuclei. The lamellar bone slowly acquires a mosaic pattern because of
irregular cement lines. Pain is usually due to fractures of the misshapen bone.
Benign reactive bone around an osteoid nidus (choice C) describes osteoid osteoma. This is a common lesion
that is found in young people aged 5-25 years. It is usually seen radiographically in the cortex of the diaphysis
as a small, round lesion, composed of a nidus of osteoid surrounded by woven bone. It is painful, usually at
night; the pain is relieved by aspirin. Surgery is curative.
Malignant sarcoma (choice D) would probably represent an osteosarcoma in this age and location. Clinically,
sarcomas do not present with acute pain, but rather are associated with steadily progressive pain. Radiographs
show a destructive tumor with elevated periosteum and reactive new bone formation. Treatment includes
amputation.
Well-differentiated cartilage (choice E), if located in the marrow space and not just caused by a misguided

250
biopsy attempt, would indicate a solitary chondroma or enchondroma. This is a benign tumor that is probably
hamartomatous in nature. It is asymptomatic and forms during development. It is sometimes found incidentally.

A 10-year-old boy is in a fire and sustains burns over 25% of his body. The next day, his serum urea nitrogen
(BUN) is 30 mg/dL and his serum creatinine is 0.8 mg/dL. He receives intravenous fluids throughout his course
and never has a significant drop in blood pressure or urine output. Which of the following most likely accounts for
his BUN and creatinine values?
A. Decreased renal perfusion
B. Distal urinary tract obstruction
C. Increased synthesis of urea
D. Renal glomerular disease
E. Renal tubule interstitial disease
Explanation:
The correct answer is C. This patient has elevated BUN and normal serum creatinine. Raised BUN with normal
creatinine can be seen in prerenal causes of azotemia, which can be subclassified into those due to decreased
perfusion of the kidney and those due to increased synthesis of urea. In this case, the boy's blood pressure
and urine output were maintained, so inadequate renal perfusion is unlikely. The burns themselves can cause
significant urea production, since urea is the major nitrogen-containing end product of protein catabolism, which
increases markedly in burn victims.
Decreased renal perfusion (choice A) also produces increased BUN with normal serum creatinine, but you
would expect to see urine output decrease as a consequence of the inadequate perfusion.
Distal urinary tract obstruction (choice B), due to processes such as stones, cancer, or benign prostatic
hyperplasia, causes both BUN and serum creatinine to rise, but the rise in serum urea is proportionally higher
than that of serum creatinine.
In renal glomerular disease (choice D) of sufficient severity to cause acute or chronic renal failure, creatinine
and urea usually rise in parallel.
In renal tubulointerstitial disease (choice E), notably in acute tubular necrosis, creatinine may rise
disproportionately to urea.

A 22-year-old male presents with complaints of dull lower back pain and morning stiffness. The pain was initially
episodic, but now has become persistent and bilateral. On physical examination, there is tenderness over the
costosternal junctions, spinous processes of the vertebrae, and the iliac crests. Which of the following tests would
be most likely to be helpful in establishing a diagnosis of ankylosing spondylitis?
A. C-reactive protein
B. Erythrocyte sedimentation rate

251
C. HLA typing
D. Serum alkaline phosphatase
E. Serum IgA
Explanation:
The correct answer is C. Ankylosing spondylitis is one of the spondyloarthropathies. It is a chronic inflammatory
joint disease primarily affecting the vertebrae and the sacroiliac joints, usually beginning in late adolescence or
early adulthood. Close to 90% of Caucasian patients with ankylosing spondylitis are HLA-B27 positive, so tests
for this HLA type are the most helpful of those listed in establishing the diagnosis.
C-reactive protein (choice A) and erythrocyte sedimentation rate (choice B) are non-specific markers of
inflammation that can be elevated in active ankylosing spondylitis.
Serum alkaline phosphatase (choice D) and serum IgA (choice E) can also be (usually mildly) elevated, but do
not specifically suggest ankylosing spondylitis.
A renal pathologist examining the day's kidney biopsies notes that one biopsy shows amorphous red nodules
within the glomerular mesangium in hematoxylin and eosin stained material. Congo red stain of the biopsy
demonstrates apple-green birefringence of these nodules. These nodules are most likely to be related to which
of the following?
A. Acute urinary tract infection
B. Diabetes mellitus
C. Sarcoidosis
D. Systemic lupus erythematosus
E. Tuberculosis
Explanation:
The correct answer is E. Glomerular nodules may be either the Kimmelstiel-Wilson nodules of diabetes mellitus
or may be composed of amyloid. An amyloid origin can be confirmed by staining with Congo red, which causes
the nodules to stain red with ordinary light but exhibit a distinctive "apple-green" birefringence when viewed with
polarized light. The most common type of amyloid (type AA) is due to deposition of altered immunoglobulin light
chains and is seen in diseases with chronic antigenic stimulation, such as tuberculosis.
Acute urinary tract infection (choice A) is an infection of too short a duration to cause amyloidosis.
Diabetic mellitus (choice B) is associated with glomerular nodules called Kimmelstiel-Wilson nodules, but these
will not show apple-green birefringence when viewed with polarized light.
Sarcoidosis (choice C) typically involves the lymph nodes, lungs, spleen, and to a lesser extent, the skin and
eye. It is unrelated to amyloidosis.
Systemic lupus erythematosus (choice D) can produce a wide range of renal lesions, but amyloid nodules are
not among the likely manifestations.

252

A microbiology laboratory reports growth of Staphylococcus aureus from pus drained from a breast abscess.
What is the most likely condition predisposing the patient to the development of a breast abscess?
A. Breast feeding
B. Endocarditis
C. Inflammatory breast carcinoma
D. Menopause
E. Paget's disease of the breast
Explanation:
The correct answer is A. Acute mastitis, frequently complicated by breast abscess formation, typically develops
in the postpartum period, when the nipples are first subjected to the physical stresses of breast feeding.
Bacteria enter the breast via cracks in the nipple, and flourish in the microenvironment of the lactating breast.
Acute mastitis causes redness, pain, and swelling in the affected breast; Staph. aureus is the most common
pathogen.
Although endocarditis (choice B) could send septic emboli to the breast, Staph. aureus endocarditis is more
commonly associated with bacteremia and fevers, proliferative glomerulonephritis, valvular dysfunction, and
emboli to the brain, kidneys, heart, and gut.
Inflammatory breast carcinoma (choice C) does not invoke an inflammatory response, and it is not associated
with an infection. Inflammatory breast carcinoma is characterized by tumor spread into the dermal lymphatics,
producing diffuse induration and skin tenderness with the typical peau d'orange appearance.
Mastitis arising in the perimenopausal period (choice D) is usually a chronic mastitis, caused by obstruction of
ducts due to inspissated secretions. Chronic mastitis is sterile. The breast tissue shows lymphocytes and
plasma cells surrounding dilated ducts filled by cellular debris.
Paget's disease (choice E) presents as skin changes on the breast or nipple, in association with ductal
carcinoma in the underlying breast that is percolating out into the epidermis. Paget's disease produces a
eczematous skin lesion that may be crusted or weeping, but it is not related to bacterial infection.

A patient complains to a physician of epigastric pain that fails to respond to antacids. Endoscopy demonstrates
an ulcerated mass on the greater curvature of the stomach. Genetic studies on the tumor demonstrate an
altered DCC gene. Which of the following tumor suppressor genes is found on the same chromosome as DCC?
A. BRCA-1
B. DPC
C. NF-1

253
D. NF-2
E. p53
Explanation:
The correct answer is B. The tumor is gastric carcinoma, which is associated with the DCC oncogene located
on the long arm of chromosome 18 (18q). DCC is also associated with carcinomas of colon. 18q also has the
DPC gene, associated with pancreatic cancer.
The BRCA-1 gene (choice A), associated with breast cancer and ovarian cancer, is on 17q.
The NF-1 gene (choice C), associated with neurofibromatosis type I, is on 17q.
The NF-2 gene (choice D), associated with neurofibromatosis type II, is on 22q.
The p53 gene (choice E), associated with many cancers, is on 17p.

A patient is brought to the emergency room following a seizure. Serum electrolyte studies demonstrate serum
sodium of 128 mEq/L. The urine osmolarity is higher than the serum osmolarity. Chest x-ray demonstrates a lung
mass. Which of the following forms of lung cancer is most likely to cause the described electrolyte imbalance?
A. Adenocarcinoma
B. Bronchioloalveolar carcinoma
C. Large cell carcinoma
D. Small cell carcinoma
E. Squamous cell carcinoma
Explanation:
The correct answer is D. The patient has SIADH (syndrome of inappropriate antidiuretic hormone secretion),
which can be caused by ectopic ADH secretion by small cell carcinomas of the lung, CNS disorders, chronic
pulmonary disease, and certain drugs. Features of SIADH include excessive water retention, hyponatremia
(which can lead to seizures when severe), and serum hypo-osmolarity with urine osmolarity greater than serum
osmolarity.
Associate adenocarcinoma (choice A) with peripheral lung cancer and lung cancer arising in scars.
Associate bronchioloalveolar carcinoma (choice B) with alveolar-like spaces and no link to smoking.
Large cell carcinoma (choice C) is an aggressive, undifferentiated lung neoplasm.
Associate squamous cell carcinoma (choice E) with bronchogenic origin, a strong association with smoking, and
hypercalcemia.

254
Examination of a skin lesion demonstrates very abnormal squamous cells with a high nuclear/cytoplasmic ratio
and clumped chromatin. These cells form nests within the epidermis that extend to the superficial surface of the
epithelium. In some places, nests of these cells have central areas of abnormal keratin formation. The basement
membrane is intact and no nests of cells are seen in the dermis. Which of the following terms best describes this
lesion?
A. Carcinoma in situ
B. Dysplasia
C. Invasive carcinoma
D. Metaplasia
E. Metastatic carcinoma
Explanation:
The correct answer is A. The lesion is an example of carcinoma in situ. The presence of keratin pearls and the
distribution of abnormal cells all the way to the top of the epidermis distinguish this lesion from simple dysplasia
(choice B), in which abnormal cells are seen only in the lowermost layers of the epidermis.
Invasive carcinoma (choice C) would be indicated by a disruption of the basement membrane or the presence
of nests of cells in the dermis.
The term metaplasia (choice D) is used when a normal epithelium is replaced by another mature, differentiated
epithelium.
Tumor cells in lymphatics or blood vessels would suggest progression to metastatic carcinoma (choice E).

Which of the following forms of ischemic heart disease manifests with slowly progressive heart failure, with or
without other clinical manifestations of myocardial ischemia?
A. Chronic ischemic heart disease
B. Myocardial infarction
C. Prinzmetal angina
D. Stable angina
E. Unstable angina
Explanation:
The correct answer is A. Atherosclerosis of coronary arteries is the underlying pathologic change in the great
majority of cases of ischemic heart disease. Ischemic heart disease may manifest with several clinical
syndromes depending on distribution and morphology of atherosclerotic changes and rate of progression. A
slowly progressive increase in luminal stenosis in all major coronary vessels results in diffuse ischemia of the
myocardium and dropout of scattered myocytes throughout the ventricular walls. Eventually, such a reduction

255
in myocardial mass will lead to decreased contractility and heart failure. Patients with this chronic ischemic
heart disease often come to clinical attention with signs and symptoms of congestive heart failure, sometimes
without any history of chest pain or arrhythmias.
Myocardial infarction (choice B) may be clinically silent (ie, without pain), although it is usually associated with
severe chest pain. If the infarct is extensive, cardiogenic shock ensues.
All forms of angina can be easily ruled out as correct choices since angina indicates, by definition, paroxysmal
chest pain due to reversible myocardial ischemia. Prinzmetal angina (choice C) is an uncommon form of angina
occurring at rest and caused by vasospasm. Stable angina (choice D) refers to chest pain with a predictable
pattern of onset, usually after physical efforts, emotional excitement, or any cause of increased cardiac load.
Unstable angina (choice E) manifests with chest pain in an unpredictable, inconstant pattern, often at rest and
with progressively increasing frequency. Stable angina is associated with fixed narrowing of coronary arteries
due to stable plaques; unstable angina results from disrupted atherosclerotic plaques. Disrupted plaques lead
to abrupt worsening of luminal stenosis by promoting formation of small platelet aggregates or producing small
microscopic atheroemboli. Unstable angina is a harbinger of myocardial infarction.
Which of the following would most likely be observed in the lung during an autopsy of a 2-week-old
infant who died of neonatal respiratory distress syndrome?
A. Alveoli filled with neutrophils
B. Dense fibrosis of the alveolar walls
C. Enlarged air space
D. Hyaline membranes and collapsed alveoli
E. Normal lung histology for age
Explanation:
The correct answer is D. Neonatal respiratory distress syndrome is a disease of immaturity. The
immature lung is not able to produce sufficient surfactant to prevent collapse of many alveoli.
Severe diffuse damage to alveoli causes precipitation of protein ("hyaline membranes") adjacent
to many alveolar walls. In infants that survive, particularly if oxygen was used for therapy,
the lungs eventually become heavily fibrotic (misnamed bronchopulmonary dysplasia).
Abundant neutrophils (choice A) would not be seen unless the patient had also developed
pneumonia.
Fibrosis (choice B) is a late, not early, feature of respiratory distress syndrome.
The air spaces are collapsed, not enlarged (choice C), in this condition.
The histology in these patients is usually markedly abnormal (not choice E).
A 4 year-old child develops a large erythematous rash around the site of a mosquito bite. One month
later, she is taken to a pediatrician because of a puffy face and swollen ankles. The scanty
urine sample has a reddish-brown hue, and contains both red blood cells and protein. Which of
the following distinctive features would be most likely to be seen on renal biopsy?
A. Fusion of podocyte foot processes
B. IgA in the mesangium

256
C. Linear IgG deposits
D. Onion-skinning of renal arterioles
E. Subepithelial electron dense humps
Explanation:
The correct answer is E. The disease is poststreptococcal glomerulonephritis, which can follow
either streptococcal pharyngitis (one to two weeks after the infection) or skin infection
(three to six weeks after the infection). The child is showing signs of both nephritis
(hematuria) and nephrosis (puffy face and swollen ankles).The characteristic feature of this
disease on renal biopsy is the presence of subepithelial humps, visible either by light or
electron microscopy.
Fusion of podocyte foot processes (choice A) suggests minimal change disease.
IgA in the mesangium (choice B) suggests Berger's disease.
Linear IgG deposits (choice C) suggests anti-glomerular basement membrane disease, which is
called Goodpasture's disease if it is accompanied by pulmonary damage.
Onion-skinning of renal arterioles (choice D) suggests malignant hypertension.
A 35-year-old man presents to an emergency department with an exquisitely tender prostate gland. An
acute bacterial infection is suspected. Which of the following organisms is the most common
pathogen in this setting?
A. Enterobacter
B. Escherichia
C. Klebsiella
D. Proteus
E. Pseudomonas
Explanation:
The correct answer is B. The disease is acute bacterial prostatitis; the usual route of
infection is by direct extension from a bladder or urethral source. Less commonly, hematogenous
or lymphatic spread from a distant site may occur. The most commonly isolated organism is
Escherichia coli; other common pathogens include Klebsiella,Proteus, Pseudomonas, Enterobacter,
Serratia, Enterococcus, and Staphylococcus aureus.
Enterobacter(choice A), Klebsiella(choice C), Proteus(choice D), and Pseudomonas(choice E) are
all gram-negative rods that can cause acute bacterial prostatitis, but they are not as common
as Escherichia coli.
On rectal examination, a patient is found to have a large, fungating mass protruding into the rectal
lumen. Biopsy of this mass demonstrates an invasive malignant tumor composed of glandular
structures. The development of this condition is most strongly associated with
A. diverticulitis

257
B. diverticulosis
C. juvenile polyposis syndrome
D. Peutz-Jeghers syndrome
E. ulcerative colitis
Explanation:
The correct answer is E. The disease is adenocarcinoma of the colon. Predisposing conditions
include inflammatory bowel disease (ulcerative colitis more than Crohn's disease) and
adenomatous polyps occurring either as an isolated finding or as part of familial syndromes
including familial polyposis coli, Gardner syndrome, and Turcot syndrome. Western diets high in
fat and protein and low in fiber are also thought to predispose for colon cancer.
Diverticulitis (inflamed diverticula, choice A) and diverticulosis (presence of diverticula in
colon, choice B) do not appear to predispose for colon cancer.
Neither juvenile polyposis syndrome (choice C), in which the polyps consist of mucus-filled
tubules, nor Peutz-Jeghers syndrome (choice D), in which polyps form around an arborizing tree
of connective tissue and smooth muscle, predisposes for colon cancer.
A 3-year-old child is referred to a major medical center because of an abdominal mass arising from his
right adrenal gland. Biopsy of the lesion demonstrates sheets of small cells with
hyperchromatic nuclei containing occasional pseudorosettes composed of circles of tumor cells
with central young nerve fibers arising from the tumor cells. Which of the following oncogenes
is associated with this patient's tumor?
A. erb-B2
B. c-myc
C. L-myc
D. N-myc
E. ret
Explanation:
The correct answer is D. The tumor is a neuroblastoma, which is one of the principal forms of
cancer in children. Neuroblastoma typically occurs before age 5, with many presenting before
age 2. Neuroblastoma can arise from neural crest cells throughout the body, but the adrenal
medulla is the most common site. Homer-Wright pseudorosettes are circles of tumor cells with
central young nerve fibers arising from the tumor cells. The oncogene associated with
neuroblastoma is N-myc.
erb-B2 (choice A) is associated with breast, ovarian, and gastric carcinomas.
c-myc(choice B) is associated with Burkitt's lymphoma.
L-myc(choice C) is associated with small cell carcinoma of the lung.
ret (choice E) is associated with multiple endocrine neoplasia, types II and III.

258
Which of the following clinical conditions is most likely to result in hoarseness during speech?
A. Bronchogenic carcinoma of the apical segment of the right lung
B. Mitral valve insufficiency
C. Pulmonary thromboembolism
D. Retroesophageal right subclavian artery
E. Thymoma
Explanation:
The correct answer is A. A tumor of the apical segment of the upper lobe may impinge on the
recurrent laryngeal nerve since it ascends from the superior mediastinum to the root of the
neck in a groove between the trachea and esophagus. The recurrent laryngeal nerve supplies all
intrinsic muscles of the larynx except the cricothyroid.
Mitral valve insufficiency (choice B) causes a backup of blood into the left atrium. The
enlarged left atrium exerts pressure on the esophagus, which lies posterior to it, causing
dysphagia (difficulty swallowing).
Pulmonary embolus (choice C) causes an obstruction to arterial blood flow of the lung,
resulting in infarction of the affected segment. It may also cause pleurisy, resulting in pain
conveyed by intercostal nerves.
When the right subclavian artery (choice D) arises from the arch of the aorta, rather than from
the brachiocephalic trunk, it passes posterior to the esophagus to reach the right upper
extremity. This may cause compression of the esophagus, resulting in dysphagia.
Thymoma (choice E), or tumor of the thymus gland, may cause dyspnea (difficulty breathing) due
to pressure on the trachea. It may also cause engorgement of deep and superficial veins of the
neck due to pressure on the superior vena cava.
A 10-year-old girl is brought to the doctor with malaise, pallor, low-grade fever, and weight loss. She
says that she has had several nose bleeds recently. Her complete blood count shows a markedly
elevated white blood cell count. Bone marrow aspirate shows 35% myeloblasts and 55%
erythroblasts. According to the FAB classification, which acute myelogenous leukemia does this
patient have?
A. M2
B. M3
C. M4
D. M5
E. M6
Explanation:
The correct answer is E. The FAB classification (French American British) sorts AML into seven
categories (M1-M7). If the marrow has greater than 30% blasts, of which more than 50% are
erythroblasts, then the diagnosis is M6. The symptoms described are the typical presenting

259
features of AML. Only half of AML patients actually present with elevated WBCs.
M2 (choice A), or myeloblastic leukemia with maturation, is characterized by greater than 30%
blasts, with more than 50% of the blasts being myeloblasts and promyelocytes. Mature forms are
seen as well. Eosinophils may be heavy in some cases.
M3 (choice B), or hypergranular promyelocytic leukemia, is characterized by greater than 30%
blasts. The majority of cells have a promyelocytic appearance with kidney-shaped nuclei. Auer
rods are usually seen in the cytoplasm.
M4 (choice C), or myelomonocytic leukemia, is characterized by greater than 30% blasts, with
promonocytes and monocytes accounting for more than 20% of the bone marrow aspirate.
M5 (choice D), or monocytic leukemia, is divided into differentiated and poorly differentiated.
There are more than 30% blasts; the granulocyte component is less than 10% of marrow cells. The
monocytes can be detected cytochemically with the fluoride-sensitive esterase reaction.
After an upper respiratory infection, a 33-year-old man develops severe lower back pain. Over the next
several days, severe, generalized muscle weakness occurs, accompanied by distal paresthesia. On
physical examination, weakness is evident, but there is no appreciable sensory loss. Nerve
conduction studies show evidence of demyelination. Over the next 2 months, the patient recovers
with minimal residual sequelae. The mechanism underlying this patient's disease is similar to
the mechanism associated with which of the following diseases?
A. Creutzfeldt-Jakob disease
B. Friedreich ataxia
C. Huntington disease
D. Multiple sclerosis
E. Progressive multifocal leukoencephalopathy
Explanation:
The correct answer is D. The patient has Guillain-Barr syndrome, also known as acute
idiopathic inflammatory polyneuropathy. This condition, which typically follows an upper
respiratory or other infection by several days to a month, is due to an autoimmune attack on
the myelin of peripheral nerves. In this respect, it is most similar to multiple sclerosis,
which is an autoimmune attack on the myelin in the brain and spinal cord. Most cases of
Guillain-Barr syndrome resolve spontaneously. A few patients have recurrences, and rare
patients die during the acute episode of respiratory muscle failure (artificial ventilation may
be required).
Creutzfeldt-Jakob disease (choice A) is an Alzheimer-like condition caused by a prion (protein
infectious agent).
Friedreich ataxia (choice B) is an autosomal recessive disorder associated with spinocerebellar
degeneration. Ataxia and paralysis are seen beginning in adolescence.
Huntington disease (choice C) is an autosomal dominant degeneration of the caudate and frontal
lobes characterized by movement disorder and dementia.
Progressive multifocal leukoencephalopathy (choice E) is due to papovavirus (JC and SV40) brain
infection, which causes demyelination by killing oligodendroglia.

260

A 54-year-old man presents to a physician with headaches. A complete blood count demonstrates a
hematocrit of 62%. The peripheral smear shows normocellular erythrocytes, with increased
reticulocytes and nucleated red cells. Bone marrow biopsy demonstrates increased numbers of
erythrocytic precursors. Cancer of which of the following organs would be most likely to cause
these findings?
A. Colon
B. Kidney
C. Ovary
D. Prostate
E. Thyroid
Explanation:
The correct answer is B. The hematologic finding is erythrocytosis, which can be caused by
abnormal erythropoietin secretion by renal cell carcinoma (i.e., a paraneoplastic syndrome).
Absolute erythrocytosis also occurs in a number of other conditions, such as hypoxia, other
types of renal disease, some tumors (e.g., hepatocellular carcinoma, meningioma,
pheochromocytoma, cerebellar hemangioblastoma, adrenal adenoma), androgen therapy, Bartter's
syndrome, or in polycythemia vera.
Cancers of the colon (choice A), prostate (choice D), and thyroid (choice E) do not usually
produce inappropriate hormones.
Cancers of the ovary (choice C) can produce male or female sex steroids, but do not produce
erythropoietin.
A 55-year-old diabetic man has a long history of renal failure. He is currently waiting for an organ for
transplant, and has been on hemodialysis in the interim. If this man develops amyloid deposits
around his joints, they are likely to be composed of which of the following substances?
A. Amyloid-associated protein
B. Amyloid light chains
C. Beta2 microglobulin
D. Calcitonin precursors
E. Prealbumin (transthyretin)
Explanation:
The correct answer is C. Amyloid deposits appear similar by light and electron microscopy, but
actually represent a heterogeneous population of proteins that have a common tertiary structure
forming a beta-pleated sheet. Questions about the type of amyloid seen in different disease
states (even though the type is usually not determined clinically) are favorites on examinations.
Here are two tables to help you out:
Systemic amyloidosis: Chronic active disease AA amyloid from serum amyloid- associated
(SAA) protein from liver (choice A) Myelomas and related diseases AL amyloid from immunoglobulin

261
light chain (choice B) Chronic hemodialysis Beta2 microglobulin (related to high plasma levels)
(choice C) Nephropathic hereditary forms (familial Mediterranean fever) AA amyloid from SAA
(choice A) Cardiomyopathic hereditary forms (senile systemic amyloidosis)
Prealbumin/transthyretin (choice E) Neuropathic hereditary syndromes Prealbumin/transthyretin
(choice E)
Localized amyloidosis: Senile cardiac amyloidosis Atrial natriuretic peptiderelated fibrils Cerebral amyloid in Alzheimer disease, Down syndrome Cerebral amyloid in
Alzheimer disease, Down syndrome Medullary carcinoma of thyroid Calcitonin precursors (choice D)
Isolated, massive, nodular deposits (skin, lung, urogenital tract) AL from light chains (choice
B)
A 22-year-old college student comes to the emergency room complaining of right upper quadrant abdominal
pain for several hours. He admits to drinking excessive amounts of alcohol 3 days ago. Over the
course of the last few days he recalls that he ingested the contents of an entire bottle of
acetaminophen. On exam he is afebrile, with normal vital signs and slight tenderness in the right
upper quadrant. Which of the following is the most likely set of liver function enzymes in this
patient?
Aspartate aminotransferase (AST) Alanine aminotransferase (ALT) Alkaline
phosphatase Amylase
A. High High High Normal
B. High High Normal Normal
C. High Low High High
D. Normal Normal High Normal
E. Normal High Normal High
Explanation:
The correct answer is B. This patient is suffering from hepatic injury as a result of
acetaminophen overdose. Acetaminophen, when broken down by the liver, produces a hepatotoxic
metabolite that can cause liver necrosis. When ingested in large quantities, massive destruction
may occur, potentially resulting in liver failure and mandatory liver transplant for survival.
This patient's liver is even more susceptible to hepatotoxicity from acetaminophen after an
alcohol binge, which renders it less able to metabolize the toxic byproducts.
Both aspartate aminotransferase (AST) and alanine aminotransferase (ALT) are contained within
liver cells and are released when liver cells are destroyed, as in this case (often these enzymes
can be elevated to tens of thousands in an acute injury). Alkaline phosphatase is primarily
located within the cells of the bile ducts and biliary tree. Thus, an elevation of this enzyme
usually indicates pathology within the biliary tree (i.e., stones within the bile ducts, cancers
in the biliary tree, or other obstructive processes causing damage in the biliary tree). Amylase
is located within several different types of cells, but is used primarily in the diagnosis of
pancreatitis, in which it becomes elevated. Therefore, in this patient, both the AST and ALT would
likely be very high. It is unlikely that there would be any abnormality in the levels of alkaline
phosphatase or amylase.
A child presents with "freckles" all over his body, including the buccal mucosa, lips, palms, soles, and
skin not exposed to sun. Which of the following additional findings would most likely be
present?

262
A. Colonic polyps
B. Desmoid tumors
C. Epidermoid cysts
D. Osteomas of the jaw
E. Pigmented ocular fundus
Explanation:
The correct answer is A. The widespread "freckles" (spots of melanin pigmentation) described in
this case are associated with hamartomatous colonic polyps in Peutz-Jeghers syndrome. The
polyps in Peutz-Jeghers syndrome do not progress to colon cancer. Interestingly, Peutz-Jeghers
syndrome is associated with an increased potential to develop carcinomas of the pancreas,
breast, ovary, uterus, and lung.
All of the other features listed are components of Gardner's syndrome, a variant of familial
adenomatous polyposis syndrome, which carries a greatly increased risk of colon cancer.
A high school basketball player passes out in the middle of a game. He is rushed to the emergency room,
where he regains consciousness. He claims that just before he fainted, he had difficulty
breathing and experienced palpitations. On physical exam, he has a bifid apical impulse and a
coarse systolic murmur at the left sternal border. The echocardiogram reveals ventricular
hypertrophy with asymmetric septal thickening. Which of the following would you expect on
microscopic examination of his heart muscle?
A. Aschoff bodies
B. Disorganization of myofibrils
C. Infiltration by inflammatory cells
D. Localized fibrous scarring
E. Structures resembling poorly formed vessels
Explanation:
The correct answer is B. Microscopic examination of heart muscle from a patient with
hypertrophic cardiomyopathy reveals short, thick myofibrils arranged in circular patterns
admixed with normal tissue. On electron microscopy, myofibrils and myofilaments appear
disarrayed.
Aschoff bodies (choice A) appear in rheumatic myocarditis and consist of degenerating material
and leukocytes. These occur along with Anitschkow myocytes, which contain "ribbon-like" nuclei
and eosinophilic cytoplasm.
Infiltration by inflammatory cells (choice C) would be expected in association with an
infectious process such as acute bacterial endocarditis.
Localized fibrous scarring (choice D) is associated with myocardial healing after infarction.
Structures resembling poorly formed vessels (choice E) are found in cardiac myxoma, the most
common primary tumor of the heart. The tumor cells are derived from primitive multipotent

263
mesenchymal cells. They may be sessile or pedunculated masses.
An elderly woman is found dead in her home. Autopsy demonstrates an area of pallor on the apex of her
heart. On microscopic examination, there is coagulative necrosis of the muscles accompanied by
a modest neutrophilic infiltrate, which is focally heavy in some areas. No macrophages or
fibroblasts are seen. Approximately how long before her death did this woman experience a
myocardial infarction?
A. 2 hours
B. 18 hours
C. 3 days
D. 10 days
E. 10 weeks
Explanation:
The correct answer is C. The findings suggest a 2-4 day old myocardial infarction (MI). An MI
becomes visible grossly after about 12 hours (sometimes earlier) as an area of pallor and
eventually, cyanosis. Neutrophils infiltrate the tissue anywhere from 12 to 72 hours after
infarction, with increasing evidence of coagulative necrosis.
At 2 hours (choice A), no changes would likely be evident in the infarcted myocardium.
At about 18 hours (choice B), "waviness" of some myocardial fibers may be apparent
microscopically, and subtle changes in the appearance of the myofibrils producing alternating
densely-colored (contracted) and lightly-colored (expanded) areas can be observed.
At 5 to 10 days (choice D), both macrophages and neutrophils are present and granulation tissue
with fibroblasts is growing in from the edges.
After 7 weeks (choice E), the area of infarction is typically completely replaced by scar
tissue.
A patient with rheumatoid arthritis mentions to her physician that after many years without dental
problems, she has recently had seven dental caries filled. This finding suggests that she
should be evaluated for which of the following diseases?
A. Oral squamous cell carcinoma
B. Polyarteritis nodosa
C. Sjgren's syndrome
D. Systemic lupus erythematosus
E. Thyrotoxicosis
Explanation:
The correct answer is C. Rheumatoid arthritis can coexist with a variety of autoimmune diseases
(including those listed in the answers), but is most frequently associated with Sjgren's
syndrome. Sjgren's syndrome classically presents with keratoconjunctivitis (dry eyes) and

264
xerostomia (dry mouth, often resulting in dental caries and fissures in the oral mucosa). These
symptoms are due to autoimmune involvement with subsequent scarring of the salivary and
lacrimal glands. Parotid gland enlargement is common, as is vasculitis, Raynaud's phenomenon,
hyperviscosity syndrome, and peripheral neuropathy.
The development of oral squamous cell carcinoma (choice A) is not related to the presence of
dental caries.
Polyarteritis nodosa (choice B) is a systemic necrotizing vasculitis. Patients typically
present with low-grade fever, weakness, and weight loss. Abdominal pain, hematuria, renal
failure, hypertension, and leukocytosis may occur.
Systemic lupus erythematosus (choice D) is an autoimmune disease characterized by vasculitis,
rash, renal disease, hemolytic anemia, and neurologic disturbances.
Thyrotoxicosis (choice E) is not related to the development of dental caries.
Following a week of binge drinking, a patient is admitted with severe upper abdominal pain with
radiation to the back. He describes the pain as steady, but notes that it is more severe when
he lays down on his back. His blood pressure starts to drop, but no source of bleeding is
identified. The patient's white count is within normal limits. Which of the following
laboratory tests will most likely be diagnostic?
A. Antinuclear antibodies
B. IgG levels
C. Reticulocyte count
D. Serum acid phosphatase
E. Serum amylase
Explanation:
The correct answer is E. This is a classic presentation of acute pancreatitis. The other
classic presentation is as a complication of gallstones occluding the pancreatic duct. Many
cases of acute pancreatitis are comparatively mild, but severe cases can cause life-threatening
shock secondary to release of vasoactive substances from the damaged pancreas that is difficult
to treat. Acute pancreatitis is diagnosed by demonstrating elevated serum amylase levels.
Antinuclear antibodies (choice A) are present in a number of autoimmune diseases, such as
systemic lupus erythematosus.
IgG levels (choice B) are not affected by acute pancreatitis.
The reticulocyte count (choice C) is an index of bone marrow function that is used to evaluate
the etiologies of some anemias.
Acid phosphatase (choice D) is produced by the prostate, osteoclasts, muscle, and certain blood
cells, among others.
A 62-year-old man with a 40-pack-year history of cigarette smoking, who had a malignant melanoma on his
neck 5 years ago, presents to the emergency room with acute intestinal obstruction. Laparoscopic
examination of his abdomen demonstrates multiple, large, white masses on the serosa of his small
bowel. This patient most likely has which of the following?

265

A. An inflammatory lesion
B. Metastatic lung cancer
C. Metastatic melanoma
D. Primary intestinal lymphoma
E. Pseudomyxoma peritonei
Explanation:
The correct answer is C. The obvious answer is correct. The serosal surface of the intestines
is a common metastatic site for melanoma, and is an unusual site for other tumors not located
in the abdomen or pelvis (tumors of the abdominal and pelvic organs can seed the peritoneal
fluid by direct extension, producing tumor throughout the abdominal cavity). An important
feature of melanoma is that the metastatic lesions are often not obviously pigmented, and may
even be amelanotic when observed microscopically. The production of visible melanin pigment
requires that the tumor cells retain a sophisticated level of differentiation, and many of the
more aggressive (which are more likely to metastasize) components of the tumor may have lost
this function. The immunohistochemical tumor markers S-100 and HMB-45 can be very helpful in
these cases in establishing that the masses are indeed melanoma.
An inflammatory etiology (choice A) is unlikely, given the presence of the serosal masses.
Metastatic lung cancer (choice B) would be unlikely to metastasize to the bowel serosa.
Primary intestinal lymphomas (choice D) are quite rare compared with metastatic melanoma in a
patient with previous melanoma.
Pseudomyxoma peritonei (choice E) is associated with seeding of the abdomen with gelatinous
implants rather than discrete masses, often from a mucinous cystadenoma or cystadenocarcinoma
of the ovary or appendix.
A 68-year-old, well-developed, well-nourished black male presents to the emergency department
complaining of shortness of breath. He denies chest pain. He has no significant past medical
history and takes no medications. A chest x-ray shows clear lung fields, mild cardiomegaly and
a widened thoracic aorta with linear calcifications. An MRI of the chest shows aortic
dilatation in the thorax, extending proximally, with atrophy of the muscularis and wrinkling of
the intimal surface. What is the most likely etiology of this condition?
A. Atherosclerosis
B. Hypertension
C. Marfan's syndrome
D. Syphilis infection
E. Takayasu's arteritis
Explanation:
The correct answer is D. Although rare now because of advances in treatment, syphilitic
aortitis and aneurysm are still seen, especially in underserved populations. This complication

266
generally occurs 10 to 40 years after initial infection. The vasa vasorum of the aorta
undergoes obliterative endarteritis, leading to atrophy of the muscularis and elastic tissues
of the aorta and dilatation. Linear calcifications are often seen in the ascending aorta by xray. The intimal wrinkling or "tree barking" is also a common feature. Syphilitic aneurysm can
be associated with respiratory distress, cough, congestive heart failure and rarely, rupture.
Atherosclerosis (choice A) is the most common cause of aortic aneurysms. These are most often
located in the abdominal aorta, distal to the renal arteries. Intimal wrinkling and linear
calcifications are not seen.
Hypertension (choice B) is usually responsible for dissecting aneurysms located within 10 cm of
the aortic valve. Patients present with sudden chest pain, which is usually severe and tearing
in nature. The chronic hypertension causes a cystic medial necrosis, allowing the separation of
vessel layers.
Marfan's syndrome, an autosomal dominant connective tissue disorder (choice C) is also
associated with dissecting aneurysms, usually of the ascending aorta. The patients are often
very tall with arachnodactyly and ligamentous laxity. Their life-span is generally shortened.
This patient's description and age are not consistent with this diagnosis.
Takayasu's arteritis (choice E) is a syndrome characterized by ocular disturbances and weak
pulses in the arms. It occurs most frequently in young females. It is considered a giant cell
arteritis, and does not cause aneurysms.
An 18-year-old is brought to the emergency room following an automobile accident. Serum enzyme studies
demonstrate a thousand-fold elevation of creatine kinase (CK). Which of the following tissues
is most likely to be the source of the enzyme?
A. Bone
B. Brain
C. Heart
D. Kidney
E. Skeletal muscle
Explanation:
The correct answer is E. Creatine kinase occurs in several different forms, or isoenzymes, in
the body. Most tissues contain a mix of creatine kinase isoenzymes, but one species often
predominates. The MB isoenzyme of creatine kinase is associated with heart damage; the MM
isoenzyme is associated with muscle damage; and the BB isoenzyme is associated with brain
damage. Surgery and trauma are commonly implicated as causes of sufficient skeletal muscle
trauma to markedly elevate creatine kinase. If isoenzyme studies are done, the CK-MM (skeletal
muscle isoenzyme) isoenzyme will be elevated much more markedly than the CK-MB isoenzyme.
Bone (choice A) is not a significant source of CK.
Brain (choice B) can occasionally be a source of creatine kinase (BB isoenzyme), but would not
usually be the major source following an automobile accident.
Heart damage (choice C), as in myocardial infarction (unlikely at this age), can elevate
creatine kinase, but would not usually be the major source in an automobile accident. If the
dilemma arises as to whether an automobile accident was possibly secondary to a myocardial

267
infarction, the ratio of CK-MB to total CK may be helpful in demonstrating a cardiac
contribution (CK-MB is characteristic of heart muscle).
Kidney (choice D) is not a major source of creatine kinase.
In the course of a laryngoscopic examination for hoarseness, a small lesion is found on the true vocal
cord of a 57-year-old male smoker. On biopsy, severe squamous dysplasia is noted. If untreated,
this lesion may progress to which of the following?
A. Adenocarcinoma
B. Lymphoepithelioma
C. Mucoepidermoid carcinoma
D. Squamous cell carcinoma
E. Squamous papilloma
Explanation:
The correct answer is D. Squamous cell carcinoma is the most frequent type of cancer of the
larynx. As with squamous cell carcinoma of the uterine cervix, the development of laryngeal
carcinoma is related to an orderly sequence of morphologic changes. These begin with epithelial
hyperplasia, proceed through increasingly severe degrees of dysplasia up to in situ carcinoma,
and culminate with invasive carcinoma. Cigarette smoking is the most important risk factor for
the development of laryngeal carcinoma.
Adenocarcinoma (choice A) and mucoepidermoid carcinoma (choice C) are rare forms of laryngeal
cancer. Squamous dysplasia is not a precursor of either type of tumor.
Lymphoepithelioma (choice B) is a form of squamous cell carcinoma that most frequently occurs
in the nasopharynx, although it has been reported in the larynx as well. Its name is derived
from the fact that the tumor is rich in lymphocytes. This tumor occurs frequently in southern
China and certain regions in Africa. Epstein-Barr virus is implicated in its pathogenesis.
Squamous papilloma (choice E) is a benign laryngeal neoplasm caused by human papillomavirus
types 6 and 11. It is not associated with squamous dysplasia.
A patient presents to the emergency room with malaise. The physician notices slight jaundice. Serum
chemistries reveal marked elevation of both aspartate aminotransferase (AST) and alanine
aminotransferase (ALT), with the AST/ALT ratio being 2.5. Alkaline phosphatase is near normal.
Serum copper and serum iron are normal. Liver damage from which of the following diseases most
likely accounts for these findings?
A. Alcoholic hepatitis
B. Biliary cirrhosis
C. Hemochromatosis
D. Hepatitis A
E. Wilson's disease
Explanation:

268

The correct answer is A. Elevated AST and ALT with a AST/ALT ratio of greater than 1.5 is
strongly suggestive of alcoholic hepatitis. Microscopic features of this condition include
swollen and necrotic hepatocytes, neutrophil infiltration, Mallory (hyaline) bodies, fatty
change, and fibrosis around the central vein.
Biliary cirrhosis (choice B) would be associated with elevated alkaline phosphatase.
Hemochromatosis (choice C) would be associated with elevated serum iron levels.
In viral hepatitis (choice D), both AST and ALT are elevated, but the ratio is usually less
than 1.5.
In Wilson's disease (choice E), serum copper levels are usually raised, and ceruloplasmin (the
copper-carrying protein) levels are lowered.
A 47-year-old executive consults his physician with complaints of feeling tired and several months of
abdominal pain and "dark-colored" urine. The physician does a physical examination and notes
that the patient is slightly jaundiced, and his gallbladder is palpable, but not tender. Which
of the following disorders is most likely, given this presentation?
A. Acute cholecystitis
B. Amyloidosis
C. Hepatic cirrhosis
D. Hepatoma
E. Pancreatic cancer
Explanation:
The correct answer is E. Carcinoma of the pancreas is often associated with vague, non-specific
symptoms such as abdominal pain, but it may be initially asymptomatic. Jaundice is found in the
majority of patients with cancers of the head of the pancreas, along with an enlarged, palpable
gallbladder (Courvoisier's sign) without significant tenderness or other signs of
cholecystitis.
A 35-year-old roofer presents to his primary care physician complaining of dyspnea and chronic dry
cough. Chest x-ray reveals pulmonary hyperinflation with "honeycombing" and calcified parietal
pleural plaques. The most likely diagnosis is
A. anthracosis
B. asbestosis
C. berylliosis
D. byssinosis
E. silicosis
Explanation:
The correct answer is B. This question concerns a USMLE-favorite topic&mdash;pneumoconioses. Be

269
sure to know the classic clues associated with each of the answer choices to this question.
Asbestosis is a disease caused by a family of fibrous silicates commonly found in shipyards,
insulation, and roofing industries. Many years after exposure, patients complain of dyspnea and
chronic dry cough, along with recurrent respiratory infections (especially viral) and weight
loss. Classic pathological findings include lower lobe interstitial fibrosis with septal wall
widening, worse near the periphery of the lung. Chest x-ray often reveals hyperinflation of the
normal parenchyma leading to "honeycombing." Calcified parietal pleural plaques are also
commonly present. Secondary bronchiectasis may complicate the picture.
Anthracosis (choice A) is due to the inhalation of carbonaceous particles by city dwellers,
cigarette smokers, and miners. Deposition of carbon dust can be seen as black pigment in lung
parenchyma, pleura, and lymph nodes. When isolated, it is not associated with systemic disease.
Berylliosis (choice C) is due to heavy exposure to airborne beryllium or its salts. Because of
its high tensile strength and resistance to heat and fatigue, beryllium is used in the
electronic, ceramic, aerospace, and nuclear energy industries. Disease due to beryllium probably
represents a type IV hypersensitivity reaction, with noncaseating granuloma formation and
eventual fibrosis. There is increased risk of bronchogenic carcinoma.
Byssinosis (choice D) is a type of hypersensitivity pneumonitis that can occur with exposure to
cotton, linen, or hemp exposure. It is associated with histamine-related bronchospasm.
Silicosis (choice E) occurs with prolonged exposure to silica dust (mining, glass production,
sand blasting, farming, and road construction). This insidious disease can progress to
respiratory failure and death, and is associated with increased risk for tuberculosis. Classic
x-ray findings include calcified lymph nodes that produce an "eggshell" pattern. The disease
initially involves the upper lobes and perihilar region. Pleural involvement creates dense
fibrous plaques and adhesions that may obliterate the pleural cavities. Uninvolved parenchyma
tends to be hyperinflated and emphysematous.
An 80-year-old woman dies after a long history of progressive memory loss, apraxia, and recurrent
episodes of confusion. In the last months of life she was bedridden and unable to recognize
familiar faces and objects. The pathologist identifies numerous flame-shaped intracytoplasmic
inclusions in neurons of the neocortex and hippocampus. These consist of paired helical
filaments (PHFs) on electron microscopy. Which of the following biochemical changes most likely
accounts for the development of PHFs in this condition?
A. Abnormal degradation of amyloid precursor protein (APP)
B. Abnormal phosphorylation of tau
C. Accumulation of advanced glycosylation end (AGE) products
D. Increased expression of APP
E. Precipitation of insoluble &alpha;-tubulin
Explanation:
The correct answer is B. This patient has all of the characteristic clinical features of
dementia (memory loss, apraxia, and confusion). The intraneuronal inclusions found at autopsy
are neurofibrillary tangles. These appear as intracytoplasmic aggregates with a typical flame
shape and are composed of bundles of PHFs twisted around each other. PHFs result from abnormal
phosphorylation of a microtubule-associated protein called tau. Normally, tau is a soluble
protein that regulates tubulin polymerization. Excessive or abnormal phosphorylation converts
tau into an insoluble protein that precipitates as PHFs, filling the cytoplasm of neurons and

270
disrupting axoplasmic transport.
APP is a normal transmembrane protein of obscure function; the gene for APP is on chromosome
21. A&beta; amyloid is a 40 amino acid residue protein that originates from APP and forms the
core of senile plaques. Abnormal degradation of APP (choice A) is thought to be the crucial
event leading to formation of A&beta; amyloid in sporadic and familial Alzheimer disease, while
increased expression of APP (choice D) leads to accumulation of A&beta; amyloid in the brains
of patients with trisomy 21. Neither mechanism is the direct cause of PHF formation.
Advanced glycosylation end (AGE) products (choice C) are insoluble proteins (mostly collagen
and other long-lived proteins) that accumulate progressively in vessel walls as a result of
glycosylation. This is a mechanism underlying damage in diabetes mellitus and aging, but not in
PHF formation.
The function of tubulin, which is the main component of microtubules, is probably altered in
Alzheimer disease as a result of abnormal phosphorylation of tau and formation of PHFs.
However, precipitation of insoluble &alpha;-tubulin (choice E) has not been shown to occur in
Alzheimer disease.
A neonate has a large, purplish, slightly raised lesion on the skin of the forehead near the temporal
area. As the child grows, the lesion grows as well and becomes increasingly prominent. This
lesion may be a component of which of the following syndromes or diseases?
A. Buerger's disease
B. Milroy's disease
C. Osler-Weber-Rendu disease
D. Sturge-Weber syndrome
E. Von Hippel-Lindau disease
Explanation:
The correct answer is D. The lesion is a port-wine stain, which is a purple, flat
telangiectasia (vascular ectasia) that tends to occur on the head and to grow with the body.
These lesions, particularly when they lie in the distribution of the trigeminal nerve, may be
associated with angiomatous masses of the nearby leptomeninges as part of the Sturge-Weber
syndrome. Other features of Sturge-Weber syndrome include mental retardation, seizures, and
hemiplegia.
Buerger's disease (choice A) is a vasculitis especially affecting young men who are heavy
smokers.
Milroy's disease (choice B) is a congenital lymphatic malformation.
Osler-Weber-Rendu disease (choice C) is associated with multiple telangiectasias, but they are
usually small, rather than large.
Von Hippel-Lindau disease (choice E) is associated with cavernous hemangiomas, rather than a
port-wine stain.
A 3-year-old child is evaluated by a pediatrician for poor growth despite excessive food intake. The
mother reports that the child's stools are bulky, foul-smelling, and difficult to flush because
they float. Determination of which of the following would most likely be diagnostic in this

271
case?
A. Na+ in cerebrospinal fluid
B. Na+ in serum
C. Na+ in sweat
D. Na+ in urine
E. Na+ in whole blood
Explanation:
The correct answer is C. This is one of three common presentations for cystic fibrosis, which
is a common genetic recessive disorder in the Caucasian population. The two other common
presentations are meconium ileus in infancy and multiple respiratory tract infections, often in
later childhood. Cystic fibrosis was the first genetic disease for which a specific lab test
was developed: the sweat test. This test is still used diagnostically; in its modern form, in
which pilocarpine is used to stimulate sweat secretion. The basic underlying defect in cystic
fibrosis is an abnormality of the chloride channel, but both the Na+ and Cl- content of the
sweat of affected children is increased compared to normal.
The Na+ content of CSF (choice A), serum (choice B), urine (choice D), and whole blood (choice
E) is normal in children with cystic fibrosis.
A 45-year-old woman presents to her physician because of a severe "sore throat." Physical examination
demonstrates fever and an extremely tender, enlarged thyroid gland, but no throat erythema.
Serum thyroid studies demonstrate a mild degree of hyperthyroidism. Two months later, the
patient is asymptomatic, and thyroid function tests have returned to normal. She never again
experiences difficulty with her thyroid function. Which of the following was the most likely
cause of her hyperthyroidism?
A. Diffuse nontoxic goiter
B. Graves disease
C. Hashimoto's thyroiditis
D. Subacute granulomatous thyroiditis
E. Subacute lymphocytic thyroiditis
Explanation:
The correct answer is D. This patient most likely has subacute granulomatous (de Quervain's)
thyroiditis, which frequently develops after a viral infection. Microscopically, it is
characterized by microabscess formation within the thyroid, eventually progressing to
granulomatous inflammation with multinucleated giant cells. Clinically, patients may experience
fever, sudden painful enlargement of the thyroid, and/or symptoms of transient hyperthyroidism.
The disease usually abates within 6 to 8 weeks.
Diffuse nontoxic goiter (choice A) by definition does not produce hyperthyroidism.
The hyperthyroidism of Graves disease (choice B) does not spontaneously remit.

272
Hashimoto's thyroiditis (choice C) can cause transient hyperthyroidism, but then goes on to
cause hypothyroidism.
Subacute lymphocytic thyroiditis (choice E) can cause transient hyperthyroidism, but is
characteristically painless.
A macroscopic hepatic change known as nutmeg liver is indicative of
A. acute left-sided heart failure
B. acute right-sided heart failure
C. alcohol toxicity
D. chronic congestive heart failure
E. liver cirrhosis
Explanation:
The correct answer is D. Chronic heart failure results in blood stasis in the central veins and
central sinusoids of hepatic lobules, with subsequent central hemorrhagic necrosis. Thus, the
red central regions compared with the surrounding tan-brown viable parenchyma impart the
mottled appearance of a nutmeg to the liver cut surface.
Acute left-sided heart failure (choice A) gives rise to acute pulmonary edema, with
extravasation of plasma and red blood cells into alveolar spaces.
Acute right-sided heart failure (choice B) leads to acute congestion of the liver, which does
not acquire the typical nutmeg appearance as in chronic congestion.
Alcohol toxicity (choice C) leads to a number of hepatic alterations. Fatty change results from
alteration in lipoprotein metabolism, leading to accumulation of fats as a large single droplet
within the cytoplasm of hepatocytes (microvesicular steatosis). Alcoholic hepatitis is an acute
response to alcohol abuse and is associated with hepatocyte necrosis and formation of Mallory
bodies. Long-standing alcohol toxicity may cause a perturbation of the liver architecture
leading to liver cirrhosis.
Liver cirrhosis (choice E) is an end-stage condition that may be caused by a number of chronic
insults, such as alcohol toxicity, viral hepatitis B or C, and hemochromatosis. It results from
simultaneous degeneration and regeneration of the liver parenchyma with formation of broad
scars connecting portal spaces. The normal liver architecture is entirely lost and replaced by
regenerating nodules. The nodular pattern is the distinguishing macroscopic feature that allows
easy diagnosis, even on gross examination.
A 32-year-old male with a history of chronic drug abuse presents to his primary care physician
complaining of uncontrollable shaking in his hands. He moves very slowly and walks with a stooped
posture and shuffling gait. Physical exam reveals cogwheel rigidity, a pill-rolling tremor, and
masked facies. His condition deteriorates, and he eventually dies. Structures similar to which of
the following would be expected on autopsy?
A. Hirano bodies
B. Lewy bodies
C. Lipofuscin granules

273

D. Negri bodies
E. Neurofibrillary tangles
Explanation:
The correct answer is B. This patient has irreversible parkinsonism induced by a contaminant of
illicit drugs&ndash;methylphenyltetrahydropyridine (MPTP), a meperidine analog. The parkinsonism
is caused by the drug's destruction of dopaminergic neurons in the substantia nigra. The clinical
signs and symptoms described in the question are classically associated with parkinsonism. Lewy
bodies are neuronal inclusions that are characteristic of Parkinson's disease. The latest
research suggests that structures resembling immature Lewy bodies appear in MPTP-induced
parkinsonism.
Here's an important point on strategy: always take note of the patient's age and descriptors,
such as "chronic drug user." They often provide important clues to the correct answer.
Hirano bodies (choice A) are intraneuronal, eosinophilic rod-like inclusions in the hippocampus
that are associated with Alzheimer's disease. They may occur in normal elderly brains as well.
Lipofuscin granules (choice C) are pigmented cytoplasmic inclusions that commonly accumulate with
aging. They are believed to be residual bodies derived from lysosomes.
Negri bodies (choice D) are intracytoplasmic inclusions that are pathognomonic for rabies. They
are found in the pyramidal cells of the hippocampus, the brainstem, and the Purkinje cells of the
cerebellum.
Neurofibrillary tangles (choice E) consist of intracytoplasmic degenerated neurofilaments and are
seen in patients with Alzheimer's disease. Amyloid plaques are also commonly seen in Alzheimer's.
A 32-year-old woman with systemic lupus erythematosus (SLE) and chronic renal failure manifests rapidly
progressive weakness. On physical examination, she appears pale and has slightly yellow sclerae
and an enlarged spleen. Blood tests reveal severe anemia and mild, mostly unconjugated,
hyperbilirubinemia. Coombs test is positive at 37 C but negative at 0-4 C. This patient
developed anemia because of
A. bone marrow aplasia
B. IgG directed against red blood cells
C. IgM directed against red blood cells
D. renal failure
E. spleen sequestration
Explanation:
The correct answer is B. First, what type of anemia is it? It must be a hemolytic form, since
it is associated with unconjugated hyperbilirubinemia (hence the yellow sclerae), resulting
from increased destruction of red blood cells. Increased erythrocyte destruction is the cause
(not the effect) of splenomegaly. Furthermore, a positive Coombs test implies that hemolysis is
mediated by antibodies attached to red blood cells. Thus, the correct choice must be either IgG
or IgM. Since the Coombs test is positive at warm temperature (37 C), the antibody is a warm
agglutinin. Warm agglutinins are virtually always of IgG type and may be triggered by a variety

274
of disorders, including lymphomas, drugs, and autoimmune diseases such as SLE. IgG-coated red
cells are then sequestered by the spleen, where hemolysis occurs, thus explaining splenomegaly.
By contrast, cold agglutinins are IgM (choice C) and can be demonstrated by Coombs test at cold
temperature (0-4 C). Cold agglutinins are usually triggered by Mycoplasma pneumoniae infection
or lymphomas.
Bone marrow aplasia (choice A) is due to failure or suppression of myeloid stem cells, with
decreased production of red blood cells, platelets, and leukocytes (pancytopenia). Most
commonly, this condition is caused by chemical agents (especially drugs such as
chloramphenicol, phenylbutazone, alkylating agents, and antimetabolites) or total body
irradiation.
Renal failure (choice D) causes normochromic, normocytic anemia because of decreased synthesis
of erythropoietin. Erythropoietin administration is currently the standard treatment for this
form of anemia. Although renal failure is a frequent consequence of SLE, there is no evidence
that it plays a role in this patient's anemia.
Spleen sequestration (choice E) may cause anemia in case of massive splenomegaly because of an
exaggeration of the normal role of the spleen as repository of blood cells.
A primiparous woman at term experiences placental abruption and is rushed to the operating room for
emergency Cesarean section. Although the newborn recovers well from the delivery, the mother
develops shortness of breath, cyanosis, and copious bleeding from her surgical wounds. Levels
of which of the following blood components is expected to rise in this setting?
A. Factor V
B. Fibrin degradation products
C. Fibrinogen
D. Plasminogen
E. Platelets
Explanation:
The correct answer is B. DIC or consumptive coagulopathy represents pathological activation of
the coagulation system by another underlying disease, with consequent consumption and depletion
of the cellular and humoral components of the coagulation cascade. The fibrinolytic mechanisms
are also activated, and an uncontrolled cycle of bleeding and clotting develops. As a
consequence, levels of all clotting proteins (choices A and C) become depleted, platelet counts
drop (choice E), and the fibrinolytic proteins are depleted as well (choice D). Fibrin
degradation products (choice B), which are normally low in the serum, rise markedly in this
disease; identification of these proteins can be an important indicator of DIC.
A 38-year-old male has paroxysmal hypertension. He is subsequently found to have medullary carcinoma of
the thyroid, pheochromocytoma, and mucosal neuromas. Parathyroid involvement is not noted. What
is the most likely diagnosis?
A. MEN type I
B. MEN type II
C. MEN type III

275
D. Sipple's syndrome
E. Wermer's syndrome
Explanation:
The correct answer is C. MEN III, also known as MEN IIb, is characterized by medullary thyroid
carcinoma, pheochromocytoma, and mucosal neuromas.
Wermer's syndrome is also called MEN type I (choices A and E). It is characterized by
pancreatic (insulinoma), pituitary, and parathyroid involvement.
Sipple's syndrome, or MEN type II (choices B and D), is similar to MEN III, but it has
parathyroid involvement (tumor or adenoma) as opposed to neuromas.
Bilateral ovarian masses are identified on pelvic examination of a 40-year-old woman. Ultrasound
examination reveals multiloculated cystic masses involving both ovaries. The patient is treated
with total abdominal hysterectomy with removal of both adnexa. Pathologic examination
demonstrates papillary carcinoma producing serous fluid. Which of the following tumor markers
would be most useful in monitoring for recurrence?
A. Alpha-fetoprotein
B. Bombesin
C. CA-125
D. PSA
E. S-100
Explanation:
The correct answer is C. The tumors are serous papillary cystadenocarcinomas of the ovaries.
These tumors express CA-125 and are apparently derived from the surface epithelium of the
ovaries.
Alpha-fetoprotein (choice A) is not produced by this type of ovarian tumor, but can be produced
by testicular tumors, and less commonly, by ovarian tumors with a yolk sac tumor component.
Bombesin (choice B) is a marker for neuroblastoma, small cell carcinoma, gastric carcinoma, and
pancreatic carcinoma.
PSA (choice D) is a marker for prostatic carcinoma.
S-100 (choice E) is a marker for melanoma, neural tumors, and astrocytomas.
A middle-aged woman presents to her physician with fatigue, pruritus, and steatorrhea. Further studies
demonstrate sclerosing cholangitis and the presence of antimitochondrial antibodies. This
patient, if left untreated, is also at increased risk of developing which of the following?
A. Markedly decreased PaO2
B. Markedly decreased serum uric acid
C. Markedly increased serum cholesterol

276

D. Markedly increased serum pyruvic acid


E. Markedly increased serum pH
Explanation:
The correct answer is C. The patient's disease is primary biliary cirrhosis, which is an
autoimmune disease characterized by sclerosing cholangitis and cholangiolitis. The most helpful
clue in the question stem is the antimitochondrial antibody, which is present in over 90% of
the patients. The course is usually slowly progressive over 5-25 years. Patients are sometimes
diagnosed when routine blood tests demonstrate an elevated alkaline phosphatase. In addition to
the features listed in the question, patients with advanced disease tend to develop profound
hypercholesterolemia and xanthomas.
In a hospital cardiac care unit, there are three patients with different cardiac conditions: a 52-yearold man with dilated cardiomyopathy, an 18-year-old girl with mitral valve prolapse, and a 30year-old man with infective endocarditis of the mitral valve. Which of the following features do
all these patients most likely share?
A. Decreased compliance
B. Depressed myocardial contractility
C. Infectious etiology
D. Mitral valve stenosis
E. Risk of systemic thromboembolism
Explanation:
The correct answer is E. Systemic thromboembolism may develop in each of these patients.
Vegetations associated with infective endocarditis may undergo fragmentation and result in
systemic thromboembolism. Stasis develops in dilated ventricles, which predisposes to formation
of thrombi attached to the ventricular walls (mural thrombi). Mural thrombi may also form
within the left atrium in the presence of mitral valve prolapse. Thromboemboli may originate
from mural thrombi.
Decreased compliance (choice A) is a pathophysiologic alteration present in a variety of
cardiac disorders in which there is impediment to expansion or relaxation of ventricular walls,
such as restrictive cardiomyopathy, hypertrophic cardiomyopathy, and constrictive pericarditis.
This feature is not present in any of the conditions described in the question.
Depressed myocardial contractility (choice B) results from conditions that impair myocardial
inotropism, such as dilated cardiomyopathy and ischemic heart disease. Depressed inotropism is
not present in infective endocarditis or mitral valve prolapse.
Of the three conditions in the question stem, only infective endocarditis is definitely related
to an infectious etiology (choice C), usually bacteria. Recall that mitral valve prolapse is
due to myxomatous degeneration of the mitral valve, sometimes associated with Marfan syndrome.
The etiology of dilated cardiomyopathy is heterogeneous, and most cases are idiopathic. Of the
remaining cases, viral infections, toxic insults (especially alcohol), metabolic disorders
(hemochromatosis), pregnancy, and genetic influences are the underlying causes.
Mitral valve stenosis (choice D) may develop as a result of vegetations forming on the mitral

277
valve and occluding the valvular orifice. Endocarditis of the mitral valve more often leads to
mitral insufficiency because of destruction of valve leaflets or rupture of chordae tendineae.
On the contrary, both mitral valve prolapse (usually clinically silent) and dilated
cardiomyopathy may lead to mitral valve insufficiency and regurgitation.
A teenager brought to the emergency room is noted to be hyperventilating. Blood gas studies demonstrate
both a respiratory alkalosis and a metabolic acidosis. This presentation is most suggestive of
poisoning with which of the following agents?
A. Acetaminophen
B. Carbon monoxide
C. Lead
D. Mercury
E. Salicylates
Explanation:
The correct answer is E. It is worth learning the presentations of the common poisons listed in
the answer choices, because a timely diagnosis can sometimes save a patient's life. The
presentation of the teenager in the question is typical for salicylate poisoning, including
poisoning with aspirin. Fatalities are typically due to dehydration and hypokalemia.
In contrast to the acute toxicity of salicylates, acetaminophen (choice A) poisoning produces
nausea, vomiting, abdominal pain, and shock in some cases; it can cause irreversible hepatic
failure (which can be prevented or ameliorated by administration of N-acetylcysteine) 2-6 days
after ingestion.
Carbon monoxide (choice B) causes hypoxia, and a cherry red coloration of the lips and mucos
membranes may be evident.
Lead and mercury poisoning are both usually seen in their chronic forms, although mercury
ingestion can produce an acute toxicity with renal tubular necrosis and necrosis of
gastrointestinal epithelium. Chronic lead poisoning (choice C) can cause anemia (with
basophilic stippling of RBCs), neuropathy, and abdominal pain. Chronic mercury poisoning
(choice D) causes CNS atrophy, gingivitis, gastritis, and renal tubular changes.
A 60-year-old man with a 5-year history of stable angina pectoris sustains a head injury in an
automobile accident and dies. The family is contacted and consents to an autopsy. Microscopic
examination of a section of heart muscle would most likely reveal which of the following
pathologic findings?
A. Coagulative necrosis of the subendocardial muscle
B. Dense scar localized to the distribution of one coronary artery
C. Focal fibrosis and subendocardial myocardial vacuolization
D. Heavy neutrophilic infiltrate adjacent to a large area of coagulative necrosis
E. Transmural coagulative necrosis
Explanation:

278

The correct answer is C. Repeated episodes of stable angina pectoris typically cause gradual
loss of myocytes, which is seen pathologically as small patches of fibrosis and vacuolization
of damaged myocytes, typically in subendocardial locations (which are relatively poorly
perfused). The other answers describe features that may be seen in true myocardial infarctions,
of either the subendocardial or transmural type.
An elderly woman complains to her physician that her eyes and mouth are always dry. She also has
complaints of arthritis. What form of arthritis would most likely be diagnosed in this patient?
A. Gonococcal arthritis
B. Gouty arthritis
C. Osteoarthritis
D. Pseudogout
E. Rheumatoid arthritis
Explanation:
The correct answer is E. Dry eyes and mouth in an elderly woman are probably due to Sjgren's
syndrome, which is an autoimmune attack on the salivary and tear glands. Sjgren's syndrome is
associated, in some cases, with rheumatoid arthritis.
Gonococcal arthritis (choice A) is a systemic complication of gonorrhea, typically observed in
young women who have had a recent sexual encounter.
Gout (choice B) and pseudogout (choice D) can cause arthritis secondary to crystal deposition
in joints.
Osteoarthritis (choice C) appears to be due to repetitive low-level trauma to joints.
Biopsy of an ulcerated gastric lesion of a 60-year-old smoker demonstrates glands containing cells with
enlarged, hyperchromatic nuclei below the muscularis mucosa. Two tripolar mitotic figures are
noted. With which of the following infectious agents has this type of lesion been most strongly
associated?
A. Epstein-Barr virus
B. Helicobacter pylori
C. Human papilloma virus
D. Molluscum contagiosum virus
E. Schistosoma haematobium
Explanation:
The correct answer is B. The patient has gastric carcinoma, which has been strongly linked, in
at least some studies, to prior gastric infection with Helicobacter pylori. H. pylori has also
been implicated in the etiologies of gastric peptic ulcer, chronic gastritis, and
(questionably) gastric lymphoma.

279
Epstein-Barr virus (choice A) has been linked to African Burkitt's lymphoma and nasopharyngeal
carcinoma.
Human papilloma virus (choice C) has been linked to a variety of warts, condyloma, and genital
cancers.
Molluscum contagiosum virus (choice D) is a poxvirus that causes small tumor-like papules of
the skin.
Schistosoma haematobium(choice E) has been linked to bladder cancer.
A 20-year-old black woman with a history of multiple small bowel resections for Crohn's disease presents
complaining of fatigue and dyspnea on exertion. Her physical examination is notable for pallor
and a wide-based, unsteady gait. Her lab studies reveal a hemoglobin of 10.0 g/dl with a mean
corpuscular volume of 120 m3. Examination of the peripheral blood smear shows macrocytosis,
anisocytosis, poikilocytosis, and neutrophils with 6-8 nuclear lobulations. The most likely
cause of this patient's anemia is
A. beta-thalassemia trait
B. folate deficiency
C. iron deficiency
D. sickle cell trait
E. vitamin B12 deficiency
Explanation:
The correct answer is E. The patient has a macrocytic anemia with hypersegmented neutrophils.
This picture is consistent with either folate or vitamin B12 deficiency. The history of small
bowel resections and the presence of neurological signs on physical examination tip the
diagnosis to B12 deficiency.
Beta-thalassemia trait (choice A) causes a microcytic, not a macrocytic, anemia.
Folate deficiency (choice B) causes a similar picture on peripheral blood smear but is not
associated with neurological signs and symptoms.
Iron deficiency (choice C) causes a microcytic, not a macrocytic, anemia.
Sickle cell trait (choice D) does not cause anemia and is associated with a normal peripheral
blood smear under usual physiologic conditions.
A 35-year-old man returns to the U.S. after a recent trip to Central America. A few days later, he
develops altered consciousness and extreme irritability. All diagnostic investigations and
supportive therapy are unsuccessful, and the patient becomes comatose and dies. Autopsy reveals
elongated eosinophilic intracytoplasmic inclusions within the pyramidal neurons of the
hippocampus and Purkinje neurons of cerebellum. Which of the following is the most likely
diagnosis?
A. CMV encephalitis
B. Herpes encephalitis

280
C. HIV encephalitis
D. Lewy body disease
E. Rabies encephalitis
Explanation:
The correct answer is E. Rabies is still endemic in many areas of the world, including regions
of Central and South America. The etiologic agent is a virus transmitted to humans by the bite
of a rabid dog or other animal. Exposure to vampire bats (found in Central America) may lead to
infection without a bite. The disease manifests with extraordinary excitability, headache,
aversion to water, convulsions, and coma. In this case, the clinical information provided would
not be sufficient to make a correct choice, but the postmortem finding of elongated
intracytoplasmic neuronal inclusions (Negri bodies) is diagnostic of rabies. Negri bodies are
filled with virus particles and are most often found in the hippocampus and cerebellar cortex.
CMV encephalitis (choice A) affects fetuses (in utero infection) and immunocompromised
patients. The most distinctive microscopic feature is the presence of cytomegalic cells, ie,
abnormally large cells with a large intranuclear purple inclusion and granular basophilic
inclusion in the cytoplasm.
Herpes encephalitis (choice B) could be confused with rabies within the clinical context
provided here, but it is not associated with Negri bodies. It is caused by Herpes simplex virus
type 1 and frequently leads to hemorrhagic necrosis of the temporal lobes.
HIV encephalitis (choice C) is frequently seen in AIDS patients and is due to direct
involvement of the CNS by HIV. Dementia, incontinence, and seizures are frequent
manifestations. Pathologic features in the brain include "microglial nodules," which are
collections of lymphocytes, histiocytes, and microglial cells with variable numbers of
multinucleated giant cells. While microglial nodules are often encountered in viral
encephalitides, multinucleated giant cells are characteristic of HIV encephalitis.
Lewy body disease (choice D) is a dementing disorder characterized clinically by progressive
loss of memory, parkinsonism, and visual/auditory hallucinations, and pathologically by Lewy
bodies (round intracytoplasmic neuronal inclusions that contain alpha-synuclein) in the
substantia nigra and neocortex. The clinical features are obviously different from rabies. Lewy
bodies are also characteristic of Parkinson disease.
A four-month-old child develops cardiac failure and dies three months later. At autopsy, the heart has
no obvious congenital defects, but the cardiac chambers are small and covered with thick, white
endocardium. Histological sections of the heart demonstrate regular bands of elastic tissue in
the thick fibrous endocardium. Which of the following is the most likely diagnosis?
A. Coxsackievirus myocarditis
B. Cardiac amyloidosis
C. Endocardial fibroelastosis
D. Idiopathic subaortic stenosis
E. Loeffler's endocarditis
Explanation:

281
The correct answer is C. This disease is endocardial fibroelastosis, which is suspected of
being related to intrauterine virus infection, possibly with mumps virus. The endocardium
thickens secondary to a marked increase in fibrous and elastic tissue, which may extend into
the ventricle. The left ventricle is most commonly involved, but other chambers may also be
affected. A variety of other findings may also be present, including mural thrombi, flattened
trabeculae, and abnormal (usually stenosed) valves. The disease occurs in infantile and
adolescent forms.
Coxsackie virus myocarditis (choice A) would produce a flabby myocardium, patchy areas of
hemorrhage, and four chamber dilatation.
In cardiac amyloidosis (choice B), extracellular amyloid deposits would be seen in histological
sections.
Idiopathic subaortic stenosis (choice D) produces a hypertrophic cardiomyopathy.
Loeffler's endocarditis (choice E) is characterized by a prominent eosinophilic infiltrate with
no increase in elastic tissue.
A 75-year-old woman complains to a physician of facial pain, headache, and intermittent visual symptoms.
Both temporal arteries are biopsied. If the biopsies show abnormal vessels, which of the
following would be the most likely pathological finding?
A. Atherosclerotic plaque
B. Fibrinoid necrosis
C. Focal granulomatous inflammation
D. Fungal hyphae
E. Leukocytoclastic vasculitis
Explanation:
The correct answer is C. The suspected disease is temporal arteritis, also known as giant cell
arteritis, because of the focal granulomatous inflammation (with giant cells) that
characterizes the process. The cranial arteries (branches of the carotid system) are most often
involved. Vigorous attempts at diagnosis, which may require biopsying 2-cm lengths of each
temporal artery, are indicated because the process can be patchy. Correct diagnosis is
essential because untreated temporal arteritis can involve the ophthalmic artery, leading to
irreversible blindness.
Atherosclerosis (choice A) is almost never seen in the distal (lower blood pressure) part of
the temporal artery.
Fibrinoid necrosis (choice B) is a feature of polyarteritis nodosa, not temporal arteritis.
Fungal hyphae (choice D) would suggest a fungal infection rather than temporal arteritis.
Leukocytoclastic vasculitis (choice E) is a feature of hypersensitivity angiitis.
A 40-year-old woman has had several episodes of rheumatic fever as a child. She is currently afebrile
and feels well, and has come to a hospital for monitoring echocardiography. Which of the
following would be most likely to be seen in this patient's mitral valve?

282
A. Ballooning of valve leaflets
B. "Fish mouth" valve
C. Irregular beads of calcification in annulus
D. Large bulky vegetation with adjacent leaflet perforation
E. Tiny vegetations along line of closure of valve leaflet
Explanation:
The correct answer is B. Acute rheumatic fever is a multisystem inflammatory disease that can
follow group A streptococcal pharyngitis. The heart is a primary target of this disease and may
be damaged sufficiently to develop permanent sequelae. The most important of these sequelae is
chronic rheumatic heart disease with valvular damage. The damage most often involves the mitral
and/or aortic valves. The resulting thickened, blunted cardiac valve leaflets, often with
fibrous bridging between valve leaflets and calcification, frequently take on a "fish mouth" or
"button hole" stenotic morphology.
Associate ballooning of valve leaflets (choice A) with mitral valve prolapse.
Associate irregular, beadlike calcifications on the annulus (choice C) with calcification of
the mitral annulus, seen in elderly individuals.
Associate large vegetations and leaflet perforation (choice D) with acute bacterial
endocarditis, which usually involves healthy, rather than previously damaged, valves.
Associate tiny vegetations along line of closure (choice E) with marantic (nonbacterial
thrombotic) endocarditis, most typically seen at autopsy of patients who died after protracted
illness.
A 23-year-old construction worker presents to the emergency room after falling down on his outstretched
right hand. X-ray reveals a fracture of one of the wrist bones. The ER physician is concerned
about the risk for avascular necrosis. Which of the following bones was fractured?
A. Capitate
B. Lunate
C. Pisiform
D. Scaphoid
E. Trapezoid
Explanation:
The correct answer is D. Suspect fracture of the scaphoid (navicular) bone in any young adult
who has "fallen on an outstretched hand" (classic clue). The physician was concerned about the
risk of avascular necrosis because, in some people, the blood supply of the bone is located
distally and a fracture would deprive the proximal region of the bone of its arterial
nourishment. Osteoarthritis is a common complication of scaphoid fractures that do not heal
properly.
The lunate bone (choice B) is commonly dislocated in patients who have fallen on an

283
outstretched hand. Median nerve injury is a frequently associated occurrence.
The capitate (choice A), pisiform (choice C), and trapezoid (choice E) bones of the wrist are
less commonly associated with falls on an outstretched hand.
A 30-year-old pregnant woman complains to her physician of feeling very tired during her pregnancy. A
complete blood count with differential reveals a hematocrit of 30%, with hypersegmented
neutrophils and large, hypochromic red cells. Deficiency of which of the following would be most
likely to produce these findings?
A. Ascorbic acid
B. Calcium
C. Copper
D. Folate
E. Iron
Explanation:
The correct answer is D. The patient has a megaloblastic anemia, which can be due to deficiency
of folate or B12. Pregnancy increases the need for folate and other nutrients used by both baby
and mother, and may "unmask" a borderline dietary deficiency. For this reason, most
obstetricians recommend vitamin supplements for pregnant women.
Ascorbic acid (choice A) is vitamin C, and its deficiency predisposes for capillary fragility
and oral lesions.
Calcium deficiency (choice B) predisposes for osteoporosis/osteopenia.
Copper deficiency (choice C) is rare; when it occurs, it may cause a hypochromic anemia,
neutropenia, osteoporosis, or hypotonia.
Iron deficiency (choice E) causes a microcytic, hypochromic anemia, with reduced mental and
physical performance.
A 30-year-old female presents to her physician with fever, night sweats, and weight loss. Physical exam
reveals supraclavicular and lower cervical adenopathy. Lymph node biopsy reveals the presence
of Reed-Sternberg cells with lacunae surrounding the nuclei of the cells. Which of the
following variants of Hodgkin's disease is the most likely?
A. Lymphocyte depletion
B. Lymphocyte predominance
C. Mixed cellularity
D. Nodular sclerosis
Explanation:
The correct answer is D. The nodular sclerosis variant of Hodgkin's disease is more common in
women and is associated with the presence of lacunar cells [Reed-Sternberg (RS) cells with
nuclei surrounded by a clear space] and fibrous bands in the lymph nodes. It has a good

284
prognosis.
Lymphocyte depletion (choice A) is associated with scarcity of lymphocytes, multiple RS cells,
fibrosis, and necrosis. It has a poor prognosis.
Lymphocyte predominance (choice B) is the least common form of Hodgkin's. It is associated with
an abundance of lymphocytes and histiocytes and scant RS cells.
Mixed cellularity (choice C) is the most common form of Hodgkin's. It is associated with the
presence of neutrophils, lymphocytes, eosinophils, plasma cells, and histiocytes. Many classic
RS cells may be identified.
Examination of a peripheral blood smear demonstrates a leukemia composed of small mature lymphocytes
without blast forms. Which of the following is the most likely age of this patient?
A. 1 year
B. 5 years
C. 20 years
D. 45 years
E. 65 years
Explanation:
The correct answer is E. Different leukemias tend to affect populations of different ages. The
disease described is chronic lymphocytic leukemia (CLL), which is a disease of older adults.
Both the one year-old (choice A) and the five year-old (choice B) would be most likely to have
acute lymphocytic leukemia (ALL).
The 20 year-old (choice C) would be most likely to have acute myelocytic leukemia (AML).
The 45 year-old (choice D) would be likely to have either AML or chronic myelogenous leukemia
(CML).
A 10-year-old boy has a small tumor in the wall of the right lateral ventricle. A biopsy of this tumor
is consistent with subependymal giant cell astrocytoma. Which of the following lesions may also
be present in this patient?
A. Caf-au-lait spots
B. Cortical tubers
C. Hemangioblastoma
D. Lisch nodules
E. Schwannoma of the 8th cranial nerve
Explanation:
The correct answer is B. This item tests your knowledge of neurocutaneous syndromes, a group of
hereditary conditions characterized by concomitant neoplastic or hamartomatous lesions

285
predominantly affecting the skin and nervous system. Subependymal giant cell astrocytoma is a
peculiar astrocytic tumor that grows from the walls of the lateral ventricles. It is
pathognomonic of tuberous sclerosis, which is caused by mutations of TS1 or TS2 genes. Tuberous
sclerosis manifests with multiple hamartomatous lesions in the skin, CNS, and visceral organs.
Cortical tubers are malformed (hamartomatous) nodules of the cortex, probably resulting from
faulty cortical development. Other lesions include shagreen patches and ash-leaf spots on the
skin, cardiac myomas, and renal angiomyolipomas.
Caf-au-lait spots (choice A) are found in both types of neurofibromatosis. Lisch nodules
(choice D) are small pigmented nodular lesions of hamartomatous nature that are present in the
iris of patients with neurofibromatosis type 1. Schwannomas of the 8th cranial nerve (choice
E), especially when bilateral, are typically associated with neurofibromatosis type 2.
Hemangioblastoma (choice C) is a vascular tumor of unknown histologic origin that frequently
develops in the cerebellum of patients with von Hippel-Lindau syndrome. Renal cell carcinomas
are also common in this disease. Von Hippel-Lindau syndrome is caused by autosomal dominant
mutations of the VHL gene, a tumor suppressor gene. You may recall that mutations of VHL gene
are also found in the majority of sporadic renal cell carcinomas.
A 70-year-old male patient dies with severe dementia. Autopsy demonstrates marked atrophy of the frontal
and temporal lobes, with relative sparing of the rest of the brain. Which of the following
microscopic features would be most useful in establishing the diagnosis?
A. Enlarged presynaptic axon terminals surrounding a central core of extracellular
amyloid-like substance
B. Intracytoplasmic spherules composed of paired helical filaments
C. Intracytoplasmic spherules that stain brightly eosinophilic
D. Intranuclear and intracytoplasmic inclusion bodies in enlarged cells
E. Small cytoplasmic vacuoles containing a central granule
Explanation:
The correct answer is B. Selective frontal and temporal atrophy in a demented patient suggests
Pick's disease. Microscopically, severe neuronal loss and astrocytosis are seen. The
characteristic microscopic finding in this disorder is the presence of Pick's bodies, which are
intracytoplasmic spherules composed of paired helical filaments, seen best with silver stains.
Choice A describes senile plaques. These are seen in Alzheimer disease as well as in some
normal elderly individuals.
Choice C describes Lewy bodies, seen in Parkinson's disease.
Choice D describes the inclusion bodies typically seen in glial cells and endothelial cells in
cytomegalovirus encephalopathy.
Choice E describes granulovacuolar degeneration.
A 30-year-old female is diagnosed with cervical intraepithelial neoplasia associated with a previous
viral infection. Which of the following viral products are implicated in producing this type of
dysplasia?
A. EBNA proteins

286

B. E1A and E1B proteins


C. E6 and E7 proteins
D. Large tumor antigen
Explanation:
The correct answer is C. Cervical intraepithelial neoplasia (CIN) is associated with human
papilloma virus (HPV) infection, particularly types 16 or 18. These papillomaviruses produce E6
and E7 proteins, which induce the expression of cellular p53 and p110Rb oncogenes,
respectively.
EBNA proteins (choice A) are produced by the Epstein-Barr virus and are associated with
hepatocellular carcinoma, Burkitt's lymphoma, and carcinoma of the nasopharynx.
E1A and E1B proteins (choice B) are produced by adenoviruses 12, 18, and 31. They also bind to
the p53 and p110Rb oncogenes, respectively.
Large tumor antigen (choice D) is produced by simian virus 40 (SV40), a papovavirus found in
monkeys that is known to produce sarcoma in laboratory hamsters.
A 10-year-old child develops a testicular mass and undergoes orchiectomy. On cut section, the mass shows
a variety of appearances and colors. Histologically, many different tissues are seen, including
cartilage, thyroid, and neural tissue. A small focus of clear-cut squamous cell carcinoma is
seen. Which of the following is the most appropriate classification for this tumor?
A. Dermoid cyst
B. Embryonal carcinoma
C. Immature teratoma
D. Solid mature teratoma
E. Teratoma with malignant transformation
Explanation:
The correct answer is E. This is teratoma with malignant transformation. The possibility of
malignant transformation is why even obvious mature teratomas with very well differentiated
tissues should be completely excised. Malignant transformation is more common in teratomas in
adults than in children or babies.
Dermoid cyst (choice A) is a cystic form of mature teratoma, usually found in the ovaries.
Embryonal carcinoma (choice B) is a type of adenocarcinoma rather than squamous cell carcinoma.
Immature teratoma (choice C), while clinically malignant, shows embryonal tissues and often
displays no clear-cut cytological evidence of malignancy.
Solid mature teratoma (choice D) without the added descriptor "with malignant transformation"
is by definition a benign tumor. Careful extensive sampling is required to exclude minute foci
of cancerous transformation.

287
An infant is evaluated for partial intestinal obstruction that has been present from birth.
Esophagogastroduodenal endoscopic visualization fails to demonstrate an intraluminal lesion, but
does show prominent narrowing at the level of the mid duodenum. CT studies demonstrate a mass
lesion surrounding that portion of the duodenum. Which type of tissue is most likely present
within the lesion?
A. Gastric
B. Hepatic
C. Pancreatic
D. Renal
E. Small intestinal
Explanation:
The correct answer is C. The lesion is an annular pancreas, which is a rare pancreatic
malformation, in which the pancreatic head encircles the second part of the duodenum,
potentially causing obstruction.
An increase in thickness of the muscular part of the gastric wall (choice A) can cause
congenital pyloric stenosis.
While the liver is near the first and second parts of the duodenum, there is no common
developmental anomaly in which hepatic tissue (choice B) would surround the duodenum.
The kidneys (choice D) are located retroperitoneally and relatively far lateral from the
duodenum, and would not be expected to surround it.
Duplication or atresia of the duodenum itself (choice E) can occur, but would not cause the
appearance of a mass surrounding the duodenum.
A 65-year-old male with a history of smoking and alcohol abuse complains of poor appetite and difficulty
swallowing both solid and liquid foods over the course of the last 4 months. He has lost 20 lbs
and occasionally vomits blood. A mass is detected in his esophagus and is subsequently
biopsied. What is the most likely histological appearance of the biopsy?
A. Glandular epithelium associated with desmoplasia
B. Malignant tumor of mesenchymal origin
C. Squamous cell morphology
D. Tumor derived from all three germ layers
E. Tumor of epithelial origin demonstrating transitional cell morphology
Explanation:
The correct answer is C. The most common esophageal cancer is squamous cell in origin. Recall
that the esophageal mucosa consists of stratified, nonkeratinized, squamous epithelium. Smoking
and alcohol increase the risk for the development of esophageal carcinoma. Suspect cancer when
there are signs like rapid weight loss in a short period of time and symptoms such as
dysphasia, hematemesis, and anorexia.

288

Glandular epithelium (choice A) is associated with adenocarcinoma, which is often associated


with Barrett's esophagus. This is not as common as squamous cell carcinoma.
A mesenchymal tumor (choice B) is a sarcoma.
A teratoma can be derived from all three germ layers (choice D), e.g., a dermoid cyst in the
ovary.
Transitional cell morphology (choice E) is found in the urinary system and not in the
esophagus, unless it was metastatic to that location.
A 45-year-old woman has crippling arthritis of her fingers, wrists, and knees. Physical examination
demonstrates large subcutaneous nodules near the patient's elbows. Serology demonstrates a
positive rheumatoid factor. The patient's probable disease has the strongest association with
which HLA type?
A. HLA-A3
B. HLA-B27
C. HLA-DR2
D. HLA-DR3
E. HLA-DR4
Explanation:
The correct answer is E. The disease is rheumatoid arthritis, which commonly presents as
described in the question stem. Rheumatoid nodules develop in 20 to 30 percent of patients,
usually on or around joints, on extensor surfaces, or on other areas subjected to mechanical
trauma. RF stands for rheumatoid factor, and is actually anti-self IgG (usually IgM). The most
important HLA association is with HLA-DR4.
Associate HLA-A3 (choice A) with primary hemochromatosis.
Associate HLA-B27 (choice B) with psoriasis, ankylosing spondylitis, inflammatory bowel
disease, and Reiter's syndrome.
Associate HLA-DR2 (choice C) with systemic lupus erythematosus (also associated with HLA-DR3,
choice D).
Also, associate HLA-DR3 (choice D) with Type I diabetes (which has a second association with
HLA-DR4).
Which of the following is the most common site for development of embryonal rhabdomyosarcoma?
A. Gastrointestinal tract
B. Head and neck
C. Kidneys
D. Liver

289
E. Lungs
Explanation:
The correct answer is B. Embryonal rhabdomyosarcoma is the most common form of
rhabdomyosarcoma. The tumor is composed predominantly of small, round cells resembling cells
found in developing muscle. The head and neck (particularly the nose, nasopharynx, and orbit)
is the most common site for the development of embryonal rhabdomyosarcoma. Other favored sites
are the genitourinary tract (e.g., sarcoma botyroides, or embryonal rhabdomyosarcoma of the
vagina) and the extremities.
A 32-year-old woman presents with complaints of several months of burning substernal chest pain
exacerbated by large meals, cigarettes, and caffeine. Her symptoms are worse when she lies on
her back, especially when sleeping at night. Antacids often improve her symptoms. This patient
is at risk for which of the following conditions?
A. Cardiac ischemia
B. Columnar metaplasia of the distal esophagus
C. Mallory-Weiss lesion in the esophagus
D. Squamous cell carcinoma
E. Zenker's diverticulum
Explanation:
The correct answer is B. This woman likely suffers from reflux esophagitis, a condition in
which the lower esophageal sphincter (LES) does not adequately prevent acidic gastric contents
from refluxing back into the distal esophagus. Most commonly, there is a defect in the LES
mechanism itself, in addition to secondary causes such as pregnancy (due to increased abdominal
pressure) and some medications (anticholinergics, beta-2 agonists, theophylline, nitrates,
calcium-channel blockers). Symptoms can mimic cardiac chest pain and must be carefully
evaluated. Complications of reflux esophagitis include esophageal strictures, ulcerations,
laryngitis, pulmonary aspiration, and Barrett's esophagus (columnar metaplasia of the distal
esophagus). Barrett's esophagus is considered a premalignant state, with roughly a 30-fold
increase in the incidence of esophageal adenocarcinoma. The normal squamous epithelium of the
esophagus transforms into columnar epithelium similar to gastric epithelium as a result of
recurrent reflux of acidic gastric contents.
Anginal pain, signaling cardiac ischemia (choice A), is generally not burning in nature, and is
not relieved by antacids.
Mallory-Weiss lesions (choice C) are actual tears of the epithelia of the proximal stomach or
distal esophagus as a result of retching (seen in anorexics and alcoholics).
The incidence of pure squamous cell carcinoma (choice D) is not increased by acid reflux
disease.
Zenker's diverticula (choice E), the most common of esophageal diverticuli, are not true
diverticuli, but only mucosal herniations that can cause obstructive symptomatology.
A 70-year-old man presents with severe acute abdominal pain. Physical examination reveals tenderness
with guarding localized to the left lower quadrant. A complete blood count with differential
shows a white count of 18,000/mm3 with increased neutrophil band forms. Paracentesis

290
demonstrates mixed flora bacteria with many neutrophils in the peritoneal fluid. Which of the
following is the most probable source of the infection?
A. Bladder infection
B. Colonic diverticulum
C. Prostatic inflammation
D. Ruptured appendix
E. Ureteral stone
Explanation:
The correct answer is B. Diverticula that become impacted with fecaliths (undigested food
residues) can become inflamed and perforate, causing the equivalent of appendicitis, but
typically involve the left rather than right lower quadrant. This complication of
diverticulitis can produce acute peritonitis, evidenced by the abdominal tenderness, guarding,
and leukocytosis, and can kill elderly debilitated patients. Diagnostic paracentesis may be
valuable in determining if perforation has occurred.
Bladder infection (choice A), usually related to an enlarged prostate, is common in elderly
men, but does not usually cause peritonitis.
Prostatic inflammation (choice C) or infection does not usually spread to the abdomen.
Acute appendicitis (choice D) is usually a disease of young adults and is characterized by
tenderness in the right lower quadrant.
Ureteral stones (choice E) can cause severe abdominal pain but do not usually cause
peritonitis.
A 25-year-old woman develops extensive pruritic wheals following ingestion of seafood to which she was
allergic. While these lesions are usually not biopsied, a biopsy would probably show which of
the following features?
A. Dilated superficial lymphatic channels
B. Granular complement and IgG at dermal/epidermal junction
C. Microscopic blisters
D. Munro microabscesses
E. Solar elastosis
Explanation:
The correct answer is A. Urticaria (hives) are pruritic wheals that form after mast cells
degranulate and trigger localized dermal edema with dilated superficial lymphatic channels. The
mast cell degranulation is sometimes, but not always, triggered by IgE-antigen interactions.
Granular complement and IgG deposition at the dermal/epidermal junction (choice B) is a
characteristic of systemic lupus erythematosus.

291
Microscopic blisters (choice C) are a characteristic of dermatitis herpetiformis.
Munro abscesses (choice D) are a characteristic of psoriasis.
Solar elastosis (choice E) is found in actinic keratoses.
A 54-year-old African-American female patient undergoes a routine insurance physical examination. Chest
x-ray reveals bilateral hilar masses. Biopsy of the masses shows granulomata, but acid-fast and
fungal stains are negative for organisms. Which of the following diseases should be suspected?
A. Caroli's disease
B. Raynaud's disease
C. Sarcoidosis
D. Scleroderma
E. Systemic lupus erythematosus
Explanation:
The correct answer is C. Sarcoidosis is a multisystem disease characterized by non-caseating
granulomata in a variety of organs. The disease may be symptomatic (respiratory and
constitutional symptoms) or may be discovered incidentally when chest x-ray or autopsy reveals
bilateral hilar adenopathy. Definitive diagnosis is based on biopsy, which reveals noncaseating granulomata that are negative for fungi or acid-fast bacilli. Sarcoidosis is more
common in individuals of African descent.
Caroli's disease (choice A) is a congenital malformation of the bile duct system.
Raynaud's disease (choice B) is a vasospasm of vessels that causes temporary ischemia in the
hands.
Scleroderma (choice D), or progressive systemic sclerosis, is characterized by progressive
fibrosis of skin and internal organs.
Systemic lupus erythematosus (choice E) is an autoimmune disease characterized by vasculitis
(which may produce a variety of symptoms depending on the site of the lesion), rash, renal
disease, hemolytic anemia, and neurologic disturbances.

In the 25th week of pregnancy, a G1P0 woman develops increased blood pressure (160/95) and proteinuria.
Physical examination demonstrates generalized edema, and serum chemistries demonstrate hyperuricemia and
increased concentrations of liver enzymes. Which of the following usually provides definitive therapy for this
patient's medical condition
A. Anti-hypertensive medical therapy
B. Delivery
C. Low dose aspirin
D. Oxygen supplementation

292
E. Renal dialysis
Explanation:
The correct answer is B. The woman has preeclampsia, the features of which include proteinuria and increased
blood pressure. A wide variety of other features of preeclampsia can also be seen, including excessive weight
gain, generalized edema, ascites, hyperuricemia, hypocalciuria, increased plasma concentration of von
Willebrand factor and cellular fibronectin, reduced plasma concentration of anti-thrombin III, thrombocytopenia,
increased hematocrit, increased liver enzymes, intrauterine growth retardation, and intrauterine hypoxia.
Modern theories suggest that the true primary lesion may involve the endothelium, and that medical control of
hypertension (choice A) actually only treats a small part of the syndrome. At present, the only definitive therapy
is delivery of the baby, and obstetricians often play a delicate game trying to delay delivery for a premature
baby's sake as long as possible while judging the severity of the preeclampsia and its immediate risks to mother
and fetus.
Low dose aspirin (choice C) may have a modest effect in preventing pre-eclampsia, but this is not yet well
established.
Oxygen supplementation (choice D) is used in some cases of cerebral hemorrhage secondary to preeclampsia,
but does not constitute definitive therapy.
Renal dialysis (choice E) is usually not necessary in preeclampsia, although both renal cortical necrosis and
renal tubular necrosis can occasionally occur in this disorder.
A baby is born with a large red, raised discoloration of the face that persists into adulthood. This type of lesion ismost
likely a component of which of the following syndromes?
A. Arnold Chiari malformation
B. Dandy-Walker malformation
C. Neurofibromatosis
D. Sturge-Weber disease
E. Tuberous sclerosis
Explanation: The correct answer is D. The lesion is a port wine stain, which is a vascular lesion that, unlike the more
common strawberry nevus, does not usually regress with age. These lesions can be a component of Sturge-Weber disease,
which can also include similar vascular lesions of the meninges (leptomeningeal angiomatosis) and sometimes cutaneous
angiomatosis at other sites.
Arnold Chiari malformations (choice A) are a cluster of related brain and spinal cord malformations in which
there is a downward displacement of the cerebellar vermis and tonsils into the foramen magnum.
Dandy-Walker malformation (choice B) includes a distended fourth ventricle with a hypoplastic (or absent) cerebellum.
Neurofibromatosis (choice C) includes benign and malignant peripheral nerve lesions and caf au lait spots.
Tuberous sclerosis (choice E) includes cortical tubers, adenoma sebaceum of the skin, pancreatic cysts, renal
angiomyolipomas, and cardiac rhabdomyomas.
Renal biopsy demonstrates concentric, laminated thickening of arteriolar walls due to proliferation of smooth muscle cells.
This process is best described by which of the following terms?
A. Atherosclerosis

293
B. Hyaline arteriolosclerosis
C. Hyperplastic arteriolosclerosis
D. Mnckeberg's arteriosclerosis
E. Polyarteritis nodosa
Explanation:
The correct answer is C. The "onion skinning" morphology of these arterioles is a feature of hyperplastic
arteriolosclerosis. Affected vessels may also show necrotizing arteriolitis with acute vessel wall necrosis
accompanied by fibrin deposition. This type of vessel change is a hallmark of severe (malignant) hypertension,
and can damage arterioles throughout the body, notably in the kidneys, intestine, and gall bladder.
Atherosclerosis (choice A) would produce luminal narrowing by plaques.
Hyaline arteriolosclerosis (choice B) would produce partial replacement of arterial walls by homogeneous, pink
material.
Mnckeberg's arteriosclerosis (choice D) is characterized by medial calcification of smaller arteries.
Polyarteritis nodosa (choice E) is characterized by focal acute inflammation of smaller arteries.
Two bone marrow aspirates are studied under the microscope. One is taken from an adult with Hemoglobin SS disease
(sickle cell anemia) and the other is from an adult with normal Hemoglobin A. The marrow aspirate from the patient with
sickle cell anemia could be identified due to its increased
A. fat
B. iron stores
C. medullary bone
D. megakaryocytes
E. myeloid:erythroid ratio
Explanation:
The correct answer is B. Adults with sickle cell disease have undergone decades of accelerated RBC formation and
destruction, leading to accelerated erythropoiesis in the bone marrow. Consequently the bone marrow becomes hyperplastic,
with marked increases in the number of normoblasts (erythroblasts) at the expense of
marrow fat and marrow bone (choices A and C). Although the white cell and megakaryocyte lines are
undiminished (choices D and E), there is a marked increase in RBC precursors and iron stores. Iron storage
increases as a consequence of both chronic transfusions and increased dietary absorption; these increased
iron stores can be appreciated with a Prussian blue stain.
Ultrasound examination of a developing fetus demonstrates a fluid-filled sac at the base of the fetus' spine that connects to the
spinal canal and apparently contains part of the spinal cord. A dietary deficiency of which of the following is most strongly
associated with this type of lesion?
A. Calcium

294
B. Folate
C. Iron
D. Vitamin C
E. Vitamin K
Explanation:
The correct answer is B. The lesion is a neural tube defect, probably a meningomyelocele, in which both
meninges and spinal cord herniate through a bony vertebral defect. These defects most commonly occur in the
lumbosacral region, typically resulting in motor and sensory deficits in the lower extremities, and bowel and
bladder dysfunction. This condition is now known to be associated with low maternal folate during the first three
to four weeks of pregnancy, a time when many women may be unaware of their pregnancy. It is now
recommended that all women of childbearing age consume at least 400 g of folic acid daily.
Calcium deficiency (choice A) can cause osteoporosis and osteopenia.
Iron deficiency (choice C) can cause iron deficiency anemia.
Vitamin C deficiency (choice D) can cause scurvy.
Vitamin K deficiency (choice E) can cause a bleeding diathesis.
A 30-year-old patient presents to a clinician because of intermittent, severe headaches accompanied by
perspiration, palpitations, and pallor. Blood pressure on the initial examination was within normal limits, but, when
the patient came in later with a headache, it was 180/135 mm Hg. Urinary vanillylmandelic acid (VMA) levels were
elevated. Roughly, what percentage of the tumors causing this pattern is malignant?
A. 1%
B. 10%
C. 50%
D. 90%
E. 99%
Explanation:
The correct answer is B. The tumor is a pheochromocytoma that intermittently secretes epinephrine and other vasoactive
amines, producing episodes of elevated blood pressure accompanied by headache. This is the tumor to associate with the rule
of the 10's: 10% malignant, 10% bilateral, 10% extra-adrenal, 10% calcify, 10% occur in kids, and 10% are familial. It is also a
favorite target on examinations, although the incidence is quite low.
An 82-year-old man develops a round, fluid-filled cystic structure on the back of his knee that limits the knee's
mobility. This most likely represents an enlargement of which of the following structures?
A. Deep infrapatellar bursa
B. Prepatellar bursa
C. Semimembranous bursa

295
D. Superficial infrapatellar bursa
E. Suprapatellar bursa
Explanation:
The correct answer is C. The lesion is commonly called a Baker's cyst, and anatomically represents an
enlarged semimembranous bursa. Baker's cysts are more commonly seen at the extremes of age.
The deep infrapatellar bursa (choice A) is on the anterior aspect below the knee.
The prepatellar bursa (choice B) is anterior to the patella.
The superficial infrapatellar bursa (choice D) is on the anterior aspect of the leg below the knee.
The suprapatellar bursa (choice E) is on the anterior aspect of the thigh above the knee.

Biopsy of a 4 mm rough, tan, and slightly raised skin lesion on the face of a 65-year-old man demonstrates
atypical basal cells with eosinophilic cytoplasm but persistent intercellular bridges. The stratum corneum is
thickened and parakeratosis is present; the remainder of the epidermis is thinned. Which of the following features
would probably additionally be seen in the dermis?
A. Benign nevus cells
B. Blue-gray elastic fibers
C. Large numbers of spindle-shaped fibroblasts
D. Malignant nevus cells
E. Touton giant cells
Explanation:
The correct answer is B. The lesion described is an actinic keratosis, which is a common premalignant lesion
caused by solar damage, which also characteristically damages the elastic fibers (changing their color in
stained tissue to blue-gray) of the superficial dermis.
Benign nevocellular nevus cells (choice A) are found in common moles (nevocellular nevi).
Large numbers of spindle-shaped fibroblasts (choice C) are found in dermatofibromas.
Malignant nevus cells (choice D) are found in melanoma.
Touton giant cells (choice E) are found in xanthomas.

A 50-year-old man consults a physician because of a lesion on his nose. On examination, a flesh-colored lesion
about 1 cm in diameter, with a rolled edge and central ulceration, is observed. Which of the following is the most

296
likely diagnosis?
A. Basal cell carcinoma
B. Eczema
C. Psoriasis
D. Urticaria
E. Verruca vulgaris
Explanation:
The correct answer is A. The description is typical for basal cell carcinoma. These skin cancers typically occur
on sun-exposed areas of the head, neck, and upper trunk. Basal cell carcinoma only rarely metastasizes, but
can become locally mutilating if neglected. When located on the face, it may be difficult to adequately excise
without damaging facial structures.
Eczema (choice B) typically involves a larger area of skin and may cause dryness, discoloration, and thickening
of the involved area. Blistering, erythema, or oozing may also be observed.
Psoriasis (choice C) is characterized by erythematous plaques with a silvery surface.
Urticaria (choice D) causes transient, nonpitting, erythematous wheals.
Verruca vulgaris (choice E), the common wart, causes well-demarcated verrucous papules, often on the hands.

A patient with respiratory symptoms undergoes bronchoalveolar lavage. Which of the following findings would be
most suggestive that congestive heart failure is the cause of the symptoms?
A. Ciliated bronchial epithelial cells
B. Encapsulated pairs of small cocci
C. Hemosiderin-laden macrophages
D. Rhomboid-shaped crystals
E. Single hat-shaped structures that stain with silver stains
Explanation:
The correct answer is C. Hemosiderin-laden macrophages are sometimes called "heart failure cells" because
they are so often seen in congestive heart failure. The congestive heart failure causes increased pulmonary
capillary pressure with tiny hemorrhages. The pulmonary alveolar macrophages phagocytize the dead
erythrocytes and retain the iron from the hemoglobin in the form of hemosiderin.
Ciliated bronchial epithelial cells (choice A) are a normal finding.

297
The cocci described in choice B are pneumococcus, and can cause pneumonia.
Choice D describes Charcot-Leyden crystals, found in allergic asthma.
Choice E describes the Pneumocystis organism, which can infect AIDS patients.

An adult presents to a physician because of repeated episodes of fainting. EKG fails to disclose an arrhythmia.
Echocardiogram shows a mass in the left atrium that is acting like a "ball valve" to produce intermittent
obstruction of flow. Which of the following would most likely be seen on microscopic examination of the resected
mass?
A. Benign myxoid tumor
B. Benign tumor with gland formation
C. Benign tumor with striated muscle differentiation
D. Malignant tumor with gland formation
E. Malignant tumor with striated muscle differentiation
Explanation:
The correct answer is A. The most common primary cardiac tumor of adults is the benign atrial myxoma. 90% of
these lesions involve the left atrium, where they can produce intermittent obstruction when they flop onto the
mitral valve. Resection is curative.
Neither benign (choice B, adenomas) nor malignant (choice D, adenocarcinomas) primary glandular tumors
usually involve the heart. If adenocarcinoma is seen, suspect metastatic disease.
The rhabdomyoma is the benign tumor with striated muscle differentiation (choice C) that can involve the heart.
This tumor is the most common cardiac tumor of children.
Rhabdomyosarcomas are malignant tumors with striated muscle differentiation (choice E) and typically involve
the head and neck or uterus.

A patient with long-standing, progressive congestive heart failure dies in respiratory distress. The lungs at
autopsy are 3 times their normal weight. Histologically, the alveoli show a proteinaceous granular precipitate,
engorged alveolar capillaries, and hemosiderin-laden macrophages. Other inflammatory cells are inapparent.
Which of the following is the most likely diagnosis?
A. Candida pneumonia
B. Pneumococcal pneumonia
C. Pneumocystis pneumonia
D. Pulmonary edema

298
E. Pulmonary infarction
Explanation:
The correct answer is D. These are the characteristic findings of pulmonary edema. The edema fluid is
apparent as a proteinaceous granular precipitate after histologic processing of the tissue. Hemosiderin-laden
macrophages in the tissue, called "heart failure cells," are the end result of ingestion of red blood cells by
alveolar macrophages. Pulmonary edema develops in heart failure when the pulmonary venous pressure rises
and the capacity for the tissue to reabsorb the fluid into the venous system is exceeded.
In Candida pneumonia (choice A), fungal hyphae and spores would be described.
In pneumococcal pneumonia (choice B), gram-positive cocci (Streptococcus pneumoniae) would be described.
In Pneumocystis pneumonia (choice C), hat-shaped, silver-stained cysts would be described.
In pulmonary infarction (choice E), ischemic necrosis of alveoli would be described.

A 3-year-old child is brought to the emergency room by her concerned parents. They state the girl has been
complaining of a severe headache and has had two episodes of vomiting. On physical examination, there is bilateral
papilledema and an impaired level of consciousness. Emergency contrast CT scan demonstrates displacement of the ventricular
system by a multilocular "mass" with well-defined white high-attenuation rings around black low-attenuation centers. The
lesion involves the cerebellum. To which of the following conditions i this lesion most likely related?
A. Bacteremia following tooth extraction
B. Bacterial meningitis
C. Lung abscess
D. Otitis media
E. Sinusitis
Explanation:
The correct answer is D. The child has a cerebellar abscess. Abscess of the brain can be caused by any of the
choices listed, but about 60% are related to middle ear infection. Cerebellar abscess is specifically associated
with middle ear infection.
Bacteremia following tooth extraction (choice A) and blood borne bacteria from lung abscess (choice C) are
relatively uncommon causes of intracranial abscess.
Subdural empyema in infants is usually a complication of bacterial meningitis (choice B).
Both middle ear infections and sinusitis (choice E) can cause frontal and temporal lobe abscesses.

Ophthalmoscopic examination of a 5-year-old child demonstrates a retinal angioma. This finding should raise the possibility of
which of the following syndromes?

299

A. Neurofibromatosis type I
B. Neurofibromatosis type II
C. Sturge-Weber disease
D. Tuberous sclerosis
E. Von Hippel-Lindau disease
Explanation:
The correct answer is E. Retinal angioma may be a "visible" manifestation of von Hippel-Lindau disease. This
syndrome is characterized by tumors and/or cysts that can involve many structures: retinal angiomas, capillary
hemangioblastomas of the CNS (notably cerebellum, but also many other sites), renal carcinoma (up to 30% of
cases, may occur at a young age or may be bilateral), and angiomatous or cystic lesions of many other organs
(epididymis, kidneys, liver, pancreas, lung, skin).
Neurofibromatosis type I (choice A) is characterized by Lisch nodules of the iris, caf-au-lait macules, and
peripheral nerve tumors.
Neurofibromatosis type II (choice B) is characterized by acoustic neural tumors and caf-au-lait macules.
Sturge-Weber disease (choice C) is characterized by port-wine nevus of the head and leptomeningeal
angiomatosis.
Tuberous sclerosis (choice D) is characterized by adenoma sebaceum of the skin and cortical tubers.

A 45-year-old man presents to a physician with back pain and facial pain. Physical examination demonstrates coarse facial
features and kyphosis. Laboratory examination is remarkable for elevated alkaline phosphatase.
X-ray studies demonstrate skull thickening with narrowing of foramina, and bowing of the femur and tibia. Bone biopsy
reveals a mosaic pattern of bone spicules with prominent osteoid seams. Which of the following neoplasms occurs at an
increased frequency in patients with this disorder?
A. Astrocytoma
B. Hodgkin's lymphoma
C. Meningioma
D. Non-Hodgkin's lymphoma
E. Osteosarcoma
Explanation:
The correct answer is E. The phrase "mosaic pattern" of newly formed woven bone is a specific tip-off for
Paget's disease of bone, and is not seen in other bone conditions. The clinical and radiologic presentation are
typical; an increased hat size may also be a clue. In its early stages, Paget's disease is characterized by

300
osteolysis, producing patchwork areas of bone resorption with bizarre, large osteoclasts. In the middle stage of
the disease, secondary osteoblastic activity compensates with new bone formation, producing the mosaic
pattern. In late Paget's, the bones are dense and osteosclerotic. Paget's disease is suspected to be related to
prior viral infection, but the cause remains mysterious. Complications include myelophthisic anemia, high output
cardiac failure, pain secondary to nerve compression, deformities secondary to skeletal changes, and in about
1% of patients, osteosarcoma or other sarcoma, typically involving the jaw, pelvis, or femur.
An increased incidence of astrocytomas (choice A) is associated with tuberous sclerosis.
Hodgkin's lymphoma (choice B) is usually a disease of young adults, although older patients may have the
lymphocyte-depleted form.
Meningiomas (choice C) are mostly benign tumors that affect adults, especially females. There may be an
association with breast cancer, possibly related to high estrogen states.
Non-Hodgkin's lymphoma (choice D) is more common in AIDS and other immunodeficiency states, although the
incidence in the immunocompetent is increasing.

A 28-year-old man discovers a mass in his neck while buttoning his shirt collar. Physical examination reveals a 2
cm mass in one thyroid lobe, which is &ldquo;cold&rdquo; on scintiscan. Aspiration of the nodule demonstrates
small &ldquo;solid balls&rdquo; of neoplastic follicular cells. Careful examination of these tissue balls reveals that
they contain microscopic blood vessels and fibrous stroma in their centers. Which of the following is the most
likely diagnosis?
A. Follicular carcinoma
B. Hashimoto's disease
C. Medullary carcinoma
D. Papillary carcinoma
E. Thyroid adenoma
Explanation:
The correct answer is D. The distinctive cell balls described are broken-off papillary clusters, and are
considered pathognomic for papillary carcinoma of the thyroid. This is the most common form of thyroid
carcinoma. It tends to present in the 3rd to 5th decade and shows a modest female predominance. Despite its
propensity for local lymphatic intrusion (which may cause multifocality of tumor in the thyroid or cervical lymph
node metastases), the tumor generally has an excellent prognosis with 90% 20-year survival.
Follicular carcinoma (choice A) is characterized by follicular cells and colloid on aspiration, and cannot be
reliably distinguished from thyroid adenoma.
Distinctive features of Hashimoto's disease (choice B) on aspiration are lymphocytes, plasma cells, and
macrophages.
The most distinctive feature of medullary carcinoma (choice C) on aspiration is the presence of amyloid.
Thyroid adenoma (choice E) shows follicular cells and colloid on aspiration, and cannot be reliably
distinguished from follicular carcinoma.

301

A 1-year-old child with mental retardation, seizures, and facial angiofibromas develops repeated episodes of
syncope. Echocardiogram reveals a mass in the left ventricle producing intermittent obstruction. Pathologic
examination of the resected mass demonstrates a cardiac rhabdomyoma. Which of the following lesions would
this patient most likely also have?
A. Acoustic neuromas
B. Berry aneurysm
C. Cortical tubers
D. Meningiomas
E. Neurofibromas
Explanation:
The correct answer is C. The disease is tuberous sclerosis, in which cortical hamartomas known as tubers can
be associated with facial angiofibromas, cardiac rhabdomyomas (probably actually a skeletal muscle
hamartoma), seizures, mental retardation, and astrocytomas.
Associate acoustic neuromas (choice A) with neurofibromatosis type II.
Associate berry aneurysms (choice B) with adult polycystic kidney disease.
Associate meningiomas (choice D) with breast cancer and, possibly, high estrogen states.
Associate neurofibromas (choice E) with neurofibromatosis type I.

Endoscopy performed on a patient with persistent substernal pain despite antacid use demonstrates irregular
erythematous patches several centimeters above the gastroesophageal junction. Biopsy of one of these lesions
demonstrates epithelial metaplasia. Which of the following cell types was most likely observed in the involved
areas?
A. Ciliated columnar epithelium
B. Cuboidal epithelium
C. Keratinizing squamous epithelium
D. Non-ciliated columnar epithelium
E. Non-keratinizing squamous epithelium
Explanation:
The correct answer is D. The medical condition is Barrett's esophagus, in which the normally non-keratinizing

302
squamous epithelium (choice E) of the esophagus undergoes metaplasia to gastric or intestinal-like epithelium
composed of non-ciliated columnar epithelial cells. Barrett's esophagus typically develops in the setting of
chronic gastroesophageal reflux, and significantly increases the risk of later development of adenocarcinoma of
the distal esophagus.
Ciliated columnar epithelium (choice A) is found in the respiratory tract.
Cuboidal epithelium (choice B) is found in the kidney, peritoneal lining, and pleural lining.
Keratinizing squamous epithelium (choice C) is found in skin.
Non-keratinizing squamous epithelium (choice E), in addition to being the normal epithelium of the esophagus,
is found in mouth, nose, and vagina.

An elderly man comes to medical attention because of anemia and multiple infections. Physical examination is
remarkable for hepatosplenomegaly. An automated blood count demonstrates pancytopenia. Review of the
peripheral smear by a hematologist demonstrates rare, distinctive, neoplastic white cells covered by fine, hairlike
projections. Which of the following characteristics would likely be associated with these cells?
A. Birbeck granules
B. Elevated leukocyte alkaline phosphatase
C. Philadelphia chromosome
D. Positive tartrate-resistant acid phosphatase
E. Production of Bence-Jones proteins
Explanation:
The correct answer is D. The disease is hairy cell leukemia, which, unlike most forms of leukemia, is
characterized by pancytopenia rather than increased numbers of circulating cells. This disease tends to affect
older men and the characteristic cells (which are not always obviously &ldquo;hairy&rdquo;) are positive for
tartrate-resistant acid phosphatase (TRAP).
Associate Birbeck granules (choice A) with Langerhans cell histiocytosis (histiocytosis X).
Associate elevated leukocyte alkaline phosphatase (choice B) with myeloid metaplasia.
Associate Philadelphia chromosome (choice C) with chronic myeloid leukemia (and occasionally with acute
lymphocytic leukemia).
Associate production of Bence-Jones proteins (choice E) with multiple myeloma.

A pathologist reports the presence of koilocytotic atypia in a cervical biopsy from a 25-year-old woman. On the
basis of this histopathologic finding, which of the following conditions does this patient most likely have?
A. Chlamydia infection

303

B. Gonococcal infection
C. Herpes simplex virus infection
D. Human papillomavirus infection
E. Severe squamous dysplasia
F. Squamous cell carcinoma in situ
G. Trichomonas infection
Explanation:
The correct answer is D. Human papillomavirus (HPV) is the cause of condyloma, which may occur on any
mucocutaneous surface of the male or female external genitalia. Squamous epithelial cells infected by HPV
show characteristic nuclear and cytoplasmic changes, referred to as koilocytosis or koilocytotic atypia, on tissue
sections or Pap smears. These changes allow the diagnosis of HPV infection to be made. The nuclei are
condensed and have irregular contours, acquiring a "raisin-like" appearance; cytoplasm vacuolization creates a
perinuclear halo. HPV is a crucial etiopathogenetic factor in the development of squamous cell carcinoma of the
cervix. Not all types of HPV, however, are associated with cervical carcinoma. High-risk types are 16 and 18,
whereas the most frequent low-risk types are 6 and 11. HPV types 16 and 18 express proteins (E6 and E7) that
inactivate p53 and Rb tumor suppressor genes, leading to neoplastic transformation. Although HPV infection is
a risk factor for the development of squamous dysplasia and carcinoma, koilocytosis is not equivalent to severe
squamous dysplasia (choice E) or carcinoma in situ (choice F). In the current Bethesda system for diagnosing
and reporting Pap smears, koilocytosis is likened to a low-grade squamous intraepithelial lesion, ie, mild atypia.
Remember that koilocytosis (koilocytotic atypia) = HPV infection, not severe dysplasia/carcinoma.
Chlamydia infection (choice A) of the genital tract is due to Chlamydia trachomatis. Approximately half a million
cases of Chlamydia urethritis (nongonococcal urethritis) are reported each year in the U.S. The infection is
often symptomatic in men and clinically silent in women. In addition to urethritis and lymphogranuloma
venereum, C. trachomatis causes trachoma, an eye infection endemic in some developing countries.
Gonococcal infection (choice B), or gonorrhea, is due to Neisseria gonorrhoeae, a gram-negative diplococcus.
There are about 700,000 cases of gonorrhea each year in the U.S.
Infection by herpes simplex virus (choice C) is the most common sexually transmitted disease in the U.S. It is
generally due to herpes simplex type 2 virus and manifests with painless blisters, which subsequently break
down, forming painful erosions.
Trichomonas infection (choice G) is also sexually transmitted. It is caused by Trichomonas vaginalis, an
anaerobic flagellated protozoon, and manifests with itching and a frothy white discharge in women. The infection
is usually asymptomatic in men. There are approximately 3 million cases of T. vaginalis infection each year in
the U.S.

A 52-year-old woman has long-standing rheumatoid arthritis (RA) and is being treated with corticosteroids and
nonsteroidal anti-inflammatory drugs (NSAIDs). Which of the following cardiac complications may arise in this
clinical setting?
A. Constrictive pericarditis

304
B. Dilated cardiomyopathy
C. Hypersensitivity myocarditis
D. Hypertrophic cardiomyopathy
E. Restrictive cardiomyopathy
Explanation:
The correct answer is E. What links rheumatoid arthritis (RA) to restrictive cardiomyopathy? Amyloid!
Long-standing inflammatory conditions such as RA are associated with deposition of a form of amyloid known
as AA (amyloid-associated protein), which may involve kidneys, heart, liver, skeletal muscle, and skin, for
example. Amyloid deposition in the myocardium results in decreased compliance and impaired diastolic filling,
i.e., restrictive cardiomyopathy. The myocardium has a rigid and waxy texture. This form of amyloid, as well as
any other biochemical form, can be visualized on tissue section by staining with Congo red, which acquires a
characteristic apple-green birefringence under polarized light.
Constrictive pericarditis (choice A) is due to any pathologic process that results in fibrous thickening of the
pericardium, with resultant impaired compliance. Clinically, therefore, this condition manifests with a picture
similar to restrictive cardiomyopathy because of impaired diastolic filling. Constrictive pericarditis is usually
caused by previous episodes of acute pericarditis, especially hemorrhagic, suppurative, and caseous
pericarditis.
Dilated cardiomyopathy (choice B) is characterized by massive ventricular dilatation and may be caused by
genetic alterations, myocarditis, toxic insults (alcohol), metabolic disorders (hemochromatosis), etc. Most cases
are idiopathic. The main pathophysiologic alteration is impaired contractility.
You may be tempted to think that this patient is prone to developing hypersensitivity myocarditis (choice C), but
this form of myocardial disease has been reported after treatment with some antihypertensive agents,
antibiotics, and diuretics -- not with corticosteroids or NSAIDs. Furthermore, myocarditis manifests acutely with
arrhythmias and heart failure, and chronically with dilated cardiomyopathy and congestive heart failure.
Most cases of hypertrophic cardiomyopathy (choice D) are familial and due to mutations in one of the genes
encoding proteins of the sarcomeres, most frequently &beta;-myosin heavy chain. This form of cardiomyopathy
leads to asymmetric hypertrophy of the left ventricle, with predominant thickening of the interventricular septum.
In this case, too, impairment of ventricular compliance is the basic mechanism leading to reduced diastolic filling
and heart failure.

Following an upper respiratory infection, a 7-year-old child develops multiple petechial hemorrhages over her
entire body. Her mother takes her to the physician, who does a complete blood count, revealing marked
thrombocytopenia. Which of the following diagnoses is most appropriate for this individual?
A. Bernard-Soulier syndrome
B. Idiopathic thrombocytopenic purpura
C. Thrombasthenia
D. Thrombotic thrombocytopenic purpura
E. Von Willebrand's disease

305

Explanation:
The correct answer is B. The process described is commonly called acute idiopathic thrombocytopenic purpura
(ITP), even though the autoimmune basis has been clearly established (some authors use "immune
thrombocytopenic purpura,&rdquo; so that the initials still work). The thrombocytopenia in this disorder appears
to be secondary to splenic destruction of opsonized platelets, and usually follows a viral upper respiratory tract
infection or exanthem. The acute form of ITP is usually explosive, but self-limited; a chronic form in adults may
respond to steroid therapy or splenectomy.
In Bernard-Soulier syndrome (choice A), platelets are unable to adhere to collagen.
Thrombasthenia (choice C), is associated with a functional defect in platelets (they do not aggregate).
In thrombotic thrombocytopenic purpura (choice D), platelets are consumed in intravascular platelet
microthrombi.
In von Willebrand's disease (choice E), deficient von Willebrand factor produces platelet dysfunction, but
thrombocytopenia is not prominent.

A 20-year-old man has a testicular mass, which on orchiectomy shows a malignant tumor with yolk sac
differentiation. Which of the following tumor markers would probably be most useful in monitoring the patient for
recurrent or metastatic disease?
A. Alpha-fetoprotein
B. Bombesin
C. Carcinoembryonic antigen (CEA)
D. Prostate-specific antigen (PSA)
E. S-100
Explanation:
The correct answer is A. Of the markers listed, alpha-fetoprotein is the best marker for non-seminomatous germ
cell tumors of the testes, such as this patient's yolk sac tumor. Alpha-fetoprotein is also a good marker for
hepatocellular carcinomas.
Bombesin (choice B) is a marker for neuroblastoma, small cell carcinomas, gastric carcinomas, and pancreatic
carcinomas.
CEA (choice C, carcinoembryonic antigen) is a nonspecific marker, but has the advantage that it is produced by
about 70% of colorectal and pancreatic cancers. It is also produced by some gastric and breast cancers.
PSA (choice D, prostatic specific antigen) is a marker for prostatic carcinoma.
S-100 (choice E) is a marker for melanoma, neural tumors, and astrocytomas.

306

A 61-year-old woman with leukemia abruptly develops an intensely itchy rash. Physical examination
demonstrates multiple erythematous patches of the distal arms and legs, some of which involve the palms and
soles. Some of the patches show central clearing with surrounding erythematous rings. Which of the following is
the most likely diagnosis?
A. Erythema migrans chronicum
B. Erythema multiforme
C. Kaposi's sarcoma
D. Psoriasis
E. Urticaria
Explanation:
The correct answer is B. The most specific clue in the description is the presence of erythematous patches with
central clearing, known clinically as "target lesions," which are associated with erythema multiforme. Both
erythema multiforme and its severe, life-threatening version, known as Stevens-Johnson syndrome, are
produced by immune complex deposition in dermal blood vessels. In approximately 50% of patients, no specific
precipitating cause is identified. In the remainder of patients, however, a variety of causes have been
implicated, including certain infections (herpes simplex, enteroviruses, Mycoplasma pneumoniae, Chlamydia,
histoplasmosis), drugs (penicillin, sulfonamides, phenytoin, aspirin, corticosteroids, cimetidine, allopurinol, oral
contraceptives), neoplasia (leukemia, lymphoma, multiple myeloma, internal malignancy), sarcoidosis, and
foods (notably emulsifiers in margarine).
Erythema migrans chronicum (choice A) also produces an annular erythematous rash with central clearing, but
usually affects the thigh, groin, and axilla; it is associated with Lyme disease.
Kaposi's sarcoma (choice C) causes purple lesions with no target lesions.
Psoriasis (choice D) causes erythematous plaques with silvery scale but does not produce target lesions.
Urticaria (choice E) causes wheals that are intensely pruritic, but does not produce target lesions.

A jaundiced, 43-year-old alcoholic male presents to the emergency room complaining of bright red blood in his
last two stools. He denies pain on defecation or changes in his bowel habits. Which of the following would be the
most likely finding on sigmoidoscopic examination?
A. Colorectal carcinoma
B. Diverticulitis
C. Hemorrhoids
D. Hyperplastic polyps
E. Ulcerative colitis

307

Explanation:
The correct answer is C. The patient's jaundice indicates that he has significant alcoholic liver disease. Hepatic
fibrosis or cirrhosis both produce vascular injury and portal hypertension, which leads to the development of
portosystemic shunts-most typically causing esophageal varices, caput medusae, and hemorrhoids. Bright red
blood in stools is a classic presentation of hemorrhoids.
Colorectal carcinoma (choice A) generally develops in an older population, and produces occult blood loss
rather than frankly bloody stools.
Although diverticulosis (choice B) may be an asymptomatic condition, diverticulitis is associated with significant
abdominal distress, which may be accompanied by blood loss and diarrhea and/or constipation.
Hyperplastic polyps (choice D) are asymptomatic, non-neoplastic, polypoid growths, generally less than 5mm in
diameter. They do not produce bleeding or any increased risk of carcinoma.
Ulcerative colitis (choice E) is an inflammatory bowel disease that produces mucoid diarrhea accompanied by
lower abdominal pain and cramps. Painless bleeding is unlikely to be ulcerative colitis.

SPACE FOR PHOTO


The cells and related structures indicated by the arrow are reduced in number in which of the following disorders?

A. Hirschsprung's disease
B. Kwashiorkor

308

C. Polio
D. Schwannoma
E. Scurvy
Explanation:
The correct answer is A. These are autonomic motor ganglion cells and nerve fibers surrounded by smooth muscle
of the muscularis externus. The close proximity of the smooth muscle and the orientation of the fibers are keys to
deciding that this is a myenteric plexus. Ganglion cells usually have a spherical cell body, although they are
considered multipolar, with a round nucleus and a prominent "owl's eye" nucleolus. The nerve fibers typically appear
wavy in light microscopic views. The reduction in number of these cells and associated fibers leads to the lack of
peristaltic control in the small and large bowel. The aganglionic bowel segment is narrowed because the lack of
peristalsis keeps stool from moving into that segment. The distal rectum is always involved, and the lesion can extend
proximally anywhere from a few centimeters past the rectum all the way up to the small intestine. The bowel proximal
to the lesion is usually dilated.
In kwashiorkor (choice B), a condition of extreme malnutrition, protein-secreting cells atrophy. The cells in this figure
do not contain protein granules. In such cells, the rough endoplasmic reticulum would also be abundant, and this is
not evident here.
In polio (choice C), motor neurons are lost from the anterior horn of the spinal cord.
Schwann cells produce schwannomas (choice D). Schwann cells are present among the nerve fibers, but they are
not abundant, and they are not specifically affected in Hirschsprung's disease.
Vitamin C deficiency leads to scurvy (choice E), characterized by abnormal collagen hydroxylation. The wavy fibers
do not have the typical bundle arrangement of collagen, but rather, have an appearance that is characteristic of
nerve.

A 38-year-old female presents to the physician with complaints of excessive thirst and urination for the past 4
weeks. Her appetite has been normal and she has not had diarrhea. Blood chemistry shows mildly elevated
glucose and glucagon. Physical examination reveals tenderness in the left upper quadrant and an erythematous
necrotizing skin eruption on her legs. Radiographic studies show a tumor in the pancreas. Which of the following
cells is responsible for this lesion?
A. Acinar cell
B. Alpha cell
C. Beta cell
D. Delta cell
E. G cell
Explanation:
The correct answer is B. The symptoms described are typical for a patient with glucagon excess. Glucagon is

309
secreted by alpha cells of the pancreatic islets of Langerhans. Increased levels are rare, usually due to
carcinoma (70%) or adenoma (30%) of the islets. Two-thirds of patients with carcinoma have liver metastasis at
the time of diagnosis. Increased glucagon causes a syndrome similar to diabetes mellitus, due to antagonism of
insulin effects. Patients also have migratory skin rashes, alopecia, hyperpigmentation of the skin, and glossitis.
Diagnosis is made by measuring glucagon.
The acinar cell (choice A), the main exocrine cell of the pancreas, contains digestive enzymes. Increases in the
release of these enzymes (especially amylase and lipase) occur with pancreatitis. Acute pancreatitis is
considered an emergent medical condition. Chronic pancreatitis is associated with fibrosis and atrophy of the
acinar structures.
Beta cells (choice C) are responsible for insulin release. An adenoma of beta cells (insulinoma) would cause
hyperinsulinism. 70% are solitary and 10% are multiple. Insulinomas cause hypoglycemia, dizziness, confusion,
and excessive sweating. Glucose needs to be given promptly to avoid coma and death. Diagnosis is made by
finding increased insulin and hypoglycemia.
Delta cells (choice D) are islet cells that secrete somatostatin. Tumors producing increased somatostatin are
usually malignant. Clinically, a diabetes mellitus-like syndrome occurs along with diarrhea. Diagnosis is made
from elevated serum somatostatin levels, but can be difficult because of the hormone's short half-life.
The G cell (choice E) is the islet cell that secretes gastrin. Gastrin excess is usually associated with
gastrinomas, 70% of which are malignant. Zollinger-Ellison syndrome ensues, causing low gastric and duodenal
pH, mucosal ulceration, and diarrhea. Diagnosis is made from high serum gastrin levels.

The patient shown in the pedigree above did poorly in school, then became increasingly forgetful and irrational.
Over a period of years, he developed ataxia and began posturing. Eventually, he was demented and unable to
care for himself. At autopsy, extensive cortical demyelination is observed. Microscopic examination of the areas of
demyelination reveals numerous macrophages containing crystals that stain a light brown color with toluidine
blue. This presentation is probably due to a deficiency of
A. arylsulfatase A
B. galactocerebroside &beta;-galactosidase
C. glucocerebrosidase
D. hexosaminidase A
E. sphingomyelinase
Explanation:
The correct answer is A. The presentation and autopsy findings are consistent with metachromatic
leukodystrophy. The crystals in the macrophages that stained brown with toluidine blue are sulfatides, which
accumulate in this disorder. The color shift seen in the toluidine blue stain is termed metachromasia (hence the
term metachromatic leukodystrophy). The cause of metachromatic leukodystrophy is deficiency of arylsulfatase
A.
Galactocerebroside &beta;-galactosidase (choice B) deficiency produces Krabbe's disease. In this disorder,
multinucleated cells derived from macrophages (globoid cells) are seen around blood vessels. Electron

310
microscopy of the macrophages reveals the presence of linear inclusions.
Glucocerebrosidase (choice C) deficiency results in Gaucher's disease, characterized by accumulation of
glucocerebrosides. Gaucher's cells are cells distended with material that resembles crumpled tissue paper.
Hexosaminidase A (choice D) deficiency is associated with Tay Sachs disease, characterized by accumulation of
GM2 ganglioside in the central and autonomic nervous systems.
Sphingomyelinase deficiency (choice E) produces Niemann-Pick disease, in which sphingomyelin accumulates,
especially in cells of the mononuclear phagocytic system.

An IV drug abuser develops an aggressive form of nephrotic syndrome that does not respond to steroids. A renal
biopsy is performed. Which of the following histological diagnoses will most likely be made from the biopsy tissue?
A. Focal segmental glomerulosclerosis
B. IgA nephropathy
C. Lipoid nephrosis
D. Membranoproliferative glomerulonephritis
E. Membranous glomerulonephritis
Explanation:
The correct answer is A. You should be aware of a specific association between focal segmental
glomerulosclerosis and both IV drug abuse and HIV nephropathy. This disorder usually presents as an
aggressive form of nephrotic syndrome; the prognosis is generally poor, with rare cases responding to steroid
therapy. Histologically, there is focal (affecting only some glomeruli) and segmental (affecting only a portion of
each glomerular capillary tuft) sclerosis and hyalinization with focal deposits of IgM and C3.
In IgA nephropathy (choice B), there are mesangial deposits of IgA and mesangial proliferation.
Lipoid nephrosis (choice C), or minimal change disease, is the most frequent cause of nephrotic syndrome in
children. Light microscopic changes are generally minimal or absent.
Membranoproliferative glomerulonephritis (choice D) is characterized by proliferation of glomerular cells and
thickening of capillary loops. It is not particularly associated with drug abuse.
Membranous glomerulonephritis (choice E) is an important cause of nephrotic syndrome in adults, characterized
by diffuse thickening of the walls of glomerular capillaries.

A 54-year-old man comes to the emergency room four hours after the onset of severe chest pain radiating to the
left arm. Examination of the levels of which of the following serum markers would best aid in the evaluation of this
individual's chest pain?
A. Aspartate aminotransferase (AST)

311
B. Creatine kinase-MB isozyme (CK-MB)
C. Lactate dehydrogenase-1 isozyme (LDH1)
D. Total creatine kinase
E. Troponin
Explanation:
The correct answer is E. Cardiac-specific forms of troponin-T and troponin-I are not normally detectable in the
blood, but may increase 20-fold following a myocardial infarction. Slight amino acid differences between cardiac
and skeletal muscle forms of troponin allow specific association of the troponin with heart muscle damage,
rather than skeletal muscle damage. Troponin is the best serum marker for myocardial infarction for the first
eight hours, and because levels of the cardiac troponins remain elevated for 7-10 days, it may be useful for
evaluation of small, CK-negative infarctions for several days after the event.
AST (choice A) is a nonspecific marker for cardiac, liver, and skeletal muscle. Due to its lack of specificity, it is
much less useful as a marker of myocardial infarction than CK-MB or troponin .
Both total creatine kinase (choice D) and its more cardiac specific form CK-MB (choice B) are most useful from
8 to 24 hours after infarction, typically with peaks at 12-18 hours.
LDH1(choice C), the cardiac specific form of lactic dehydrogenase, is the test of choice 2 to 7 days after a
suspected myocardial infarction.

A 37-year-old man presents with an unexplained 30 lb. weight loss and poorly-defined ill health over the past six
months. The physician suspects cancer, but does not know which type of primary tumor is involved. A panel of
tumor markers is ordered, demonstrating markedly elevated alpha-fetoprotein. Which of the following tumors
would most likely produce an elevation of this substance?
A. Colorectal and pancreatic cancers
B. Liver and yolk sac tumors
C. Melanoma, neural tumors, and astrocytomas
D. Neuroblastoma, small cell carcinomas, gastric carcinomas, and pancreatic carcinomas
E. Prostatic carcinoma
Explanation:
The correct answer is B. Alpha-fetoprotein (AFP) is a normal fetal protein that can be produced in the adult by
hepatocellular carcinomas and nonseminomatous germ cell tumors of the testes. The germ cell tumors most
likely to be associated with elevated AFP are yolk sac tumors and mixed germ cell tumors with a yolk sac
component.
CEA is associated with both colorectal and pancreatic cancers (choice A).
S100 is the tumor marker associated with melanoma, neural tumors, and astrocytomas (choice C).

312

Bombesin is a tumor marker associated with neuroblastoma, small cell carcinomas, gastric carcinomas, and
pancreatic carcinomas (choice D).
Prostate specific antigen is often markedly elevated in cases of prostatic carcinoma (choice E).

A 75-year-old female with a long history of cigarette smoking is found to have a small tumor at the periphery of
her right upper lobe. Initially, the tumor was believed to be a Stage I carcinoma (T1 NO MO), but after surgery it
is found to be Stage II (T1 N1 MO). What is found at surgery that changed the staging?
A. Involvement of the chest wall
B. Positive bronchial lymph nodes
C. Small cell histology
D. Tumor at the carina
E. Tumor size greater than 3 cm
Explanation:
The correct answer is B. Although it is nearly impossible to memorize the staging rules for all the different kinds
of tumors, there are several basic principles common to all. The TNM stage of a tumor is determined by the
tumor size and extent (T for tumor), lymph node involvement (N for nodes), and metastasis (M for metastasis).
N0 always means no lymph nodes are involved. An NI lesion has positive nodes, and only choice B, positive
bronchial lymph nodes, changes the patient's nodal status.
Involvement of the chest wall is a feature of tumor size and extent-thus, it is a component of the T in
TNM-staging. T2 lesions involve the chest wall (choice A).
Histological features of the tumor, such as small cell morphology (choice C), are not considered in tumor stage,
but rather in tumor grade. Small cell tumors are considered high-grade carcinomas.
Tumor extending to the carina reflects the tumor size and extent-thus, it is a component of the T in
TNM-staging: T3 lesions involve the carina (choice D).
Tumor size is a consideration in the T of TNM-staging. Tumors greater than 3 cm (choice E) are at least T2
lesions.

A 30-year-old pregnant woman of Jewish descent presents to a physician with painful oral ulcers. Physical
examination demonstrates widespread erosions of her mucous membranes. Close examination reveals a friable
mucosa, but no well-defined aphthous ulcers. Biopsy of perilesional mucosa demonstrates acantholysis; direct
immunofluorescence demonstrates an intraepidermal band of IgG and C3. Which of the following is the most
likely diagnosis?
A. Bullous pemphigoid
B. Dermatitis herpetiformis

313

C. Herpes simplex I
D. Herpes simplex II
E. Pemphigus vulgaris
Explanation:
The correct answer is E. This is pemphigus vulgaris, in which autoantibody directed against transmembrane
cadherin adhesion molecules induces acantholysis (breakdown of epithelial cell-cell connections) with resulting
intraepidermal blister formation. It may develop spontaneously or following triggers such as drugs (thiols,
penicillamine), physical injury (burns), cancer, pregnancy, other skin diseases, and emotional stress.
Pemphigus vulgaris is a relatively rare blistering disease; it is seen more commonly in patients with Jewish or
Mediterranean heritage. In addition to the usually prominent oral ulcers, uncomfortable skin erosions can also
occur when the blisters rupture rapidly and are not observed. The epidermis at the edge of these erosions is
often easily disrupted by sliding pressure (Nikolsky sign).
Bullous pemphigoid (choice A) is characterized by deeper blisters, occurring at the dermal-epidermal junction.
Dermatitis herpetiformis (choice B) is characterized by severe, intense pruritus and groups of papules and
vesicles.
Herpes simplex I (choice C) or II (choice D) can show multinucleated giant cells on scrapings of the ulcer base.

A 35-year-old woman with a ten-year history of vague arthralgias and fevers has antibody studies performed that
demonstrate autoantibody directed against double-stranded DNA. Later, she develops a photosensitive rash over
her nose and cheeks. Biopsy of the rash would most likely demonstrate which of the following?
A. Fibroblastic-like cells in a storiform pattern
B. Granular complement and IgG at the dermal-epidermal junction
C. Multiple horn cysts
D. Pautrier microabscesses
E. Sawtooth dermal/epidermal junction
Explanation:
The correct answer is B. The disease is systemic lupus erythematosus; the most useful specific clue to the
diagnosis is the presence of autoantibodies to double-stranded DNA (one type of antinuclear antibody or ANA).
The characteristic "butterfly" facial rash of lupus is due to deposition of antibodies and complement at the
dermal/epidermal junction.
Cells similar to fibroblasts growing in a storiform ("pinwheel") pattern (choice A) are characteristic of
dermatofibrosarcoma protuberans, a slow-growing type of fibrosarcoma.
Horn cysts (choice C) are lamellated collections of keratin that occur in seborrheic keratoses, a type of benign
epithelial tumor of the skin.

314

Pautrier microabscesses (choice D) are a feature of mycosis fungoides, a cutaneous T-cell lymphoma.
A sawtooth dermal/epidermal junction (choice E) is a feature of lichen planus, an inflammatory skin condition.

A demented elderly man is transferred from a nursing home to the emergency department because of
generalized edema. The patient is afebrile, and his blood pressure is within normal limits. Blood samples are sent
to the laboratory for analysis, yielding a serum creatinine of 2.0 mg/dL and serum urea nitrogen of 65 mg/dL.
Which of the following is the most likely diagnosis?
A. Acute tubular necrosis
B. Congestive heart failure
C. Low protein intake
D. Prostatic hyperplasia
E. Severe liver disease
Explanation:
The correct answer is D. This elderly patient with oliguria and edema has high serum creatinine (normal 0.6-1.2
mg/dL) and high serum urea nitrogen (normal 7-18 mg/dL). The serum urea nitrogen/serum creatinine ratio is
32, greater than the normal 12 to 20 for individuals on a normal diet. High ratios with elevated creatinine levels
can occur in postrenal obstruction (probably benign prostatic hyperplasia in this elderly man) and in prerenal
azotemia that is superimposed on renal disease (but not pure prerenal azotemia, as would occur in simple
congestive heart failure). The reason for the high ratio is that urine flow obstruction causes back pressure on
the renal tubules, favoring back diffusion of urea into the blood from the tubules.
Acute tubular necrosis (choice A) is typically characterized by a low urea nitrogen/creatinine ratio.
Congestive heart failure (choice B) generally causes prerenal azotemia. This is associated with a high urea
nitrogen/creatinine ratio, mainly because of an increased plasma urea nitrogen, with the creatinine level usually
near normal.
Low protein intake (choice C) and severe liver disease (choice E) can cause a low urea nitrogen/creatinine
ratio.

A 30-year-old male presents to the dermatologist with silvery, scaling plaques on his elbows and knees. His
mother has been afflicted with the same condition in the past. The most likely diagnosis is
A. acne rosacea
B. acne vulgaris
C. pemphigus vulgaris
D. pityriasis rosea

315

E. psoriasis vulgaris
Explanation:
The correct answer is E. Psoriasis vulgaris usually appears on the nails, knees, elbows, and scalp. It does not
generally affect the mucous membranes. Lesions are well-demarcated coral-colored plaques with a white or
silvery scale (classic clue). Histologically, epidermal hyperplasia causing thickening and lengthening of the rete
ridges is apparent, as is thinning of the epidermis over the dermal papillae. There is a recognized genetic
component to this condition. Peak incidence is at age 30.
Here's a point-saving strategy: It is easy to get confused on this question because of the similar looking answer
choices, especially under time pressure. Be extra careful in such cases to mark the proper choice on your
answer grid.
Acne rosacea (choice A) affects the central face. Erythema, telangiectasias, acneform lesions (papules, cysts,
pustules), and rhinophyma (telangiectasias and hyperplasia of nasal soft tissue) are found in various
combinations. It is common from ages 30-50. Women are affected three times more frequently than men, but
the syndrome is more severe in the latter.
Acne vulgaris (choice B) causes comedones, papules, and cysts. It may be related to hormones, drugs, diet,
irritants, and genetic factors. Allergy to Propionibacterium acnes has been found to contribute to this condition.
Pemphigus vulgaris (choice C) starts with small vesicles, usually on the oral or nasal mucosa, then spreads to
other parts of the body. Bullae are delicate and flaccid. The condition is due to autoantibodies to intercellular
junctions between keratinocytes. Nikolsky's sign (production of blistering by light stroking or rubbing of the skin)
is positive. Pemphigus is most common from ages 40-60.
Pityriasis rosea (choice D) presents first with a red, scaling, "herald patch" approximately 4 cm in diameter. It is
followed within days by eruption in a "short-sleeve turtleneck" distribution. The classic clue to the diagnosis is
the appearance of crops of small, pink, oval patches in a "fir tree configuration" on the flexural lines.

A 38-year-old man presents to the emergency room with multiple arm and leg fractures following a minor fall.
Physical examination reveals a slight weakness of facial muscles on the left. A complete blood count shows a mild
anemia. X-ray studies reveal a generalized bony widening with partial obliteration of marrow spaces. The patient's
disorder is most likely due to abnormal function of which of the following cell types?
A. Granulocytic stem cells
B. Megakaryocytes
C. Plasma cells
D. Osteoblasts
E. Osteoclasts
Explanation:
The correct answer is E. The disease is osteopetrosis (Albers-Schonberg disease), which is a group of
hereditary diseases in which impaired osteoclast function leads to reduced bone resorption. The abnormal

316
osteoclasts frequently are enlarged, with bizarre shapes. The bones become thick and brittle; other features
include anemia secondary to marrow loss and cranial nerve deficits secondary to narrowing of bony ostea. An
autosomal recessive, severe form of the disease produces death in childhood. A relatively benign, autosomal
dominant form presents in adulthood.
Abnormal proliferation of granulocytic stem cells (choice A) can produce myelocytic leukemias.
Megakaryocyte abnormalities (choice B) can produce platelet disorders, but not deficient bone resorption.
In multiple myeloma, neoplastic plasma cells (choice C) can cause lytic bone lesions characterized by excessive
resorption of bone.
Abnormally low osteoclast, rather than osteoblast (choice D), function is apparently the problem in
osteopetrosis.

A four-year-old child develops steroid-sensitive nephrotic syndrome. Renal biopsy studies demonstrate normal
appearing glomeruli by light microscopy and fusion of foot processes by electron microscopy. Which of the
following proteins would be present in the urine in the highest concentration?
A. Albumin
B. Ceruloplasmin
C. IgA
D. Kappa light chain
E. Lambda light chain
Explanation:
The correct answer is A. Severe, persistent, proteinuria is required to produce nephrotic syndrome. The renal
histology described is that of minimal change disease, which produces a proteinuria that is relatively selective
for albumin.
Ceruloplasmin (choice B) is a copper-binding normal serum protein that could be spilled into the urine in many
types of glomerulonephritis that cause nonselective proteinuria.
IgA (choice C) can also be spilled into urine in non-selective proteinurias.
Both kappa and lambda light chains (choices D and E) are freely filtered at the glomerulus, but are usually
present in only trivial amounts in patients who do not have multiple myeloma or related diseases.

A 24-year-old man has progressive, painless enlargement of neck lymph nodes. Routine chest film followed by
CT scan demonstrates marked enlargement of mediastinal nodes. No nodules are seen in the liver or lungs.
When evaluating the biopsy of one of the involved nodes, the pathologist should specifically look for which of the
following?
A. Abnormal plasma cells

317

B. Giant platelets
C. Immature neutrophil precursors
D. Melanin pigment
E. Reed-Sternberg cells
Explanation:
The correct answer is E. This is a classic presentation of Hodgkin's disease, which is a form of lymphoma
characterized by neoplastic proliferation of Reed-Sternberg cells admixed with variable numbers of reactive
lymphocytes, neutrophils, and eosinophils. The cell of origin of the Reed-Sternberg cells is still disputed. These
cells have a distinctive appearance with a large double nucleus that contains paired, large, nucleoli which are
often red, producing an "owl's eye" effect.
Abnormal plasma cells (choice A) would be a feature of multiple myeloma or some B-cell leukemias and
lymphomas, which are not as likely in this patient.
Giant platelets (choice B) are a feature seen in several myeloproliferative disorders (notably essential
thrombocytopenia), which do not cause lymphadenopathy.
Immature neutrophil precursors (choice C) would most likely be a feature of a myeloid leukemia, which would not
cause a lymphadenopathy.
Melanin pigment (choice D) would be a feature of malignant melanoma, which would probably have caused lung
or liver metastases if it were at such an advanced stage as to have caused massive lymphadenopathy.

A surgeon operates on a patient with a pheochromocytoma involving the adrenal gland. He requests examination
of intraoperative frozen sections by the pathology laboratory. Which of the following criteria can be used to determine if
the lesion is benign or malignant?
A. Blood vessel invasion
B. Cannot be determined by microscopic examination
C. Hemorrhage and necrosis
D. Nuclear pleomorphism
E. Number of mitoses
Explanation:
The correct answer is B. Pheochromocytomas, and their related counterparts in extra-adrenal sites called
paragangliomas, are notorious because the only reliable indicator of metastatic potential is the presence of
distant metastases. Very malignant-appearing tumors may not metastasize and benign-appearing tumors may
produce metastases. In practice, there is no way for the pathologist to make the decision on frozen (or
permanent) section analysis unless metastasis has already occurred and can be documented. These tumors
should all be considered "potentially malignant."

318

At autopsy, the spleen of a patient is noted to have a thickened capsule and many small, scarred areas.
Microscopic examination of the scarred areas reveals fibrosis with hemosiderin and calcium
deposition. This type of spleen is usually seen in conjunction with which of the following
disorders?
A. Hepatic cirrhosis
B. Hodgkin's disease
C. Rheumatoid arthritis
D. Sickle cell anemia
E. Trauma
Explanation:
The correct answer is A. The spleen shows the changes of chronic congestive splenomegaly, typically
associated with hepatic cirrhosis. The described small scars are called Gandy-Gamma nodules, and are
thought to be the result of organization of old hemorrhages.
Hodgkin's disease (choice B) produces large splenic nodules in which Reed-Sternberg cells can be found
surrounded by mature lymphocytes, eosinophils, and neutrophils.
Rheumatoid arthritis (choice C) and many other chronic inflammatory disorders induce reactive
hyperplasia of the spleen with formation of many large germinal centers in the splenic follicles.
Sickle cell anemia (choice D) produces many small (often triangularly shaped) infarctions in
the spleen.
Trauma (choice E) can cause splenic rupture.
A 30-year-old man with an undiagnosed cardiac anomaly has been unable to do strenuous exercise
throughout his life. Auscultation demonstrates an immediate diastolic murmur, which has been be
present since childhood. The patient has never been cyanotic. Which of the following disorders
would most likely cause the patient's findings?
A. Bicuspid aortic valve
B. Common pattern transposition of the great arteries
C. Patent ductus arteriosus

319
D. Tricuspid atresia
E. Truncus arteriosus
Explanation:
The correct answer is A. Isolated bicuspid aortic value is a very common congenital anomaly,
which may be completely asymptomatic or may cause modest noncyanotic obstruction related to
stenosis. It is associated with an immediate diastolic murmur, related to some degree of
aortic insufficiency that is often recognized in early childhood. Surgical correction of even
mild to moderate stenosis or insufficiency is now recommended to prevent long-term damage to
the heart muscle.
The common pattern of transposition (choice B) of the great vessels produces early cyanosis.
Uncorrected patent ductus arteriosus (choice C) produces a continuous ("machinery") murmur and late
cyanosis.
Tricuspid atresia (choice D), which is usually accompanied by an atrial septal defect,
produces early cyanosis.
Truncus arteriosus (choice E) produces early cyanosis.
A 60-year-old male presents to the dermatologist with a complaint of tender blisters on his arm and
flank. Physical examination reveals blisters and flaccid bullae; a few have ruptured, leaving
red, sore, denuded areas. Which of the following findings would suggest the diagnosis of
pemphigus vulgaris as opposed to bullous pemphigoid?
A. Eosinophils within bullae
B. IgA deposits on basement membrane
C. IgG autoantibody activity
D. Negative Nikolsky sign
E. Oral mucosal lesions
Explanation:
The correct answer is E. Pemphigus vulgaris is associated with chronic, severe bullae formation
on the skin and oral mucosa. Mucosal lesions are extremely rare in bullous pemphigoid. This can
be used clinically to guide therapy, although a skin biopsy should be taken to confirm the
diagnosis. Both diseases are characterized by formation of tender bullae that can rupture,
leaving red, raw areas. Pemphigus vulgaris patients eventually become febrile and lose weight,
and if untreated, most will die within one year. Bullous pemphigoid lesions tend to heal and
the patients do very well. This prognostic difference is an important distinction in
dermatologic medicine.
Eosinophils within blisters (choice A) provide an important clue supporting bullous pemphigoid
as the diagnosis which must be ascertained with histologic examination. The vesicles in
pemphigus vulgaris mostly contain rounded acantholytic keratinocytes "floating" within.
IgA deposits seen with immunofluorescence on the basement membrane (choice B) is characteristic
of a third bullae-forming disease known as dermatitis herpetiformis. Vesicles form between the
dermal-epidermal junction. The disease is associated with gluten sensitivity and celiac

320
disease.
IgG autoantibody activity (choice C) is common to both pemphigus vulgaris and bullous pemphigoid.
Nikolsky's sign consists of separation of the epidermis upon manual stroking of the skin.
Bullous pemphigoid is characterized by a negative Nikolsky's sign (choice D), but Nikolsky's
sign is positive in pemphigus vulgaris due to the IgG-mediated destruction of intercellular
bridges between keratinocytes.
A five-year-old girl is brought to a pediatrician because she is developing breasts. Physical
examination shows large hyperpigmented macules with irregular margins on one side of her back.
Which of the following bony abnormalities would most be likely associated with these symptoms?
A. A combination of osteitis fibrosa cystica and osteomalacia
B. A generalized thinning of bony spicules
C. Excessive bony deposition with obliteration of marrow
D. Formation of new bone beneath the periosteum of phalanges
E. Multiple localized whorls of connective tissue
Explanation:
The correct answer is E. McCune-Albright syndrome is the combination of precocious sexual development,
irregularly shaped ("coast of Maine") pigmented skin macules, and polyostotic fibrous
dysplasia. The latter condition is characterized by local bony defects containing unmineralized
whorls of connective tissue.
Renal osteodystrophy refers to a constellation of bony changes occurring in the context of
chronic renal failure. Features may include those of osteitis fibrosa cystica (marrow fibrosis,
bony resorption and cyst formation within the marrow cavity) and osteomalacia (choice A).
Osteomalacia results from vitamin D deficiency in adults, and is characterized by a generalized
thinning of bony spicules (choice B).
Osteopetrosis is characterized by excessive bony deposition that can obliterate marrow (choice
C), causing anemia.
Formation of new bone beneath the periosteum of the phalanges (choice D) is characteristic of
hypertrophic osteoarthropathy, which may be associated with thoracic cancer, chronic lung
disease, and chronic liver disease.
During a pelvic examination, a gynecologist discovers an adnexal mass in a 32-year-old woman. Resection
of the mass demonstrates a malignant ovarian tumor. The tumor suppressor gene strongly
associated with this tumor is also associated with cancer of which other organ or tissue?
A. Breast
B. Colon
C. Pancreas
D. Peripheral nerve

321
E. Stomach
Explanation:
The correct answer is A. The tumor suppressor gene most strongly associated with ovarian carcinoma is
BRCA-1, which is also associated with breast cancer. This tumor suppressor gene is located on 17q.
Associate APC (5q), DCC (18q), and p53 (17p) with colon cancer (choice B).
Associate DPC (18q) with pancreatic cancer (choice C).
Associate NF-1 (17q) with neurofibromatosis type I, which is associated with peripheral nerve
tumors (choice D).
Associate DCC (18q) with stomach cancer (choice E).
A 30-year-old woman presents to a physician with rather non-specific complaints of fatigue, malaise,
low-grade fever, and arthralgias. Screening serum chemistries demonstrate an elevated BUN. The
urine shows proteinuria. Testing for which of the following will be most helpful for making the
diagnosis?
A. Anticentromeric antibody
B. Antimitochondrial antibody
C. Antinuclear antibody
D. Anti-TSH receptor antibody
E. Rheumatoid factor
Explanation:
The correct answer is C. Systemic lupus erythematosus is a favorite on board examinations, and
should be considered in any 15- to 45-year-old woman with chronic, poorly defined symptoms.
Clues that help are malar rash, anti-double-stranded DNA antibodies, and renal involvement.
Antinuclear antibody is a good screening test (it is also positive in some other autoimmune
diseases) and should be used first in working this patient up. Renal involvement is unusual in
most other autoimmune diseases.
Anticentromeric antibody (choice A) is a marker for the CREST form of scleroderma.
Antimitochondrial antibody (choice B) is a marker for primary biliary cirrhosis.
Anti-TSH receptor antibody (choice D) is a marker for Graves' disease.
Rheumatoid factor (choice E) is a marker for rheumatoid arthritis.
An adult with newly diagnosed tuberculosis is most likely to have which of the following types of
lesions?
A. A single lesion in a lung apex
B. A single lesion in the gastrointestinal tract
C. A single lesion subjacent to the pleura

322

D. Both a lesion subjacent to the pleura in the lower part of an upper lobe and active disease in the
mediastinal lymph nodes
E. Multiple tiny masses throughout the body
Explanation:
The correct answer is A. This is something of a trick question, but it stresses an important
fact many medical students do not realize. Primary tuberculosis infection characteristically
involves the lung subjacent to the pleura in either the lower part of the upper lobe or the
upper part of a lower lobe of one lung. The mediastinal nodes are also usually involved (choice
D), rather than having a single lesion subjacent to the pleura (choice C) without lymph node
involvement. However, the "trick" part of this question rests on the fact that roughly 80% of
newly diagnosed pulmonary tuberculosis cases in adults are actually due to reactivation of an
often clinically unsuspected infection acquired years-to-decades previously. The reinfection
site usually is in the apex of the lung.
You should also be aware that the lungs are not the only site where tuberculosis can occur (it
can occur throughout the body), and isolated gastrointestinal involvement (choice B) is (uncommonly)
also seen.
Miliary tuberculosis (rare) is a widely disseminated and dangerous form of tuberculosis
characterized by small lesions throughout the body (choice E).
A 40-year-old woman has the following family and personal history: her mother died of breast cancer at
age 64, she smokes one pack/day, she drinks five or more cups of coffee/day, she has no
children, and she takes birth control pills. Which of the following is the most significant
risk factor for breast cancer in this patient?
A. Birth control pills
B. Caffeine consumption
C. Cigarette smoking
D. Family history
E. Nulliparity
Explanation:
The correct answer is D. Positive family history is definitively the most significant risk
factor of developing breast cancer among the ones mentioned in the list. About 5% to 10% of
cases are attributable to inheritance of autosomal dominant genes. Most hereditary cases of
breast cancer are due to two genes, BRCA1 and BRCA2, which are tumor suppressor genes probably
involved in DNA repair. There are additional rare "susceptibility" genes, namely those causing
Li-Fraumeni syndrome (mutations of p53), Cowden syndrome (mutations of a gene on chromosome
10), and ataxia-telangiectasia (mutations of ATM gene). Note, however, that 80% to 90% of women
with breast cancer do not have a positive family history.
The role of exogenous estrogens, including those administered with birth control pills (choice A) or
postmenopausal replacement therapy, is still debated. If there is any associated risk for
breast cancer, this is minimal.
There is no convincing evidence linking caffeine consumption or cigarette smoking (choices B

323
and C) to breast cancer.
Breast cancer is more frequent in nulliparous women than in multiparous women. Nulliparity
(choice E) is therefore a risk factor, but its role is considered less important than family
history. Similar to a long duration of reproductive life and late age at first intercourse,
nulliparity seems to increase breast cancer risk by increasing exposure to endogenous estrogen
during the menstrual cycle.
Biopsy of a small, rounded rectal polyp demonstrates glands and sawtooth crypts composed of a
proliferation of goblet and columnar epithelial cells. No atypia is seen. This polyp is best
classified as which of the following?
A. Hyperplastic polyp
B. Peutz-Jeghers polyp
C. Tubular adenoma
D. Tubulovillous adenoma
E. Villous adenoma
Explanation:
The correct answer is A. This is a hyperplastic polyp; these polyps comprise 90% of all colonic
polyps and have no malignant potential.
Peutz-Jeghers polyps (choice B) also have no malignant potential, but tend to be larger and
have a complex branching pattern.
Tubular adenomas, tubulovillous adenomas, and villous adenomas (choices C, D, and E) are all
true neoplastic polyps containing dysplastic epithelium; the malignant potential of these
polyps increases with size and the percentage of the polyp which has a villous configuration.
A 52-year-old Caucasian male presents with sinus pain and drainage, bloody nasal discharge, and nasal
mucosal ulceration. On laboratory examination, the man is found to have proteinuria, hematuria,
and red blood cell casts. A biopsy of the upper airway tissue reveals granulomatous inflammation
with necrosis. Renal biopsy confirms the existence of glomerulonephritis. Blood tests show the
presence of antineutrophil cytoplasmic antibodies (c-ANCA). What is the most likely diagnosis?
A. Allergic angiitis
B. Goodpasture's syndrome
C. Non-infectious granulomatous disease
D. Tumors of the upper airway
E. Wegener's granulomatosis
Explanation:
The correct answer is E. Antineutrophil cytoplasmic antibodies yielding a cytoplasmic
immunofluorescence pattern (c-ANCA) are found in over 90% of patients with Wegener's granulomatosis. The
classic clinicopathologic complex of Wegener's granulomatosis usually provides ready differentiation
from other disorders in which c-ANCA is positive. c-ANCA is not considered pathognomonic, but has a very

324
strong association with Wegener's and is not found in any of the other diseases listed.
c-ANCA is generally negative in patients with allergic angiitis (choice A), non-infectious
granulomatous disease (choice C), and tumors of the upper airway (choice D).
c-ANCA is typically negative in Goodpasture's syndrome (choice B), but p-ANCA have been
detected in some patients with Goodpasture's syndrome.
A tall man with gynecomastia and testicular atrophy has a testicular biopsy that shows sparse,
completely hyalinized seminiferous tubules with a complete absence of germ cells and only rare
Sertoli cells. Leydig cells are present in large clumps between the hyalinized tubules. Which
of the following genetic disorders should be suspected?
A. Testicular feminization syndrome
B. Trisomy 18
C. Trisomy 21
D. 45, XO
E. 47, XXY
Explanation:
The correct answer is E. The testicular changes described are those observed in Klinefelter's
syndrome, most often due to 47, XXY genetics.
Testicular feminization syndrome (choice A) is due to a genetically determined unresponsiveness to
testosterone that produces a phenotypic female in an individual with 46, XY chromosomes.
Trisomy 18 (choice B) is Edwards' syndrome, characterized by facial features that are small and
delicate.
Trisomy 21 (choice C) is Down syndrome, the most common trisomy. Characteristics include
oblique palpebral fissures, epicanthal folds, endocardial cushion defects, simian creases, and
high-arched palate, among other anomalies.
Turner's syndrome, 45, XO (choice D), produces a sterile but phenotypic female individual of
short stature with webbing of the neck.
A 54-year-old woman presents with complaints of abdominal fullness and early satiety. She denies a
change in bowel habits and says that constipation is her normal state. A radiographic bowel
series shows an "apple core" lesion in her sigmoid colon. Which of the following markers is
expected to be elevated in this patient?
A. Alpha-fetoprotein (AFP)
B. Carcinoembryonic antigen (CEA)
C. Human chorionic gonadotropin (hCG)
D. Lactate dehydrogenase (LDH)
E. Prostatic acid phosphatase (PAP)

325
Explanation:
The correct answer is B. CEA is a glycoprotein (200,000 daltons) that is found in the
gastrointestinal mucosal cells and pancreatobiliary system secretions. It becomes elevated when breaks
in the mucosal basement membrane occur due to tumor growth. Some other conditions can also cause
elevations of CEA (e.g., cirrhosis, pancreatitis). The patient described above has colonic carcinoma,
and the "apple core" lesion is the classic radiographic finding of this tumor. Although CEA is not used
for screening because of the large number of false positives, it can be used to guide treatment in known
cases. It returns to normal levels 30-45 days after tumor resection. If it begins to rise again, tumor
recurrence is likely.
AFP (choice A), or alpha-fetoprotein, is an alpha globulin (70,000 daltons) made by the liver
and yolk sac of the human fetus. It is elevated with hepatocellular carcinoma and germ cell
neoplasms. It is not particularly specific.
hCG (choice C), or human chorionic gonadotropin, is a glycoprotein secreted by trophoblastic
epithelium of the placenta. It is useful (sensitive and specific) for germ cell tumors of the testis and
ovary.
LDH (choice D), or lactate dehydrogenase, is a very non-specific test that can be elevated in
many conditions, including malignancy.
PAP (choice E), or prostatic acid phosphatase, is a sensitive marker used for detecting prostatic
adenocarcinoma.
A 54-year-old man is admitted to the hospital with cough and dyspnea, which over a period of years has
led to marked respiratory embarrassment and cyanosis. Chest x-ray film of the lungs shows
bilateral lower lobe ground-glass infiltrates. Wedge biopsy of the lung demonstrates airspaces
filled with macrophages containing lipid, periodic acid-Schiff (PAS)-positive granules, and
lamellar bodies. There is an accompanying interstitial pneumonitis, hyperplasia of the septal
lining epithelial cells, and desquamation of epithelial cells into alveoli. The lamellar bodies
within the macrophages are composed of which of the following?
A. Amyloid
B. Calcitonin
C. Fibrin
D. Hemosiderin
E. Surfactant
Explanation:
The correct answer is E. The disease is desquamative interstitial pneumonitis (DIP), which is
an idiopathic process related to idiopathic interstitial fibrosis. DIP may respond to steroid
therapy but may also progress to end-stage lung disease. The lamellar (layered) bodies within
macrophages contain surfactant derived from type II pneumocytes.
Amyloid (choice A) is deposited extracellularly and is not part of the DIP process.
Calcitonin (choice B) is found in medullary carcinoma of the thyroid.
Fibrin deposition (choice C) can be part of the DIP process, but occurs as eosinophilic
extracellular deposits, rather than as intracellular lamellar bodies.

326

Hemosiderin (choice D) can accumulate in pulmonary macrophages, usually in the setting of


congestive heart failure with microhemorrhages, and produces golden brown granules.
Over a one month period, a 35-year-old man develops symptoms of hyperthyroidism with tachycardia and
heat intolerance. Thyroid biopsy demonstrates a heavy mononuclear cell infiltrate with
multinucleated giant cells, follicular disruption, and loss of colloid. About one month later,
the patient develops symptoms of hypothyroidism. To which of the following is this patient's
condition thought to be most closely related?
A. Blocking TSH receptor autoantibodies
B. Carcinoma
C. Lymphoma
D. Stimulating TSH receptor autoantibodies
E. Viral infection
Explanation:
The correct answer is E. The patient has subacute thyroiditis, also known as granulomatous
giant cell or de Quervain's thyroiditis. This condition is thought to be due to viral infection
of the thyroid gland, and may follow an upper respiratory infection. Early in the process, the
follicular destruction causes inappropriate release of thyroid hormone with resulting
hyperthyroidism. Later, the destruction can proceed to the point that hypothyroidism occurs.
Antibodies that block the TSH receptor (choice A) are a feature of Hashimoto's disease.
Thyroid carcinoma (choice B) usually does not produce hyperthyroidism.
Lymphoma (choice C) of the thyroid is more likely to occur in the setting of Hashimoto's disease.
Stimulating TSH receptor antibodies (choice D) are a feature of Graves disease.
A 3-year-old boy with an enlarging, left-sided, abdominal mass undergoes diagnostic biopsy. The tumor
reveals a variety of cellular patterns: dense immature islands of epithelial cells, ribbons of
spindled fibroblast-like stromal cells, and poorly formed tubular structures. This triphasic
histology is most suggestive of which of the following childhood neoplasms?
A. Embryonal rhabdomyosarcoma
B. Ewing's sarcoma
C. Hodgkin's disease
D. Neuroblastoma
E. Wilms' tumor
Explanation:
The correct answer is E. Wilms' tumor, a neoplasm of the embryonic renal blastema, contains
both a neoplastic epithelial and stromal component. In addition to organizing as islands of
stromal or epithelial cells, these cells characteristically organize into recognizable

327
glomerular and/or tubular structures that may represent a minority or sizeable component of the
tumor.
Rhabdomyosarcoma (choice A) is a malignancy of skeletal muscle origin, and the cells are
recognized by their cytoplasmic content of eosinophilic muscle proteins (thin and thick
filaments). Rhabdomyosarcoma has an embryonal, alveolar, and pleomorphic variant, but none
demonstrates a triphasic histology.
Ewing's sarcoma (choice B) appears as a monotonous mass of primitive, small, round cells with a
thin rim of clear cytoplasm. Ewing's sarcoma is a tumor of bone; presentation as an abdominal
mass is distinctly unusual.
Hodgkin's disease (choice C) is a lymphoid malignancy that presents with lymphadenopathy,
typically in the cervical or mediastinal nodes. The cellular component contains varying amounts
of inflammatory cells and Reed-Sternberg cells.
Neuroblastoma (choice D) frequently presents as an abdominal mass in children; however, it does
not have an epithelial component. The cell population of neuroblastoma ranges from primitive,
small, blue, round cells to well-differentiated neuritic cells resembling ganglion cells.
A 60-year-old man presents to a physician because of difficulty in reading and coming down stairs, which
he attributes to an inability to "look down." Physical examination reveals that the patient
looks around by moving his head rather than his eyes and also shows a distinctive axial
rigidity of neck, trunk, and proximal limb muscles. He shows poverty of movement and dysarthric
speech. Mentally, the patient responds very slowly but has better memory and intellect than are
initially apparent. Which of the following pathologic findings of the brain would most likely
be present?
A. Depigmentation of the substantia nigra and locus ceruleus
B. Diffuse cortical atrophy with relative sparing of primary motor and sensory areas
C. Selective frontal and temporal lobe atrophy
D. Striking degeneration of the caudate nucleus
E. Widespread neuronal loss and gliosis in subcortical sites
Explanation:
The correct answer is E. The disease is progressive supranuclear palsy, a degenerative disorder
characterized by ophthalmoplegia, pseudobulbar palsy, axial dystonia, and bradykinesia. The
presentation described in the question is typical. The pathologic changes consist of widespread
neuronal loss and gliosis in subcortical sites with sparing of the cerebral and cerebellar
cortices.
Pigmented neurons in the substantia nigra (pars compacta) and locus ceruleus (choice A) selectively
degenerate in Parkinson's disease.
In Alzheimer's disease, there is diffuse cortical atrophy (choice B), especially over the
association cortex of frontal, temporal, and parietal lobes, with relative sparing of primary sensory
and motor areas.
Selective frontal and temporal lobe atrophy (choice C) is characteristic of Pick's disease.
The caudate nucleus and putamen undergo severe atrophy (choice D) in Huntington's disease. Cortical

328
atrophy occurs to a lesser extent.
A 2-year-old girl is evaluated because of deteriorating vision. On physical examination, a whitish hue
is noted in the right pupil, along with strabismus. Palpation of the eye elicits apparent pain
and tenderness. The patient is referred to an ophthalmologist, who schedules surgery, and
removes a multifocal tumor from the right eye. The pathology report notes the presence of
rosettes composed of cuboidal-to-columnar cells surrounding a central lumen in the tumor. In all
likelihood, this child's condition stems from a mutation in a gene on chromosome
A. 5
B. 7
C. 9
D. 13
E. 21
Explanation:
The correct answer is D. The description of this case is classic for retinoblastoma, the most
common malignant eye tumor of childhood. Retinoblastoma presents as in the question, and microscopically
is composed of neuroepithelial cells that form characteristic rosettes (Flexner-Wintersteiner rosettes).
Familial retinoblastoma is often bilateral or multifocal, as in this case. A preexisting mutation of a
tumor suppressor gene, Rb, located on chromosome 13 (13q14) is present, but causes no symptoms. A
second, somatic mutation in a retinal cell leads to loss of a nuclear protein that prevents the retinal
cell from exiting G1, allowing development of the tumor. In cases of sporadic retinoblastoma, mutations
must occur in both chromosomes 13 of a somatic cell, a rare event, hence sporadic retinoblastoma is
always unifocal and unilateral.
A 25-year-old man presents to the emergency room with severe abdominal pain. Guaiac test on stool
demonstrates occult blood. The patient is taken to exploratory laparotomy, at which a small
area of infarcted small bowel is found and surgically removed. Histologic studies on the
removed section of bowel demonstrate a recent thrombus occluding a small muscular artery. The
adjacent vessel wall shows fibrinoid necrosis with a mixed inflammatory infiltrate containing
neutrophils, eosinophils, and mononuclear cells. Which of the following is the most likely
cause of the small bowel infarction?
A. Atherosclerosis
B. Cystic medial necrosis
C. Mnckeberg's arteriosclerosis
D. Polyarteritis nodosa
E. Wegener's granulomatosus
Explanation:
The correct answer is D. The lesion described is that of polyarteritis nodosa. This condition
is often considered to be the prototype for the vasculitides, and characteristically affects
small or medium-sized muscular arteries, with a predilection for the gastrointestinal tract and
kidney (causing hematuria, albuminuria, or renal failure). Patients are typically young adult
males, although both sexes and all ages may have the condition. Characteristically, vascular

329
lesions of different stages may be present. Corticosteroid and cyclophosphamide therapy has
improved a formerly poor prognosis.
Atherosclerosis (choice A) can cause bowel infarction, but the histologic description would be
that of a plaque.
Cystic medial necrosis (choice B) predisposes for dissecting aortic aneurysm.
Mnckeberg's arteriosclerosis (choice C) is a benign condition characterized by ring-like
calcifications of vessel walls.
Wegener's granulomatosus (choice E) affects the lungs, upper airways, and kidneys.
Which of the following is most likely to cause a hypochromic microcytic anemia?
A. Folate deficiency
B. Hereditary spherocytosis
C. Iron deficiency anemia
D. Sickle cell anemia
E. Vitamin B12 deficiency
Explanation:
The correct answer is C. Conditions that produce microcytic anemia include iron deficiency,
thalassemia minor, anemia of chronic disease, and the anemia produced by erythrocyte fragmentation.
Folate deficiency (choice A) usually produces a macrocytic anemia.
Hereditary spherocytosis (choice B) usually produces an anemia with cells of normal volume.
Sickle cell anemia (choice D) usually produces an anemia with cells of normal volume.
Vitamin B12 deficiency (choice E) usually produces a macrocytic anemia.
A 28-year-old female presents to the doctor complaining of syncopal episodes that last a few minutes.
She is not taking any medications and has no previous medical history. EEG and EKG studies are performed
and are unremarkable. An echocardiogram shows a single ball-shaped mass dangling in the left atrium near
the mitral valve. The most likely diagnosis is
A. angiosarcoma
B. mesothelioma
C. myxoma
D. rhabdomyoma
E. rhabdomyosarcoma
Explanation:
The correct answer is C. The vignette illustrates a typical presentation for a tumor of the

330
heart. Primary cardiac tumors are rare and usually require an intensive work-up to pinpoint the
diagnosis. 75% of primary cardiac tumors are benign and among these, myxoma is the most common.
The tumors are usually single; the most common location is the left atrium. They may cause
syncopal episodes or even shock and death due to obstruction by a "ball valve" mechanism.
Angiosarcoma (choice A) is a malignant tumor of vascular origin that can occur as a primary
cardiac tumor. It is the most common malignant primary cardiac tumor, but it is still very
rare. Angiosarcoma usually affects the right side of the heart.
Mesothelioma (choice B) is a benign tumor of mesothelial origin that can rarely present as a
primary cardiac tumor. It is usually a small intramyocardial tumor that presents with
disturbances of the conduction system of the heart.
Rhabdomyoma (choice D) is a benign tumor of muscle origin. It can occur as a primary cardiac
tumor, typically in infants and children, in whom it may be associated with tuberous sclerosis.
It usually occurs in the ventricles.
Rhabdomyosarcoma (choice E) is a malignant neoplasm that can also occur as a rare primary
cardiac tumor. It is of muscle origin and usually affects the right heart.
The day after playing in the high grasses of a neighbor's fields, an 8-year-old boy develops a weeping,
vesicular, erythematous, and itchy rash on his arms, legs, and neck. Which of the following best
describes this inflammatory response?
A. Erythema nodosum
B. Pemphigus
C. Psoriasis
D. Spongiotic dermatitis
E. Urticaria
Explanation:
The correct answer is D. Spongiotic dermatitis is seen in cases of contact dermatitis, such as
poison ivy exposure. The accumulation of inflammatory cells in the superficial dermis causes
marked edema, which splays epidermal keratinocytes apart and giving a spongy appearance to
intercellular bridges. Grossly, the skin has a weepy appearance with frequent blistering.
Erythema nodosum (choice A) is a form of panniculitis, which is chronic inflammation in the
subcutaneous fat lobules. Erythema nodosum presents as painful erythematous nodules, often with
fever and malaise. It is associated with infections and drug reactions, and is not a contact
dermatitis.
Pemphigus (choice B) is a genetic blistering disorder due to the production of antibodies to
the intercellular cement substances in skin and mucous membranes.
Psoriasis (choice C) is a common chronic inflammatory disease causing plaques and scales, typically on
elbows, knees, and scalp. The pathogenesis of psoriasis is still unclear; it may be a
complement-mediated autoimmune process.
Urticaria (choice E) is an IgE-driven hypersensitivity process, usually occurring in adults
between 20 and 40 years old. Urticaria is characterized by wheals (edematous pruritic plaques),
and typically affects the trunk and distal extremities.

331

A 30-year-old woman presents complaining of nosebleeds. She also notes easy bruising and excessively
heavy bleeding during her periods. The patient denies taking any medications. Physical
examination is remarkable for scattered petechiae. The spleen is normal-sized. Laboratory
examination is remarkable for a platelet count of 45,000/microliter and a bleeding time of 17
minutes. The bone marrow shows an increased number of megakaryocytes. Antinuclear antibody is
negative. Autoantibodies directed against which of the following antigens would likely be found
in this patient's serum?
A. Acetylcholine receptor
B. Erythrocyte membrane protein
C. Glycoprotein IIb/IIIa
D. Intrinsic factor
E. Type IV collagen
Explanation:
The correct answer is C. The history of nosebleeds and menorrhagia, the petechiae,
thrombocytopenia and increased bleeding time all suggest a platelet disorder. The decreased platelet
count suggests a thrombocytopenic disorder rather than a platelet function disorder. The absence of
antinuclear antibody argues against systemic lupus erythematosus (a significant cause of
thrombocytopenia). The negative drug history rules out drug-associated thrombocytopenia. After other
causes have been ruled out, a tentative diagnosis of idiopathic thrombocytopenic purpura can be made.
This disorder, also called immune thrombocytopenic purpura, is an acquired thrombocytopenia caused by
formation of autoantibodies directed against the platelet membrane proteins glycoprotein IIb/IIIa,
followed by splenic destruction of opsonized platelets. The disease typically occurs in women from 20-40
years of age. Splenectomy and immunosuppressive therapy can generally control the thrombocytopenia.
Antibodies to the acetylcholine receptor (choice A) are seen in myasthenia gravis.
Antibodies to erythrocyte membrane proteins (choice B) are seen in autoimmune hemolytic anemia.
Antibodies to intrinsic factor (choice D) are seen in pernicious anemia.
Antibodies to Type IV collagen (choice E) are seen in Goodpasture's syndrome.
A 37-year-old woman has cyclical premenstrual pain. Her breasts have a "lumpy-bumpy" texture on
palpation. A biopsy is performed. The histopathologic features include small cysts lined by
epithelial cells with apocrine metaplasia, calcium deposits, areas of fibrosis, increased
number of acini (adenosis), and foci of florid hyperplasia of ductal epithelium. Which of these
changes increase the risk of breast cancer?
A. Adenosis
B. Apocrine metaplasia
C. Calcium deposits
D. Cysts
E. Epithelial hyperplasia

332
Explanation:
The correct answer is E. Fibrocystic changes usually come to clinical attention by causing pain
(often cyclical, in premenstrual phase), palpable lumps, or mammographic densities and calcifications. A
"lumpy-bumpy" texture is caused by cysts and fibrosis. Fibrocystic changes, per se, do not increase the
risk of developing breast cancer, unless there are associated proliferative changes. Epithelial
hyperplasia is defined as an increase in the number of epithelial cell layers in the ductal epithelium.
Florid epithelial hyperplasia leads to an increased risk of developing carcinoma, especially if there is
associated cellular atypia (atypical ductal hyperplasia).
Adenosis (choice A) refers to an increase in the number of acini and can be observed in
fibrocystic changes as well as in other breast conditions, such as sclerosing adenosis.
Physiologic adenosis is part of the changes that occur during pregnancy. Adenosis is not
associated with increased risk of breast cancer, except as a component of sclerosing adenosis.
Apocrine metaplasia (choice B) describes a benign change of breast epithelial cells that come
to resemble the apocrine epithelium of sweat glands. Apocrine cells have abundant granular
eosinophilic cytoplasm. Apocrine metaplasia is seen frequently in fibrocystic changes, as well
as in normal breast.
Calcium deposition (choice C) is a nonspecific finding that may occur in a number of both
benign and malignant breast changes, including fibrocystic changes, ductal carcinoma in situ,
and invasive carcinoma. Calcification is not clinically significant except for its diagnostic
value. Mammographic detection of calcium may serve to guide biopsy procedures.
Cysts (choice D) are frequent in fibrocystic changes and result from dilatation of ducts. The
nodularity of breasts with fibrocystic changes is due in part to the presence of cysts of
various sizes. A classic gross description is that of blue-dome cysts, which appear brown to
blue because of their turbid fluid content. The epithelial lining may undergo apocrine
metaplasia or produce papillary projections. There is no direct link between cyst formation and
breast cancer.
A patient has had years of intermittent diarrhea and abdominal pain, but has never consulted a
physician. Eventually, he begins to pass fecal material in his urine and he seeks medical
attention. Which of the following diseases is most likely to cause this complication?
A. Celiac disease
B. Crohn's disease
C. Diverticulitis
D. Ulcerative colitis
E. Whipple's disease
Explanation:
The correct answer is B. Passing fecal material in urine strongly suggests the possibility of
a fistula between the bowel and bladder. Of the diseases listed, only Crohn's disease (a type
of inflammatory bowel disease) commonly produces fistulas. Fistulas are produced in Crohn's
because the disease affects the entire thickness of the bowel wall, rather than being
restricted to the mucosa (e.g., ulcerative colitis).
Celiac disease (choice A) is a mucosal disorder of the small intestine caused by intolerance
to certain components of gluten from wheat and other grains.

333

Diverticulitis (choice C) can cause bowel perforation with peritonitis but does not usually
cause fistula formation.
Ulcerative colitis (choice D) is much less commonly associated with fistula formation than is
Crohn's disease.
Whipple's disease (choice E) is a small intestinal disorder caused by infection with
Tropheryma whippelii.
A 29-year-old man presents with hemoptysis and hematuria. Renal biopsy shows inflammation of the
glomeruli. X-ray shows focal pulmonary infiltrates. Which of the following is likely to be found
with an immunofluorescence study of the glomeruli and alveoli?
A. Electron-dense humps on epithelial side of basement membrane
B. Linear IgG deposits on alveolar and glomerular basement membranes
C. Mesangial deposition of IgA
D. Spike and dome IgG deposits on the glomerular basement membrane
Explanation:
The correct answer is B. This is a classic case of Goodpasture's syndrome. It is autoimmune in
origin and is characterized by hemorrhagic pneumonitis leading to hemoptysis and
glomerulonephritis progressing to renal failure. Most cases involve the presence of antibasement membrane antibodies. It occurs mostly in young men. Under immunofluorescence, there
are linear deposits of IgG on alveolar and glomerular basement membranes.
Electron-dense humps (choice A) are found under electron microscopy, not with
immunofluorescence. These humps are found in poststreptococcal glomerulonephritis;
immunofluorescence in this case would show C3, IgM, and IgG granular deposits.
Mesangial deposition of IgA (choice C) occurs in Berger's disease or IgA nephropathy.
Spike and dome IgG deposits (choice D) are found in membranous nephropathy, a cause of the nephrotic
syndrome.
A patient presents to a physician because of a markedly inflamed and painful great toe. Physical
examination additionally demonstrates small nodules on the patient's external ear. Aspiration of
the metatarsal-phalangeal joint of the affected toe demonstrates needle-shaped, negatively
birefringent crystals. Of which of the following are the crystals most likely composed?
A. Bile pigments
B. Calcium pyrophosphate
C. Cystine
D. Monosodium urate
E. Struvite
Explanation:

334
The correct answer is D. The patient has gout, which is due to precipitation of monosodium
urate crystals in joint spaces (notably the great toe) and soft tissues (causing tophi, which
are often found on the external ears).
Bile pigments (choice A) are found in some gallstones.
Calcium pyrophosphate (choice B) crystals are deposited in pseudogout, which classically
affects the knee or other large joints.
Cystine (choice C) and struvite (choice E) can form kidney stones.
Which of the following features characterizes apoptosis but not necrosis?
A. Disaggregation of polyribosomes
B. Eosinophilia
C. Inflammation
D. Karyolysis
E. Peripheral aggregation of chromatin
Explanation:
The correct answer is E. Apoptosis is a form of cell death that serves to eliminate unwanted
cells during development, maintain cell numbers in intact organs or tissues, and eliminate
immune cells after an immune response has faded. Apoptosis also occurs in response to noxious
agents and in the aging process. The process is the result of execution of an internal program
mediated by the expression of a number of different genes. In the process of apoptosis, cells
shrink and cytoplasmic organelles become more densely packed. Cytoplasmic blebs may form, and
apoptotic bodies (membrane-bound cellular fragments) can be produced. The most characteristic
feature of apoptosis is a distinctive peripheral aggregation of chromatin, sometimes
accompanied by breaking up of the nucleus into several fragments. In necrosis, the chromatin
may become more pale (karyolysis; choice D), or form irregular clumps, and the nucleus itself
may shrink into a dense pyknotic body.
Disaggregation of polyribosomes (choice A) is characteristic of the initial stages of cellular
injury and necrosis.
Eosinophilia (choice B) characterizes both apoptosis and necrosis.
Inflammation (choice C) is typically absent in apoptosis, in contrast to necrosis.
A man living in southern Japan contracts HTLV-1 infection through sexual contact. Twenty-five years
later he develops generalized lymphadenopathy with hepatosplenomegaly, a skin rash,
hypercalcemia, and an elevated white blood count. This man has most likely developed which of
the following?
A. AIDS
B. Autoimmunity
C. Delayed hypersensitivity reaction
D. Leukemia

335

E. Recurrent infection
Explanation:
The correct answer is D. HTLV-1, or human T-cell lymphotrophic virus 1, is an enveloped,
single-stranded, RNA retrovirus endemic to southern Japan and the Caribbean basin, but
sporadically seen in the United States. Infection by HTLV-1 can lead to T-cell leukemia 20-30
years after the infection. The HTLV-associated T-cell leukemia generally presents as described
above, and is very aggressive, progressing to death in under 1 year.
AIDS (choice A), acquired immunodeficiency syndrome, is transmitted through the HIV virus.
Although both viruses are RNA retroviruses and both infect CD4+ T cells, the diseases they produce are
distinct.
Autoimmune diseases (choice B) produced after infection are typified by rheumatic fever after
infection with Streptococcus. There is speculation that EBV virus (Epstein-Barr virus) may
produce the autoimmune reactions in rheumatoid arthritis, but autoimmunity plays no role in
leukemia after HTLV-1 infection.
Delayed hypersensitivity reactions (choice C) are T-cell-mediated immune responses typified by the
subcutaneous reaction to tuberculin in sensitized individuals. It characteristically occurs 2-3
days after an exposure, and is a local lymphocytic response.
Recurrent infections (choice E) occur when latent viruses are reactivated in the host at the
site of primary infection. Recurrent infections are most common with the herpesviruses,
especially HSV-1, HSV-2, and VZV (varicella-zoster virus).
A 25-year-old female presents with a 12-month history of palpitations, intermittent diarrhea, anxiety,
and a 1-month history of "bulging of both eyes." What is the most likely cause of her symptoms?
A. Graves' disease
B. Hashimoto's thyroiditis
C. Multinodular toxic goiter
D. Papillary carcinoma
E. Subacute thyroiditis
Explanation:
The correct answer is A. Graves' disease is the most common cause of hyperthyroidism in a young
female and is the only one that causes exophthalmos ("bulging of both eyes"). Grave's disease
is an autoimmune disorder in which a thyroid-stimulating IgG immunoglobulin (TSI) binds to the
TSH receptors causing increased release of thyroid hormone. The exophthalmos is caused by
lymphocytic infiltration of the extraocular muscles.
Hashimoto's thyroiditis (choice B) results in hypothyroidism and is associated with a diffusely
enlarged thyroid gland and antimicrosomal antibodies against the thyroid parenchyma.
Multinodular toxic goiter (choice C) causes hyperthyroidism, but does not result in exophthalmos.
Papillary carcinoma (choice D) will only very rarely present as a hypersecreting nodule. Most
cases will be nonsecreting, cold nodules. There is no exophthalmos. This is the most common

336
thyroid cancer and has the best prognosis of all thyroid cancers.
Subacute thyroiditis (choice E) is an uncommon form of thyroiditis that lasts approximately 8
months and is self-limited. Early on, with destruction of thyroid tissue, there may be release
of thyroid hormone and symptoms of hyperthyroidism, but exophthalmos is generally absent.
The 10-year-old daughter of a United Nations ambassador in Turkey develops a severe sore throat, which
resolves after a few days. 10 days later, the mother notices dark stains on the child's
underwear, and takes her to the pediatrician. A urine sample is smoky in color, and red cell
casts are noted in the urinary sediment. If a renal biopsy were obtained from this child it
would probably show
A. dense deposits
B. fusion of podocyte foot processes
C. linear deposition of IgG
D. mesangial IgA deposits
E. subepithelial humps
Explanation:
The correct answer is E. The history suggests streptococcal pharyngitis, followed by poststreptococcal
glomerulonephritis. Subepithelial humps composed of proteinaceous material can be seen
projecting outward from the outer capillary wall to the urinary space by light, or especially,
electron microscopy. Granular deposits of IgG and C3 can be seen by immunofluorescence
microscopy.
Dense deposits (choice A) are characteristic of Type II membranoproliferative glomerulonephritis.
Fusion of podocyte processes (choice B) is seen in minimal change disease.
Linear deposition of IgG (choice C) is seen in Goodpasture's syndrome.
Mesangial IgA deposits (choice D) are characteristic of Berger's disease.
A 40-year-old woman has severe, disabling rheumatoid arthritis. Rheumatoid factor is positive. What
would a biopsy of the synovium of her knee most likely reveal?
A. A nearly normal synovium with scattered inflammatory cells
B. A non-proliferative synovitis with abscess formation
C. A non-proliferative synovitis with many neutrophils
D. A proliferative synovitis with many eosinophils, neutrophils, and plasma cells
E. A proliferative synovitis with many lymphocytes, macrophages, and plasma cells
Explanation:
The correct answer is E. Severe rheumatoid arthritis causes a proliferative synovitis with
extensive damage to the synovium of the joint. The synovial membrane becomes markedly thickened (pannus
formation), with edematous villous projections that extend into the joint space. The intense

337
inflammatory infiltrate that is present is typically composed of plasma cells, lymphocytes, and
macrophages.
A 3-year-old child is seen by a pediatrician because he has developed multiple isolated lesions on his
face and neck. Physical examination reveals many lesions up to 4 cm in diameter with golden
crusts, while in other sites small blisters and weeping areas are seen. Which of the following
is the most likely diagnosis?
A. Aphthous ulcers
B. Erysipelas
C. Herpes simplex I
D. Impetigo
E. Measles
Explanation:
The correct answer is D. This is impetigo, which is typically seen in preschool children with
poor hygiene, particularly in the summer in warm climates. The characteristic lesion has a
large golden crust. Most cases are caused by Staphylococcus aureus; Streptococcus pyogenes is
occasionally implicated. Impetigo is highly infectious, and mini-epidemics can occur in daycare
settings. The initial treatment is typically with penicillins and topical preparations.
Methicillin-resistant strains are presently rare in this setting, but can occur.
Aphthous ulcers (choice A), commonly known as "canker sores," are painful, shallow ulcers of
the oral cavity.
Erysipelas (choice B) is a different type of skin infection, often caused by Streptococcus
pyogenes (also sometimes others including Staphylococcus), and is characterized by large erythematous
patches.
Herpes simplex I (choice C) causes tiny oral and perioral vesicles, but not large golden crusts.
Measles (choice E) causes a blotchy erythematous rash.
A 6-year-old girl manifests acute vomiting and nuchal rigidity. MRI reveals a tumor in the posterior
fossa consisting of a large cyst with a nodular mass attached to its wall (cyst with "mural
nodule"). Histologic examination shows elongated astrocytes with long bipolar processes and
numerous Rosenthal fibers. Which of the following is the most likely diagnosis?
A. Astrocytoma, WHO grade II
B. Ependymoma
C. Glioblastoma multiforme
D. Medulloblastoma
E. Pilocytic astrocytoma
Explanation:
The correct answer is E. Location, gross appearance, and histologic features are typical of pilocytic

338
astrocytoma, an astrocytic tumor with an indolent growth pattern that usually affects children.
Cerebellar and hypothalamic regions are the most frequent locations. The presence of Rosenthal
fibers is an important histopathologic clue. Rosenthal fibers are corkscrew-shaped, intensely
eosinophilic structures deriving from accumulation of &alpha;&beta;-crystallin within
astrocytic processes. Rosenthal fibers are found in pilocytic astrocytoma, reactive gliosis
(especially around tumors and vascular malformations), and a rare leukodystrophy known as
Alexander disease. Pilocytic astrocytomas have excellent prognosis in the posterior fossa, but
complete surgical excision is difficult with tumors in the hypothalamic region. The tumor is
classified as a grade I astrocytoma in the WHO system. Remember: a tumor in the cerebellum
appearing as a cyst with a mural nodule and containing abundant Rosenthal fibers = Pilocytic
astrocytoma.
Astrocytoma, grade II (choice A) and glioblastoma (choice C) are both astrocytomas, ie, tumors
of astrocytic origin. Glioblastoma is grade IV according to the WHO system. As their grades
suggest, grade II astrocytoma is less anaplastic (ie, less malignant) than grade IV
astrocytoma. These tumors grow in a diffuse manner, do not possess a cystic component, and
usually arise in the white matter of the centrum semiovale.
Ependymoma (choice B) originates from the ependyma and develops near the ventricular walls (4th
ventricle in children). Ependymomas of the 4th ventricle fill the ventricular cavity but can be
easily differentiated from pilocytic astrocytomas in their histologic appearance: ependymal
cells organized around small vessels (perivascular pseudorosettes) or around small lumina (true
ependymal rosettes), mimicking the primordial ependymal canal.
Medulloblastoma (choice D) refers to a malignant neoplasm of childhood that presumably originates from
immature glioneuronal precursors. It is located in the midline posterior fossa and infiltrates
the cerebellar vermis. This tumor is solid (not cystic) and consists of undifferentiated small
cells arranged in patternless sheets.
A 36-year-old male presents with a complaint of passing dark-reddish urine. He states that yesterday he
played racquetball for 4 hours, which was the first time he had exercised in 4 months. He awoke
this morning with sore muscles and discolored urine. Physical examination is unremarkable. The
urine is reddish-brown in color; dipstick test for blood is positive, the pH is 5.1, the
specific gravity 1.03. Microscopic examination of the urine reveals no red blood cells. What is
the most likely etiology for this presentation?
A. Hemolyzed blood in the urine
B. Ingestion of foods that contained red dye
C. Myoglobinuria
D. Nephrolithiasis
E. Prostatitis
Explanation:
The correct answer is C. The appearance of a dark-reddish urine is suggestive of a number of
conditions, including hematuria; however, there are no red blood cells present on microscopic
examination. Furthermore, the dipstick test for blood is positive, which is suggestive of
myoglobinuria. Myoglobin is a pigment that, if present in the urine, will produce a positive
dipstick test, even in the absence of red blood cells. Myoglobinuria can be caused when an
unconditioned individual partakes in intense physical exertion. This extreme physical activity
leads to muscle cell breakdown and the release of myoglobin into the blood, which is then
filtered by the kidneys and excreted into the urine.

339

If there was hemolyzed blood in the urine (choice A), a few red blood cells should have been
detected on urinalysis.
Ingestion of foods that contained red dye (choice B) can discolor the urine; however, it would
not produce a positive dipstick test.
Nephrolithiasis (choice D) generally results in the appearance of blood cells in the urine.
Prostatitis (choice E), an inflammation of the prostate gland, is generally caused by an
infection. Prostatitis produces symptoms of dysuria, urinary frequency, and cloudy urine; hematuria is
rare.
A woman who is heterozygous for glucose-6-phosphate dehydrogenase (G6PD), a polymorphic enzyme
transcribed from the X chromosome, develops chronic myeloid leukemia. Restriction fragment length
polymorphism (RFLP) studies on the tumor cells for G6PD reveal that only a single form of the enzyme is
transcribed. This finding supports which of the following features of neoplasia?
A. Genetic mutation
B. Monoclonality
C. Mosaicism
D. Oncogene activation
E. Point mutation
Explanation:
The correct answer is B. G6PD is an enzyme involved in carbohydrate metabolism-it is neither an
oncogene nor a tumor suppressor, and its altered expression in tumor cells is solely due to its
location on the X chromosome. Each somatic cell expresses only one X chromosome, and thus only
one of the two isotypes of G6PD. The presence of only one form of G6PD in the tumor cells
reflects the monoclonal origin of the cells. Cells contained in the neoplasm express only a
single isotype of G6PD because they are all derived from a single progenitor cell.
Genetic mutation (choice A) does not underlie the monotypic expression of G6PD in the tumor
cells. The allele for G6PD not expressed in the monoclonal cell line was inactivated in
embryogenesis (in the Barr body). Although genetic mutation is a requirement of neoplasia, it
is not demonstrated by this phenomenon.
Mosaicism (choice C) is the expression of two or more different phenotypes heterogeneously
throughout an organ or organism. Mosaicism usually reflects an asymmetry of mitotic division in
embryogenesis.
G6PD is not an oncogene (choice D), it is a metabolic enzyme expressed constitutively in all
cells. Altered expression of G6PD does not produce any growth advantage in a cell.
Although point mutations (choice E) underlie much of the genetics involved in neoplasia, no
point mutation is required for monoclonal expression of G6PD.
A patient presents to a physician because of troubling "heartburn" and difficulty swallowing. Esophageal
motility studies demonstrate a near absence of smooth muscle peristalsis and lower esophageal
sphincter tone. No mass lesions are noted and the esophagus is not dilated. Which of the

340
following findings would most likely also be present?
A. Anemia
B. Atrophic glossitis
C. Hourglass-shaped stomach on barium swallow
D. Massively dilated colon
E. Thick skin
Explanation:
The correct answer is E. Near complete absence of muscle tone and peristalsis is characteristic
of involvement of the esophagus with scleroderma, which causes replacement of muscle by dense
connective tissue. Similar changes in the dermis cause the skin to be thickened. The thickened,
shiny skin of the hands may cause them to resemble claws.
Anemia (choice A) and atrophic glossitis (choice B) are associated with esophageal webs in
Plummer-Vinson syndrome.
An hourglass-shaped stomach within the thoracic cavity (choice C) is a feature of a sliding
hiatal hernia.
A massively dilated esophagus (megaesophagus) can be caused by Chagas' disease, a trypanosomal
disease that can also cause massive dilation of the colon (choice D).
A strict vegetarian is not getting sufficient vitamin D in his diet, and he develops osteomalacia. This
disease is characterized by which of the following changes in his bones?
A. Decreased osteoblasts
B. Increased osteoclast activity
C. Increased osteoid
D. Marrow fibrosis
E. Sparse bony trabeculae
Explanation:
The correct answer is C. The hallmark of osteomalacia is widened osteoid seams. Although the
trabeculae are normal in number and size, they do not mineralize effectively, and the rim of
uncalcified osteoid is much larger than normal.
Osteoclast activity is normal in osteomalacia. Increased osteoclast activity (choice B) is
typical of hyperparathyroidism and Paget's disease of bone.
There are increased numbers of osteoblasts in osteomalacia (compare to choice A), which lay
down increased quantities of osteoid in an effort to strengthen the bone. Unfortunately, the
increased osteoid is not mineralized and the bone remains soft and weak.
The bone marrow space is normal in osteomalacia. Marrow fibrosis (choice D) is more typical of
hematopoietic disorders, although hyperparathyroidism may also lead to bone marrow fibrosis.

341

Although poorly mineralized, the bony trabeculae are of normal abundance in osteomalacia.
Sparse trabeculae (choice E) is characteristic of osteoporosis.
A 35-year-old woman notices a change in the appearance of a mole on her neck. Physical examination
reveals that the lesion is an irregular, nodular, superficial mass with a variegated
appearance. Biopsy demonstrates a primary malignant tumor. Which of the following factors is
most predictive of the patient's long term prognosis?
A. Circumference of lesion
B. Darkness of lesion
C. Degree of color variation
D. Depth of lesion
E. Sharpness of border between lesion and adjacent skin
Explanation:
The correct answer is D. The lesion is a malignant melanoma. Melanomas can develop either de
novo or in an existing mole. Sunlight exposure is a significant risk factor and fair-skinned
persons are at increased risk of developing melanoma. The most significant factor for long term
prognosis is the depth of the lesion, since the superficial dermis lies about 1 mm under the
skin surface, and penetration to this depth is associated with a much higher incidence of
metastasis than is seen with a more superficial location.
The circumference of the lesion (choice A) is much less important than depth, since one form of
melanoma (superficial spreading) can still have good prognosis despite large size, if it has
not extended to the depth of the superficial dermal lymphatic bed.
The darkness (choice B) or degree of variation in color (choice C) do not have prognostic
significance once melanoma is diagnosed.
Irregularity, or fuzziness at the border (choice E) of a mole-like lesion is a good clue to
potential malignancy, but does not affect prognosis once a melanoma is diagnosed.
A 45-year-old man presents to a physician with flank pain and hypertension. Serum chemistries
demonstrate slightly increased blood urea nitrogen and creatinine. Hematuria is noted on
urinalysis. Ultrasound studies demonstrate markedly enlarged kidneys with irregular margins and
many fluid-filled spaces of varying sizes. Excessive secretion of which of the following
hormones would most likely account for the patient's hypertension?
A. ACTH
B. Cortisol
C. Parathormone
D. Renin
E. Thyroxine
Explanation:

342
The correct answer is D. The disease is adult polycystic kidney disease, which is an autosomal
dominant condition that typically manifests in middle age. While the kidneys appear very badly
deformed, they function surprisingly well because the cystic spaces actually only affect 10% or
less of the nephrons. Pressure exerted by the cysts can somewhat compromise blood flow to some
glomeruli, which is probably why hypertension, as a consequence of renin secretion, is such a
problem for these patients.
ACTH (choice A) can stimulate cortisol (choice B) secretion, and the cortisol (in high amounts)
can have enough mineralocorticoid activity to cause hypertension. However, this would be more
likely with a pituitary tumor, adrenal tumor, or with exogenous corticosteroid use.
Parathormone (choice C) regulates calcium metabolism and does not usually affect blood
pressure.
Thyroxine excess (choice E) can cause hypertension as a consequence of its effects on the heart
and vasculature; this might be seen in the setting of hyperthyroidism due to thyroid disease.
A 45-year-old woman with a history of thyroid disease presents to her physician with a anterior neck
mass. Biopsy demonstrates non-Hodgkin's lymphoma. Which of the following thyroid conditions
most likely preceded the development of lymphoma in this patient?
A. Follicular thyroid carcinoma
B. Graves disease
C. Hashimoto's thyroiditis
D. Nodular goiter
E. Papillary thyroid carcinoma
Explanation:
The correct answer is C. Hashimoto's disease (autoimmune thyroiditis) confers a 60- to-80 fold
increased risk of developing thyroid lymphoma compared to normal controls. This increased risk
is not surprising, considering the histology exhibited in Hashimoto's, in which sheets of
benign but activated lymphocytes infiltrate and eventually destroy the thyroid gland, producing
eventual thyroid failure.
Neither follicular (choice A) nor papillary (choice E) thyroid carcinomas predispose for
lymphoma.
Lymphoma is not particularly related to Graves disease (choice B) or benign nodular goiter
(choice D).
A patient with intestinal malabsorption is found to markedly improve when flour products (bread,
noodles, etc.) are removed from his diet. At the height of the patient's disease, marked
histologic changes would be seen at which of the following sites?
A. Distal large bowel
B. Distal small bowel
C. Entire large bowel
D. Entire small bowel

343

E. Proximal small bowel


Explanation:
The correct answer is E. The patient has celiac disease, which is apparently an acquired
hypersensitivity to the gluten (such as gliadin) in wheat. Unlike tropical sprue (which may be
related to enterotoxigenic E. coli infection), which involves the entire small bowel, celiac
sprue is usually limited to the proximal small bowel. This may occur because the gluten
antigens have not yet been digested at this point in their journey through the bowel. A glutenfree diet usually restores the small bowel mucosa.
Accumulation of which of the following substances indicates aging at a cellular level?
A. Beta-carotene
B. Bilirubin
C. Hemosiderin
D. Lipofuscin
E. Melanin
Explanation:
The correct answer is D. Lipofuscin is a brown pigment that accumulates with aging. It is
believed to be produced from the peroxidation of lipids. Lipofuscin accumulation does not
necessarily impair the ability of the cell to function, and can be found in the hearts and
livers of healthy elderly patients.
Beta-carotene (choice A) is a carotenoid ingested in the diet (found in yellow vegetables such
as squash, pumpkins, and carrots) and converted to vitamin A. Excessive beta-carotene can cause
a benign yellow-orange discoloration of the skin in a condition known as carotenemia.
Bilirubin (choice B) is a pigment derived from the metabolism of the heme group of hemoglobin.
As hemoglobin is broken down, it first forms biliverdin, which is subsequently converted to
bilirubin. Bilirubin can be conjugated (to glucuronide) or unconjugated. The conjugated form
(also called the direct reacting portion) accumulates in biliary obstructions. The unconjugated
form of bilirubin (indirect-reacting) accumulates in hemolytic processes.
Hemosiderin (choice C) is the storage form of iron and stains blue with Prussian blue.
Hemosiderin accumulation from breakdown of red cells is seen in chronic passive congestion of
the lung (inside hemosiderin-laden macrophages called "heart failure cells"). Hemosiderin
deposition is also seen in hemochromatosis, a disorder characterized by abnormal iron storage.
Hemochromatosis is seen in patients with increased iron uptake from the GI tract, and in
patients receiving repeated blood transfusion therapy.
Melanin (choice E) is a brown-black pigment made by melanocytes in the skin. Melanin is also
found in the iris, giving the eye its color. Neuromelanin is a type of melanin found in
catecholamine neurons in the brain.
A 25-year-old man presents to a physician because of multiple small nodules on his lips. The clinician
notes that the patient has a marfanoid habitus. Biopsy of one of these nodules demonstrates a
mucosal neuroma. Which of the following screening tests is indicated?

344
A. Pentagastrin-stimulated calcitonin
B. Serum gastrin
C. Serum insulin
D. Serum parathyroid hormone
E. Serum vasoactive intestinal peptide
Explanation:
The correct answer is A. The presence of mucosal neuromas, particularly when multiple and when
the patient has a marfanoid syndrome, is a marker for multiple endocrine neoplasia type IIb (MEN
IIb). MEN IIb predisposes for medullary carcinoma of the thyroid and pheochromocytoma.
Pentagastrin-stimulated calcitonin studies may suggest the presence of thyroid C cell
hyperplasia, which may precede frank medullary carcinoma.
High serum gastrin (choice B) suggests pancreatic or duodenal gastrinomas, which may be a
component of MEN I.
High serum insulin (choice C) or vasoactive intestinal peptide (choice E) also suggest
pancreatic endocrine tumors, which may be a component of MEN I.
Parathyroid hyperplasia or adenomas with parathyroid hormone secretion (choice D) can be seen in
MEN I and MEN IIa, but are not part of MEN IIb.
An adult patient presents with persistent headaches. A CT scan of the head demonstrates a 2-cm spherical
mass at the junction of the white and gray matter of the lateral aspect of the cerebral
hemisphere. Which of the following would most likely produce this lesion?
A. Astrocytoma
B. Ependymoma
C. Glioblastoma multiforme
D. Meningioma
E. Metastatic carcinoma
Explanation:
The correct answer is E. 70% of adult brain tumors occur above the tentorium (70% of childhood
tumors occur below the tentorium). Statistically, metastases > astrocytomas (including
glioblastoma) > meningioma > pituitary tumor. Location at the junction of cortical gray and
white matter is also typical for metastatic disease, as is the round shape.
Astrocytomas (choice A) typically arise in the white matter and have an irregular shape.
Ependymomas (choice B) are uncommon and arise from the ependymal lining of the ventricles.
Glioblastoma multiforme (choice C) is an aggressive form of astrocytoma that can cause a
"butterfly lesion", crossing between the cerebral hemispheres.
Meningiomas (choice D) can cause spherical lesions, and are usually located on the surface of

345
the brain.
A 62-year-old female is evaluated for uterine bleeding. In the course of her workup, an endometrial
biopsy reveals marked endometrial hyperplasia. A left ovarian mass is identified, which appears
to be solid and about 4 cm in diameter on ultrasound. The ovarian tumor is most likely to be
a(n)
A. corpus albicans
B. endometrioid carcinoma
C. endometriotic cyst
D. granulosa cell tumor
E. teratoma
Explanation:
The correct answer is D. Granulosa cell tumors arise from the granulosa cells of the ovarian
follicle. Typically occurring in postmenopausal women, granulosa cell tumors are solid, yellow
masses that frequently produce estrogens. Consequently, granulosa cell tumors often present
with abnormal uterine bleeding, and they are considered a risk factor for endometrial
carcinoma.
Corpora albicantia (choice A) are the fibrotic remnants of corpora lutea. The corpus albicans
is small, usually less than 1 cm, and does not produce an ovarian mass or hormonal changes that
might cause bleeding.
Endometrioid carcinoma (choice B) of the ovary is an epithelial malignancy that resembles
endometrial carcinoma by light microscopy. It is a solid and/or cystic tumor that arises (like
serous or mucinous carcinoma) in the absence of any appreciable hormonal imbalance.
An endometriotic cyst (choice C) is a focus of endometriosis within an organ other than the
uterus. Endometriotic cysts are commonly called "chocolate" cysts due to the appearance of
brown, semi-solid hemorrhage with the lining wall of endometrial glands and stroma.
Endometriotic cysts do not appear solid, and they commonly regress after menopause.
Teratomas (choice E) usually appear solid and cystic on diagnostic studies, due to the multiple
tissue components present within. Most teratomas contain hair and sebaceous material among
other things, but they do not produce estrogens. There are rare cases of teratomas containing
predominantly thyroid tissue or carcinoids that may be hormonally active.
A 79-year-old man complains of pain in the upper portion of his neck on swallowing. He occasionally
regurgitates undigested food shortly after eating. Which of the following is the most likely
etiology of his problems?
A. Mallory-Weiss tears
B. Plummer-Vinson syndrome
C. Schatzki rings
D. Traction diverticula
E. Zenker's diverticulum

346

Explanation:
The correct answer is E. This is the classic presentation of Zenker's diverticulum, which is a
false diverticulum formed by herniation of the mucosa at a point of weakness at the junction of
the pharynx and esophagus in the posterior hypopharyngeal wall. Zenker's diverticulum is also
associated with halitosis, and if the diverticulum fills completely with food, it can cause
dysphagia or obstruction of the esophagus.
Mallory-Weiss tears (choice A) are mucosal tears at the gastroesophageal junction secondary to
repeated, forceful vomiting. They are often seen in alcoholics.
Plummer-Vinson syndrome (choice B) is the triad of dysphagia (due to esophageal webs in the
upper esophagus), atrophic glossitis, and iron-deficiency anemia.
Schatzki rings (choice C) are mucosal rings found in the distal esophagus at the squamocolumnar
junction.
In contrast to a Zenker's diverticulum, the usually asymptomatic traction diverticula (choice
D) are true diverticula involving all of the layers of the esophagus. They are typically caused
by adherence of the esophagus to a scarred mediastinal structure.
A 72-year-old woman complains of constipation and abdominal pain. Over a period of 48 hours, her
symptoms worsen, and she is transported to the hospital for laparoscopic evaluation of an acute
abdomen. At laparoscopy, a volvulus is noted. Which of the following is the most likely
location for the volvulus?
A. Appendix and cecum
B. Ascending colon
C. Descending colon
D. Sigmoid colon
E. Transverse colon
Explanation:
The correct answer is D. Typically, an elderly patient with a volvulus develops an "acute
abdomen", and is found at laparoscopy or laparotomy to have a twisted (and potentially
infarcted) segment of bowel, usually in the poorly supported sigmoid colon. Most other parts of
the colon are held in place by the posterior peritoneal membrane, and the transverse colon is
stretched so that it can not twist. In contrast, intussusception tends to occur either in
babies and young children or in patients with a mass lesion that gets dragged by peristalsis
into the adjacent section of large or small intestine.
A 7-year-old child with slowly developing cardiac outflow obstruction is found on echocardiographic
study to have a mass lesion in the left ventricle. Surgical removal of the mass demonstrates a
benign tumor composed of cells resembling striated muscle. This child should also be evaluated
for which of the following central nervous system lesions?
A. Berry aneurysm
B. Glioblastoma multiforme

347
C. Infarction
D. Meningioma
E. Tubers
Explanation:
The correct answer is E. The child has a rhabdomyoma, which is the most common primary cardiac
tumor in children. These tumors are composed of cells that resemble skeletal muscle cells, and
are particularly common in children with tuberous sclerosis.
You should associate berry aneurysms (choice A) with adult polycystic kidney disease.
Glioblastoma multiforme (choice B) is a high-grade astrocytoma, and is a relatively common
childhood CNS malignancy. Glioblastoma is not associated with cardiac tumors.
Embolization of atrial myxomas, not rhabdomyomas, may cause CNS infarction (choice C).
Meningiomas (choice D) are not associated with cardiac tumors and are more common in adults.
These usually benign tumors tend to grow at the periphery of the brain, and may compress it,
but do not penetrate it.
A 57-year-old woman with a 30-year x 2 pack/day history of cigarette smoking undergoes bronchoscopy.
Biopsy of bronchial tissue shows replacement of the normal pseudostratified ciliated columnar
epithelium with stratified squamous epithelium. This change represents
A. dysplasia
B. hyperplasia
C. malignant transformation
D. metaplasia
E. necrosis and repair
Explanation:
The correct answer is D. Squamous metaplasia is a frequent airway response to chronic
irritation from cigarette smoking. It represents the replacement of one differentiated tissue
with another mature, differentiated tissue. It is believed that the squamous epithelium is more
resistant to injurious agents, and so represents an adaptive response.
Dysplasia (choice A) is characterized by pleomorphism (the individual cells may vary widely in
appearance) and by loss of normal tissue architecture, rather than by replacement with another
mature epithelium (squamous).
Hyperplasia (choice B) refers to growth of a tissue or organ by cellular proliferation.
Malignant transformation (choice C) is the multistep process by which normal cells progress to
the "cancer" phenotype. Metaplasia is not malignant transformation, although the stimuli
producing metaplasia may also be associated with the development of malignancy.
Necrosis and repair (choice E) typically result in re-epithelialization and/or fibrosis, not
metaplasia.

348

A patient develops an ovarian mass that is picked up on pelvic examination. Resection of the mass
demonstrates a lymphoma composed of small lymphocytes with interspersed macrophages, producing
a starry sky pattern. This lesion would be most likely to be associated with which of the
following?
A. abl-bcr hybrid
B. bcl-2 activation
C. c-myc activation
D. t(9,22)
E. t(14,18)
Explanation:
The correct answer is C. The disease is Burkitt's lymphoma (the major clue is "starry sky"
pattern), which occurs as a jaw lesion in epidemic form in Africa (associated with Epstein-Barr
virus) and in a sporadic form that usually involves the pelvic or abdominal organs. Burkitt's
lymphoma is associated with c-myc activation due to a t(8,14) translocation that places the cmyc-containing segment of chromosome 8 near an actively transcribed gene for immunoglobulin
heavy chains.
t(9,22) (choice D) and abl-bcr hybrid (choice A) are associated with chronic myeloid leukemia
(CML).
t(14, 18) (choice E) and bcl-2 (choice B) are associated with follicular lymphomas, not
Burkitt's lymphoma.
A 43-year-old man presents to a urologist because of a lesion on his penis. Physical examination reveals
an 8-mm opaque, gray-white, relatively flat penile plaque. A biopsy is performed on the lesion;
the pathology report indicates the presence of clearly dysplastic squamous epithelium. Which of
the following is the most likely diagnosis?
A. Bowenoid papulosis
B. Bowen's disease
C. Condyloma acuminatum
D. Erythroplasia of Queyrat
E. Giant condyloma
Explanation:
The correct answer is B. The lesion described is the form of in situ penile carcinoma known as
Bowen's disease. Bowen's disease is thought to carry a 10% risk of progressing to invasive
squamous cell carcinoma, if left untreated.
Bowenoid papulosis (choice A) is a form of in situ penile carcinoma that is characterized
clinically by multiple, reddish-brown, papular lesions.
Condyloma acuminatum (choice C) is a benign lesion that resembles the common wart. Condylomata

349
are associated with papillomavirus infection.
Erythroplasia of Queyrat (choice D) is a form of in situ penile carcinoma that produces a soft,
red plaque.
Giant condyloma (choice E) is an uncommon variant of condyloma acuminatum that is locally
aggressive, but does not usually metastasize.
Which of the following types of breast cancer has the best prognosis?
A. Ductal carcinoma
B. Inflammatory carcinoma
C. Lobular carcinoma
D. Medullary carcinoma
E. Tubular carcinoma
Explanation:
The correct answer is E. Breast adenocarcinoma occurs in a variety of morphologic subtypes,
some of which have a significantly worse or better prognosis than the most common invasive
ductal carcinoma. Tubular carcinoma occurs in women younger than 50 and has an excellent
prognosis. It consists of well-formed tubules, which are so well differentiated that the tumor
is sometimes mistaken for a benign lesion. Axillary metastases are present in fewer than 10% of
cases at the time of diagnosis. Of the histologic types of breast cancer listed here, the
tubular variant is certainly associated with the best chances of survival.
Invasive ductal carcinoma (choice A) and invasive lobular carcinoma (choice C) are the main
morphologic types of breast carcinoma. Together, they constitute 80% to 90% of all cases. The
prognosis mainly depends on the staging, related to size of the primary cancer and evidence of
lymphohematogenous spread to regional nodes and distant organs. Grading (that is degree of
differentiation of primary tumor) has a minor influence on prognosis.
Whenever breast cancer of any histologic type grows very rapidly, infiltrating and occluding
the dermal lymphatics, acute swelling and redness of the breast may develop, leading to a
clinical picture referred to as inflammatory carcinoma (choice B). Inflammatory carcinoma,
therefore, is not a true morphologic subtype but a clinical definition. Any form of breast
cancer manifesting as &ldquo;inflammatory carcinoma&rdquo; is likely to be extremely
aggressive.
Medullary carcinoma (choice D) has a slightly better prognosis than ductal or lobular types. It
tends to occur in younger women, especially those with mutations of BRCA1. Grossly, this tumor
is soft and well circumscribed. Histologically, this carcinoma is surrounded by a marked
lymphoplasmacytic reaction, which may account for its propensity to grow more slowly than other
forms of cancer. Medullary carcinoma, however, has a worse prognosis than tubular carcinoma.
A 13-year-old girl is brought to pediatrician because her mother says "she just sits around." Physical
examination reveals akinesia and rigidity without other obvious neurologic deficits. Her eyes
are unusual, with thin greenish-brown rings around the outer edge of the cornea. Serum liver
enzymes are moderately elevated. Which of the following medications would likely be most
effective in treating this patient?
A. Deferoxamine

350

B. Edetate (EDTA)
C. Lithium
D. Penicillamine
E. Phenobarbital
Explanation:
The correct answer is D. The disease is Wilson's disease. This is an autosomal recessive
abnormality of copper metabolism that can present with either neurologic (psychiatric symptoms,
bradykinesia and rigidity, tremors, or chorea) or hepatic (hepatitis, cirrhosis, or
asymptomatic liver function test abnormalities) findings. The corneal rings described in the
question stem are the pathognomic Kayser-Fleischer rings due to copper deposition. Treatment is
with life-long administration of the copper-chelating agent, penicillamine.
Deferoxamine (choice A) is used parenterally to chelate circulating iron.
EDTA (choice B) is an effective chelator of divalent (and trivalent) cations such as lead. It
is administered as the Na2CaEDTA salt to avoid hypocalcemia.
Lithium (choice C) is used to treat bipolar disorder.
Phenobarbital (choice E) is used to treat epilepsy.
A 35-year-old man with severe arthritis and a decrease in mobility of his lower back undergoes HLA
testing, which demonstrates HLA-B27. With which of the following diseases is the HLA-B27
haplotype strongly associated?
A. Budd-Chiari syndrome
B. Goodpasture's syndrome
C. Reiter's syndrome
D. Reye's syndrome
E. Sjgren's syndrome
Explanation:
The correct answer is C. The patient has ankylosing spondylitis, which is very strongly
associated with HLA-B27. HLA-B27 is also associated with Reiter's syndrome, which can be
remembered as the syndrome in which you "can't see (anterior uveitis conjunctivitis), can't pee
(urethritis), and can't climb a tree (arthritis)."
Budd-Chiari syndrome (choice A) is liver disease secondary to occlusion of the inferior vena
cava or hepatic veins.
Goodpasture's syndrome (choice B) is pulmonary hemorrhage and glomerular disease secondary to
antibodies to basement membrane.
Reye's syndrome (choice D) is an often fatal liver disease that can follow viral infection.

351
Sjgren's syndrome (choice E) is an autoimmune disease that damages salivary glands and tear
glands.
A 63-year-old woman is seen in the emergency room after falling on an icy sidewalk and fracturing her
hip. She is found to have a low bone density, thought to be due to a malignant tumor. She also
has a history of recurrent pneumonia over the last 6 months. Lab work demonstrates a normal
white blood cell count, but decreased platelets and serum albumin with an elevated erythrocyte
sedimentation rate (ESR). Serum electrophoresis indicates an M-protein spike band of IgG kappa.
The urine will most likely show the presence of Bence-Jones proteins composed of
A. IgG heavy chains
B. kappa and lambda light chains of a 60:40 ratio
C. kappa light chains
D. lambda light chains
E. monomer IgG
Explanation:
The correct answer is C. The patient suffers from multiple myeloma, a neoplastic proliferation
of plasma cells (or their precursors) found within the bone marrow. These malignant cells are
responsible for the production of excessive amounts of immunoglobulin (usually IgG or IgA).
Plasma cells synthesize a greater amount of light chains than heavy chains. The intact
immunoglobulins are not excreted by the kidney, but light chains are filtered at the glomerulus
and found in the urine. In normal individuals, polyclonal light chains will be found in the
urine, reflecting the polyclonal population of plasma cells producing the light chains. In
monoclonal proliferations such as multiple myeloma, the light chains will be monoclonal. This
patient is making a monoclonal population of kappa light chains and excreting them in the urine
as Bence-Jones proteins. It is not unusual for patients with myeloma to have recurring
bacterial infections, particularly pneumococcal pneumonia. Remember, this patient is making
decreased levels of normal immunoglobulins of all isotypes, thus making her susceptible to the
bacteria to which she is exposed. Infiltration of bone by the myeloma cells may lead to
pathological fractures.
Bence Jones proteins are light chains, not heavy chains (choice A).
The serum protein electrophoresis showed a monoclonal spike of kappa chains, so the Bence-Jones
proteins will be monoclonal kappa chains. Rarely, a biclonal plasma cell dyscrasia can produce
Bence-Jones proteins composed of both kappa and lambda light chains (choice B), but this is not
the case here.
Bence-Jones proteins composed of lambda light chains (choice D) would not be produced by a
monoclonal myeloma producing kappa chains in the serum.
Bence-Jones proteins are light chains of the immunoglobulin molecule and not the intact monomer
IgG (choice E), which is composed of two light chains and two heavy chains.
Three to five days after a viral illness, a 4-year-old child develops pernicious vomiting, lethargy
progressing to coma, and hepatomegaly. A liver biopsy would likely show
A. hemosiderin
B. microvesicular steatosis

352

C. normal histology
D. Periodic acid-Schiff (PAS)-positive cytoplasmic granules
E. rhodamine-positive cytoplasmic granules
Explanation:
The correct answer is B. The child has Reye syndrome, which is a postviral derangement of
metabolism associated with enlarged, distorted mitochondria in many tissues. Most significant
clinically are the effects on the liver and brain. In the liver, there is a reduction in the
activity of many key biochemical pathways, notably the citric acid cycle, urea cycle, and beta
oxidation of fatty acids. The disruption of these pathways produces the very extensive
microvesicular steatosis (fatty change) that is the characteristic liver finding. In the brain,
edema is a prominent feature, and the astrocytes show mitochondrial disruptions by electron
microscopy. Many of the first described cases of Reye syndrome occurred in children treated
with aspirin, but the role of salicylates in the pathogenesis of this condition has not been
clearly established. Seventy-five percent of patients have a mild clinical course; the
remainder develop coma, liver failure, or permanent neurologic impairments. Death may result
from either hepatic or CNS damage. Therapy is supportive.
Hemosiderin (choice A) is seen in hemochromatosis and other conditions with increased hepatic
iron stores.
PAS-positive cytoplasmic granules (choice D) are a feature of &alpha;-1-antitrypsin deficiency.
Rhodamine-positive cytoplasmic granules (choice E) reflect copper accumulation in patients with
Wilson disease.
During a bitterly cold winter, an elderly couple is found dead in their apartment. All of their windows
are closed and their leaky old furnace is on full. Which of the following is the primary
mechanism by which the toxin involved led to the death of this couple?
A. Decreasing intracellular calcium
B. Inhibition of cytochrome oxidase
C. Inhibition of Na+/K+ ATPase
D. Irreversibly binding to hemoglobin
E. Stimulation of cellular apoptosis
Explanation:
The correct answer is D. This couple died of carbon monoxide poisoning. Carbon monoxide has
approximately 240 times the affinity for hemoglobin than does oxygen. In a sense, the
hemoglobin-CO dissociation curve is shifted very far to the left compared to the hemoglobin-O2
dissociation curve. This means that the binding of hemoglobin to carbon monoxide is virtually
irreversible. (The carbon monoxide that cigarette smokers inhale is cleared only when senescent
red cells are phagocytized in the spleen and the hemoglobin is degraded.) In addition, the
carbon monoxide shifts the hemoglobin-O2 dissociation curve to the left, making the unloading
of O2 to the tissues very difficult. When too much hemoglobin is tied up with carbon monoxide,
the person dies. The carbon monoxide-hemoglobin complex has a bright red color; a distinctive
feature of carbon monoxide poisoning that can be helpful either at autopsy or in living

353
patients is that this color makes the skin and organs also appear bright cherry red.
Generally, intracellular calcium levels increase, rather than decrease (choice A), with
cellular injury or death.
Cytochrome oxidase is inhibited by cyanide (choice B).
Ouabain is an example of a poison that inhibits Na+/K+ ATPase (choice C).
Apoptosis (programmed cell death) is stimulated (choice E) by certain genes (e.g., p53, ced
3,4), glucocorticoids, and aging.
A third-world patient develops muscle weakness and atrophy with fasciculations and hyporeflexia. The
patient's condition continues to deteriorate, and his legs are eventually permanently
paralyzed. The virus that can cause these problems is usually spread by which of the following
routes?
A. Dirt contact with open wound
B. Injection
C. Mosquitoes
D. Oral-fecal
E. Puncture wound of the foot
Explanation:
The correct answer is D. The disease is poliomyelitis, which is caused by the poliovirus, a
picorna virus. The virus is spread via the fecal-oral route and can then cause paralysis by
infecting the alpha-motor neurons of the anterior horn of the spinal cord. Early symptoms
include malaise, headache, fever, nausea, abdominal pain, and sore throat.
Bacterial and fungal infections can be spread by dirt contact with an open wound (choice A).
AIDS is an example of a disease spread by injection (choice B) or exchange of body fluids.
Malaria is an example of a disease spread by mosquitoes (choice C).
Tetanus is the classic example of disease spread by a puncture wound of the foot (choice E).
A 6-cm length of rectosigmoid colon containing a 2-cm diameter sessile polyp is surgically removed. On
sectioning, the lesion shows finger-like papillae with cores of scant lamina propria. The
surfaces of the papillae are covered by dysplastic columnar epithelium with considerable
nuclear pleomorphism. No glandular structures are seen in the base of the lesion or in the
adjacent muscle tissue. The margins of the specimen are free of dysplastic epithelium. What
further therapy does this person require?
A. Adjunct chemotherapy
B. Complete colectomy
C. Radiation therapy
D. Resection of regional lymph nodes

354

E. No further therapy is required


Explanation:
The correct answer is E. The patient has a villous adenoma, which has been adequately treated
by complete resection. If dysplastic epithelium had been found at the surgical resection
margins, re-excision to remove the dysplastic epithelium would be required, since villous
adenomas are considered to be a premalignant condition.
Adjunct chemotherapy (choice A) or radiation therapy (choice C) are not indicated since villous
adenoma is a premalignant condition rather than an actual malignancy.
Complete colectomy (choice B) is typically performed for inflammatory bowel disease rather than
for carcinoma of the colon, and would certainly not be indicated for the treatment of villous
adenoma.
The presence of clearly invasive glands would have indicated that the patient had a true
adenocarcinoma and evaluation of regional lymph nodes (choice D), would be required to stage
the extent of the disease.
A 62-year-old man with a long history of cigarette smoking develops high blood pressure, a moon face,
and central obesity. Serum ACTH is increased, but MRI studies of the pituitary and hypothalamus
fail to demonstrate any tumors. A chest x-ray film reveals a small tumor in the right upper
lobe, and a biopsy is performed. Histologically, the tumor is composed of sheets of anaplastic
cells with a high nuclear/cytoplasmic ratio. The tumor is immunoreactive to antibodies directed
against ACTH. Which of the following is the most likely diagnosis?
A. Adenocarcinoma
B. Carcinoid tumor
C. Hamartoma
D. Small cell carcinoma
E. Squamous cell carcinoma
Explanation:
The correct answer is D. Small cell carcinoma of the lung is composed of extremely
undifferentiated (ie, anaplastic) cells, with high nuclear/cytoplasmic ratio. This neoplasm is
very aggressive and tends to metastasize so early in its course that even small tumors are
considered inoperable by the time of clinical diagnosis. One of the most interesting
characteristics of this type of tumor is the frequent association with paraneoplastic
syndromes, the most common of which is due to ectopic production of ACTH. Paraneoplastic
syndromes also occur with other types of lung tumors.
Adenocarcinomas of the lung (choice A) are composed of mucin-producing neoplastic cells
arranged in a glandular pattern. Its histologic features make this tumor easily distinguishable
from small cell carcinoma.
Carcinoid tumors of the lung (choice B) are histologically similar to carcinoid tumors of the
gastrointestinal tract. They are composed of cords and islands of uniform cuboidal cells.
Carcinoid syndrome may be produced because of the ability of the tumor to produce and release
serotonin.

355

A hamartoma (choice C) is not a true neoplasm, but rather a malformative lesion consisting of
normal lung tissues arranged in haphazard manner. Hamartomas are often discovered incidentally
on chest x-ray films as "coin lesions" and contain cartilage, smooth muscle, and cystlike
spaces lined by bronchial epithelium.
Squamous cell carcinomas (choice E) are characterized by neoplastic squamous epithelium with
evidence of extracellular and/or intracellular keratin production. The most frequent
paraneoplastic syndrome accompanying pulmonary squamous cell carcinomas is hypercalcemia.
Finally, remember that cigarette smoking is a strong risk factor for all types of lung cancer,
especially squamous cell carcinoma and small cell carcinoma.
A patient presents to a physician with ill-defined complaints of malaise. The physician orders an
erythrocyte sedimentation rate, which yields a value lower than normal. Which of the following
conditions would most likely produce a decreased erythrocyte sedimentation rate?
A. Cancer
B. Infection
C. Polycythemia
D. Pregnancy
E. Systemic lupus erythematosus
Explanation:
The correct answer is C. The erythrocyte sedimentation rate (ESR) is a simple, cheap, but
nonspecific test based on the rate at which red cells will settle in blood. A decreased
erythrocyte sedimentation rate (cells settle out slower) can be seen in sickle cell anemia
(because of the altered cell shape), polycythemia (because there are many cells), and
congestive heart failure (the mechanism is not obvious).
Diseases associated with increased antibody formation tend to markedly increase the ESR by
causing the erythrocytes to stick to each other better and consequently settle faster.
Conditions with elevated ESR include cancer (choice A), infection (choice B), pregnancy (choice
D) and connective tissue diseases such as systemic lupus erythematosus (choice E).
A 42-year-old man previously diagnosed with kidney stones complains of gnawing, burning epigastric pain.
On questioning, he also notes moderate to severe diarrhea. Measurement of the patient's basal
gastric acid output reveals that it is markedly elevated. These symptoms are likely the result
of which of the following neoplastic syndromes?
A. Familial polyposis coli
B. MEN I
C. MEN IIA
D. MEN IIB
E. MEN III
Explanation:

356

The correct answer is B. Pancreatic islet tumors, which may produce gastrin and secondarily
Zollinger-Ellison syndrome, are a feature of multiple endocrine neoplasia type I. Other
features of MEN I are tumors of the parathyroid (the resulting hypercalcemia and hypercalciuria
leads to kidney stones), adrenal cortex, and pituitary gland. In general, the tumors in the MEN
syndromes may be expressed at different times in a patient's life, and not all patients may
exhibit the full syndromes.
Endocrine tumors are not a feature of familial polyposis coli (choice A), which is instead
characterized by colonic polyps and colon cancer.
MEN type IIA (choice C) features tumors of the adrenal medulla (pheochromocytoma), medullary
carcinoma of the thyroid, and parathyroid hyperplasia or adenoma.
MEN type IIB (choice D) closely resembles type IIA but also includes mucosal neuromas, and less
often includes parathyroid diseases.
MEN type III (choice E) is now considered to be the same as MEN type IIB.
A 70-year-old woman has a bilateral hip replacement for osteoarthritis. Which of the following
pathological changes will the removed femoral heads most likely demonstrate?
A. Marked synovial proliferation with pannus formation
B. Multiple small fractures in the cartilage
C. Multiple white flecks in the synovium
D. No visible change
E. Pus covering the articular surface
Explanation:
The correct answer is B. Osteoarthritis is characterized by mechanical, rather than
inflammatory, damage to the joint. The damage usually begins as multiple small fractures of the
cartilage tips of the involved bones, which can lead to wearing down of the cartilage to expose
the underlying bone. In long-standing cases, the articular ends of the bones may develop a
mushroom-like or flattened deformation.
Synovial proliferation with pannus formation (choice A) suggests chronic inflammatory joint
disease such as rheumatoid arthritis.
White flecks (choice C) in a joint suggests the crystals of gout or pseudogout.
The presence of pus (choice E) would suggest an acute inflammatory (possibly infectious)
etiology.
Examination, at autopsy, of the brain of a man who died in an intensive care unit demonstrates
bilateral, linear, parasagittal areas of liquefactive necrosis in the junctional zone between
the anterior and middle cerebral arterial systems. Which of the following is the most likely
etiology?
A. Bacterial infection
B. Fungal infection

357

C. Ischemia
D. Tumor
E. Viral infection
Explanation:
The correct answer is C. The distinctive areas of necrosis described in the question stem are
border zone (watershed) infarcts. They occur as the result of severe ischemia, which most
profoundly affects the relatively poorly perfused areas at the boundaries between major
arterial territories. The areas of necrosis described in the question stem are those most
commonly observed, although similar infarcts involving border zones in the brain stem are
sometimes also seen.
Bacterial infection (choice A) tends to produce either meningitis, cerebritis, or localized
infections such abscesses.
Fungal infection (choice B) tends to produce either meningitis, vasculitis, granulomas, or
abscesses.
Tumor (choice D) tends to produce localized masses.
Viral infection (choice E) may have a variety of patterns, including meningitis, diffuse
encephalitis, or localized necrosis, but would not be expected to produce a distinctive linear
pattern.
A 59-year-old woman with advanced, metastatic lung cancer develops profound fatigue and weakness and
alternating diarrhea and constipation. Physical examination demonstrates hyperpigmentation of
skin, even in areas protected from the sun. Tumor involvement of which endocrine organ is most
strongly suggested by this patient's presentation?
A. Adrenal gland
B. Endocrine pancreas
C. Ovaries
D. Pituitary gland
E. Thyroid gland
Explanation:
The correct answer is A. This is Addison disease, in which severe adrenal disease produces
adrenocortical insufficiency. Causes include autoimmune destruction, congenital adrenal
hyperplasia, hemorrhagic necrosis, and replacement of the glands by either tumor (usually
metastatic) or granulomatous disease (usually tuberculosis). The symptoms can be subtle and
nonspecific (such as those illustrated), so a high clinical index of suspicion is warranted.
Skin hyperpigmentation is a specific clue that may be present on physical examination,
suggesting excess pituitary ACTH secretion. (The ACTH precursor has an amino acid sequence
similar to MSH, melanocyte stimulating hormone.) Most patients have symptoms (fatigue,
gastrointestinal distress) related principally to glucocorticoid deficiency. In some cases,
however, mineralocorticoid replacement may also be needed for symptoms of salt wasting with
lower circulating volume.

358

Except in the case of primary pancreatic cancer, complete tumor replacement of the endocrine
pancreas (choice B) would be uncommon. In any event, pancreatic involvement would be associated
with diabetes mellitus.
Involvement of the ovaries (choice C) by metastatic tumor (classically gastric adenocarcinoma)
would produce failure of menstruation.
Involvement of the pituitary gland (choice D) could produce Addisonian symptoms, but the
pigmented skin suggests a primary adrenal problem rather than pituitary involvement.
Replacement of the thyroid gland (choice E) by tumor would cause hypothyroidism with lesser
degrees of fatigue and cold intolerance; this is an unusual cause of hypothyroidism and is less
likely to be tested than Addison disease caused by bilateral adrenal metastasis.

Anda mungkin juga menyukai